You are on page 1of 180

Academy ForumIAS | Dashboard 18/02/24, 8:41 PM

 Go Back

Test Completed In 01:39:34


Question: 1 Mark Flag  Previous Next 

Correct Answer : c
Your Answer : b

Consider the following rulers of ancient India:

1. Ajatashatru of Magadha

2. Prasenjit of Kosala

3. Bimbisara of Magadha

4. Mahapadma Nanda

How many of the above rulers were contemporaries of Buddha?

प्राचीन भारत के िनम्निलिखत शासकों पर िवचार करें:


1. मगध का अजातशत्रु
2. कोसल का प्रसेनिजत
3. मगध का िबिम्बसार
4. महापद्म नन्द

उपरोक्त शासकों में से िकतने बुद्ध के समकालीन थे?


Only one केवल
a एक
Only two केवल
b दो
Only three केवल
c तीन
All four सभी
d चार

Explanation
Option c is the correct answer

Gautama Buddha was born in Lumbini, Nepal, in 563 BCE.

https://academy.forumias.com/beta/lms/viewSolutions Page 1 of 180


Academy ForumIAS | Dashboard 18/02/24, 8:41 PM

Statement 1 is correct: Ajatashatru (Ruled - 492 BCE - 460 BCE) was contemporary to Lord Buddha. He was son of
Bimbisara. Lord Buddha died during his reign. He organized first Buddhist council immediately after Buddha's death.

Statement 2 is correct: Prasenjit of Kosala was contemporary to Lord Buddha. Prasenjit was the king of Kosala, a
kingdom in northern India. He was a contemporary of the Buddha and is mentioned in several Buddhist texts like
Samyutta nikaya. He was also a patron of the Buddha and is said to have invited him to Kosala on several occasions.

Statement 3 is correct: Bimbisara of Magadha (544-492 BCE) was contemporary to Buddha. Bimbisara was the king
of Magadha from 544 to 492 BCE. He is said to have invited the Buddha to his court and to have given him land for a
monastery.

Statement 4 is incorrect: Mahapadma Nanda was not contemporary to Lord Buddha. Mahapadma Nanda was the
first ruler of the Nanda dynasty, which ruled over Magadha in ancient India from 345 BCE.

Source: Poonam Dalal Dahiya - 'Ancient and Medieval India' chapter 4 " THE MAHAJANAPADAS OF THE INDO-
GANGETIC PLAIN (c. 600- 300 BCE)" Pg 130

िवकल्प c सही उत्तर है


गौतम बुद्ध का जन्म 563 ईसा पूवर् में नेपाल के लुंिबनी में हुआ था।
कथन 1 सही है: अजातशत्रु (शासनकाल - 492 ईसा पूवर् - 460 ईसा पूवर्) भगवान बुद्ध के समकालीन थे। वह िबिम्बसार का पुत्र था। इसके
शासनकाल के दौरान भगवान बुद्ध की मृत्यु हो गई। उन्होंने बुद्ध की मृत्यु के तुरत
ं बाद पहली बौद्ध पिरषद का आयोजन िकया।
कथन 2 सही है: कोशल का प्रसेनिजत भगवान बुद्ध का समकालीन था। प्रसेनिजत उत्तरी भारत के एक राज्य कोशल का राजा था। वह बुद्ध के
समकालीन थे और उनका उल्लेख संयुक्त िनकाय जैसे कई बौद्ध ग्रंथों में िकया गया है। वह बुद्ध का संरक्षक भी था और कहा जाता है िक उसने
कई अवसरों पर उन्हें कोसल में आमंित्रत िकया था।
कथन 3 सही है: मगध के िबिम्बसार (544-492 ईसा पूवर्) भगवान बुद्ध के समकालीन थे। िबिम्बसार 544 से 492 ईसा पूवर् तक मगध का
राजा था। ऐसा कहा जाता है िक उसने बुद्ध को अपने दरबार में आमंित्रत िकया था और उन्हें एक मठ के िलए जमीन दी थी।
कथन 4 गलत है: महापद्म नंद भगवान बुद्ध के समकालीन नहीं थे। महापद्म नंद, नंद वंश के पहले शासक थे, िजन्होंने 345 ईसा पूवर् से प्राचीन
भारत में मगध पर शासन िकया था।

Source: Poonam Dalal Dahiya - 'Ancient and Medieval India' chapter 4 " THE MAHAJANAPADAS OF THE
INDO-GANGETIC PLAIN (c. 600– 300 BCE)" Pg 130

Question Attempted / Student

Question: 2 Mark Flag  Previous Next 

Correct Answer : b
Your Answer : a

With reference to ancient India, consider the following statements about Megalithic sites:

https://academy.forumias.com/beta/lms/viewSolutions Page 2 of 180


Academy ForumIAS | Dashboard 18/02/24, 8:41 PM

1. Brahmagiri and Adichanallur are important megalithic sites in India.

2. They were exclusively located in Deccan and South India.

3. Iron tools and weapons were found in some of these sites.

How many of the statements given above are correct?

प्राचीन भारत के संदभर् में, महापाषाण स्थलों के बारे में िनम्निलिखत कथनों पर िवचार करें:
1. ब्रह्मिगिर और आिदचनल्लूर भारत में महत्वपूणर् महापाषाण स्थल थे।
2. वे िवशेष रूप से दक्कन और दिक्षण भारत में िस्थत थे।
3. इनमें से कुछ स्थलों पर लोहे के उपकरण और हिथयार पाए गए।

ऊपर िदए गए कथनों में से िकतने सही हैं?

Only one केवल


a एक
Only two केवल
b दो
All three
c तीनों
None कोई
d नहीं

Explanation
Option b is the correct answer

Megalithic sites are a type of archaeological site found in India. They are characterized by large stone structures, often
built in circular or oval shapes, that were used to bury the dead.

Statement 1 is correct: Brahmagiri and Adichanallur are important megalithic sites. At Brahmagiri, one skeleton was
buried with 33 gold beads, 2 stone beads, 4 copper bangles, and one conch shell. Other skeletons have only a few pots.
These finds suggest that there was some difference in status amongst the people who were buried. Some were rich, others
poor, some chiefs, others followers.

https://academy.forumias.com/beta/lms/viewSolutions Page 3 of 180


Academy ForumIAS | Dashboard 18/02/24, 8:41 PM

Statement 2 is incorrect: Megalithic burials in India were prevalent throughout the Deccan, south India, in the
north-east and Kashmir. Megalithic burials are found throughout Kashmir, and they are often associated with the region's
ancient cultures. The north-east of India is another region where megalithic burials sites are common.

Statement 3 is correct: In megalithic sites, the dead were generally buried with distinctive pots, which are called Black
and Red Ware. Tools and weapons of iron and sometimes, skeletons of horses, horse equipment and ornaments of stone
and gold have been found in these sites.

Source: https://ncert.nic.in/textbook.php?fess1=4-11

िवकल्प b सही उत्तर है


महापाषाण स्थल भारत में पाए जाने वाले एक प्रकार के पुराताित्वक स्थल हैं। इनकी िवशेषता बड़ी पत्थर की संरचनाएँ हैं, जो अक्सर
गोलाकार या अंडाकार आकार में बनाई जाती हैं, िजनका उपयोग मृतकों को दफनाने के िलए िकया जाता था।

कथन 1 सही है: ब्रह्मिगिर और आिदचनल्लूर महत्वपूणर् महापाषाण स्थल हैं। ब्रह्मिगिर में, एक कंकाल को 33 सोने के मोितयों, 2 पत्थर के
मोितयों, 4 तांबे की चूिड़यों और एक शंख के साथ दफनाया गया था। अन्य कंकालों में केवल कुछ ही बतर्न हैं। इन खोजों से पता चलता है िक
िजन लोगों को दफनाया गया था, उनकी िस्थित में कुछ अंतर था। कुछ अमीर थे, कुछ गरीब, कुछ मुिखया, कुछ अनुयायी।

https://academy.forumias.com/beta/lms/viewSolutions Page 4 of 180


Academy ForumIAS | Dashboard 18/02/24, 8:41 PM

कथन 2 गलत है: भारत में महापाषाण शवाधान पूरे दक्कन, दिक्षण भारत, उत्तर-पूवर् और कश्मीर में प्रचिलत थी। मेगािलिथक शवाधान पूरे
कश्मीर में पाई जाती हैं, और वे अक्सर क्षेत्र की प्राचीन संस्कृितयों से जुड़ी होती हैं। भारत का उत्तर-पूवर् एक और क्षेत्र है जहां
महापाषाणकालीन अंत्येिष्ट स्थल आम हैं।
कथन 3 सही है: महापाषाण स्थलों में, मृतकों को आम तौर पर िविशष्ट बतर्नों के साथ दफनाया जाता था, िजन्हें काले और लाल बतर्न कहा
जाता है। इन स्थलों से लोहे के औजार और हिथयार और कभी-कभी घोड़ों के कंकाल, घोड़े के उपकरण और पत्थर और सोने के आभूषण पाए
गए हैं।
Source: https://ncert.nic.in/textbook.php?fess1=4-11

Question Attempted / Student

Question: 3 Mark Flag  Previous Next 

Correct Answer : a
Your Answer : a

With reference to Indus Valley Civilisation (IVC), consider the following statements:

1. IVC people practised irrigation for agriculture.

2. Sugarcane was familiar to IVC people.

Which of the statements given above is/are correct?

िसं धु घाटी सभ्यता (IVC) के संदभर् में, िनम्निलिखत कथनों पर िवचार करें:

https://academy.forumias.com/beta/lms/viewSolutions Page 5 of 180


Academy ForumIAS | Dashboard 18/02/24, 8:41 PM

1. िसं धु घाटी सभ्यता के लोग कृिष के िलए िसं चाई से पिरिचत थे।
2. िसं धु घाटी सभ्यता के लोग गन्ने से पिरिचत थे।

ऊपर िदए गए कथनों में से कौन सा/से सही है/हैं?

1 only केवल
a 1
2 only केवल
b 2
Both 1 and 2 1 और 2
c दोनों
Neither 1 nor 2 न तो 1 और न ही
d 2

Explanation
Option a is the correct answer

The Indus Valley Civilization (IVC), also known as the Harappan Civilization, was a Bronze Age civilization in the
Indian subcontinent. It flourished in the basins of the Indus River, one of the major rivers of Asia, and its tributaries.

Statement 1 is correct: They were aware of the importance of irrigation for growing crops, and they developed
several methods for bringing water to their fields like wells, tanks etc. These methods allowed them to grow a wide variety
of crops, including wheat, barley, rice etc.

Statement 2 is incorrect: Sugarcane was not known to the people of the Indus Valley Civilization. Sugarcane is a
tropical plant that is native to Southeast Asia and IVC people were far away from Southeast Asia region.

िवकल्प a सही उत्तर है


िसं धु घाटी सभ्यता (आईवीसी), िजसे हड़प्पा सभ्यता के नाम से भी जाना जाता है, भारतीय उपमहाद्वीप में कांस्य युग की सभ्यता थी। यह
एिशया की प्रमुख निदयों में से एक िसं धु नदी और उसकी सहायक निदयों के घािटयों में फली-फूली।
कथन 1 सही है: वे फसल उगाने के िलए िसं चाई के महत्व से अवगत थे, और उन्होंने अपने खेतों में पानी लाने के िलए कई तरीके िवकिसत
िकए जैसे कुएं , टैंक आिद। इन तरीकों ने उन्हें गेहू,ं जौ, चावल आिद सिहत िविभन्न प्रकार की फसलों उगाने में मदद की।
कथन 2 गलत है: िसं धु घाटी सभ्यता के लोगों को गन्ने के बारे में जानकारी नहीं थी। गन्ना एक उष्णकिटबंधीय पौधा है जो मूल रूप से दिक्षण
पूवर् एिशया उगाया जाता था और िसं धु घाटी सभ्यता लोग दिक्षण पूवर् एिशया क्षेत्र से बहुत दू र थे।

Question Attempted / Student

Question: 4 Mark Flag  Previous Next 

https://academy.forumias.com/beta/lms/viewSolutions Page 6 of 180


Academy ForumIAS | Dashboard 18/02/24, 8:41 PM

Correct Answer : b
Your Answer : b

With reference to Gupta Empire, consider the following personalities:

1. Harisena

2. Vasubandhu

3. Kalidasa

How many of the poets/authors given above were contemporary to Samudragupta?

गुप्त साम्राज्य के संदभर् में, िनम्निलिखत व्यिक्तत्वों पर िवचार करें:


1. हिरसेना
2. वसुबन्धु
3. कािलदास

ऊपर िदए गए किवयों/लेखकों में से िकतने समुद्रगुप्त के समकालीन थे?

Only one केवल


a एक
Only two केवल
b दो
All three
c तीनों
None कोई
d नहीं

Explanation
Option b is the correct answer

SamudraGupta (c. 335-375 CE) was an emperor of the Gupta Empire. Samudra Gupta was a great warrior and conqueror.
He expanded the Gupta Empire to include most of northern and central India. He also defeated the Shakas, the Hunas, and
the Yavanas.

Statement 1 is correct: Harisena was contemporary to Samudra Gupta. Harisena was a poet and minister in the
court of the Gupta emperor, Samudragupta. He composed the "Allahabad Prashasti" in praise of Samudragupta.

Statement 2 is correct: Vasubandhu was contemporary to Samudra Gupta. Vasubandhu was a Buddhist philosopher.
Vasubandhu was minister at the court of Samudra Gupta. His greatest work is considered Abhidharmakosa.

https://academy.forumias.com/beta/lms/viewSolutions Page 7 of 180


Academy ForumIAS | Dashboard 18/02/24, 8:41 PM

Statement 3 is incorrect: Kalidasa was contemporary to Chandragupta II. Kalidasa was a Sanskrit poet and dramatist
who flourished in the 5th century CE. He is considered one of the greatest poets in the Sanskrit language. His some of
the famous work includes Abhijananashakuntalam, Malavikagnimitram,Vikramorvasiyam etc.

Source: Poonam Dalal Dahiya - 'Ancient and Medieval India' chapter 8 " THE GUPTAS AND THE VAKATAKAS (c.
300-600 CE) " Pg 330

िवकल्प b सही उत्तर है


समुद्रगुप्त (लगभग 335-375 ई.) गुप्त साम्राज्य के एक सम्राट थे। समुद्र गुप्त एक महान योद्धा और िवजेता था। उन्होंने गुप्त साम्राज्य का
िवस्तार करते हुए अिधकांश उत्तरी और मध्य भारत को इसमें शािमल कर िलया। उन्होंने शक, हूण और यवनों को भी हराया।
कथन 1 सही है: हिरसेन समुद्र गुप्त के समकालीन थे। हिरसेन गुप्त सम्राट समुद्रगुप्त के दरबार में एक किव और मंत्री थे। उन्होंने समुद्रगुप्त की
प्रशंसा में "इलाहाबाद प्रशिस्त" की रचना की।
कथन 2 सही है: वसुबंधु समुद्र गुप्त के समकालीन थे। वसुबन्धु एक बौद्ध दाशर्िनक थे। वसुबन्धु समुद्रगुप्त के दरबार में मंत्री थे। उनका सबसे
बड़ा काम अिभधमर्कोश माना जाता है।
कथन 3 गलत है: कािलदास चंद्रगुप्त िद्वतीय के समकालीन थे। कािलदास एक संस्कृत किव और नाटककार थे जो 5वीं शताब्दी ई.पू. में फले-
फूले। उन्हें संस्कृत भाषा के महानतम किवयों में से एक माना जाता है। उनके कुछ प्रिसद्ध कायोर्ं में अिभज्ञानशाकुंतलम, मालिवकािग्निमत्रम,
िवक्रमोवर्िशयम आिद शािमल हैं।

Source: Poonam Dalal Dahiya - 'Ancient and Medieval India' chapter 8 " THE GUPTAS AND THE VAKATAKAS
(c. 300-600 CE) " Pg 330

Question Attempted / Student

Question: 5 Mark Flag  Previous Next 

Correct Answer : d
Your Answer : b

Consider the following statements regarding the Indian Space Research Organisation (ISRO)'s XPoSat Mission':

Statement-I: It is a space observatory capable of detecting diverse electromagnetic waves from celestial sources.

Statement-II: It contributes to an improved understanding of astronomical phenomena such as black holes and neutron
stars.

Which one of the following is correct in respect of the above statements?

भारतीय अंतिरक्ष अनुसंधान संगठन (ISRO) के 'XPoSat िमशन' के संबंध में िनम्निलिखत कथनों पर िवचार करें:
कथन-I: यह एक अंतिरक्ष वेधशाला है जो आकाशीय स्रोतों से िविवध िवद्युत चुम्बकीय तरंगों का पता लगाने में सक्षम है।
कथन-II: यह ब्लैक होल और न्यूट्रॉन िसतारों जैसी खगोलीय घटनाओं की बेहतर समझ में योगदान देता है।

https://academy.forumias.com/beta/lms/viewSolutions Page 8 of 180


Academy ForumIAS | Dashboard 18/02/24, 8:41 PM

उपरोक्त कथनों के संबंध में िनम्निलिखत में से कौन सा सही है?

a
Both Statement-I and Statement-II are correct and Statement-II is the correct explanation for Statement-I. कथन-I और
कथन-II दोनों सही हैं और कथन-II कथन-I की सही व्याख्या है।
b
Both Statement-I and Statement-II are correct and Statement-II is not the correct explanation for Statement-I. कथन-I
और कथन-II दोनों सही हैं और कथन-II कथन-I के िलए सही व्याख्या नहीं है।
Statement-I is correct but Statement-II is incorrect. कथन-I सही है लेिकन कथन-II गलत
c है।
Statement-I is incorrect but Statement-II is correct. कथन-I गलत है लेिकन कथन-II सही
d है।

Explanation
Option d is the correct answer.

XPoSat (X-ray Polarimeter Satellite) is India's first dedicated polarimetry mission to study various dynamics of bright
astronomical X-ray sources in extreme conditions. The spacecraft will carry two scientific payloads in a low earth orbit -
POLIX (Polarimeter Instrument in X-rays) and XSPECT (X-ray Spectroscopy and Timing) to measure X-rays with
different energy ranges.

Statement-I is incorrect: The XPoSat mission is capable of detecting X-rays only, not diverse electromagnetic waves.
In contrast, the AstroSat mission, launched by ISRO in 2015, conducts multi-wavelength observations spanning radio,
optical, infrared (IR), ultraviolet (UV), and X-ray wavelengths.

Statement-II is correct: XPoSat is India's first dedicated polarimetry mission to study various dynamics of bright
astronomical x-ray sources in extreme conditions. This mission is instrumental in examining the X-ray radiation emitted
by nearby black hole and neutron star systems, with a specific focus on analyzing the polarization of the
radiation. Polarized X-rays have a preferred electric field orientation during propagation, determined by the motion
of the electric charge generating the wave. Measuring this polarization aids astronomers in studying objects emitting
polarized X-rays, like pulsars or areas around black holes, providing insights into their nature.

Source: https://www.isro.gov.in/XPoSat.html#:~:text=XPoSat (X-ray Polarimeter Satellite,in a low earth orbit.

https://economictimes.indiatimes.com/tech/technology/isro-to-launch-advanced-astronomy-observatory-xposat-
today/articleshow/106425369.cms?from=mdr#:~:text=ET that the-,mission,-will help study

https://www.thehindu.com/sci-tech/science/what-is-it-xposat-isros-x-ray-eye-in-the-
sky/article67697737.ece#:~:text=Premium&text=At 9.10 am on the,their polarisation from earth-orbit.

िवकल्प d सही उत्तर है।


XPoSat (एक्स-रे पोलािरमीटर सैटेलाइट) चरम िस्थितयों में उज्ज्वल खगोलीय एक्स-रे स्रोतों की िविभन्न गितशीलता का अध्ययन करने के
िलए भारत का पहला समिपर् त पोलािरमेट्री िमशन है। अंतिरक्ष यान िविभन्न ऊजार् श्रेिणयों के साथ एक्स-रे को मापने के िलए पृथ्वी की िनचली
कक्षा में दो वैज्ञािनक पेलोड - POLIX (एक्स-रे में पोलािरमीटर उपकरण) और XSPECT (एक्स-रे स्पेक्ट्रोस्कोपी और टाइिमं ग) ले जाएगा।

https://academy.forumias.com/beta/lms/viewSolutions Page 9 of 180


Academy ForumIAS | Dashboard 18/02/24, 8:41 PM

कथन-I गलत है: XPoSat िमशन केवल एक्स-रे का पता लगाने में सक्षम है, िविवध िवद्युत चुम्बकीय तरंगों का नहीं। इसके िवपरीत, 2015
में इसरो द्वारा लॉन्च िकया गया एस्ट्रोसैट िमशन, रेिडयो, ऑिप्टकल, इं फ्रारेड (आईआर), पराबैंगनी (यूवी), और एक्स-रे तरंग दैध्यर् में फैले बहु-
तरंग दैध्यर् अवलोकन करता है।
कथन-II सही है: XPoSat चरम िस्थितयों में उज्ज्वल खगोलीय एक्स-रे स्रोतों की िविभन्न गितशीलता का अध्ययन करने के िलए भारत का
पहला समिपर् त पोलािरमेट्री िमशन है। यह िमशन पास के ब्लैक होल और न्यूट्रॉन स्टार िसस्टम द्वारा उत्सिजर् त एक्स-रे िविकरण की जांच करने
में सहायक है, िजसमें िविकरण के ध्रुवीकरण का िवश्लेषण करने पर िवशेष ध्यान िदया गया है। ध्रुवीकृत एक्स-रे में प्रसार के दौरान एक
पसंदीदा िवद्युत क्षेत्र अिभिवन्यास होता है, जो तरंग उत्पन्न करने वाले िवद्युत आवेश की गित से िनधार्िरत होता है। इस ध्रुवीकरण को मापने से
खगोलिवदों को ध्रुवीकृत एक्स-रे उत्सिजर् त करने वाली वस्तुओ,ं जैसे पल्सर या ब्लैक होल के आसपास के क्षेत्रों का अध्ययन करने में मदद
िमलती है, िजससे उनकी प्रकृित के बारे में जानकारी िमलती है।
Source: https://www.isro.gov.in/XPoSat.html#:~:text=XPoSat%20(X%2Dray%20Polarimeter%20Satellite,in%20a%20low%
https://economictimes.indiatimes.com/tech/technology/isro-to-launch-advanced-astronomy-observatory-xposat-
today/articleshow/106425369.cms?from=mdr#:~:text=ET%20that%20the-,mission,-will%20help%20study

https://www.thehindu.com/sci-tech/science/what-is-it-xposat-isros-x-ray-eye-in-the-
sky/article67697737.ece#:~:text=Premium&text=At%209.10%20am%20on%20the,their%20polarisation%20from%20eart

Question Attempted / Student

Question: 6 Mark Flag  Previous Next 

Correct Answer : b
Your Answer : a

Consider the following pairs about the ambassadors/travelers and the rulers, under whom they visited India:

Ambassadors/Travellers Visited suring the reign


of

1. Fa-Hien Chandragupta Maurya

2. Hiuen Tsang Harshavardhana

3. Megasthenes Chandragupta II

4. Deimachus Bindusara

How many of the above pairs are correctly matched?

याित्रयों और शासकों के बारे में िनम्निलिखत युग्मों पर िवचार करें, िजनके अधीन उन्होंने भारत का दौरा िकया:

राजदू त/यात्री शासनकाल के दौरान यात्रा

फ़ािहयान चन्द्रगुप्त मौयर् के शासनकाल में

https://academy.forumias.com/beta/lms/viewSolutions Page 10 of 180


Academy ForumIAS | Dashboard 18/02/24, 8:41 PM

ह्वेनसांग हषर्वद्धर् न के शासनकाल में

मेगस्थनीज चंद्रगुप्त िद्वतीय के शासनकाल में

डाइमाचस िबन्दुसार के शासनकाल में

उपरोक्त में से िकतने युग्म सही सुमेिलत हैं?

Only one केवल एक


a युग्म
Only two केवल दो
b युग्म
Only three केवल तीन
c युग्म
All four सभी चार
d युग्म

Explanation
Option b is the correct answer.

Pair 1 is incorrect: Fa-Hien visited the court of Chandragupta II. Fa-Hien was a Chinese Buddhist monk who travelled
to India in the 5th century CE. Fa-Hien's visit is recorded in his travelogue "Fo-Kwo-Ki".

Pair 2 is correct: Hiuen Tsang visited the court of Harshavardhana. Hiuen Tsang was a Chinese Buddhist monk who
travelled to India in the 7th century CE. Harshavardhana was a patron of Buddhism, and Hiuen Tsang spent several months
at his court. During this time, he met with other Buddhist scholars and translated many Buddhist texts into Chinese.

Pair 3 is incorrect: Megasthenes was a Greek ambassador who visited the court of Chandragupta Maurya, the
founder of the Mauryan Empire. Megasthenes account of India is one of the earliest and most comprehensive descriptions
of the country by a foreign visitor. He compiled his description in a book called "Indica".

Pair 4 is correct: Deimachus was a Greek ambassador who visited the court of the Indian emperor Bindusara. He
was sent by the Antiochus I of Syria to establish diplomatic relations with the Mauryan Empire. Deimachus's visit is
significant because it marks the beginning of a period of increased contact between the Greek and Indian worlds.

Source: Poonam Dalal Dahiya - 'Ancient and Medieval India' chapter 6 " THE MAURYA EMPIRE (c. 324-187 BCE)" Pg
220

िवकल्प b सही उत्तर है


युग्म 1 गलत है: फाह्यान ने चंद्रगुप्त िद्वतीय के दरबार का दौरा िकया। फ़ा-िहयान एक चीनी बौद्ध िभक्षु थे िजन्होंने 5वीं शताब्दी ईस्वी में भारत
की यात्रा की थी। फ़ा- िहयान की यात्रा उनके यात्रा वृतांत "फो-क्वो-की" में दजर् है।
युग्म 2 सही है: ह्वेन त्सांग ने हषर्वद्धर् न के दरबार का दौरा िकया। ह्वेन त्सांग एक चीनी बौद्ध िभक्षु थे िजन्होंने 7वीं शताब्दी ईस्वी में भारत की
यात्रा की थी। हषर्वद्धर् न बौद्ध धमर् के संरक्षक थे और ह्वेन त्सांग ने उनके दरबार में कई महीने िबताए थे। इस दौरान उन्होंने अन्य बौद्ध िवद्वानों से
मुलाकात की और कई बौद्ध ग्रंथों का चीनी भाषा में अनुवाद िकया।

https://academy.forumias.com/beta/lms/viewSolutions Page 11 of 180


Academy ForumIAS | Dashboard 18/02/24, 8:41 PM

युग्म 3 गलत है: मेगस्थनीज एक यूनानी राजदू त था िजसने मौयर् साम्राज्य के संस्थापक चंद्रगुप्त मौयर् के दरबार का दौरा िकया था। भारत के
बारे में मेगस्थनीज का िववरण िकसी िवदेशी आगंतुक द्वारा देश के सबसे शुरुआती और सबसे व्यापक िववरणों में से एक है। उन्होंने अपना
िववरण “इिण्डका” नामक पुस्तक में संकिलत िकया।
युग्म 4 सही है: डाइमाचस एक यूनानी राजदू त था जो भारतीय सम्राट िबं दुसार के दरबार में आया था। उन्हें सीिरया के एं िटओकस प्रथम द्वारा
मौयर् साम्राज्य के साथ राजनियक संबंध स्थािपत करने के िलए भेजा गया था। डेमाचस की यात्रा महत्वपूणर् है क्योंिक यह ग्रीक और भारतीयों
के बीच बढ़ते संपकर् की अविध की शुरुआत का प्रतीक है।

Source: Poonam Dalal Dahiya - 'Ancient and Medieval India' chapter 6 " THE MAURYA EMPIRE (c. 324–187
BCE)" Pg 220

Question Attempted / Student

Question: 7 Mark Flag  Previous Next 

Correct Answer : a
Your Answer : a

Consider the following regions:

1. Kamrup

2. Taxila

3. Suvarnagiri

4. Ujjain

How many of the following regions did not form part of Ashoka's empire?

िनम्निलिखत क्षेत्रों पर िवचार करें:


1. कामरूप
2. तक्षिशला
3. सुवणर्िगिर
4. उज्जैन

िनम्निलिखत में से िकतने क्षेत्र अशोक के साम्राज्य का िहस्सा नहीं थे?

a Only one केवल एक


b Only two केवल दो
c Only three केवल तीन
d All four सभी चार

https://academy.forumias.com/beta/lms/viewSolutions Page 12 of 180


Academy ForumIAS | Dashboard 18/02/24, 8:41 PM

Explanation
Option a is the correct answer.

Option 1 is correct. Kamrup was not part of the Mauryan Empire. Kamrup is the modern region situated between
two rivers, the Manas and the Barnadi in Western Assam. The history of the Kamrup region dates back to the 4th
century under Kamarupa Kingdom. The kingdom was successively ruled by three dynasties - the Varman, the
Mlechchha (Mech) and the Pala dynasties.

Option 2 is incorrect. Taxila was part of the Mauryan Empire. It was conquered by Alexander in 327 BC and later on
came under the rule of the Mauryan dynasty. Under the king Asoka, the city reached its peak in terms of development.
Taxila after this saw the most creative period under the rule of Gandhara. In the next 200 years, Taxila became a center of
great learning.

Option 3 is incorrect. Suvarnagiri was part of the Mauryan Empire, which was ruled by Emperor Ashoka from
268-232 BCE. The Mauryan Empire was divided into four provinces, with Suvarnagiri serving as the capital of the
southern province. The other three provincial capitals were Tosali in the east, Ujjain in the west, and Taxila in the north.

Option 4 is incorrect. During the reign of Ashoka, Ujjain was part of the Mauryan empire. From the 6th to the 4th
centuries BCE, Ujjain, also known as Avantika, was the capital of the Aryan Avanti state. Ujjain was the seat of Ashoka,
the last Mauryan ruler, in the second century BCE. Ashoka was the viceroy of the Maurya empire's western regions, and
Ujjain was the capital of that province.

Source: Poonam Dalal Dahiya Ch-6

िवकल्प a सही उत्तर है।

https://academy.forumias.com/beta/lms/viewSolutions Page 13 of 180


Academy ForumIAS | Dashboard 18/02/24, 8:41 PM

िवकल्प 1 सही है : कामरूप मौयर् साम्राज्य का िहस्सा नहीं था। कामरूप पिश्चमी असम में दो निदयों मानस और बरनाडी के बीच िस्थत
आधुिनक क्षेत्र है। कामरूप क्षेत्र का इितहास कामरूप साम्राज्य के तहत चौथी शताब्दी का है। राज्य पर क्रिमक रूप से तीन राजवंशों - वमर्न,
म्लेच्छ (मेक) और पाल राजवंशों का शासन था।
िवकल्प 2 ग़लत है : तक्षिशला मौयर् साम्राज्य का िहस्सा था। इस पर 327 ईसा पूवर् में िसकंदर ने कब्ज़ा कर िलया था और बाद में यह मौयर्
वंश के शासन में आ गया। राजा अशोक के अधीन, शहर िवकास के मामले में अपने चरम पर पहुंच गया। इसके बाद तक्षिशला ने गांधार के
शासन का सबसे रचनात्मक काल देखा। अगले 200 वषोर्ं में तक्षिशला महान िशक्षा का केंद्र बन गया।
िवकल्प 3 ग़लत है. सुवणर्िगिर मौयर् साम्राज्य का िहस्सा था, िजस पर 268-232 ईसा पूवर् तक सम्राट अशोक का शासन था। मौयर् साम्राज्य
को चार प्रांतों में िवभािजत िकया गया था, िजसमें सुवणर्िगिर दिक्षणी प्रांत की राजधानी थी। अन्य तीन प्रांतीय राजधािनयाँ पूवर् में तोसाली,
पिश्चम में उज्जैन और उत्तर में तक्षिशला थीं।

िवकल्प 4 गलत है : अशोक के शासनकाल के दौरान, उज्जैन मौयर् साम्राज्य का िहस्सा था। छठी से चौथी शताब्दी ईसा पूवर् तक, उज्जैन,
िजसे अवंितका के नाम से भी जाना जाता है, आयर् अवंती राज्य की राजधानी थी। दू सरी शताब्दी ईसा पूवर् में उज्जैन अंितम मौयर् शासक
अशोक की गद्दी थी। अशोक मौयर् साम्राज्य के पिश्चमी क्षेत्रों का सूबेदार था और उज्जैन उस प्रांत की राजधानी थी।

Source: Poonam Dalal Dahiya Ch-6

Question Attempted / Student

Question: 8 Mark Flag  Previous Next 

Correct Answer : b

https://academy.forumias.com/beta/lms/viewSolutions Page 14 of 180


Academy ForumIAS | Dashboard 18/02/24, 8:41 PM

Your Answer :

With regard to the Satavahanas, consider the following statements:

1. The founder of the Satavahana dynasty was Simuka

2. The Sanjan Copper plate inscription describes the major achievements of Shatavahana rulers.

3. Kalyani on the west Deccan was an important port of the Satavahana kingdom.

How many of the above statements are correct?

सातवाहनों के संबंध में, िनम्निलिखत कथनों पर िवचार करें:


1. सातवाहन वंश का संस्थापक िसमुक था
2. संजन ताम्रपत्र िशलालेख में शतवाहन शासकों की प्रमुख उपलिब्धयों का वणर्न है।
3. दक्कन के पिश्चम में कल्याणी सातवाहन साम्राज्य का एक महत्वपूणर् बंदरगाह था।

उपरोक्त में से िकतने कथन सही हैं?

a Only one केवल एक


b Only two केवल दो
c All three तीनों
d None कोई नहीं

Explanation
Option b is the correct answer.

Option 1 is correct. Simuka is considered the founder of the Satavahana dynasty, and he played a significant role in
establishing the dynasty in the Deccan region.

Option 2 is incorrect. Sanjan copper plate inscription is an important source of information about Rashtrakuta
ruler Amoghavarsha I. The Sanjan inscriptions of 871 claim that Amoghavarsha I made a great effort to overthrow the
kingdom of the Dravidas and that the mobilization of his armies struck terror in the hearts of the kings of Kerala, Pandya,
Chola, Kalinga, Magadha, Gujarat, and Pallava.

Option 3 is correct. An important port of the Satavahanas was Kalyani on the western Deccan. Gandakasela,
Ganjam on the east coast were the other important seaports.

Source: Poonam Dalal Dahiya Ch-7

िवकल्प b सही उत्तर है


िवकल्प 1 सही है : िसमुक को सातवाहन राजवंश का संस्थापक माना जाता है और उसने दक्कन क्षेत्र में राजवंश की स्थापना में महत्वपूणर्
भूिमका िनभाई थी।

https://academy.forumias.com/beta/lms/viewSolutions Page 15 of 180


Academy ForumIAS | Dashboard 18/02/24, 8:41 PM

िवकल्प 2 ग़लत है : संजन ताम्रपत्र िशलालेख राष्ट्रकूट शासक अमोघवषर् प्रथम के बारे में जानकारी का एक महत्वपूणर् स्रोत है। 871 के संजन
िशलालेखों का दावा है िक अमोघवषर् प्रथम ने द्रिवड़ों के राज्य को उखाड़ फेंकने का एक बड़ा प्रयास िकया और उसकी सेनाओं की लामबंदी ने
केरल, पांड्य, चोल, किलं ग, मगध, गुजरात और पल्लव के राजाओं में दहशत पैदा कर िदया।
िवकल्प 3 सही है : सातवाहनों का एक महत्वपूणर् बंदरगाह पिश्चमी दक्कन में कल्याणी था। पूवीर् तट पर गंडकसेला, गंजाम अन्य महत्वपूणर्
बंदरगाह थे।

Source: Poonam Dalal Dahiya Ch-7

Question Attempted / Student

Question: 9 Mark Flag  Previous Next 

Correct Answer : b
Your Answer : c

With regard to Ancient Indian History, consider the following statements about Kanishka:

1. The Besnagar Inscription provides valuable information about him.

2. He is believed to have introduced the Saka era in 78 AD.

3. The fourth Buddhist Council has been held under his patronage.

How many of the above statements are correct?

प्राचीन भारतीय इितहास के संबंध में, किनष्क के बारे में िनम्निलिखत कथनों पर िवचार करें:
1. बेसनगर िशलालेख उसके बारे में बहुमूल्य जानकारी प्रदान करता है।
2. ऐसा माना जाता है िक उन्होंने 78 ईस्वी में शक युग की शुरुआत की थी।
3. चतुथर् बौद्ध संगीित का आयोजन इनके संरक्षण में हुआ है।

उपरोक्त में से िकतने कथन सही हैं?

a Only one केवल एक


b Only two केवल दो
c All three तीनों
d None कोई नहीं

Explanation
Option b is the correct answer.

https://academy.forumias.com/beta/lms/viewSolutions Page 16 of 180


Academy ForumIAS | Dashboard 18/02/24, 8:41 PM

Kanishka (127-150 CE) was the Kushan dynasty's most powerful ruler. The Kushan empire expanded from Central Asia to
the northern Indian subcontinent during his reign. Kanishka is well-known for being a great conqueror, religious and
cultural patron, and Mahayana Buddhist convert.

Option 1 is incorrect. The Rabatak Inscription in modern Baghran province, Afghanistan, is a significant source of
information about Kanishka.

The Besnagar Pillar Inscription of Heliodorus is a Prakrit inscription from the late 2nd century BCE. It provides
information about the history of the Bhāgavata Sect in India and the strong Vaishnavite movement in north India.

Option 2 is correct. Kanishka, the emperor of the Kushan dynasty in the second century, is believed to have
introduced the Saka era in 78 AD. The Saka era is also known as the Saka calendar or Sakabda. The Saka era is called
after the Saka people, a nomadic population from Central Asia. Kanishka is thought to have chosen the Saka era to honor
the Saka people and their contributions to the region.

Option 3 is correct. Passa, a Buddhist monk, advised Kanishka to start the fourth Buddhist Council. Vasumitra
presided over the council, which was held in Kundalvana, Kashmir, with Avaghosa as his deputy. The purpose of the
council was to organize the Sarvastivadin Abhidharma works, which had been translated from Prakrit into Sanskrit. In
addition, the council divided Buddhism into two sects: Mahayana and Hinayana.

Knowledge Base:

1) Vikram Samvat is a calendar based on the moon's movement and has 354 days in a year. It is named after King
Vikramaditya and marks the beginning of the Vikrama era, which prevailed in India and Nepal. The period starts at 57
B.C. and ends in 78 A.D.

Source: Poonam Dalal Dahiya Ch-7

िवकल्प b सही उत्तर है।


किनष्क (127-150 ई.) कुषाण वंश का सबसे शिक्तशाली शासक था। उनके शासनकाल के दौरान कुषाण साम्राज्य का िवस्तार मध्य एिशया
से उत्तरी भारतीय उपमहाद्वीप तक हुआ। किनष्क एक महान िवजेता, धािमर् क और सांस्कृितक संरक्षक और महायान बौद्ध धमार्ंतिरत होने के िलए
प्रिसद्ध है।
िवकल्प 1 ग़लत है : आधुिनक बाघरान प्रांत, अफगािनस्तान में रबातक िशलालेख किनष्क के बारे में जानकारी का एक महत्वपूणर् स्रोत है।
हेिलयोडोरस का बेसनगर स्तंभ िशलालेख दू सरी शताब्दी ईसा पूवर् के अंत का एक प्राकृत िशलालेख है। यह भारत में भागवत संप्रदाय के
इितहास और उत्तर भारत में मजबूत वैष्णव आं दोलन के बारे में जानकारी प्रदान करता है।
िवकल्प 2 सही है : माना जाता है िक दू सरी शताब्दी में कुषाण वंश के सम्राट किनष्क ने 78 ईस्वी में शक युग की शुरुआत की थी। शक संवत
को शक कैलेंडर या सकब्दा के नाम से भी जाना जाता है। शक युग का नाम मध्य एिशया की खानाबदोश आबादी शक लोगों के नाम पर रखा
गया है। ऐसा माना जाता है िक किनष्क ने शक लोगों और क्षेत्र में उनके योगदान का सम्मान करने के िलए शक युग को चुना था।
िवकल्प 3 सही है : एक बौद्ध िभक्षु पासा ने किनष्क को चौथी बौद्ध पिरषद शुरू करने की सलाह दी। वसुिमत्र ने पिरषद की अध्यक्षता की, जो
कश्मीर के कुंडलवन में आयोिजत की गई थी, िजसमें अवघोष उनके प्रितिनिध थे। पिरषद का उद्देश्य सवार्िस्तवािदन अिभधमर् कायोर्ं को
व्यविस्थत करना था, िजनका प्राकृत से संस्कृत में अनुवाद िकया गया था। इसके अलावा, पिरषद ने बौद्ध धमर् को दो संप्रदायों में िवभािजत
िकया: महायान और हीनयान।
ज्ञानधार:
1) िवक्रम संवत चंद्रमा की गित पर आधािरत एक कैलेंडर है और इसमें वषर् में 354 िदन होते हैं। इसका नाम राजा िवक्रमािदत्य के नाम पर रखा
गया है और यह िवक्रम युग की शुरुआत का प्रतीक है, जो भारत और नेपाल में प्रचिलत था। अविध 57 ईसा पूवर् से शुरू होती है और 78 ईस्वी
में समाप्त होती है।

https://academy.forumias.com/beta/lms/viewSolutions Page 17 of 180


Academy ForumIAS | Dashboard 18/02/24, 8:41 PM

Source: Poonam Dalal Dahiya Ch-7

Question Attempted / Student

Question: 10 Mark Flag  Previous Next 

Correct Answer : b
Your Answer : b

Which one of the following best reflects the meaning of the term 'Domestic Systemically Important Banks (D-SIBs)'?

िनम्निलिखत में से कौन सा 'घरेलू प्रणालीगत महत्त्वपूणर् बैंक (D-SIBs)' शब्द के अथर् का सवार्िधक उपयुक्त वणर्न है?

a
Banks that have secured customer confidence owing to their robust performance. वे बैंक िजन्होंने अपने मजबूत प्रदशर्न के
कारण ग्राहकों का िवश्वास सुरिक्षत िकया है।
b
Banks that have the capacity to disrupt the financial system if they face failure. ऐसे बैंक जो िवफलता का सामना करने पर
िवत्तीय प्रणाली को बािधत करने की क्षमता रखते हैं।
c
Banks that exhibit a higher return on investment and lower Non-Performing Assets (NPAs). वे बैंक जो िनवेश पर अिधक
िरटनर् और कम गैर-िनष्पािदत पिरसंपित्तयां (एनपीए) प्रदिशर् त करते हैं।
d
Banks that majorly operate within India and are resilient to external shocks. वे बैंक जो मुख्य रूप से भारत के भीतर संचािलत
होते हैं और बाहरी झटकों के प्रित लचीले होते हैं।

Explanation
Option b is the correct answer.

The Reserve Bank of India (RBI) has listed State Bank of India, HDFC Bank, and ICICI Bank as Domestic Systemically
Important Banks (D-SIBs). SBI moved from bucket 3 to 4, HDFC Bank shifted from bucket 1 to 2, while ICICI Bank
retained its bucket 1 status. RBI places these banks in appropriate buckets depending upon their Systemic Importance
Scores (SISs). Based on the bucket in which a D-SIB is placed, an additional Common Equity Requirement (it is a
measurement of a bank's core equity capital, compared with its total risk-weighted assets) has to be applied to it.

Option b is correct: Domestic Systemically Important Banks (D-SIBs) are financial institutions whose potential failure
could significantly disrupt the financial system. This financial risk emerges given the size, cross-jurisdictional activities,
complexity, lack of substitutability, and interconnectedness of the D-SIBs. These Systemically Important Banks (SIBs) are
often labeled as 'Too Big to Fail (TBTF)' due to the critical role they play in the stability of the financial system.

https://academy.forumias.com/beta/lms/viewSolutions Page 18 of 180


Academy ForumIAS | Dashboard 18/02/24, 8:41 PM

Option a, c and d are incorrect: Domestic Systemically Important Banks (D-SIBs) are not identified based on
customer confidence, return on investment, or resilience to external shocks. Instead, they are primarily categorized
based on their potential to adversely impact a nation's economy in the event of their failure.

Source: https://www.thehindu.com/business/rbi-tags-sbi-icici-bank-hdfc-bank-as-systemically-important-
banks/article66330854.ece

िवकल्प b सही उत्तर है


भारतीय िरजवर् बैंक (आरबीआई) ने भारतीय स्टेट बैंक, एचडीएफसी बैंक और आईसीआईसीआई बैंक को घरेलू प्रणालीगत रूप से महत्वपूणर्
बैंकों (डी-एसआईबी) के रूप में सूचीबद्ध िकया है। एसबीआई बकेट 3 से 4 पर आ गया, एचडीएफसी बैंक बकेट 1 से 2 पर आ गया, जबिक
आईसीआईसीआई बैंक ने बकेट 1 का दजार् बरकरार रखा। आरबीआई इन बैंकों को उनके प्रणालीगत महत्व स्कोर (एसआईएस) के आधार पर
उिचत श्रेिणयों में रखता है। उस बकेट के आधार पर िजसमें डी-एसआईबी रखा गया है, एक अितिरक्त सामान्य इिक्वटी आवश्यकता (यह बैंक
की कुल जोिखम-भािरत पिरसंपित्तयों की तुलना में इसकी मुख्य इिक्वटी पूंजी का माप है) को इस पर लागू िकया जाना है।
िवकल्प b सही है: घरेलू प्रणालीगत रूप से महत्वपूणर् बैंक (डी-एसआईबी) िवत्तीय संस्थान हैं िजनकी संभािवत िवफलता िवत्तीय प्रणाली को
महत्वपूणर् रूप से बािधत कर सकती है। यह िवत्तीय जोिखम डी-एसआईबी के आकार, अंतर-क्षेत्रािधकार गितिविधयों, जिटलता, प्रितस्थापन
क्षमता की कमी और परस्पर जुड़ाव को देखते हुए उभरता है। िवत्तीय प्रणाली की िस्थरता में महत्वपूणर् भूिमका िनभाने के कारण इन व्यविस्थत
रूप से महत्वपूणर् बैंकों (एसआईबी) को अक्सर 'टू िबग टू फेल (टीबीटीएफ)' के रूप में लेबल िकया जाता है।
िवकल्प a, c और d गलत हैं: घरेलू प्रणालीगत महत्वपूणर् बैंकों (डी-एसआईबी) की पहचान ग्राहकों के िवश्वास, िनवेश पर िरटनर् या बाहरी
झटकों के प्रित लचीलेपन के आधार पर नहीं की जाती है। इसके बजाय, उन्हें मुख्य रूप से उनकी िवफलता की िस्थित में िकसी देश की
अथर्व्यवस्था पर प्रितकूल प्रभाव डालने की उनकी क्षमता के आधार पर वगीर्कृत िकया जाता है।

Source: https://www.thehindu.com/business/rbi-tags-sbi-icici-bank-hdfc-bank-as-systemically-important-
banks/article66330854.ece

Question Attempted / Student

Question: 11 Mark Flag  Previous Next 

Correct Answer : a
Your Answer : c

Consider the following pairs with regard to the important ports and control of the various dynasties:

Port Controlled by

1. Korkai Pandyas

2. Tondi Cholas

3. Muchiri Kakatiyas

4. Motupalli Cheras

https://academy.forumias.com/beta/lms/viewSolutions Page 19 of 180


Academy ForumIAS | Dashboard 18/02/24, 8:41 PM

How many of the above pairs are correctly matched?

िविभन्न राजवंशों के महत्वपूणर् बंदरगाहों और िनयंत्रण के संबंध में िनम्निलिखत युग्मों पर िवचार करें:

बंदरगाह िनयंत्रण

1. कोरकाई पंड्या द्वारा

2. टोंडी चोल द्वारा

3. मुिचरी काकतीय द्वारा

4. मोटु पल्ली चेरस द्वारा

उपरोक्त में से िकतने युग्म सही सुमेिलत हैं?

a Only one केवल एक


b Only two केवल दो
c Only three केवल तीन
d All four सभी चार

Explanation
Option a is the correct answer.

Option a is correct. Korkai was a major port and the capital of the early Pandyan Kingdom in ancient South India.
It was located at the confluence of the Tamiraparani and the Bay of Bengal, and was famous for pearl fishery and chank
diving. Korkai was also a central hub for trade with the Roman Empire.

Options b and c are incorrect. Muchiri (Muziris) and Tondi was a major port city of the Cheras, located on the
Malabar Coast in present-day Kerala. Muchiri was a significant trading center from at least the 1st century BC, and was
known for exporting pepper, cardamom, and other spices.

Option d is incorrect. Motupalli is one of India's oldest port towns, dating back to at least the 2nd century AD. It
became prominent during the Kakatiyas' medieval kingdom, which ruled the region from the 12th to 14th centuries.

1) Poompuhar, also known as Puhar and Kaveripattinam, is considered to be the Chola Empire's important port town.

Arabiansea

Source : Poonam Dalal Dahiya Ch-7

https://academy.forumias.com/beta/lms/viewSolutions Page 20 of 180


Academy ForumIAS | Dashboard 18/02/24, 8:41 PM

िवकल्प a सही उत्तर है।


िवकल्प a सही है : कोरकाई प्राचीन दिक्षण भारत में एक प्रमुख बंदरगाह और प्रारंिभक पांिडयन साम्राज्य की राजधानी थी। यह तमीरापारानी
और बंगाल की खाड़ी के संगम पर िस्थत था, और मोती मछली पकड़ने और चैंक डाइिवं ग के िलए प्रिसद्ध था। कोरकाई रोमन साम्राज्य के साथ
व्यापार का एक केंद्रीय केंद्र भी था।
िवकल्प b और c गलत हैं। मुिचरी (मुिज़िरस) और टोंडी चेरों का एक प्रमुख बंदरगाह शहर था, जो वतर्मान केरल में मालाबार तट पर िस्थत
था। मुिचरी कम से कम पहली शताब्दी ईसा पूवर् से एक महत्वपूणर् व्यापािरक केंद्र था, और काली िमचर्, इलायची और अन्य मसालों के िनयार्त
के िलए जाना जाता था।
िवकल्प d गलत है. मोटु पल्ली भारत के सबसे पुराने बंदरगाह शहरों में से एक है, जो दू सरी शताब्दी ईस्वी पूवर् का है। यह काकतीय
मध्यकालीन साम्राज्य के दौरान प्रमुख हो गया, िजसने 12वीं से 14वीं शताब्दी तक इस क्षेत्र पर शासन िकया।

1) पूमपुहार, िजसे पुहार और कावेरीपिट्टनम के नाम से भी जाना जाता है, चोल साम्राज्य का महत्वपूणर् बंदरगाह शहर माना जाता है।

Source: : Poonam Dalal Dahiya Ch-7

Question Attempted / Student

Question: 12 Mark Flag  Previous Next 

Correct Answer : c
Your Answer : c

It is a classical Sanskrit play that is traditionally attributed to the ancient Indian playwright Vishakadatta. The play is a
historical drama that revolves around the political events during the rise of Chandragupta Maurya, the founder of the
Maurya Empire.

The passage above refers to which of the following texts?

यह एक शास्त्रीय संस्कृत नाटक है िजसका श्रेय परंपरागत रूप से प्राचीन भारतीय नाटककार िवशाखदत्त को िदया जाता है। यह नाटक एक
ऐितहािसक नाटक है जो मौयर् साम्राज्य के संस्थापक चंद्रगुप्त मौयर् के उदय के दौरान हुई राजनीितक घटनाओं के इदर्-िगदर् घूमता है।

उपरोक्त पिरच्छे द िनम्निलिखत में से िकस ग्रन्थ को संदिभर् त करता है?

https://academy.forumias.com/beta/lms/viewSolutions Page 21 of 180


Academy ForumIAS | Dashboard 18/02/24, 8:41 PM

a Rajatarangini राजतरंिगणी
b Parishishtaparvan पिरिशष्टपवर्ण
c Mudrarakshasa मुद्राराक्षस
d Kadambari कादंबरी

Explanation
Option c is the correct answer.

Vishakhadatta was a Sanskrit poet and playwright who existed during the Gupta dynasty. He was a well-known personality
and is regarded as one of the best poets and playwrights of his day.

Option a is incorrect. The Rajatarangini (literally, River of Kings) is an epic poem (mahakavya/prabandha) composed in
the classical language, Sanskrit, in 1148-50 CE in Kashmir. It was composed by a Kashmiri poet, named Kalhana.

Option b is incorrect. Hemachandra's "Parishishtaparvan" is a Jain supplementary text to the "Mahabharata,"


offering anecdotes, teachings, and diverse content. It reflects Jain perspectives on Hindu epics, showcasing Hemachandra's
contributions to medieval Indian literature and philosophy through his prolific writings and Jain adaptations of ancient
texts.

Option c is correct. Mudrarakshasa is a classical Sanskrit play by Vishakadatta, centers on the political rise of
Chandragupta Maurya. It is believed to have been composed in the 4th-5th century CE, it delves into the historical
drama of the Maurya Empire, emphasizing Chanakya's strategic brilliance as the chief advisor.

Option d is incorrect. Banabhatta, a scholar from the 7th century CE during the Gupta Empire, wrote
"Kadambari," an early novel. It's a romantic story featuring Prince Chandrapeeda and Princess Kadambari, woven
within a complex narrative with court intrigue and divine elements. The novel, known for its rich language, blends prose
and poetry and explores life's transience and spiritual quests. A parrot named Shuka narrates the story to comfort a
grieving queen, incorporating philosophical themes of fate and meaning.

Source: Poonam Dalal Dahiya- Ch- 6. The Maurya Empire

िवकल्प c सही उत्तर है।


िवशाखदत्त एक संस्कृत किव और नाटककार थे जो गुप्त राजवंश के दौरान अिस्तत्व में थे। वह एक प्रिसद्ध व्यिक्तत्व थे और उन्हें अपने समय
के सवर्श्रेष्ठ किवयों और नाटककारों में से एक माना जाता है।
िवकल्प a ग़लत है : राजतरंिगणी (शािब्दक रूप से, राजाओं की नदी) एक महाकाव्य (प्रबंध) है, जो 1148-50 ईस्वी में कश्मीर में शास्त्रीय
भाषा, संस्कृत में िलखी गई थी। इसकी रचना कल्हण नामक कश्मीरी किव ने की थी।
िवकल्प b ग़लत है : हेमचंद्र का "पिरिशष्टपवर्न" "महाभारत" का एक जैन पूरक ग्रन्थ है, जो उपाख्यानों, िशक्षाओं और िविवध सामग्री की
पेशकश करता है। यह िहं दू महाकाव्यों पर जैन दृिष्टकोण को दशार्ता है, हेमचंद्र के िवपुल लेखन और प्राचीन ग्रंथों के जैन रूपांतरण के माध्यम
से मध्ययुगीन भारतीय सािहत्य और दशर्न में उनके योगदान को प्रदिशर् त करता है।
िवकल्प c सही है : मुद्राराक्षस िवशाखादत्त का एक शास्त्रीय संस्कृत नाटक है, जो चंद्रगुप्त मौयर् के राजनीितक उत्थान पर केंिद्रत है। ऐसा माना
जाता है िक इसकी रचना चौथी-पांचवीं शताब्दी ईस्वी में की गई थी, यह मौयर् साम्राज्य के ऐितहािसक नाटक का िववरण देता है, िजसमें मुख्य
सलाहकार के रूप में चाणक्य की रणनीितक प्रितभा पर जोर िदया गया है।

https://academy.forumias.com/beta/lms/viewSolutions Page 22 of 180


Academy ForumIAS | Dashboard 18/02/24, 8:41 PM

िवकल्प d गलत है : गुप्त साम्राज्य के दौरान 7वीं शताब्दी ईस्वी के िवद्वान बाणभट्ट ने एक प्रारंिभक उपन्यास "कादंबरी" िलखा था। यह
राजकुमार चंद्रपीड और राजकुमारी कादंबरी की एक रोमांिटक कहानी है, जो दरबारी सािज़श और दैवीय तत्वों के उल्लेख की गई है। यह
उपन्यास, जो अपनी समृद्ध भाषा के िलए जाना जाता है, गद्य और पद्य का िमश्रण है और जीवन की क्षणभंगुरता और आध्याित्मक खोजों की
पड़ताल करता है। शुका नाम का एक तोता एक दुखी रानी को सांत्वना देने के िलए कहानी सुनाता है, िजसमें भाग्य और अथर् के दाशर्िनक
िवषयों को शािमल िकया गया है।

Source: Poonam Dalal Dahiya- Ch- 6. The Maurya Empire

Question Attempted / Student

Question: 13 Mark Flag  Previous Next 

Correct Answer : a
Your Answer : d

Consider the following statements about Nalanda University:

1. It was founded by Gupta ruler Kumaragupta I.

2. The prominent scholars Chanakya and Panini studied in this university.

Which of the statements given above is/are correct?

नालंदा िवश्विवद्यालय के बारे में िनम्निलिखत कथनों पर िवचार करें:


1. इसकी स्थापना गुप्त शासक कुमारगुप्त प्रथम ने की थी।
2. प्रमुख िवद्वान चाणक्य और पािणिन ने इसी िवश्विवद्यालय में अध्ययन िकया था।

ऊपर िदए गए कथनों में से कौन सा/से सही है/हैं?

a 1 only केवल 1
b 2 only केवल 2
c Both 1 and 2 1 और 2 दोनों
d Neither 1 nor 2 न तो 1 और न ही 2

Explanation
Option a is the correct answer.

Nalanda University, established in the 5th century CE during the Gupta period, was a renowned ancient center of Buddhist
learning in India, fostering intellectual pursuits and attracting scholars worldwide.

https://academy.forumias.com/beta/lms/viewSolutions Page 23 of 180


Academy ForumIAS | Dashboard 18/02/24, 8:41 PM

Statement 1 is correct. In the fifth century AD, Kumargupta-I, commonly known as Shakraditya, established Nalanda
University. Many scholars and students, notably Hiuen Tsang, came to study and gather Buddhist writings at the university.

Statement 2 is incorrect. While Chanakya and Panini were renowned scholars in ancient India, they are not historically
associated with Nalanda University. They were associated with Taxila University.

Knowledge Base:

Some notable figures associated with Nalanda University include:

1) Nagarjuna: A Mahayana philosopher and one of the Seventeen Nalanda Masters

2) Dinnaga: A Buddhist logician and one of the Seventeen Nalanda Masters

3) Xuanzang: A Chinese traveler

4) Aryabhata: An Indian mathematician

5) Harshavardhana: An Indian emperor

6) Atiśa: A Buddhist master

Several notable personalities associated with Taxila University include:

1) Chanakya (Kautilya): Chanakya, also known as Kautilya or Vishnugupta, was a teacher and mentor to Chandragupta
Maurya.

2) Panini: An ancient Sanskrit grammarian and linguist, Panini is known for his work "Ashtadhyayi," a comprehensive and
systematic description of Sanskrit grammar.

3) Jivaka: Jivaka, also known as Jivaka Kumar Bhaccha, was a renowned physician and a contemporary of Gautama
Buddha. He is considered one of the earliest known surgeons in the world.

4) Charaka: Charaka was an ancient Indian physician and the author of the "Charaka Samhita," an important Ayurvedic
treatise.

5) Dhanvantari: an ancient physician and surgeon, is considered the god of Ayurveda.

Source: Poonam Dalal Dahiya- Ch- 8.

िवकल्प a सही उत्तर है।


गुप्त काल के दौरान 5वीं शताब्दी ईस्वी में स्थािपत नालंदा िवश्विवद्यालय, भारत में बौद्ध िशक्षा का एक प्रिसद्ध प्राचीन केंद्र था, जो बौिद्धक
गितिविधयों को बढ़ावा देता था और दुिनया भर के िवद्वानों को आकिषर् त करता था।
कथन 1 सही है : पांचवीं शताब्दी ईस्वी में, कुमारगुप्त-प्रथम, िजसे आमतौर पर शक्रािदत्य के नाम से जाना जाता है, ने नालंदा िवश्विवद्यालय
की स्थापना की। कई िवद्वान और छात्र, िवशेष रूप से ह्वेन त्सांग, िवश्विवद्यालय में बौद्ध लेखन का अध्ययन करने और जानकारी इकट्ठा करने
के िलए आए थे।
कथन 2 गलत है : जबिक चाणक्य और पािणिन प्राचीन भारत में प्रिसद्ध िवद्वान थे, वे ऐितहािसक रूप से नालंदा िवश्विवद्यालय से जुड़े नहीं हैं।
वे तक्षिशला िवश्विवद्यालय से जुड़े थे।
ज्ञानधार:
नालंदा िवश्विवद्यालय से जुड़ी कुछ उल्लेखनीय हिस्तयों में शािमल हैं:
1) नागाजुर्न: एक महायान दाशर्िनक और सत्रह नालंदा गुरुओं में से एक

https://academy.forumias.com/beta/lms/viewSolutions Page 24 of 180


Academy ForumIAS | Dashboard 18/02/24, 8:41 PM

2) िदन्नागा: एक बौद्ध तकर्शास्त्री और सत्रह नालंदा गुरुओं में से एक


3) ह्वेनसांग: एक चीनी यात्री
4) आयर्भट्ट: एक भारतीय गिणतज्ञ
5) हषर्वधर्न: एक भारतीय सम्राट
6) आितशा: एक बौद्ध गुरु
तक्षिशला िवश्विवद्यालय से जुड़ी कई उल्लेखनीय हिस्तयों में शािमल हैं:
1) चाणक्य (कौिटल्य) : चाणक्य, िजन्हें कौिटल्य या िवष्णुगुप्त के नाम से भी जाना जाता है, चंद्रगुप्त मौयर् के िशक्षक और गुरु थे।
2) पािणनी: एक प्राचीन संस्कृत व्याकरणिवद् और भाषािवद्, पािणिन को उनके काम "अष्टाध्यायी" के िलए जाना जाता है, जो संस्कृत
व्याकरण का एक व्यापक और व्यविस्थत िववरण है।
3) जीवक: जीवक, िजन्हें जीवक कुमार भच्चा के नाम से भी जाना जाता है, एक प्रिसद्ध िचिकत्सक और गौतम बुद्ध के समकालीन थे। उन्हें
दुिनया के सबसे शुरुआती ज्ञात सजर्नों में से एक माना जाता है।
4) चरक: चरक एक प्राचीन भारतीय िचिकत्सक और एक महत्वपूणर् आयुवेर्िदक ग्रंथ "चरक संिहता" के लेखक थे।
5) धन्वंतिर: एक प्राचीन िचिकत्सक और सजर्न, आयुवेर्द के देवता माने जाते हैं।

Source: Poonam Dalal Dahiya- Ch- 8.

Question Attempted / Student

Question: 14 Mark Flag  Previous Next 

Correct Answer : b
Your Answer : b

With reference to Harsha Vardhana's contribution towards literature, consider the following statements:

1. Harsha authored Sanskrit plays Nagananda and Priyadarshika.

2. The ancient poet Banabhatta was patronized by Harsha.

3. Buddhacarita, a Buddhist text, written during Harsha's period.

How many of the above statements are correct?

सािहत्य के प्रित हषर् वधर्न के योगदान के संदभर् में, िनम्निलिखत कथनों पर िवचार करें:
1. हषर् ने संस्कृत नाटक नागानंद और िप्रयदिशर् का की रचना की।
2. प्राचीन किव बाणभट्ट को हषर् का संरक्षण प्राप्त था।
3. बुद्धचिरत, एक बौद्ध ग्रंथ, जो हषर् के काल में िलखा गया था।

उपरोक्त में से िकतने कथन सही हैं?

https://academy.forumias.com/beta/lms/viewSolutions Page 25 of 180


Academy ForumIAS | Dashboard 18/02/24, 8:41 PM

a Only one केवल एक


b Only two केवल दो
c All three तीनों
d None कोई नहीं

Explanation
Option b is the correct answer.

Harsha Vardhana, commonly known as Harsha, was a prominent ruler of North India during the 7th century CE. Harsha
patronised literary and cultural activities. It is said the state spent a quarter of its revenue for such activities.

Statement 1 is correct. Harsha himself was an author and wrote three Sanskrit plays, Nagananda, Ratnavali, and
Priyadarshika. One-fourth of his revenue went for patronizing scholars.

Statement 2 is correct. Harshavardhana patronized the Sanskrit scholar Banabhatta. Banabhatta wrote the biography of
Harsha known as Harshacharita in Sanskrit. This gives us the genealogy of Harsha.

Statement 3 is incorrect. Ashvaghosha (80AD - 150AD) (Harsha lived during 7th century CE) was a philosopher and poet
who is regarded as the pioneer of Sanskrit play. Buddhacarita, poetic narrative of the life of the Buddha by the
Sanskrit poet Ashvaghosha, one of the finest examples of Buddhist literature.

Source: Poonam Dalal Dahiya- Ch- 9

िवकल्प b सही उत्तर है


हषर् वधर्न, िजसे आमतौर पर हषर् के नाम से जाना जाता है, 7वीं शताब्दी ईस्वी के दौरान उत्तर भारत का एक प्रमुख शासक था। हषर् ने
सािहित्यक एवं सांस्कृितक गितिविधयों को संरक्षण िदया। ऐसा कहा जाता है िक राज्य ने अपने राजस्व का एक चौथाई िहस्सा ऐसी
गितिविधयों पर खचर् िकया।
कथन 1 सही है : हषर् स्वयं एक लेखक थे और उन्होंने तीन संस्कृत नाटक नागानंद, रत्नावली और िप्रयदिशर् का िलखे। उसके राजस्व का एक-
चौथाई भाग िवद्वानों को संरक्षण देने में चला जाता था।
कथन 2 सही है : हषर्वद्धर् न ने संस्कृत िवद्वान बाणभट्ट को संरक्षण िदया। बाणभट्ट ने हषर् की जीवनी संस्कृत में हषर्चिरत के नाम से िलखी।
इससे हमें हषर् की वंशावली िमलती है।
कथन 3 ग़लत है : अश्वघोष (80 ई. - 150 ई.) (हषर् 7वीं शताब्दी ई. के दौरान हुए) एक दाशर्िनक और किव थे िजन्हें संस्कृत नाटक का
प्रणेता माना जाता है। बुद्धचिरत, संस्कृत किव अश्वघोष द्वारा बुद्ध के जीवन की काव्यात्मक कथा, बौद्ध सािहत्य के बेहतरीन उदाहरणों में से
एक है।

Source: Poonam Dalal Dahiya- Ch- 9

Question Attempted / Student

Question: 15 Mark Flag  Previous Next 

https://academy.forumias.com/beta/lms/viewSolutions Page 26 of 180


Academy ForumIAS | Dashboard 18/02/24, 8:41 PM

Correct Answer : c
Your Answer : d

Recently the Hon'ble Prime Minister inaugurated the Maharishi Valmiki International Airport in Ayodhya, Uttar Pradesh.
Which one of the following statements correctly represents the teachings of 'Maharishi Valmiki'?

हाल ही में माननीय प्रधान मंत्री ने उत्तर प्रदेश के अयोध्या में महिषर् वािल्मकी अंतरार्ष्ट्रीय हवाई अड्डे का उद्घाटन िकया। िनम्निलिखत में से कौन सा कथन
'महिषर् वािल्मकी' की िशक्षाओं का सही प्रितिनिधत्व करता है?

a
Achieving salvation was possible only through conducting elaborate rituals. िवस्तृत अनुष्ठानों के आयोजन से मोक्ष प्राप्त
करना संभव था।
b
Individuals must renounce all their wealth and desire to achieve salvation. व्यिक्तयों को मोक्ष प्राप्त करने के िलए अपनी
सारी संपित्त और इच्छा का त्याग करना होगा।
c
Salvation lies in the moderate approach in one's pursuit of religion, material wealth, and personal desires. मोक्ष िकसी
व्यिक्त के धमर्, भौितक धन और व्यिक्तगत इच्छाओं की पूितर् में मध्यम दृिष्टकोण में िनिहत है।
d
Individuals' knowledge about Atman and Brahman was a crucial path to attain Salvation. आत्मा और ब्रह्म के बारे में
व्यिक्तयों का ज्ञान मोक्ष प्राप्त करने का एक महत्वपूणर् मागर् था।

Explanation
Option c is the correct answer.

Recently the Prime Minister inaugurated the Maharishi Valmiki International Airport in Ayodhya, Uttar Pradesh. Valmiki
is referred to as Adi Kavi, which translates to the 'first/original poet' in Sanskrit. He is credited with composing the
Ramayana, one of the earliest and most revered epic poems in the Sanskrit literary tradition.

Option a is incorrect: Valmiki's teachings underscore that religion extends beyond mere rituals and ceremonies. He
criticized the wasteful use of wealth in meaningless rituals, observing that such practices are against public welfare and
humanity at large.

Option b is incorrect: Valmiki did not advocate for the renunciation of the world and singing hymns. He criticized
those who abandoned material wealth in pursuit of religion.

Option c is correct: Valmiki proposed the middle path or moderate approach to attain salvation. (moksha).
According to him, salvation lies in advancing on the road of religion, wealth, and desire. He believed that religion leads
to wealth, and wealth, in turn, gives rise to desires. However he cautioned against immoral physical lust, recognizing it
as a degenerate and impure facet of desire. According to him, individuals consumed by excessive lust tend to forget their
religious principles, lose their wealth, and ultimately ruin themselves. Thus it is true that he advocated a moderate
approach to one's pursuit of religion, material wealth, and personal desires to attain salvation.

https://academy.forumias.com/beta/lms/viewSolutions Page 27 of 180


Academy ForumIAS | Dashboard 18/02/24, 8:41 PM

Option d is incorrect: Valmiki did not place much emphasis on Individuals' knowledge about Atman and Brahman to
attain salvation.

Source:

https://infra.economictimes.indiatimes.com/news/aviation/pm-modi-inaugurates-maharishi-

valmiki-international-airport-in-ayodhya/106430238#:~:text=Varanasi (UP)Prime Minister Narendra,Valmiki international


airport in Ayodhya.&text=VARANASI: Prime Minister Narendra Modi,facilities available at the airport.

http://www.rbu.ac.in/home/avz9s6ve0gq6/public_html/ckfinder/userfiles/files/MousumiDas_BA6-DSE6_1_09062020.pdf
(pg no 3 & 4)

िवकल्प c सही उत्तर है।


हाल ही में प्रधानमंत्री ने उत्तर प्रदेश के अयोध्या में महिषर् वािल्मकी अंतरार्ष्ट्रीय हवाई अड्डे का उद्घाटन िकया। वािल्मकी को आिद किव कहा जाता है,
िजसका संस्कृत में अनुवाद "प्रथम/मौिलक किव" होता है। उन्हें रामायण की रचना करने का श्रेय िदया जाता है, जो संस्कृत सािहित्यक परंपरा में
सबसे प्रारंिभक और सबसे प्रितिष्ठत महाकाव्य में से एक है।
िवकल्प a ग़लत है: वािल्मकी की िशक्षाएँ इस बात पर ज़ोर देती हैं िक धमर् केवल अनुष्ठानों और समारोहों से परे है। उन्होंने िनरथर्क अनुष्ठानों में धन
के व्यथर् उपयोग की आलोचना की और कहा िक ऐसी प्रथाएं बड़े पैमाने पर सावर्जिनक कल्याण और मानवता के िखलाफ हैं।
िवकल्प b गलत है: वािल्मकी ने संसार को त्यागने और भजन गाने की वकालत नहीं की। उन्होंने उन लोगों की आलोचना की िजन्होंने धमर् की खोज में
भौितक संपदा को त्याग िदया।
िवकल्प c सही है: वािल्मकी ने मोक्ष प्राप्त करने के िलए मध्यम मागर् या मध्यम दृिष्टकोण का प्रस्ताव रखा। (मोक्ष). उनके अनुसार, मोक्ष धमर्, धन
और इच्छा के मागर् पर आगे बढ़ने में है। उनका मानना था िक धमर् धन की ओर ले जाता है और धन, बदले में, इच्छाओं को जन्म देता है। हालाँिक उन्होंने
अनैितक शारीिरक वासना के प्रित आगाह िकया, इसे इच्छा का एक पितत और अशुद्ध पहलू माना। उनके अनुसार, अत्यिधक वासना से ग्रस्त व्यिक्त
अपने धािमर् क िसद्धांतों को भूल जाते हैं, अपना धन खो देते हैं और अंततः खुद को बबार्द कर लेते हैं। इस प्रकार यह सच है िक उन्होंने िकसी व्यिक्त के
धमर्, भौितक धन और मोक्ष प्राप्त करने की व्यिक्तगत इच्छाओं की पूितर् के िलए एक उदारवादी दृिष्टकोण की वकालत की।
िवकल्प d गलत है: वािल्मकी ने मोक्ष प्राप्त करने के िलए आत्मा और ब्रह्म के बारे में व्यिक्तयों के ज्ञान पर अिधक जोर नहीं िदया।
Source:
https://infra.economictimes.indiatimes.com/news/aviation/pm-modi-inaugurates-maharishi-valmiki-
international-airport-
in-ayodhya/106430238#:~:text=Varanasi%20(UP)Prime%20Minister%20Narendra,Valmiki%20international%20
airport%20in%20Ayodhya.&text=VARANASI%3A%20Prime%20Minister%20Narendra%20Modi,facilities%20available%20at%20th

http://www.rbu.ac.in/home/avz9s6ve0gq6/public_html/ckfinder/userfiles/files/MousumiDas_BA6-
DSE6_1_09062020.pdf (pg no 3 & 4)

Question Attempted / Student

Question: 16 Mark Flag  Previous Next 

https://academy.forumias.com/beta/lms/viewSolutions Page 28 of 180


Academy ForumIAS | Dashboard 18/02/24, 8:41 PM

Correct Answer : b
Your Answer : b

Consider the following Pallava rulers and the works/events related with them:

Rulers Works/events related to


them

1. Mahendravarman Constructed
Mamallapuram rock-cut
temples

2. Narasimhavarman I Hiuen Tsang visited


Kanchipuram during his
reign

3. Nandivarman II Constructed Kailashnatha


Temple of Kanchipuram

How many of the above pairs are correctly matched?

िनम्निलिखत पल्लव शासकों और उनसे संबंिधत कायोर्ं/घटनाओं पर िवचार करें:

शासकों उनसे संबंिधत कायर्/घटनाएँ

1. महेंद्रवमर्न मामल्लापुरम में चट्टानों को काटकर


बनाए गए मंिदरों का िनमार्ण कराया

2. नरिसं हवमर्न प्रथम ह्वेन त्सांग ने इसके शासनकाल के


दौरान कांचीपुरम का दौरा िकया

3. नंिदवमर्न िद्वतीय कांचीपुरम के कैलाशनाथ मंिदर का


िनमार्ण कराया

उपरोक्त में से िकतने युग्म सही सुमेिलत हैं?

a Only one केवल एक


b Only two केवल दो
c All three तीनों
d None कोई नहीं

Explanation
Option b is the correct answer.

The Pallava dynasty ruled the Deccan, also known as Tondaimandalam, from 275 CE until 897 CE. During their reign, the
Pallavas were instrumental in creating Southern Indian history and legacy. The dynasty rose to prominence with the
demise of the Satavahana dynasty, for whom they had previously served as feudatories.

https://academy.forumias.com/beta/lms/viewSolutions Page 29 of 180


Academy ForumIAS | Dashboard 18/02/24, 8:41 PM

Option 1 is correct. Mahendravarman I was a Pallava king who ruled from 600 to 630 CE. He was a patron of art and
literature, contributing to the development of the Mamallapuram rock-cut temples.

Option 2 is correct. Narasimhavarman I, also known as Mamalla, ruled from 630 to 668 CE. Hiuen Tsang, the Chinese
Buddhist monk and traveler, visited Kanchipuram (Kanchi) during his reign, noting the grandeur of the city.

Option 3 is incorrect. Narasimhavarman II, also known as Rajasimha and Rajamalla, was a Pallava emperor who ruled
from 690 to 725 CE. He is credited with building the Shore Temple Complex, Mamallapuram's Isvara and Mukunda
Temples, Panamalai's Talagirisvara Temple, and Kanchi's Kailasanatha Temple. He is also credited with the
construction of Nagipattinam's Buddhist Vihara, also known as the 'China-pagoda.'

Source: Poonam Dalal Dahiya- Ch- 9

िवकल्प b सही उत्तर है


पल्लव राजवंश ने 275 ईस्वी से 897 ईस्वी तक दक्कन, िजसे टोंिडमंडलम के नाम से भी जाना जाता है, पर शासन िकया। अपने शासनकाल
के दौरान, पल्लवों ने दिक्षणी भारतीय इितहास और िवरासत के िनमार्ण में महत्वपूणर् भूिमका िनभाई। यह राजवंश सातवाहन राजवंश के अंत के
साथ प्रमुखता से उभरा, िजनके िलए वे पहले सामंतों के रूप में कायर् करते थे।
िवकल्प 1 सही है : महेंद्रवमर्न प्रथम एक पल्लव राजा था िजसने 600 से 630 ई. तक शासन िकया था। वह कला और सािहत्य के संरक्षक थे,
िजन्होंने मामल्लापुरम रॉक-कट मंिदरों के िवकास में योगदान िदया।
िवकल्प 2 सही है : नरिसं हवमर्न प्रथम, िजसे मामल्ल के नाम से भी जाना जाता है, ने 630 से 668 ई. तक शासन िकया। चीनी बौद्ध िभक्षु
और यात्री ह्वेन त्सांग ने शहर की भव्यता को देखते हुए अपने शासनकाल के दौरान कांचीपुरम (कांची) का दौरा िकया था।
िवकल्प 3 ग़लत है : नरिसम्हावमर्न िद्वतीय, िजन्हें राजिसम्हा और राजमल्ला के नाम से भी जाना जाता है, एक पल्लव सम्राट थे िजन्होंने 690
से 725 ईस्वी तक शासन िकया था। उन्हें शोर मंिदर पिरसर, मामल्लापुरम के ईश्वर और मुकुंद मंिदर, पनामालाई के तलिगरीश्वर मंिदर और
कांची के कैलासनाथ मंिदर के िनमार्ण का श्रेय िदया जाता है। उन्हें नागीपिट्टनम के बौद्ध िवहार के िनमार्ण का श्रेय भी िदया जाता है, िजसे
'चीन-पैगोडा' भी कहा जाता है।

Source: Poonam Dalal Dahiya- Ch- 9

Question Attempted / Student

Question: 17 Mark Flag  Previous Next 

Correct Answer : d
Your Answer :

Consider the following statements about the administration of Chalukyas of Badami?

1. Pattala was the smallest unit of administration.

2. All officials received payment in cash.

3. Bhukti (provinces) were supervised by Bhojapati.

https://academy.forumias.com/beta/lms/viewSolutions Page 30 of 180


Academy ForumIAS | Dashboard 18/02/24, 8:41 PM

How many of the above statements are correct?

बादामी के चालुक्यों के प्रशासन के बारे में िनम्निलिखत कथनों पर िवचार करें?


1. पट्टाल प्रशासन की सबसे छोटी इकाई थी।
2. सभी अिधकािरयों को नकद भुगतान प्राप्त हुआ।
3. भुिक्त (प्रान्तों) की देखरेख भोजपित द्वारा की जाती थी।

उपरोक्त में से िकतने कथन सही हैं?

a Only one केवल एक


b Only two केवल दो
c All three तीनों
d None कोई नहीं

Explanation
Option d is the correct answer.

The Chalukya of Badami ruled over the Deccan Plateau in southern India from the 6th to the 12th centuries. The Chalukya
dynasty was founded by Pulakesin I in the 6th century.

Option 1 is incorrect: Pattala refers to a group of villages or settlements and was not the smallest unit of
administration. It was bigger unit than village. The Bhojapati acted as the head of a Pattala, responsible for its
administration and well-being.

Option 2 is incorrect: Under Chalukyas administration, officials were paid by giving them grants of rent- free land.
These grants were typically made in return for military service or other administrative duties.

Option 3 is incorrect: Bhukti (provinces) was under the supervision of Uparika. The uparika was appointed by the
king and was usually a member of the Chalukya royal family or a close associate of the king.

Source: Poonam Dalal Dahiya - 'Ancient and Medieval India' chapter 9 " EARLY MEDIEVAL INDIA: THE AGE OF
REGIONAL

CONFIGURATIONS (c. 600-1200 CE) " Pg 418

िवकल्प d सही उत्तर है।


बादामी के चालुक्यों ने 6वीं से 12वीं शताब्दी तक दिक्षणी भारत में दक्कन के पठार पर शासन िकया। चालुक्य वंश की स्थापना छठी शताब्दी
में पुलकेिशन प्रथम ने की थी।
िवकल्प 1 गलत है: पट्टाला गाँवों या बिस्तयों के समूह को संदिभर् त करता है और प्रशासन की सबसे छोटी इकाई नहीं थी। यह गाँव से भी बड़ी
इकाई थी। भोजपित पट्टाल के मुिखया के रूप में कायर् करता था, जो इसके प्रशासन और कल्याण के िलए िजम्मेदार था।
िवकल्प 2 गलत है: चालुक्य प्रशासन के तहत, अिधकािरयों को लगान-मुक्त भूिम का अनुदान देकर भुगतान िकया जाता था। ये अनुदान आम
तौर पर सैन्य सेवा या अन्य प्रशासिनक कतर्व्यों के बदले में िदए जाते थे।

https://academy.forumias.com/beta/lms/viewSolutions Page 31 of 180


Academy ForumIAS | Dashboard 18/02/24, 8:41 PM

िवकल्प 3 गलत है: भुिक्त (प्रांत) उपरीका की देखरेख में था। उपिरक की िनयुिक्त राजा द्वारा की जाती थी और वह आमतौर पर चालुक्य शाही
पिरवार का सदस्य या राजा का करीबी सहयोगी होता था।
Source: Poonam Dalal Dahiya - 'Ancient and Medieval India' chapter 9 " EARLY MEDIEVAL INDIA: THE AGE
OF REGIONAL

CONFIGURATIONS (c. 600-1200 CE) " Pg 418

Question Attempted / Student

Question: 18 Mark Flag  Previous Next 

Correct Answer : b
Your Answer : b

With reference to the Rasthrakuta dynasty, consider the following statements:

1. Dantidurga was the founder of the Rashtrakuta dynasty.

2. Kailasa Temple at Ellora was built by the Rashtrakuta king Krishna I.

3. The Aihole inscription was issued by king Amoghavarsha of Rasthrakuta dynasty.

How many of the above statements are correct?

राष्ट्रकूट वंश के संदभर् में, िनम्निलिखत कथनों पर िवचार करें:


1. दंितदुगर् राष्ट्रकूट वंश का संस्थापक था।
2. एलोरा का कैलास मंिदर राष्ट्रकूट राजा कृष्ण प्रथम द्वारा बनवाया गया था।
3. ऐहोल िशलालेख राष्ट्रकूट वंश के राजा अमोघवषर् द्वारा जारी िकया गया था।

उपरोक्त में से िकतने कथन सही हैं?

a Only one केवल एक


b Only two केवल दो
c All three तीनों
d None कोई नहीं

Explanation
Option b is the correct answer

https://academy.forumias.com/beta/lms/viewSolutions Page 32 of 180


Academy ForumIAS | Dashboard 18/02/24, 8:41 PM

The Rashtrakuta dynasty was a powerful empire that ruled over much of southern India from the 8th to the 10th centuries.

Option 1 is correct: The founder of the Rasthrakuta dynasty was Dantidurga. Dantidurga overthrew then-reigning
Chalukya king, Kirtivarman II and established the Rashtrakuta dynasty.

Option 2 is correct: The Kailasa Temple at Ellora was built by the Rashtrakuta king Krishna I.

Option 3 is incorrect: The Aihole inscription was written by Ravikirti in honour of Pulakeshin II, the ruler of the
Badami Chalukya dynasty. It is one of the earliest surviving records of the Chalukya dynasty and provides valuable
information about their history and culture.

Source: Poonam Dalal Dahiya - 'Ancient and Medieval India' chapter 9 "- Early Medieval India: The Age of
Regional configurations (c. 600-1200 CE) "

िवकल्प b सही उत्तर है


राष्ट्रकूट राजवंश एक शिक्तशाली साम्राज्य था िजसने 8वीं से 10वीं शताब्दी तक दिक्षणी भारत के अिधकांश भाग पर शासन िकया।
िवकल्प 1 सही है: राष्ट्रकूट वंश का संस्थापक दंितदुगर् था। दिन्तदुगर् ने तत्कालीन चालुक्य राजा, कीितर् वमर्न िद्वतीय को उखाड़ फेंका और
राष्ट्रकूट राजवंश की स्थापना की।
िवकल्प 2 सही है: एलोरा में कैलासा मंिदर राष्ट्रकूट राजा कृष्ण प्रथम द्वारा बनाया गया था।
िवकल्प 3 गलत है: एहोल िशलालेख बादामी चालुक्य वंश के शासक पुलकेिशन िद्वतीय के सम्मान में रिवकीितर् द्वारा िलखा गया था। यह
चालुक्य वंश के सबसे पुराने जीिवत अिभलेखों में से एक है और उनके इितहास और संस्कृित के बारे में बहुमूल्य जानकारी प्रदान करता है।

Source: Poonam Dalal Dahiya - 'Ancient and Medieval India' chapter 9 “ Early Medieval India: The Age of
Regional configurations (c. 600–1200 CE) "

Question Attempted / Student

Question: 19 Mark Flag  Previous Next 

Correct Answer : b
Your Answer :

With reference to the society under Imperial Cholas, the terms Idangai and Valangai refers to-

शाही चोलों के अधीन समाज के संदभर् में, इदंगई और वलंगई शब्द का तात्पयर् है-

a Types of lands भूिम के प्रकार


b Caste groups जाित समूह
c Types of taxes करों के प्रकार
d Public assemblies सावर्जिनक सभाएँ

https://academy.forumias.com/beta/lms/viewSolutions Page 33 of 180


Academy ForumIAS | Dashboard 18/02/24, 8:41 PM

Explanation
Option b is the correct answer

The imperial Cholas were a powerful and prosperous dynasty. They controlled a large empire that stretched from the
Deccan Plateau to the southern tip of India including Sri Lanka and Malay Imperial Chola period witnessed the
emergence and prominence of the Idangai and Valangai groups. Idangai were caste groupings consisting mainly
artisanal and trading groups. Valangai were caste groupings consisting mainly of agricultural groups. This dichotomy
solidified and formalized social divisions based on caste. Each group encompassed numerous castes and sub-castes,
creating a more layered and rigid caste hierarchy.

Source: Poonam Dalal Dahiya - 'Ancient and Medieval India' chapter 9 " Early Medieval India: The Age of
Regional configurations (c. 600-1200 CE) " Pg 434

िवकल्प b सही उत्तर है


शाही चोल एक शिक्तशाली और समृद्ध राजवंश थे। उन्होंने एक बड़े साम्राज्य को िनयंित्रत िकया जो दक्कन के पठार से लेकर श्रीलंका सिहत
भारत के दिक्षणी िसरे तक फैला हुआ था और मलय इं पीिरयल चोल काल में इदंगई और वलंगई समूहों का उद्भव और प्रमुखता देखी गई।
इदंगई जाित समूह थे िजनमें मुख्य रूप से कारीगर और व्यापािरक समूह शािमल थे। वलंगई जाित समूह थे िजनमें मुख्य रूप से कृिष समूह
शािमल थे। इस द्वं द्व ने जाित के आधार पर सामािजक िवभाजन को मजबूत और औपचािरक बना िदया। प्रत्येक समूह में कई जाितयाँ और
उपजाितयाँ शािमल थीं, िजससे एक अिधक स्तिरत और कठोर जाित पदानुक्रम का िनमार्ण हुआ।

Source: Poonam Dalal Dahiya - 'Ancient and Medieval India' chapter 9 " Early Medieval India: The Age of
Regional configurations (c. 600–1200 CE) " Pg 434

Question Attempted / Student

Question: 20 Mark Flag  Previous Next 

Correct Answer : c
Your Answer : c

Which of the following is the most likely consequence if a person is declared as a designated terrorist under the Unlawful
Activities (Prevention) Amendment Act, 2019?

यिद िकसी व्यिक्त को गैरकानूनी गितिविधयां (रोकथाम) संशोधन अिधिनयम, 2019 के तहत नािमत आतंकवादी घोिषत िकया जाता है, तो
िनम्निलिखत में से कौन सा सबसे संभािवत पिरणाम होगा?

https://academy.forumias.com/beta/lms/viewSolutions Page 34 of 180


Academy ForumIAS | Dashboard 18/02/24, 8:41 PM

a
The person's assets are automatically frozen by the government. व्यिक्त की संपित्त सरकार द्वारा स्वचािलत रूप से जब्त कर ली
जाती है।
The person loses their Indian citizenship automatically. व्यिक्त अपनी भारतीय नागिरकता स्वतः खो देता
b है।
c
The person could be detained for a maximum of 180 days without the possibility of bail. व्यिक्त को जमानत की संभावना
के िबना अिधकतम 180 िदनों तक िहरासत में रखा जा सकता है।
d
The person once notified as a designated terrorist under the act, he/she can get themselves de notified only through
judicial means. िजस व्यिक्त को एक बार अिधिनयम के तहत नािमत आतंकवादी के रूप में अिधसूिचत कर िदया गया, वह केवल
न्याियक माध्यमों से ही खुद को डी-नोिटफाइड करवा सकता है

Explanation
Option c is the correct answer.

Recently, Gangster Goldy Brar, the mastermind behind the murder of Punjabi singer Sidhu Moosewala, was declared a
designated terrorist by the Centre under the Unlawful Activities (Prevention) Act (UAPA).

Option a is incorrect: The UAPA amendment act does not specify automatic freezing of assets, unlike the UN
designation of a global terrorist which involves sanctions such as freezing of assets, travel bans and an embargo against
procuring arms.

Option b is incorrect: There is no such provision in UAPA that stipulates the automatic suspension of Indian
citizenship of the person who is designated as terrorists under the same act.

Option c is correct: Typically, an individual accused of a crime can seek bail after 15 days of arrest, and in some cases,
even earlier if the court deems it appropriate. According to Section 167 of the Code of Criminal Procedure (CrPC), the
maximum statutory detention period is 90 days. However, in instances involving terror charges, Section 43D(2) of the
Unlawful Activities (Prevention) Act (UAPA), 1967, extends this period to a maximum of 180 days.

Option d is incorrect: The act provides avenues for individuals to challenge the designation, including applying for
removal through the government, review committee intervention, or moving the courts to contest the government's
decision. The act empowers the Central government to remove a name upon individual application, and the review
committee, comprising a chairperson (a retired or sitting judge of a High Court) and three other members. can order the
government to delete the designation if it deems the order flawed. Additionally, individuals can challenge the government's
decision by moving the courts.

Source: https://indianexpress.com/article/explained/explained-law/goldy-brar-designated-terrrorist-uapa-sidhu-moosewala-
9090652/

https://timesofindia.indiatimes.com/india/why-getting-bail-in-uapa-case-is-so-
di!icult/articleshow/102426429.cms?from=mdr

िवकल्प c सही उत्तर है।

https://academy.forumias.com/beta/lms/viewSolutions Page 35 of 180


Academy ForumIAS | Dashboard 18/02/24, 8:41 PM

हाल ही में, पंजाबी गायक िसद्धू मूसेवाला की हत्या के मास्टरमाइं ड गैंगस्टर गोल्डी बराड़ को केंद्र द्वारा गैरकानूनी गितिविधयां (रोकथाम)
अिधिनयम (यूएपीए) के तहत नािमत आतंकवादी घोिषत िकया गया था।
िवकल्प a गलत है: यूएपीए संशोधन अिधिनयम वैिश्वक आतंकवादी के संयुक्त राष्ट्र पदनाम के िवपरीत, संपित्त की स्वचािलत फ्रीिजं ग को
िनिदर् ष्ट नहीं करता है िजसमें संपित्त की जब्त, यात्रा प्रितबंध और हिथयारों की खरीद के िखलाफ प्रितबंध जैसे शतर् शािमल हैं।
िवकल्प b गलत है: यूएपीए में ऐसा कोई प्रावधान नहीं है जो उसी अिधिनयम के तहत आतंकवादी के रूप में नािमत व्यिक्त की भारतीय
नागिरकता के स्वत: िनलंबन को िनधार्िरत करता हो।
िवकल्प c सही है: आमतौर पर, िकसी अपराध का आरोपी व्यिक्त िगरफ्तारी के 15 िदनों के बाद जमानत मांग सकता है, और कुछ मामलों में,
इससे पहले भी, अगर अदालत उिचत समझे। दंड प्रिक्रया संिहता (सीआरपीसी) की धारा 167 के अनुसार, अिधकतम वैधािनक िहरासत अविध
90 िदन है। हालाँिक, आतंकी आरोपों से जुड़े मामलों में, गैरकानूनी गितिविधयाँ (रोकथाम) अिधिनयम (यूएपीए), 1967 की धारा 43d(2),
इस अविध को अिधकतम 180 िदनों तक बढ़ाती है।
िवकल्प d गलत है: अिधिनयम व्यिक्तयों को पदनाम को चुनौती देने के िलए अवसर प्रदान करता है, िजसमें सरकार के माध्यम से हटाने के
िलए आवेदन करना, समीक्षा सिमित के हस्तक्षेप, या सरकार के फैसले को चुनौती देने के िलए अदालतों में जाना शािमल है। समीक्षा सिमित में
एक अध्यक्ष (उच्च न्यायालय का एक सेवािनवृत्त या मौजूदा न्यायाधीश) और तीन अन्य सदस्य शािमल होते हैं। यिद वह आदेश को त्रुिटपूणर्
मानती है तो सरकार को पदनाम हटाने का आदेश दे सकती है। इसके अितिरक्त, व्यिक्त अदालतों के तहत सरकार के फैसले को चुनौती दे
सकते हैं।
Source: https://indianexpress.com/article/explained/explained-law/goldy-brar-designated-terrrorist-uapa-sidhu-
moosewala-9090652/

https://timesofindia.indiatimes.com/india/why-getting-bail-in-uapa-case-is-so-
difficult/articleshow/102426429.cms?from=mdr

Question Attempted / Student

Question: 21 Mark Flag  Previous Next 

Correct Answer : b
Your Answer : b

With reference to the Chola administration, consider the following statements:

1. Toll taxes were often imposed at the royal ports.

2. Kadammai was a tax imposed on the land.

3. No evidence of non-agricultural tax has been found during the Chola period.

How many of the statements given above are correct?

चोल प्रशासन के संदभर् में, िनम्निलिखत कथनों पर िवचार करें:


1. रॉयल पोटर् पर अक्सर टोल टैक्स लगाया जाता था।

https://academy.forumias.com/beta/lms/viewSolutions Page 36 of 180


Academy ForumIAS | Dashboard 18/02/24, 8:41 PM

2. कदम्मई भूिम पर लगाया जाने वाला कर था।


3. चोल काल में गैर-कृिष कर का कोई साक्ष्य नहीं िमला है।

ऊपर िदए गए कथनों में से िकतने सही हैं?

Only one केवल


a एक
Only two केवल
b दो
All three
c तीनों
None कोई
d नहीं

Explanation
Option b is the correct answer.

The Chola Empire was a powerful and prosperous empire that ruled over much of southern India from the 9th to the 13th
centuries. The empire was founded by Vijayalaya Chola in 850 CE, and its capital was at Thanjavur.

Statement 1 is correct: During the Chola period, royal ports were consciously developed, and tolls were levied by
royal agents. Exemption from tolls also formed part of the policy of encouragement of trade venture.

Statement 2 is correct: The existence of a regular taxation system is indicated by a statistical analysis of in Chola
inscriptions. The major land tax called kadammai was uniform as also there were many other smaller ones related to
land. kadammai was based on the productivity of the land and was collected in the form of grains or other agricultural
produce.

Statement 3 is incorrect: In the Chola empire Taxes, both central and local, have been identified. Evidence about
both agricultural and non-agricultural taxes has been found. Increase in non-agricultural taxes over time has also been
recognised in various inscriptions.

Source: Poonam Dalal Dahiya - 'Ancient and Medieval India' chapter 9 " Early Medieval India: The Age of
Regional configurations (c. 600-1200 CE) "

िवकल्प b सही उत्तर है


चोल साम्राज्य एक शिक्तशाली और समृद्ध साम्राज्य था िजसने 9वीं से 13वीं शताब्दी तक दिक्षणी भारत के अिधकांश भाग पर शासन िकया।
साम्राज्य की स्थापना िवजयालय चोल ने 850 ई. में की थी और इसकी राजधानी तंजावुर में थी।
कथन 1 सही है: चोल काल के दौरान, रॉयल पोटर् को सचेत रूप से िवकिसत िकया गया था, और रॉयल एजेंटों द्वारा टोल लगाया गया था।
टोल से छूट भी व्यापार उद्यम को प्रोत्साहन की नीित का िहस्सा बनी।
कथन 2 सही है: चोल िशलालेखों के सांिख्यकीय िवश्लेषण से एक िनयिमत कराधान प्रणाली के अिस्तत्व का संकेत िमलता है। कदम्मई
नामक प्रमुख भूिम कर एक समान था और भूिम से संबंिधत कई अन्य छोटे कर भी थे। कदम्मई भूिम की उत्पादकता पर आधािरत थी और इसे
अनाज या अन्य कृिष उपज के रूप में एकत्र िकया जाता था।

https://academy.forumias.com/beta/lms/viewSolutions Page 37 of 180


Academy ForumIAS | Dashboard 18/02/24, 8:41 PM

कथन 3 गलत है: चोल साम्राज्य में केंद्रीय और स्थानीय दोनों तरह के करों की पहचान की गई है। कृिष एवं गैर-कृिष दोनों प्रकार के करों के
प्रमाण िमले हैं। िविभन्न िशलालेखों में समय के साथ गैर-कृिष करों में वृिद्ध को भी मान्यता दी गई है।

Source: Poonam Dalal Dahiya - 'Ancient and Medieval India' chapter 9 " Early Medieval India: The Age of
Regional configurations (c. 600–1200 CE) "

Question Attempted / Student

Question: 22 Mark Flag  Previous Next 

Correct Answer : b
Your Answer :

In the context of administration of the Imperial Cholas, the term Velaikkarar refers to:

शाही चोलों के प्रशासन के संदभर् में, वेलैक्करार शब्द का तात्पयर् है:

a Tax collectors in Brahmadeyas. ब्रह्मादेय में कर संग्राहक।


b Personal troop to defend the king राजा की रक्षा के िलए व्यिक्तगत सेना
c District level judges िजला स्तर के न्यायाधीश
d Royal treasurer. शाही कोषाध्यक्ष।

Explanation
Option b is the correct answer.

The Cholas maintained a regular standing army consisting of elephants, cavalry, infantry and navy. About seventy
regiments were mentioned in the inscriptions.

1) The royal troops were called Kaikkolaperumpadai. Within this there was a personal troop to defend the king known
as Velaikkarar.

2) Attention was given to the training of the army and military cantonments called kadagams existed.

3) The Cholas paid special attention to their navy. The naval achievements of the Tamils reached its climax under the
Cholas.

Source: old tamilnadu scert 11th Chapter 13

िवकल्प b सही उत्तर है

https://academy.forumias.com/beta/lms/viewSolutions Page 38 of 180


Academy ForumIAS | Dashboard 18/02/24, 8:41 PM

चोलों ने हािथयों, घुड़सवार सेना, पैदल सेना और नौसेना से िमलकर एक िनयिमत स्थायी सेना बनाए रखी। िशलालेखों में लगभग सत्तर
रेजीमेंटों का उल्लेख िकया गया है।
1) शाही सैिनकों को कैक्कोलापेरम्पडाई कहा जाता था। इसके भीतर राजा की रक्षा के िलए एक िनजी सेना थी िजसे वेलैक्करर के नाम से
जाना जाता था।
2) सेना के प्रिशक्षण पर ध्यान िदया गया और कडगम नामक सैन्य छाविनयाँ अिस्तत्व में थीं।
3) चोलों ने अपनी नौसेना पर िवशेष ध्यान िदया। चोलों के शासनकाल में तिमलों की नौसैिनक उपलिब्धयाँ अपने चरमोत्कषर् पर पहुँ च गईं।

Source: old tamilnadu scert 11th Chapter 13

Question Attempted / Student

Question: 23 Mark Flag  Previous Next 

Correct Answer : b
Your Answer : b

With reference to the status of society and economy under the Vijayanagar Empire in, consider the following statements:

1. The practice of slavery was completely absent in Vijayanagar empire.

2. The Cannanore seaport was a significant trading hub during the Vijayanagar empire.

3. According to Abdur Razzaq, a Persian ambassador, major cities were well connected by roads in Vijayanagar empire.

How many of the above statements are correct?

िवजयनगर साम्राज्य के तहत समाज और अथर्व्यवस्था की िस्थित के संदभर् में, िनम्निलिखत कथनों पर िवचार करें:
1. िवजयनगर साम्राज्य में गुलामी की प्रथा पूणर्तः अनुपिस्थत थी।
2. िवजयनगर साम्राज्य के दौरान कन्नानोर बंदरगाह एक महत्वपूणर् व्यापािरक केंद्र था।
3. फ़ारसी राजदू त अब्दुर रज्जाक के अनुसार, िवजयनगर साम्राज्य में प्रमुख शहर सड़कों से अच्छी तरह जुड़े हुए थे।

उपरोक्त में से िकतने कथन सही हैं?

https://academy.forumias.com/beta/lms/viewSolutions Page 39 of 180


Academy ForumIAS | Dashboard 18/02/24, 8:41 PM

Only one केवल


a एक
Only two केवल
b दो
All three
c तीनों
None कोई
d नहीं

Explanation
Option b is the correct answer

The Vijayanagara Empire was a Hindu empire that existed in southern India. It was founded by Harihara I and Bukka Raya
I, two brothers.

Statement 1 is incorrect: Slavery was prevalent during their time. Historians have also highlighted the presence of
slavery during their time. Paes mentions that there were slaves to maintain the beautiful houses of the rich. Nicolo Conti
also refers to the prevalence of slavery.

Statement 2 is correct: The Cannanore seaport was an important port located on the Malabar coast of India. It was
a major trading hub for the Vijayanagara Empire, and it was also a centre for the export of spices and other goods. The
port was also a major source of revenue for the empire.

Statement 3 is correct: Abdur Razaak, the Persian ambassador who visited the Zamorin of Kochi and the
Vijayanagar court during this time states that Devaraya II controlled a vast area. He elaborated about conditions of
roads and forts. He mentioned that major cities were well connected by roads in Vijayanagar empire.

Source: Poonam Dalal Dahiya - 'Ancient and Medieval India' Chapter 12 " PROVINCIAL KINGDOMS OF MEDIEVAL
INDIA "

िवकल्प b सही उत्तर है


िवजयनगर साम्राज्य एक िहं दू साम्राज्य था जो दिक्षणी भारत में अिस्तत्व में था। इसकी स्थापना दो भाइयों हिरहर प्रथम और बुक्का राया प्रथम
ने की थी।
कथन 1 गलत है: उनके समय में गुलामी प्रचिलत थी। इितहासकारों ने भी अपने समय में गुलामी की मौजूदगी पर प्रकाश डाला है। पेस ने
उल्लेख िकया है िक अमीरों के खूबसूरत घरों की देखभाल के िलए दास होते थे। िनकोलो कोंटी भी गुलामी की व्यापकता का उल्लेख करते हैं।
कथन 2 सही है: कन्नानोर बंदरगाह भारत के मालाबार तट पर िस्थत एक महत्वपूणर् बंदरगाह था। यह िवजयनगर साम्राज्य के िलए एक प्रमुख
व्यापािरक केंद्र था और यह मसालों और अन्य वस्तुओ ं के िनयार्त का भी केंद्र था। बंदरगाह साम्राज्य के िलए राजस्व का एक प्रमुख स्रोत भी था।
कथन 3 सही है: इस दौरान कोिच्च के ज़मोिरन और िवजयनगर दरबार का दौरा करने वाले फ़ारसी राजदू त अब्दुर रज़ाक का कहना है िक
देवराय िद्वतीय ने एक िवशाल क्षेत्र को िनयंित्रत िकया था। उन्होंने सड़कों और िकलों की िस्थित के बारे में िवस्तार से बताया। उन्होंने उल्लेख
िकया िक िवजयनगर साम्राज्य में प्रमुख शहर सड़कों से अच्छी तरह जुड़े हुए थे।

Source: Poonam Dalal Dahiya - 'Ancient and Medieval India' Chapter 12 " PROVINCIAL KINGDOMS OF
MEDIEVAL INDIA "

https://academy.forumias.com/beta/lms/viewSolutions Page 40 of 180


Academy ForumIAS | Dashboard 18/02/24, 8:41 PM

Question Attempted / Student

Question: 24 Mark Flag  Previous Next 

Correct Answer : a
Your Answer : a

Consider the following statements regarding ancient and medieval India:

Statement-I: Borobudur temple in Indonesia features intricate reliefs depicting the life of Buddha, similar to those found
during Gupta period.

Statement-II: The monsoon winds across the Bay of Bengal facilitated India's connection with Southeast Asia.

Which one of the following is correct in respect of the above statements?

प्राचीन और मध्यकालीन भारत के संबंध में िनम्निलिखत कथनों पर िवचार करें:


कथन-I: इं डोनेिशया में बोरोबुदुर मंिदर में बुद्ध के जीवन को दशार्ने वाली जिटल नक्काशी है, जो गुप्त काल के दौरान पाई गई मूितर् यों के समान
है।
कथन-II: बंगाल की खाड़ी के पार मानसूनी हवाओं ने भारत को दिक्षण पूवर् एिशया से जोड़ने में मदद की।

उपरोक्त कथनों के संबंध में िनम्निलिखत में से कौन सा सही है?

a
Both Statement-I and Statement-II are correct and Statement-II is the correct explanation for Statement-I कथन-I और
कथन-II दोनों सही हैं और कथन-II, कथन-I की सही व्याख्या है
b
Both Statement-I and Statement-II are correct and Statement-II is not the correct explanation for Statement-I कथन-I
और कथन-II दोनों सही हैं और कथन-II, कथन-I के िलए सही व्याख्या नहीं है
Statement-I is correct Statement-II is incorrect कथन-I सही है कथन-II गलत
c है
Statement-I incorrect but Statement-II is correct. कथन-I गलत है लेिकन कथन-II सही
d है।

Explanation
Option a is the correct answer

https://academy.forumias.com/beta/lms/viewSolutions Page 41 of 180


Academy ForumIAS | Dashboard 18/02/24, 8:41 PM

Monsoon wind across the bay of Bengal helped India to establish early cultural contacts with Southeast Asia across the
bay of Bengal.

Statement I is correct: The Borobudur Temple in Indonesia has been influenced by Indian Gupta and post-Gupta
art. India and Southeast Asia have a long history of cultural exchange. This exchange was facilitated by the fact that
both regions are located on the Indian Ocean, which made travel between them relatively easy. One of the most important
aspects of this exchange was the spread of Buddhism from India to Southeast Asia.

Statement II is correct and statement II is the correct explanation of statement I: The monsoon winds have also
played a role in the development of trade and commerce between India and Southeast Asia along with cultural
exchanges. The winds have made it possible for ships to travel between the two regions, and this has led to a thriving trade
in goods such as spices, textiles, and metals. The winds have also helped to spread new technologies between the two
regions, such as the art of shipbuilding and navigation.

Source: Ancient and Medieval India Poonam Dalal Dahiya

https://www.worldhistory.org/Borobudur/#:~:text=Borobudur's design is a mix,roughly 3,000 bas-relief sculptures.

िवकल्प a सही उत्तर है


बंगाल की खाड़ी के पार मानसूनी हवा ने भारत को बंगाल की खाड़ी के पार दिक्षण पूवर् एिशया के साथ प्रारंिभक सांस्कृितक संपकर् स्थािपत
करने में मदद की।
कथन I सही है: इं डोनेिशया में बोरोबुदुर मंिदर भारतीय गुप्त और गुप्तोत्तर कला से प्रभािवत है। भारत और दिक्षण पूवर् एिशया में सांस्कृितक
आदान-प्रदान का एक लंबा इितहास रहा है। यह आदान-प्रदान इस तथ्य से सुगम हुआ िक दोनों क्षेत्र िहं द महासागर पर िस्थत हैं, िजससे उनके
बीच यात्रा अपेक्षाकृत आसान हो गई। इस आदान-प्रदान का सबसे महत्वपूणर् पहलू भारत से दिक्षण पूवर् एिशया तक बौद्ध धमर् का प्रसार था।
कथन II सही है और कथन II कथन I की सही व्याख्या है: मानसूनी हवाओं ने सांस्कृितक आदान-प्रदान के साथ-साथ भारत और दिक्षण पूवर्
एिशया के बीच व्यापार और वािणज्य के िवकास में भी भूिमका िनभाई है। हवाओं ने जहाजों के िलए दोनों क्षेत्रों के बीच यात्रा करना संभव बना
िदया है, और इससे मसालों, कपड़ा और धातुओ ं जैसे सामानों का व्यापार फल-फूल रहा है। हवाओं ने दोनों क्षेत्रों के बीच जहाज िनमार्ण और
नेिवगेशन की कला जैसी नई तकनीकों को फैलाने में भी मदद की है।
Source: Ancient and Medieval India Poonam Dalal Dahiya

https://www.worldhistory.org/Borobudur/#:~:text=Borobudur's%20design%20is%20a%20mix,roughly%203%2C000%20ba

Question Attempted / Student

Question: 25 Mark Flag  Previous Next 

Correct Answer : a
Your Answer : a

The term 'Acceptance of Necessity' (AON) is sometimes seen in the news with reference to

https://academy.forumias.com/beta/lms/viewSolutions Page 42 of 180


Academy ForumIAS | Dashboard 18/02/24, 8:41 PM

'आवश्यकता की स्वीकृित' (एओएन)' शब्द को कभी-कभी समाचारों में िकसके संदभर् में देखा जाता है?

Procurement of defense equipment in our country. हमारे देश में रक्षा उपकरणों की
a खरीद।
Developing solar power production in our country. हमारे देश में सौर ऊजार् उत्पादन का िवकास
b करना।
Exporting our food products to other countries. अपने खाद्य उत्पादों को अन्य देशों में िनयार्त
c करना।
d
Permitting foreign educational institutions to set up their campuses in our country. िवदेशी शैक्षिणक संस्थानों को हमारे देश
में अपने पिरसर स्थािपत करने की अनुमित देना।

Explanation
Option a is the correct answer.

The Union Ministry of Defence has recently sanctioned over ₹1.42 trillion for the acquisition of weapons systems
developed by the Defence Research & Development Organisation (DRDO). The Government of India aims to enhance the
involvement of private companies in domestic defense production to reduce the nation's reliance on imports and bolster
self-reliance.

Option a is correct: Acceptance of Necessity" (AON) is the initial approval granted by the Union Ministry of Defence
(MoD) for procuring defense equipment. It signifies the commencement of the detailed procurement process,
encompassing requests for information, proposals, evaluations, and the final awarding of contracts. Essentially, it serves as
a green light from the defense ministry, indicating the necessity of the procurement plan and allowing it to progress
through the acquisition procedure. India, under the "Aatmanirbhar Bharat" initiative, emphasizes DRDO-developed
systems for self-reliance. Reports from industry bodies indicate that 80-90% of Acceptance of Necessity (AoN) instances
involve Indian companies.

Source: https://www.thehindubusinessline.com/news/national/more-than-142000-cr-worth-aon-accorded-for-
induction-of-drdo-developed-defence-products/article67695816.ece

िवकल्प a सही उत्तर है।


केंद्रीय रक्षा मंत्रालय ने हाल ही में रक्षा अनुसंधान एवं िवकास संगठन (डीआरडीओ) द्वारा िवकिसत हिथयार प्रणािलयों की खरीद के िलए
₹1.42 िट्रिलयन से अिधक की मंजूरी दी है। भारत सरकार का लक्ष्य देश की आयात पर िनभर्रता को कम करने और आत्मिनभर्रता को बढ़ावा
देने के िलए घरेलू रक्षा उत्पादन में िनजी कंपिनयों की भागीदारी को बढ़ाना है।
िवकल्प a सही है: आवश्यकता की स्वीकृित" (एओएन) रक्षा उपकरणों की खरीद के िलए केंद्रीय रक्षा मंत्रालय (एमओडी) द्वारा दी गई
प्रारंिभक मंजूरी है। यह िवस्तृत खरीद प्रिक्रया की शुरुआत का प्रतीक है, िजसमें सूचना, प्रस्ताव, मूल्यांकन के िलए अनुरोध शािमल हैं। और
अनुबंधों का अंितम पुरस्कार। मूलतः, यह रक्षा मंत्रालय से हरी झंडी के रूप में कायर् करता है, जो खरीद योजना की आवश्यकता को दशार्ता है
और इसे अिधग्रहण प्रिक्रया के माध्यम से आगे बढ़ने की अनुमित देता है। भारत, "आत्मिनभर्र भारत" पहल के तहत, आत्मिनभर्रता के िलए
डीआरडीओ द्वारा िवकिसत प्रणािलयों पर जोर देता है। उद्योग िनकायों की िरपोटर् से पता चलता है िक आवश्यकता की स्वीकृित (एओएन) के
80-90% मामलों में भारतीय कंपिनयां शािमल हैं।

https://academy.forumias.com/beta/lms/viewSolutions Page 43 of 180


Academy ForumIAS | Dashboard 18/02/24, 8:41 PM

Source: https://www.thehindubusinessline.com/news/national/more-than-142000-cr-worth-aon-accorded-for-
induction-of-drdo-developed-defence-products/article67695816.ece

Question Attempted / Student

Question: 26 Mark Flag  Previous Next 

Correct Answer : a
Your Answer : b

With reference to the administration of the Delhi Sultanate from 13th to 16th century, consider the following:

1. Revenues from Iqta land were used for maintaining the royal court and household.

2. State-sponsored agricultural takkavi loans were given to farmers to increase agricultural production.

3. Postal services were unavailable during the Delhi Sultanate rule.

How many of the above statements are correct?

13वीं से 16वीं शताब्दी तक िदल्ली सल्तनत के प्रशासन के संदभर् में, िनम्निलिखत पर िवचार करें:
1. इक्ता भूिम से प्राप्त राजस्व का उपयोग शाही दरबार और घर के रखरखाव के िलए िकया जाता था।
2. कृिष उत्पादन बढ़ाने के िलए िकसानों को राज्य प्रायोिजत कृिष तकाकवी ऋण िदए गए।
3. िदल्ली सल्तनत शासन के दौरान डाक सेवाएँ उपलब्ध नहीं थीं।

उपरोक्त में से िकतने कथन सही हैं?

Only one केवल


a एक
Only two केवल
b दो
All three
c तीनों
None कोई
d नहीं

Explanation
Option a is the correct answer

https://academy.forumias.com/beta/lms/viewSolutions Page 44 of 180


Academy ForumIAS | Dashboard 18/02/24, 8:41 PM

Numerous invasions into India had ultimately resulted in the establishment of Delhi Sultanate which existed from A.D.
1206 to 1526. Five different dynasties - the Slave, Khalji, Tughlaq, Sayyids and Lodis - ruled under the Delhi Sultanate.

Statement 1 is incorrect: In iqta system, the land was assigned to officials as iqtas instead of payment for their
services, and not to maintain a royal court and households.

Statement 2 is correct: Firoz Tughlaq made efforts to enhance agricultural production by providing irrigational
facilities and by providing takkavi loans. Takkavi loans play an important role in helping poor farmers to meet their
urgent needs. The loans help the farmers to purchase seeds, fertilizers, and other agricultural inputs.

statement 3 is incorrect: Well well-organised postal system existed during their time. The Delhi Sultanate's postal
system was very efficient and reliable. The Delhi Sultanate's postal system was a major factor in the empire's success.
The Delhi Sultanate's postal system paved the way for the development of more advanced postal systems in later centuries.

Source: Poonam Dalal Dahiya - 'Ancient and Medieval India' Chapter 11 " THE DELHI SULTANATE"

Tamilnadu SCERT - 11th standard

िवकल्प a सही उत्तर है


भारत पर कई आक्रमणों के पिरणामस्वरूप अंततः िदल्ली सल्तनत की स्थापना हुई जो 1206 से 1526 ई. तक अिस्तत्व में थी। पाँच अलग-अलग
राजवंश - गुलाम, िखलजी, तुगलक, सैय्यद और लोदी - िदल्ली सल्तनत के अधीन शासन करते थे।
कथन 1 गलत है: इक्ता प्रणाली में, भूिम को अिधकािरयों को उनकी सेवाओं के िलए भुगतान के बजाय इक्ता के रूप में सौंपा गया था, न िक शाही
दरबार और घरों को बनाए रखने के िलए।
कथन 2 सही है: िफ़रोज़ तुगलक ने िसं चाई सुिवधाएँ प्रदान करके और तक्कवी ऋण प्रदान करके कृिष उत्पादन बढ़ाने के प्रयास िकए। ताकावी ऋण
गरीब िकसानों को उनकी तत्काल जरूरतों को पूरा करने में मदद करने में महत्वपूणर् भूिमका िनभाते थे। ऋण िकसानों को बीज, उवर्रक और अन्य कृिष
इनपुट खरीदने में मदद करते थे।
कथन 3 गलत है: उनके समय में सुव्यविस्थत डाक प्रणाली मौजूद थी। िदल्ली सल्तनत की डाक व्यवस्था अत्यंत कुशल एवं िवश्वसनीय थी। िदल्ली
सल्तनत की डाक प्रणाली साम्राज्य की सफलता का एक प्रमुख कारक थी। िदल्ली सल्तनत की डाक प्रणाली ने बाद की शतािब्दयों में और अिधक
उन्नत डाक प्रणािलयों के िवकास का मागर् प्रशस्त िकया।
Source: Poonam Dalal Dahiya - 'Ancient and Medieval India' Chapter 11 " THE DELHI SULTANATE"

Tamilnadu SCERT – 11th standard

Question Attempted / Student

Question: 27 Mark Flag  Previous Next 

Correct Answer : b
Your Answer : a

In the context of medieval India, consider the following pairs about invasion from Central Asia during Delhi Sultanate:

https://academy.forumias.com/beta/lms/viewSolutions Page 45 of 180


Academy ForumIAS | Dashboard 18/02/24, 8:41 PM

Ruler Invader from Central


Asia

1. Iltutmish Chenghiz Khan

2. Alauddin Khalji Qutlugh Khawaja

3. Muhamad bin Tughluq Timur /Taimur Lang

How many of the above pairs are correctly matched?

मध्ययुगीन भारत के संदभर् में, िदल्ली सल्तनत के दौरान मध्य एिशया से आक्रमण के बारे में िनम्निलिखत युग्मों पर िवचार करें:

शासक मध्य एिशया से आक्रमण करने वाले

1. इल्तुतिमश चंगेज खान

2. अलाउद्दीन िखलजी कुतलुग ख्वाजा

3. मुहम्मद िबन तुगलक तैमूर/तैमूर लंग

उपरोक्त में से िकतने युग्म सही सुमेिलत हैं?

Only one केवल


a एक
Only two केवल
b दो
All three सभी
c तीन
None कोई
d नहीं

Explanation
Option b is the correct answer

The Delhi Sultanate, a late medieval empire that lasted from 1206-1526, faced many invasions from nearby empires and
kingdoms. These invasions included the Mongols, Timurids, and Rajput's.

Pair 1 is correct: Iltutmish (1211-1236) was invaded by Genghis Khan. Temujin, also known as Genghis Khan, the
leader of the Mongols, launched invasions into Central Asia. He successfully defeated Jalaluddin Mangabarni, the ruler of
Khwarazm. Seeking asylum, Mangabarni crossed the Indus River and approached Iltutmish. However, Iltutmish, in a
strategic move to safeguard his empire from the Mongol threat, declined to provide shelter. Fortunately, Chengiz Khan
eventually returned home without venturing further into India. Iltutmish's diplomatic decision played a crucial role in
sparing India from the potential consequences of Chengiz Khan's aggression.

https://academy.forumias.com/beta/lms/viewSolutions Page 46 of 180


Academy ForumIAS | Dashboard 18/02/24, 8:41 PM

Pair 2 is correct: Alauddin Khalji's empire (1296-1316) was invaded by Mongol invader, Qutlugh Khwaja. Alauddin
Khalji dispatched his army on six occasions to counter the Mongols threat. The initial two endeavors proved successful,
but during the third invasion led by Khwaja, the Mongols reached Delhi, yet were thwarted from entering the capital city.
The subsequent three Mongol incursions were met with robust resistance, resulting in the death of numerous Mongol
forces. To bolster defenses, the northwestern frontier was fortified, and Gazi Malik was appointed as the Warden of
Marches to safeguard the frontier.

Pair 3 is incorrect: Muhammad bin Tughluq's empire (1325-1351) was not invaded by Timur /Taimur Lang but it
was during the reign of Nasir-ud-Din Mahmud Shah Tughluq (1394-1413 CE). Timur entered Delhi in 1398 and there
was no opposition. He sacked Delhi for three days murdering thousands of people and looting enormous wealth. He
withdrew from India in 1399 and his invasion in fact delivered a death blow to the Tughlaq dynasty.

Source: Poonam Dalal Dahiya - 'Ancient and Medieval India' Pg 543

िवकल्प b सही उत्तर है


िदल्ली सल्तनत, उत्तर मध्यकालीन साम्राज्य जो 1206-1526 तक चला, उसे आस-पास के साम्राज्यों और राज्यों से कई आक्रमणों का सामना
करना पड़ा। इन आक्रमणों में मंगोल, तैमूिरयान और राजपूत शािमल थे।
युग्म 1 सही है: इल्तुतिमश (1211-1236) पर चंगेज खान ने आक्रमण िकया था। तेमुिजन, िजसे मंगोलों के नेता चंगेज खान के नाम से भी
जाना जाता है, ने मध्य एिशया में आक्रमण शुरू िकया। उसने ख़्वारज़्म के शासक जलालुद्दीन मंगबनीर् को सफलतापूवर्क हराया। शरण की
तलाश में, मंगबनीर् ने िसं धु नदी पार की और इल्तुतिमश के पास पहुंचा। हालाँिक, इल्तुतिमश ने अपने साम्राज्य को मंगोल खतरे से बचाने के
िलए एक रणनीितक कदम के तहत आश्रय देने से इनकार कर िदया। सौभाग्य से, चेंिगज़ खान अंततः भारत में आगे बढ़े िबना घर लौट आया।
इल्तुतिमश के कूटनीितक िनणर्य ने भारत को चंगेज खान की आक्रामकता के संभािवत पिरणामों से बचाने में महत्वपूणर् भूिमका िनभाई।
युग्म 2 सही है : अलाउद्दीन िखलजी के साम्राज्य (1296-1316) पर मंगोल आक्रमणकारी कुतलुग ख्वाजा ने आक्रमण िकया था। मंगोलों के
खतरे का मुकाबला करने के िलए अलाउद्दीन िखलजी ने छह मौकों पर अपनी सेना भेजी। शुरुआती दो प्रयास सफल सािबत हुए, लेिकन
ख्वाजा के नेतृत्व में तीसरे आक्रमण के दौरान, मंगोल िदल्ली तक पहुंच गए, िफर भी उन्हें राजधानी शहर में प्रवेश करने से रोक िदया गया। बाद
की तीन मंगोल घुसपैठों को जबरदस्त प्रितरोध का सामना करना पड़ा, िजसके पिरणामस्वरूप कई मंगोल सेनाएँ मारी गईं। सुरक्षा को मजबूत
करने के िलए, उत्तर-पिश्चमी सीमा को मजबूत िकया गया, और सीमा की सुरक्षा के िलए गाजी मिलक को माचर् के वाडर्न के रूप में िनयुक्त
िकया गया।
युग्म 3 गलत है: मुहम्मद िबन तुगलक के साम्राज्य (1325-1351) पर तैमूर/तैमूर लंग ने आक्रमण नहीं िकया था, बिल्क यह नािसर-उद-दीन
महमूद शाह तुगलक (1394-1413 सीई) के शासनकाल के दौरान हुआ था। 1398 में तैमूर ने िदल्ली में प्रवेश िकया और कोई िवरोध नहीं
हुआ। उसने िदल्ली को तीन िदनों तक लूटा, हजारों लोगों की हत्या की और भारी संपित्त लूटी। वह 1399 में भारत से वापस चला गया और
उसके आक्रमण ने वास्तव में तुगलक वंश पर घातक प्रहार िकया।

Source: Poonam Dalal Dahiya - 'Ancient and Medieval India' Pg 543

Question Attempted / Student

Question: 28 Mark Flag  Previous Next 

Correct Answer : a
https://academy.forumias.com/beta/lms/viewSolutions Page 47 of 180
Academy ForumIAS | Dashboard 18/02/24, 8:41 PM

Your Answer : b

With reference to the Jagirdari system during the Mughal Empire, consider the following statements:

1. Jagirs are primarily given to nobles and military officials as a form of compensation for their services.

2. The jagirs could be passed on to the descendants of the jagirdars.

3. The size of jagirs was fixed and uniform across the Mughal Empire.

How many of the above statements are correct?

मुगल साम्राज्य के दौरान जागीरदारी व्यवस्था के संदभर् में, िनम्निलिखत कथनों पर िवचार करें:
1. जागीरें मुख्य रूप से कुलीनों और सैन्य अिधकािरयों को उनकी सेवाओं के मुआवजे के रूप में दी जाती हैं।
2. जागीरें जागीरदारों के वंशजों को दी जा सकती थीं।
3. पूरे मुगल साम्राज्य में जागीरों का आकार िनिश्चत और एक समान था।

उपरोक्त में से िकतने कथन सही हैं?

a Only one केवल एक


b Only two केवल दो
c All three तीनों
d None कोई नहीं

Explanation
Option a is the correct answer.

The Jagirdari system was an administrative structure of the Mughal Empire that awarded land revenue to officers in
exchange for a salary. These officers were known as Jagirdars and were frequently military leaders or administrators. The
Jagirdari system evolved from the Delhi Sultanate's Iqtadari system.

Statement 1 is correct. The emperor granted jagirs to nobles, including military officials, as a form of compensation
for their services. The jagirdars had the right to collect revenue from the assigned land. While jagirdars had the right to
collect revenue, they did not own the land. The ownership remained with the Mughal emperor.

Statement 2 is incorrect. The Jagirdari system was not hereditary. Jagirs were not automatically passed on to the
descendants, and reassignments were common.

Statement 3 is incorrect. The size of jagirs varied, and they could be larger or smaller based on factors such as the
nobility's service, loyalty, and the emperor's discretion.

Source: Poonam Dalal Dahiya Ch-13

िवकल्प a सही उत्तर है।

https://academy.forumias.com/beta/lms/viewSolutions Page 48 of 180


Academy ForumIAS | Dashboard 18/02/24, 8:41 PM

जागीरदारी प्रणाली मुगल साम्राज्य की एक प्रशासिनक संरचना थी जो अिधकािरयों को वेतन के बदले में भूिम राजस्व प्रदान करती थी। ये
अिधकारी जागीरदार के रूप में जाने जाते थे और अक्सर सैन्य नेता या प्रशासक होते थे। जागीरदारी व्यवस्था िदल्ली सल्तनत की इक्तादारी
व्यवस्था से िवकिसत हुई।
कथन 1 सही है : सम्राट ने सैन्य अिधकािरयों सिहत कुलीनों को उनकी सेवाओं के मुआवजे के रूप में जागीरें प्रदान कीं। जागीरदारों को
िनधार्िरत भूिम से राजस्व वसूल करने का अिधकार था। हालाँिक जागीरदारों को राजस्व एकत्र करने का अिधकार था, लेिकन उनके पास ज़मीन
का स्वािमत्व नहीं था। स्वािमत्व मुगल बादशाह के पास ही रहा।
कथन 2 गलत है : जागीरदारी प्रथा वंशानुगत नहीं थी। जागीरें स्वतः वंशजों को नहीं दी जाती थीं और पुनिनर् धार्रण आम बात थी।
कथन 3 ग़लत है : जागीरों का आकार अलग-अलग होता था, और वे कुलीनों की सेवा, वफादारी और सम्राट के िववेक जैसे कारकों के आधार
पर बड़े या छोटे हो सकते थे।

Source: Poonam Dalal Dahiya Ch-13

Question Attempted / Student

Question: 29 Mark Flag  Previous Next 

Correct Answer : a
Your Answer : c

With reference to the Mughal Empire, who among the following were referred to as Banjaras

मुगल साम्राज्य के संदभर् में, िनम्निलिखत में से िकसे बंजारा कहा जाता था

a Class of traders specialized in carrying bulk goods थोक माल ले जाने में िवशेषज्ञता रखने वाले व्यापािरयों का वगर्
b Settled community engaged primarily in farming. स्थायी समुदाय मुख्य रूप से खेती में लगे हुए थे।
c Skilled workers creating handmade products. कुशल श्रिमक हस्तिनिमर् त उत्पाद बनाते थे।
d Spiritual leaders, guiding religious practices. आध्याित्मक नेता, धािमर् क प्रथाओं का मागर्दशर्न करते थे।

Explanation
Option a is the correct answer.

In the context of the Mughal Empire, the term "Banjaras" referred to a class of traders specialized in carrying bulk
goods. The Banjaras were known for their nomadic lifestyle and their crucial role in transporting goods across long
distances. They formed caravans and played a significant role in facilitating trade by transporting commodities like grains,
salt, and other goods in large quantities.

Source: Poonam Dalal Dahiya Ch-13

https://academy.forumias.com/beta/lms/viewSolutions Page 49 of 180


Academy ForumIAS | Dashboard 18/02/24, 8:41 PM

िवकल्प a सही उत्तर है।


िवकल्प a सही है : मुगल साम्राज्य के संदभर् में, "बंजारा" शब्द थोक माल ले जाने में िवशेषज्ञता रखने वाले व्यापािरयों के एक वगर् को संदिभर् त
करता है। बंजारे अपनी खानाबदोश जीवनशैली और लंबी दू री तक माल पिरवहन में अपनी महत्वपूणर् भूिमका के िलए जाने जाते थे। उन्होंने
कारवां बनाया और बड़ी मात्रा में अनाज, नमक और अन्य वस्तुओ ं का पिरवहन करके व्यापार को सुिवधाजनक बनाने में महत्वपूणर् भूिमका
िनभाई।

Source: Poonam Dalal Dahiya Ch-13

Question Attempted / Student

Question: 30 Mark Flag  Previous Next 

Correct Answer : b
Your Answer : c

India is an important member of the 'the Square Kilometer Array (SKA) project'. Which one of the following is the
primary purpose of this project?

भारत 'स्क्वायर िकलोमीटर ऐरे (SKA) प्रोजेक्ट' का एक महत्वपूणर् सदस्य है। िनम्निलिखत में से कौन सा इस पिरयोजना का प्राथिमक उद्देश्य
है?

To detect the effectiveness of missile defense systems. िमसाइल रक्षा प्रणािलयों की प्रभावशीलता का पता
a लगाना।
b
To explore and understand the origin and evolution of the Universe ब्रह्मांड की उत्पित्त और िवकास का पता लगाना और
समझना
c
To precisely detect gravitational waves by employing trilateral Space based telescopes. ित्रपक्षीय अंतिरक्ष आधािरत दू रबीनों
का उपयोग करके गुरुत्वाकषर्ण तरंगों का सटीक पता लगाना।
d
To explore the potential for life on Earth's moon through the collection of lunar samples. चंद्र नमूनों के संग्रह के माध्यम से
पृथ्वी के चंद्रमा पर जीवन की संभावना का पता लगाना।

Explanation
Option b is the correct answer.

https://academy.forumias.com/beta/lms/viewSolutions Page 50 of 180


Academy ForumIAS | Dashboard 18/02/24, 8:41 PM

Recently the Union Cabinet has approved the Square Kilometer Array (SKA) project, allocating Rs 1,250 crore. The SKA
comprises radio telescopes in South Africa and Australia, operating in two frequency ranges, with the SKA Observatory
headquartered in the UK.

Participating countries include Australia, Canada, China, India, Italy, New Zealand, South Africa, Sweden, the
Netherlands, and the UK.

Option a is incorrect: The Square Kilometer Array (SKA) project is dedicated to the development of the world's largest
radio telescope, not related to detect the effectiveness of missile defense systems.

Option b is correct: The SKA project delves into the vast realms of the cosmos, meticulously mapping visible galaxies,
providing intricate data on the evolution of our own galaxy, searching for potential extraterrestrial life, and identifying
gravitational waves. This comprehensive exploration aims to yield profound insights into the early phases of our galaxy's
evolution while actively investigating the possibility of life beyond Earth.

Option c is incorrect: While the SKA project aims to detect gravitational waves and to study the origin of universe, black
holes etc., it does not involve establishing a trilateral space based telescope. The purpose of the evolved Laser
Interferometer Space Antenna (eLISA) project is to detect and accurately measure gravitational waves by employing
trilateral Space based telescopes.

Option d is incorrect: The SKA project is concerned with establishing a group of radio telescopes and not to explore the
potential for life on Earth's moon through the collection of lunar samples.

Source: https://www.deccanherald.com/science/space/explained-square-kilometre-array-project-worlds-
largest-radio-telescope-and-indias-role-in-it-2833808

िवकल्प b सही उत्तर है


हाल ही में केंद्रीय मंित्रमंडल ने 1,250 करोड़ रुपये आवंिटत करते हुए स्क्वायर िकलोमीटर एरे (एसकेए) पिरयोजना को मंजूरी दे दी है। SKA
में दिक्षण अफ्रीका और ऑस्ट्रेिलया के रेिडयो टेलीस्कोप शािमल हैं, जो दो फ़्रीक्वेंसी रेंज में काम करते हैं, SKA वेधशाला का मुख्यालय यूके
में है। भाग लेने वाले देशों में ऑस्ट्रेिलया, कनाडा, चीन, भारत, इटली, न्यूजीलैंड, दिक्षण अफ्रीका, स्वीडन, नीदरलैंड और यूके शािमल हैं।
िवकल्प a गलत है: स्क्वायर िकलोमीटर एरे (एसकेए) पिरयोजना दुिनया के सबसे बड़े रेिडयो टेलीस्कोप के िवकास के िलए समिपर् त है, जो
िमसाइल रक्षा प्रणािलयों की प्रभावशीलता का पता लगाने से संबंिधत नहीं है।
िवकल्प b सही है: एसकेए पिरयोजना ब्रह्मांड के िवशाल क्षेत्रों में गहराई से उतरती है, दृश्यमान आकाशगंगाओं का सावधानीपूवर्क मानिचत्रण
करती है, हमारी अपनी आकाशगंगा के िवकास पर जिटल डेटा प्रदान करती है, संभािवत अलौिकक जीवन की खोज करती है, और
गुरुत्वाकषर्ण तरंगों की पहचान करती है। इस व्यापक अन्वेषण का उद्देश्य पृथ्वी से परे जीवन की संभावना की सिक्रय रूप से जांच करते हुए
हमारी आकाशगंगा के िवकास के शुरुआती चरणों में गहन अंतदृर्िष्ट प्राप्त करना है।
िवकल्प c गलत है: जबिक एसकेए पिरयोजना का लक्ष्य गुरुत्वाकषर्ण तरंगों का पता लगाना और ब्रह्मांड की उत्पित्त, ब्लैक होल आिद का
अध्ययन करना है, इसमें ित्रपक्षीय अंतिरक्ष आधािरत दू रबीन की स्थापना शािमल नहीं है। िवकिसत लेजर इं टरफेरोमीटर स्पेस एं टीना (eLISA)
पिरयोजना का उद्देश्य ित्रपक्षीय अंतिरक्ष आधािरत दू रबीनों को िनयोिजत करके गुरुत्वाकषर्ण तरंगों का पता लगाना और सटीक रूप से मापना
है।
िवकल्प d गलत है: एसकेए पिरयोजना का संबंध रेिडयो दू रबीनों के एक समूह की स्थापना से है, न िक चंद्र नमूनों के संग्रह के माध्यम से पृथ्वी
के चंद्रमा पर जीवन की संभावना का पता लगाने से।

Source: https://www.deccanherald.com/science/space/explained-square-kilometre-array-project-worlds-largest-
radio-telescope-and-indias-role-in-it-2833808

https://academy.forumias.com/beta/lms/viewSolutions Page 51 of 180


Academy ForumIAS | Dashboard 18/02/24, 8:41 PM

Question Attempted / Student

Question: 31 Mark Flag  Previous Next 

Correct Answer : b
Your Answer : c

With reference to the Francois Bernier, a French physician and traveler, consider the following statements:

1. He arrived in India during the reign of the Mughal Emperor Jahangir.

2. His account of travels in India is titled as "Travels in the Mogul Empire".

Which of the statements given above is/are correct?

फ्रांसीसी िचिकत्सक और यात्री फ्रेंकोइस बिनर् यर के संदभर् में, िनम्निलिखत कथनों पर िवचार करें:
1. वह मुगल सम्राट जहांगीर के शासनकाल के दौरान भारत आये थे।
2. भारत में उनकी यात्रा के िववरण का शीषर्क "मुग़ल साम्राज्य में यात्राएँ " है।

ऊपर िदए गए कथनों में से कौन सा/से सही है/हैं?

a 1 only केवल 1
b 2 only केवल 2
c Both 1 and 2 1 और 2 दोनों
d Neither 1 nor 2 न तो 1 और न ही 2

Explanation
Option b is the correct answer.

François Bernier was a French doctor, political philosopher, and historian. He traveled to the Mughal Empire, like many
others, in search of opportunity. From 1656 until 1668, he lived in India for twelve years, first as a physician to Prince
Dara Shukoh, the eldest son of Emperor Shah Jahan, and later as an intellectual and scientist with Danishmand Khan, an
Armenian noble at the Mughal court.

Statement 1 is incorrect. François Bernier arrived in India in 1656, during Shah Jahan's reign (1628 -1658). He also
lived in India during the reign of the Mughal Emperor Aurangzeb (1658-1707).

1) Francisco Pelsaert, a Dutch traveler, visited India from 1620 to 1627, during the reign of Mughal Emperor
Jahangir.

https://academy.forumias.com/beta/lms/viewSolutions Page 52 of 180


Academy ForumIAS | Dashboard 18/02/24, 8:41 PM

Statement 2 is correct. Bernier's account of India, titled "Travels in the Mogul Empire," recounts his journeys from
1656-1668 during the reigns of Dara Shikoh and Aurangzeb. Drawing from personal experiences and Mughal courtiers,
Bernier portrays India as inferior to the West. His depiction highlights a society with impoverished masses ruled by a
wealthy elite.

Source: Poonam Dalal Dahiya Ch-13

िवकल्प b सही उत्तर है


फ्रांकोइस बिनर् यर एक फ्रांसीसी िचिकत्सक, राजनीितक दाशर्िनक और इितहासकार थे। अवसर की तलाश में, कई अन्य लोगों की तरह, उन्होंने
मुग़ल साम्राज्य की यात्रा की। 1656 से 1668 तक, वह बारह वषोर्ं तक भारत में रहे, पहले सम्राट शाहजहाँ के सबसे बड़े बेटे राजकुमार दारा
शुकोह के िचिकत्सक के रूप में, और बाद में मुगल दरबार में एक अमेर्िनयाई कुलीन दािनशमंद खान के साथ एक बुिद्धजीवी और वैज्ञािनक के
रूप में।
कथन 1 गलत है : फ़्राँस्वा बिनर् यर 1656 में शाहजहाँ के शासनकाल (1628-1658) के दौरान भारत आये। वह मुगल सम्राट औरंगजेब
(1658-1707) के शासनकाल के दौरान भारत में भी रहे थे।
1) फ्रांिसस्को पेलसटर्, एक डच यात्री, ने मुगल सम्राट जहांगीर के शासनकाल के दौरान 1620 से 1627 तक भारत का दौरा िकया था।
कथन 2 सही है : भारत के बारे में बिनर् यर का िववरण, िजसका शीषर्क है "ट्रैवल्स इन द मुगल एम्पायर", दारा िशकोह और औरंगजेब के
शासनकाल के दौरान 1656-1668 तक की उनकी यात्राओं का वणर्न करता है। व्यिक्तगत अनुभवों और मुगल दरबािरयों से प्रेरणा लेते हुए,
बिनर् यर ने भारत को पिश्चम की तुलना में हीन िचित्रत िकया। उनका िचत्रण एक अमीर अिभजात वगर् द्वारा शािसत गरीब जनता वाले समाज पर
प्रकाश डालता है।

Source: Poonam Dalal Dahiya Ch-13

Question Attempted / Student

Question: 32 Mark Flag  Previous Next 

Correct Answer : c
Your Answer :

With reference to the Maratha administration, the terms 'Saranjam' and 'Mokasa' refer to:

मराठा प्रशासन के संदभर् में, 'सरंजम' और 'मोकासा' शब्द संदिभर् त करते हैं:

https://academy.forumias.com/beta/lms/viewSolutions Page 53 of 180


Academy ForumIAS | Dashboard 18/02/24, 8:41 PM

Tax rate for territories beyond Maratha boundaries. मराठा सीमाओं से परे क्षेत्रों के िलए कर की
a दर।
Systematic irrigation methods. व्यविस्थत िसं चाई के
b तरीके।
Grant of land for services rendered. प्रदान की गई सेवाओं के िलए भूिम का
c अनुदान।
Administrative divisions for governance purposes. शासन उद्देश्यों के िलए प्रशासिनक
d प्रभाग।

Explanation
Option c is the correct answer.

Chhatrapati Shivaji organized a well-organized and effective army. The chief and military commander were paid with jagir
grants (Saranjam or Mokasa), while ordinary troops were paid in cash.

Option c is correct. Saranjam is a land grant given to military commanders and other officials in return for their
service. The grant is for the maintenance of troops or for military service. The holder of a Saranjam is known as the
Saranjamdar. The Saranjamdar is entitled to collect revenue from the villages included in the territory.

1) Mokasa during the Maratha administration refers to a grant of land for military service. The land was generally rent-
free.

Source: Poonam Dalal Dahiya

https://dspace.gipe.ac.in/xmlui/bitstream/handle/10973/28998/GIPE-005258.pdf?sequence=2&isAllowed=y

िवकल्प c सही उत्तर है।


छत्रपित िशवाजी ने एक सुसंगिठत एवं प्रभावशाली सेना का गठन िकया। प्रमुख और सैन्य कमांडर को जागीर अनुदान (सारंजम या मोकासा)
के साथ भुगतान िकया जाता था, जबिक सामान्य सैिनकों को नकद में भुगतान िकया जाता था।
िवकल्प c सही है : सरनजम सैन्य कमांडरों और अन्य अिधकािरयों को उनकी सेवा के बदले में िदया जाने वाला भूिम अनुदान है। अनुदान
सैिनकों के रखरखाव या सैन्य सेवा के िलए था। सरंजाम धारक को सरंजमदार कहा जाता है। सरंजमदार को क्षेत्र में शािमल गांवों से राजस्व
एकत्र करने का अिधकार था।
1) मराठा प्रशासन के दौरान मोकासा का तात्पयर् सैन्य सेवा के िलए भूिम अनुदान से है। भूिम आम तौर पर लगान-मुक्त थी।
Source: Poonam Dalal Dahiya
https://dspace.gipe.ac.in/xmlui/bitstream/handle/10973/28998/GIPE-005258.pdf?sequence=2&isAllowed=y

Question Attempted / Student

Question: 33 Mark Flag  Previous Next 

https://academy.forumias.com/beta/lms/viewSolutions Page 54 of 180


Academy ForumIAS | Dashboard 18/02/24, 8:41 PM

Correct Answer : d
Your Answer : b

With reference to Sultanate period, consider the following statements regarding Iqta system:

1. It was introduced by Qutb-ud-din Aibak in India.

2. It was inspired and based on the Amara-nayaka system of Vijaya nagara empire.

Which of the statements given above is/are correct?

सल्तनत काल के संदभर् में, इक्ता प्रणाली के संबंध में िनम्निलिखत कथनों पर िवचार करें:
1. इसे भारत में कुतुब-उद-दीन ऐबक द्वारा पेश िकया गया था।
2. यह िवजय नगर साम्राज्य की अमर-नायक प्रणाली से प्रेिरत और आधािरत थी।

ऊपर िदए गए कथनों में से कौन सा/से सही है/हैं?

a 1 only केवल 1
b 2 only केवल 2
c Both 1 and 2 1 और 2 दोनों
d Neither 1 nor 2 न तो 1 और न ही 2

Explanation
Option d is the correct answer.

In the Delhi Sultanate, the Iqta system was a land distribution and administration system. In exchange for military service,
military leaders and nobles were given the right to collect money from a certain piece of land. The Iqta holders were in
charge of managing the area and collecting taxes.

Statement 1 is incorrect. Shamsa ud-din Iltutmish of the Delhi Sultanate introduced the Iqta system in India in the
13th century. The system was based on the division of territories between Sultans and Nobility.

Statement 2 is incorrect. The amara-nayaka system was a major political innovation of the Vijayanagara Empire. It is
believed that many features of this system were derived from the iqta system of the Delhi Sultanate (not vice-versa).

Source: Poonam Dalal Dahiya Ch-11

िवकल्प d सही उत्तर है।


िदल्ली सल्तनत में इक्ता प्रणाली एक भूिम िवतरण और प्रशासन प्रणाली थी। सैन्य सेवा के बदले में, सैन्य नेताओं और रईसों को भूिम के एक
िनिश्चत टु कड़े से धन इकट्ठा करने का अिधकार िदया गया था। इक्ता धारक क्षेत्र के प्रबंधन और कर एकत्र करने के प्रभारी थे।
कथन 1 गलत है : िदल्ली सल्तनत के शम्सा उद-दीन इल्तुतिमश ने 13वीं शताब्दी में भारत में इक्ता प्रणाली की शुरुआत की। यह प्रणाली
सुल्तानों और कुलीनों के बीच क्षेत्रों के िवभाजन पर आधािरत थी।

https://academy.forumias.com/beta/lms/viewSolutions Page 55 of 180


Academy ForumIAS | Dashboard 18/02/24, 8:41 PM

कथन 2 गलत है : अमर-नायक प्रणाली िवजयनगर साम्राज्य का एक प्रमुख राजनीितक नवाचार थी। ऐसा माना जाता है िक इस प्रणाली की
कई िवशेषताएं िदल्ली सल्तनत की इक्ता प्रणाली से ली गई थीं (इसके िवपरीत नहीं)।

Source: Poonam Dalal Dahiya Ch-11

Question Attempted / Student

Question: 34 Mark Flag  Previous Next 

Correct Answer : a
Your Answer : d

With reference to modern Indian history, consider the following statements about Indian National Army:

1. The INA formation was formally announced at the Bangkok Conference of 1942.

2. INA captured Andaman and Nicobar Island from the British empire in 1943.

3. INA launched Operation U-GO against the British empire in the North-East in 1944.

How many of the statements given above are correct?

आधुिनक भारतीय इितहास के संदभर् में, भारतीय राष्ट्रीय सेना (INA) के बारे में िनम्निलिखत कथनों पर िवचार करें:
1. INA गठन की औपचािरक घोषणा 1942 के बैंकॉक सम्मेलन में की गई थी।
2. INA ने 1943 में िब्रिटश साम्राज्य से अंडमान और िनकोबार द्वीप पर कब्जा कर िलया।
3. INA ने 1944 में उत्तर-पूवर् में िब्रिटश साम्राज्य के िखलाफ ऑपरेशन यू-गो (U-GO) लॉन्च िकया।

ऊपर िदए गए कथनों में से िकतने सही हैं?

Only one केवल


a एक
Only two केवल
b दो
All three
c तीनों
None कोई
d नहीं

Explanation

https://academy.forumias.com/beta/lms/viewSolutions Page 56 of 180


Academy ForumIAS | Dashboard 18/02/24, 8:41 PM

Option a is the correct answer.

The Indian National Army (INA), also known as the Azad Hind Fauj (Free Indian Army), was a military force established
in 1942 by Indian nationalists in Southeast Asia during World War II.

Statement 1 is correct: The Japanese after defeating the British in South East Asia, took a number of Indian soldiers as
prisoners of war. A Japanese army officer persuaded captain Mohan Sigh, a prisoner of war to work in collaboration with
the Japanese for India's freedom. In march 1942 a conference of Indians was held in Tokyo, and they formed the Indian
Independence League. This was followed by a conference in Bangkok in June 1942. Rashbehari Bose was elected as
president of the league and a decision was taken to raise the INA. Captain Mohan singh was appointed the commander
of the INA which had about 40,000 Indian soldiers. This conference invited Subash Bose to lead the movement.

Statement 2 is incorrect: The Japanese intervention and rapid Japanese victories since December 1941 brought war closer
to India: between December 1941 and March 1942 Hong Kong, Borneo, Manila, Singapore, Java, Rangoon, Sumatra and
Andaman and Nicobar Islands fell into Japanese hands in quick succession. On November 6, 1943, the Japanese army
handed over control of the Andaman and Nicobar Islands to the INA. (Hence Andaman Island was not captured by
the INA)

Statement 3 is incorrect: Operation U-Go, also known as Operation C, was a major Japanese offensive launched in
March 1944 against British forces in northeastern India during World War II (not launched by INA). The operation
aimed to cut off and destroy British forces in the region, capture the strategically important towns of Imphal and Kohima,
and ultimately drive into the Brahmaputra Valley.

Source: https://egyankosh.ac.in/bitstream/123456789/20108/1/Unit-34.pdf

https://en.wikipedia.org/wiki/Operation_U-Go

From Plassey to Partition and After: A History of Modern India by Śekhara Bandyopādhyāẏa

िवकल्प a सही उत्तर है।


भारतीय राष्ट्रीय सेना (INA), िजसे आज़ाद िहं द फ़ौज (स्वतंत्र भारतीय सेना) के नाम से भी जाना जाता है, िद्वतीय िवश्व युद्ध के दौरान दिक्षण
पूवर् एिशया में भारतीय राष्ट्रवािदयों द्वारा 1942 में स्थािपत एक सैन्य बल था।
कथन 1 सही है: दिक्षण पूवर् एिशया में अंग्रेजों को हराने के बाद जापािनयों ने कई भारतीय सैिनकों को युद्ध बंदी बना िलया। एक जापानी सेना
अिधकारी ने युद्धबंदी कैप्टन मोहन िसघ को भारत की आजादी के िलए जापािनयों के साथ िमलकर काम करने के िलए राजी िकया। माचर्
1942 में टोक्यो में भारतीयों का एक सम्मेलन आयोिजत िकया गया और उन्होंने इं िडयन इं िडपेंडेंस लीग का गठन िकया। इसके बाद जून
1942 में बैंकॉक में एक सम्मेलन हुआ। रासिबहारी बोस को लीग के अध्यक्ष के रूप में चुना गया और आईएनए को खड़ा करने का िनणर्य
िलया गया। कैप्टन मोहन िसं ह को INA का कमांडर िनयुक्त िकया गया िजसमें लगभग 40,000 भारतीय सैिनक थे। इस सम्मेलन ने सुभाष
बोस को आं दोलन का नेतृत्व करने के िलए आमंित्रत िकया।
कथन 2 गलत है: िदसंबर 1941 के बाद से जापानी हस्तक्षेप और तेजी से जापानी जीत ने भारत में युद्ध को करीब ला िदया: िदसंबर 1941
और माचर् 1942 के बीच हांगकांग, बोिनर् यो, मनीला, िसं गापुर, जावा, रंगून, सुमात्रा और अंडमान और िनकोबार द्वीप समूह जापानी हाथों में आ
गए। त्विरत उत्तरािधकार में. 6 नवंबर, 1943 को जापानी सेना ने अंडमान और िनकोबार द्वीप समूह का िनयंत्रण INA को सौंप िदया। (इसिलए
अंडमान द्वीप पर आईएनए का कब्जा नहीं था)
कथन 3 गलत है: ऑपरेशन यू-गो, िजसे ऑपरेशन C के नाम से भी जाना जाता है, िद्वतीय िवश्व युद्ध के दौरान पूवोर्त्तर भारत में िब्रिटश सेना के
िखलाफ माचर् 1944 में शुरू िकया गया एक प्रमुख जापानी आक्रमण था (आईएनए द्वारा शुरू नहीं िकया गया था)। इस ऑपरेशन का उद्देश्य
क्षेत्र में िब्रिटश सेना हटाना और नष्ट करना, इं फाल और कोिहमा के रणनीितक रूप से महत्वपूणर् शहरों पर कब्जा करना और अंततः ब्रह्मपुत्र
घाटी में प्रवेश करना था।

https://academy.forumias.com/beta/lms/viewSolutions Page 57 of 180


Academy ForumIAS | Dashboard 18/02/24, 8:41 PM

Source: https://egyankosh.ac.in/bitstream/123456789/20108/1/Unit-34.pdf
https://en.wikipedia.org/wiki/Operation_U-Go

From Plassey to Partition and After: A History of Modern India by Śekhara Bandyopādhyāẏa

Question Attempted / Student

Question: 35 Mark Flag  Previous Next 

Correct Answer : b
Your Answer :

The Reserve Bank of India's recent directives relating to 'Green Deposits', popularly known as Green Deposits Framework,
command that such deposits:

1. shall be compulsorily raised by the Commercial banks and Non Banking Financial Institutions (NBFCs).

2. can be invested in short-term liquid instruments with a one-year maturity.

3. eligible to avail protection under the Deposit Insurance and Credit Guarantee Corporation (DICGC) act.

How many of the statements given above are correct?

भारतीय िरजवर् बैंक के 'ग्रीन िडपॉिजट् स' से संबंिधत हािलया िनदेर्श, िजसे ग्रीन िडपॉिजट् स फ्रेमवकर् के नाम से जाना जाता है, आदेश देता है िक
ऐसी जमाएं :
1. वािणिज्यक बैंकों और गैर बैंिकंग िवत्तीय संस्थानों (NBFC) द्वारा अिनवायर् रूप से उठाया जाएगा।
2. एक साल की पिरपक्वता वाले अल्पकािलक तरल उपकरणों में िनवेश िकया जा सकता है।
3. जमा बीमा और क्रेिडट गारंटी िनगम (DICGC) अिधिनयम के तहत सुरक्षा प्राप्त करने के पात्र।

ऊपर िदए गए कथनों में से िकतने सही हैं?

a Only one केवल एक


b Only two केवल दो
c All three तीनों
d None कोई नहीं

Explanation
Option b is the correct answer.

https://academy.forumias.com/beta/lms/viewSolutions Page 58 of 180


Academy ForumIAS | Dashboard 18/02/24, 8:41 PM

The Reserve Bank of India (RBI) recently issued updated guidelines on its Green Deposits Framework, outlining key
provisions. Green deposits are similar to regular bank deposits in all aspects except funds from these deposits are
committed to supporting environmentally friendly initiatives.

Statement 1 is incorrect: The RBI clarified that financial institutions, including banks and non-banking financial
corporations (NBFCs), are not required to raise green deposits. This means that raising green deposits is voluntary for
commercial banks and NBFCs.

Statement 2 is correct: Green deposits can be invested in short-term liquid instruments, with a one-year maturity
restriction. However, this limitation is applicable until the funds are allocated to green activities or projects.

Statement 3 is correct: The deposits raised under the framework are covered by Deposit Insurance and Credit
Guarantee Corporation (DICGC) in accordance with the Deposit Insurance and Credit Guarantee Corporation Act,
1961.

Source: https://www.livemint.com/money/personal-finance/planning-to-invest-in-green-deposits-rbi-
releases-latest-guidelines-to-explain-key-provisions-11703932993438.html

िवकल्प b सही उत्तर है


भारतीय िरजवर् बैंक (आरबीआई) ने हाल ही में अपने ग्रीन िडपॉिजट फ्रेमवकर् पर अद्यतन िदशािनदेर्श जारी िकए हैं, िजसमें प्रमुख प्रावधानों की
रूपरेखा दी गई है। ग्रीन िडपॉिजट सभी पहलुओ ं में िनयिमत बैंक जमाओं के समान हैं, िसवाय इसके िक इन जमाओं से प्राप्त धनरािश
पयार्वरण के अनुकूल पहलों का समथर्न करने के िलए प्रितबद्ध है।
कथन 1 गलत है: आरबीआई ने स्पष्ट िकया िक बैंकों और गैर-बैंिकंग िवत्तीय िनगमों (NBFC) सिहत िवत्तीय संस्थानों को ग्रीन िडपॉिजट
जुटाने की आवश्यकता नहीं है। इसका मतलब यह है िक वािणिज्यक बैंकों और NBFC के िलए ग्रीन िडपॉिजट स्वैिच्छक है।
कथन 2 सही है: हिरत जमा को एक वषर् की पिरपक्वता प्रितबंध के साथ, अल्पकािलक तरल उपकरणों में िनवेश िकया जा सकता है।
हालाँिक, यह सीमा तब तक लागू है जब तक िक धन हिरत गितिविधयों या पिरयोजनाओं के िलए आवंिटत नहीं िकया जाता है।
कथन 3 सही है: ढांचे के तहत जुटाई गई जमा रािश जमा बीमा और क्रेिडट गारंटी िनगम अिधिनयम, 1961 के अनुसार जमा बीमा और क्रेिडट
गारंटी िनगम (DICGC) द्वारा कवर की जाती है।
Source: https://www.livemint.com/money/personal-finance/planning-to-invest-in-green-deposits-rbi-releases-
latest-guidelines-to-explain-key-provisions-11703932993438.html

Question Attempted / Student

Question: 36 Mark Flag  Previous Next 

Correct Answer : b
Your Answer : a

With reference to the Sikh Empire, Adalat-i-Ala established by Maharaja Ranjit Singh is correctly described by which
option?

https://academy.forumias.com/beta/lms/viewSolutions Page 59 of 180


Academy ForumIAS | Dashboard 18/02/24, 8:41 PM

िसख साम्राज्य के संदभर् में महाराजा रणजीत िसं ह द्वारा स्थािपत अदालत-ए-आला का सही वणर्न िकस िवकल्प द्वारा िकया गया है?

a
It was a local-level court established at multiple places throughout the Sikh Empire. यह पूरे िसख साम्राज्य में कई स्थानों
पर स्थािपत एक स्थानीय स्तर का न्यायालय था।
It was a supreme judicial body of the Sikh Empire. यह िसख साम्राज्य की सवोर्च्च न्याियक संस्था
b थी।
c
It was the headquarters of the revenue department of the Sikh Empire. यह िसख साम्राज्य के राजस्व िवभाग का मुख्यालय
था।
d
It was a trade centre established in the capital to increase trade. यह व्यापार बढ़ाने के िलए राजधानी में स्थािपत एक व्यापार केंद्र
था।

Explanation
Option b is the correct answer

The Adalat-i-Ala (meaning "High Court" in English) was the Supreme Court of the Sikh Empire during the reign of
Maharaja Ranjit Singh (1780-1839). Established in Lahore, the capital of the empire, it served as the highest judicial
authority, handling appeals from lower courts across the vast Sikh territories.

The administration of justice was very simple during his reign. There was no code of law. Most of the crimes were
punished with fine and some were punished with mutilation. There was no capital punishment. Justice was
administered by the Panchayats in the villages and the Kardars in the towns. In the provincial headquarters there
were the courts of the Nazims. Above them there was the Adalat-i-Ala (Supreme Court) in Lahore. The final court
of appeal was the Maharaja himself..

Source: https://chandigarh.gov.in/sites/default/files/documents/high_court.pdf

https://trcollege.edu.in/study-material/26-history/59-rise-of-the-sikh-states.html

िवकल्प b सही उत्तर है।


िवकल्प b सही है: अदालत-ए-आला (अंग्रेजी में िजसका अथर् है "उच्च न्यायालय") महाराजा रणजीत िसं ह (1780-1839) के शासनकाल के
दौरान िसख साम्राज्य का सवोर्च्च न्यायालय था। साम्राज्य की राजधानी लाहौर में स्थािपत, यह सवोर्च्च न्याियक प्रािधकरण के रूप में कायर्
करता था, जो िवशाल िसख क्षेत्रों में िनचली अदालतों से अपील को संभालता था।
उनके शासनकाल में न्याय प्रशासन बहुत सरल था। कोई िविधक संिहता नहीं थी। अिधकांश अपराधों के िलए जुमार्ना और कुछ को अंग-भंग
करने की सजा दी गई। कोई मृत्युदंड नहीं था। न्याय का संचालन गाँवों में पंचायतें और शहरों में कारदार करते थे। प्रांतीय मुख्यालय में नािज़मों
की अदालतें थीं। इनके ऊपर लाहौर में अदालत-ए-आला (सवोर्च्च न्यायालय) था। अपील की अंितम अदालत स्वयं महाराजा थे।
Source: https://chandigarh.gov.in/sites/default/files/documents/high_court.pdf

https://trcollege.edu.in/study-material/26-history/59-rise-of-the-sikh-states.html

https://academy.forumias.com/beta/lms/viewSolutions Page 60 of 180


Academy ForumIAS | Dashboard 18/02/24, 8:41 PM

Question Attempted / Student

Question: 37 Mark Flag  Previous Next 

Correct Answer : c
Your Answer : c

Consider the following with respect to personality associated with the modern Indian history:

He was a prominent Indian reformer, educator, and social activist during the 19th century. He served as the principal of
Sanskrit College in Kolkata, India, from 1851 to 1858. His efforts led to the passage of the Widow Remarriage Act of
1856, challenging societal norms.' He introduced the study of western thought in the Sanskrit College and opened its gates
to non-Brahmin students. He wrote a book 'Hindu Widow Marriage' to bring social reforms in the country.

Which of the following personalities is referred in the above paragraph?

आधुिनक भारतीय इितहास से जुड़े व्यिक्तत्व के संबंध में िनम्निलिखत पर िवचार करें:
वह 19वीं शताब्दी के दौरान एक प्रमुख भारतीय सुधारक, िशक्षक और सामािजक कायर्कतार् थे। उन्होंने 1851 से 1858 तक भारत के
कोलकाता में संस्कृत कॉलेज के िप्रंिसपल के रूप में कायर् िकया। उनके प्रयासों के कारण 1856 का िवधवा पुनिवर् वाह अिधिनयम पािरत हुआ,
िजसने सामािजक मानदंडों को चुनौती दी। उन्होंने संस्कृत कॉलेज में पिश्चमी िवचारों का अध्ययन शुरू िकया और गैर-ब्राह्मण छात्रों के िलए
इसके द्वार खोल िदए। उन्होंने देश में सामािजक सुधार लाने के िलए “िहन्दू िवधवा िववाह” नामक पुस्तक िलखी।

उपरोक्त पिरच्छे द में िनम्निलिखत में से िकस व्यिक्तत्व का उल्लेख िकया गया है?

a Ashwini Kumar Dutta अिश्वनी कुमार दत्त


b Swami Dayanand Saraswati स्वामी दयानंद सरस्वती
c Ishwar Chandra Vidyasagar ईश्वर चंद्र िवद्यासागर
d Swami Vivekananda स्वामी िववेकानन्द

Explanation
Option c is the correct answer.

Option a is incorrect. Ashwini Kumar Dutta (1856-1923) was an Indian nationalist and social reformer who
founded the Swadesh Bandhab Samiti.

1) The Samiti's goal was to promote the use of local products and boycott foreign goods.

2) Dutta was also a delegate to the second session of the Indian National Congress in Kolkata in 1886.

https://academy.forumias.com/beta/lms/viewSolutions Page 61 of 180


Academy ForumIAS | Dashboard 18/02/24, 8:41 PM

Option b is incorrect. The Arya Samaj founded by Swami Dayanand Saraswati in 1875 undertook the task of reforming
Hindu religion in North India.

1) He considered Vedas to be infallible and the foundation of all knowledge. He rejected all those religious thoughts which
were in conflict with the Vedas.

2) He believed that every person had the right to have direct access to God.

3) It started the Shuddhi Movement to bring back those Hindus who had converted to Islam and Christianity.

4) Satyarth Prakash was his most important book.

Option c is correct. Ishwar Chandra Vidyasagar (1820-1891) was a 19th century social reformer and intellectual
activist.

1) He is known for his contributions to social justice, women's empowerment, and education.

2) He also supported John Elliot Bethune's idea to establish India's first permanent school.

3) Vidyasagar started a campaign for the remarriage of widows in 1854. His efforts led to the Hindu Widow Remarriage
Act of 1856.

4) He became the principal of the Sanskrit College in Kolkata in 1851. Vidyasagar began working as a literature
teacher at the Government Sanskrit College in 1850. The following year, he was appointed principal of the same college.
He introduced the study of western thought in the SanQkrit College and opened its gates to non-Brahmin students.

5) He wrote "Bhranti Bilas": A classic Bengali novel that satirizes the prevalent social customs and practices of the
time.

6) He wrote 'Upakramonika' and 'Byakaran Koumudi', which explained complex concepts of Sanskrit grammar in simple
to read Bengali.

Option d is incorrect: Swami Vivekananda (1863-1902) was the first spiritual leader who thought beyond religious
reforms.

1) He felt that Indian masses needed secular as well as spiritual knowledge to empower them to believe in themselves.

2) Vivekananda established the Ramakrishna mission after the name of his guru Ramakrishna Paramhansa.

3) Through his speeches and writings, he brought out the essence of Hindu culture and religion.

4) He believed in the spirit of Vedanta and the essential unity and equality of all religions.

5) He laid stress on the removal of religious superstitions, obscurantism, and outdated social customs. He tried to remove
caste rigidities, and untouchability.

Knowledge Base: Notable literary works of Ishwar Chandra Vidyasagar include:

1) "Barnaparichay": A primer introducing Bengali alphabets, designed to simplify the learning process for young children.

2) "Betar Dan": A collection of Bengali essays reflecting Vidyasagar's thoughts on various societal issues.

3) "Shakuntala": An adaptation of Kalidasa's Sanskrit drama "Abhijnanasakuntalam," showcasing Vidyasagar's literary


prowess.

https://academy.forumias.com/beta/lms/viewSolutions Page 62 of 180


Academy ForumIAS | Dashboard 18/02/24, 8:41 PM

4) "Nari Siksha": A work advocating for women's education, emphasizing the importance of educating girls for societal
progress.

5) "Kotha Mala": A collection of Bengali proverbs, showcasing Vidyasagar's interest in language and cultural expressions.

Source: A BRIEF HISTORY OF MODERN INDIA BY RAJIV AHIR

https://www.culturalindia.net/reformers/ishwar-chandra-vidyasagar.html

https://www.nios.ac.in/media/documents/SecSocSciCour/English/Lesson-06.pdf

िवकल्प c सही उत्तर है।


िवकल्प a ग़लत है. अिश्वनी कुमार दत्त (1856-1923) एक भारतीय राष्ट्रवादी और समाज सुधारक थे िजन्होंने स्वदेश बंधब सिमित की
स्थापना की।
1) सिमित का लक्ष्य स्थानीय उत्पादों के उपयोग को बढ़ावा देना और िवदेशी वस्तुओ ं का बिहष्कार करना था।
2)दत्ता 1886 में कोलकाता में भारतीय राष्ट्रीय कांग्रेस के दू सरे सत्र में भी एक प्रितिनिध थे।
िवकल्प b ग़लत है. 1875 में स्वामी दयानंद सरस्वती द्वारा स्थािपत आयर् समाज ने उत्तर भारत में िहं दू धमर् में सुधार का कायर् िकया।
1) वे वेदों को अचूक और सभी ज्ञान का आधार मानते थे। उन्होंने उन सभी धािमर् क िवचारों को अस्वीकार कर िदया जो वेदों के िवपरीत थे।
2) उनका मानना था िक प्रत्येक व्यिक्त को ईश्वर तक सीधी पहुंच का अिधकार है।
3) इसने उन िहं दुओ ं को वापस लाने के िलए शुिद्ध आं दोलन शुरू िकया जो इस्लाम और ईसाई धमर् में पिरवितर् त हो गए थे।
4) सत्याथर् प्रकाश उनकी सबसे महत्वपूणर् पुस्तक थी।
िवकल्प c सही है। ईश्वर चंद्र िवद्यासागर (1820-1891) 19वीं सदी के समाज सुधारक और बौिद्धक कायर्कतार् थे।
1) उन्हें सामािजक न्याय, मिहला सशिक्तकरण और िशक्षा में उनके योगदान के िलए जाना जाता है।
2) उन्होंने भारत के पहले स्थायी स्कूल की स्थापना के जॉन इिलयट बेथ्यून के िवचार का भी समथर्न िकया।
3) िवद्यासागर ने 1854 में िवधवाओं के पुनिवर् वाह के िलए एक अिभयान शुरू िकया। उनके प्रयासों से 1856 का िहं दू िवधवा पुनिवर् वाह
अिधिनयम बना।
4) वह 1851 में कोलकाता में संस्कृत कॉलेज के िप्रंिसपल बने। िवद्यासागर ने 1850 में सरकारी संस्कृत कॉलेज में सािहत्य िशक्षक के रूप में
काम करना शुरू िकया। अगले वषर्, उन्हें उसी कॉलेज का िप्रंिसपल िनयुक्त िकया गया। उन्होंने संस्कृत कॉलेज में पिश्चमी िवचारों का अध्ययन
शुरू िकया और गैर-ब्राह्मण छात्रों के िलए इसके द्वार खोल िदए।
5) उन्होंने "भ्रांित िबलास" िलखा: एक क्लािसक बंगाली उपन्यास जो उस समय के प्रचिलत सामािजक रीित-िरवाजों और प्रथाओं पर व्यंग्य
करता है।
6) उन्होंने 'उपक्रमोिनका' और 'ब्याकरण कौमुदी' िलखी, िजसमें संस्कृत व्याकरण की जिटल अवधारणाओं को पढ़ने में आसान बंगाली भाषा में
समझाया गया।
िवकल्प d गलत है: स्वामी िववेकानन्द (1863-1902) पहले आध्याित्मक नेता थे िजन्होंने धािमर् क सुधारों से परे सोचा।
1) उन्होंने महसूस िकया िक भारतीय जनता को खुद पर िवश्वास करने के िलए सशक्त बनाने के िलए धमर्िनरपेक्ष और आध्याित्मक ज्ञान की
आवश्यकता है।
2) िववेकानन्द ने अपने गुरु रामकृष्ण परमहंस के नाम पर रामकृष्ण िमशन की स्थापना की।
3) अपने भाषणों और लेखों के माध्यम से उन्होंने िहं दू संस्कृित और धमर् का सार सामने रखा।
4) वे वेदांत की भावना और सभी धमोर्ं की आवश्यक एकता और समानता में िवश्वास करते थे।
5) उन्होंने धािमर् क अंधिवश्वासों, रूिढ़वािदता और पुरानी सामािजक रीित-िरवाजों को हटाने पर जोर िदया। उन्होंने जाितगत कठोरता और
अस्पृश्यता को दू र करने का प्रयास िकया।
ज्ञानकोष: ईश्वर चंद्र िवद्यासागर के उल्लेखनीय सािहित्यक कायोर्ं में शािमल हैं:

https://academy.forumias.com/beta/lms/viewSolutions Page 63 of 180


Academy ForumIAS | Dashboard 18/02/24, 8:41 PM

1) "बनर्पिरचय": बंगाली वणर्माला का पिरचय देने वाला एक प्राइमर, िजसे छोटे बच्चों के िलए सीखने की प्रिक्रया को सरल बनाने के िलए
िडज़ाइन िकया गया है।
2) "बेटर दान": िविभन्न सामािजक मुद्दों पर िवद्यासागर के िवचारों को प्रितिबं िबत करने वाले बंगाली िनबंधों का एक संग्रह।
3) "शकुंतला": कािलदास के संस्कृत नाटक "अिभज्ञानशाकुंतलम" का रूपांतरण, जो िवद्यासागर की सािहित्यक क्षमता को प्रदिशर् त करता है।
4) "नारी िशक्षा": मिहलाओं की िशक्षा की वकालत करने वाला एक कायर्, सामािजक प्रगित के िलए लड़िकयों को िशिक्षत करने के महत्व पर
जोर देता है।
5) "कोठा माला": बंगाली कहावतों का एक संग्रह, जो भाषा और सांस्कृितक अिभव्यिक्तयों में िवद्यासागर की रुिच को प्रदिशर् त करता है।
Source: A BRIEF HISTORY OF MODERN INDIA BY RAJIV AHIR
https://www.culturalindia.net/reformers/ishwar-chandra-vidyasagar.html

https://www.nios.ac.in/media/documents/SecSocSciCour/English/Lesson-06.pdf

Question Attempted / Student

Question: 38 Mark Flag  Previous Next 

Correct Answer : c
Your Answer : c

In the 1985, Birsa Munda, a charismatic leader, inspired the Munda tribe of Chotanagpur to rise up against British colonial
policies. Which of the following is incorrect about the movement?

1985 में, एक किरश्माई नेता िबरसा मुंडा ने छोटानागपुर की मुंडा जनजाित को िब्रिटश औपिनवेिशक नीितयों के िखलाफ उठने के िलए प्रेिरत
िकया। आं दोलन के बारे में िनम्निलिखत में से कौन सा गलत है?

a
It was a political movement to drive out dikus and the British government and establish self ruel. यह िदकुओं और िब्रिटश
सरकार को बाहर िनकालने और स्वशासन स्थािपत करने के िलए एक राजनीितक आं दोलन था।
b
It was against the British policy which replaced the traditional khuntkari system of Munda tribes with the zamindari
system. यह िब्रिटश नीित के िखलाफ था िजसने मुंडा जनजाितयों की पारंपिरक खुंटकारी प्रणाली को जमींदारी प्रणाली में पिरणत कर
िदया था।
c
It wanted to establish a new religion with the help of Christian missionaries to remove superstitions from the Mundas.
यह मुंडाओं से अंधिवश्वास दू र करने के िलए ईसाई िमशनिरयों की मदद से एक नया धमर् स्थािपत करना चाहता था।
d
It led the British government to pass Tenancy Act of 1903, to protect the interest of the tribals. इसके कारण िब्रिटश
सरकार को आिदवािसयों के िहतों की रक्षा के िलए 1903 का काश्तकारी अिधिनयम पािरत करना पड़ा।

https://academy.forumias.com/beta/lms/viewSolutions Page 64 of 180


Academy ForumIAS | Dashboard 18/02/24, 8:41 PM

Explanation
Option c is the correct answer.

The Munda Rebellion, also known as the Ulgulan or the Tamar Revolt, was an uprising by the Munda tribal community
against British rule in the Chotanagpur plateau region during the late 19th century (1895-1900). The Mundas adopted
guerrilla warfare tactics, utilizing the terrain to their advantage. They attacked symbols of authority, including police
stations and government offices, and engaged in hit-and-run tactics.

Option a is correct. The aim of the Birsa movement was to drive out missionaries, moneylenders, Hindu landlords,
and the government and set up a Munda Raj with Birsa at its head. The movement identified all these forces as the cause of
the misery the Mundas were suffering.

Option b is correct. Before the introduction of the British policies in the areas inhabited by Qaron and Munda, their
traditional land and social systems had existed.

1) Their land system was known as 'Khuntkari system'.

2) The tribals enjoyed customary rights over their land.

3) The system was marked by the absence of the class of landlords.

4) The tribals worked on their land and paid tributes to their chiefs.

By 1874, the British replaced the traditional khuntkari system by the zamindari system. The introduction of zamindari
system created the classes of zamindars (landlords) and ryots (tenants). The tribals now had to pay rent to the landlords
and failure to do so resulted in their eviction from land.

Option c is incorrect. Birsa Munda wanted to establish a religion and he wanted it to be free from any Christian
missionaries' interference. He wanted drive out the missionaries and other britishers from the munda land.- Hence the
statement is incorrect.

Option d is correct: The Birsa Munda movement had its impact on the government attitude towards their problems.

1) The authorities felt the need to prepare the land records so that they could safeguard the tribal interests.

2) The government conducted surveys and settlement operations for the tribals between 1902 and 1910 for achieving this
purpose.

3) It decided to abolish the compulsory begar system and passed the Tenancy Act of 1903 which recognised the
Mundari Khuntkatti system.

4) The ' Government 'also passed the Chotanagpur Tenancy Act in 1908.

Source: A BRIEF HISTORY OF MODERN INDIA BY RAJIV AHIR Ch-17

िवकल्प c सही उत्तर है।


मुंडा िवद्रोह, िजसे उलगुलान या तमार िवद्रोह के नाम से भी जाना जाता है, 19वीं सदी के अंत (1895-1900) के दौरान छोटानागपुर पठार क्षेत्र
में िब्रिटश शासन के िखलाफ मुंडा आिदवासी समुदाय द्वारा िकया गया िवद्रोह था। मुंडाओं ने अपने लाभ के िलए इलाके का उपयोग करते हुए
गुिरल्ला युद्ध रणनीित अपनाई। उन्होंने पुिलस स्टेशनों और सरकारी कायार्लयों सिहत सत्ता के प्रतीकों पर हमला िकया और मारो और भागो की
रणनीित में लगे रहे।

https://academy.forumias.com/beta/lms/viewSolutions Page 65 of 180


Academy ForumIAS | Dashboard 18/02/24, 8:41 PM

िवकल्प a सही है. िबरसा आं दोलन का उद्देश्य िमशनिरयों, साहूकारों, िहं दू जमींदारों और सरकार को बाहर िनकालना और िबरसा के नेतृत्व में
मुंडा राज की स्थापना करना था। आं दोलन ने इन सभी ताकतों को मुंडाओं की दुदर्शा का कारण बताया।
िवकल्प b सही है. क़ै रोन और मुंडा के िनवास वाले क्षेत्रों में िब्रिटश नीितयों की शुरूआत से पहले, उनकी पारंपिरक भूिम और सामािजक
प्रणािलयाँ मौजूद थीं।
1) उनकी भूिम व्यवस्था को 'खुंटकारी व्यवस्था' के नाम से जाना जाता था।
2) आिदवािसयों को अपनी भूिम पर पारंपिरक अिधकार प्राप्त थे।
3) इस व्यवस्था की पहचान जमींदारों के वगर् की अनुपिस्थित थी।
4) आिदवासी अपनी भूिम पर काम करते थे और अपने मुिखयाओं को नजराना/कर देते थे।
1874 तक, अंग्रेजों ने पारंपिरक खुंटकारी प्रणाली को जमींदारी प्रणाली से बदल िदया। जमींदारी प्रथा की शुरूआत ने जमींदारों (जमींदारों)
और रैयतों (िकरायेदारों) के वगोर्ं का िनमार्ण िकया। आिदवािसयों को अब जमींदारों को लगान देना पड़ता था और ऐसा न करने पर उन्हें ज़मीन
से बेदखल कर िदया जाता था।
िवकल्प c ग़लत है. िबरसा मुंडा एक धमर् की स्थापना करना चाहते थे और वह चाहते थे िक यह िकसी भी ईसाई िमशनरी के हस्तक्षेप से मुक्त
हो। वह मुंडा भूिम से िमशनिरयों और अन्य अंग्रेजों को बाहर िनकालना चाहते थे। अतः कथन गलत है।
िवकल्प d सही है: िबरसा मुंडा आं दोलन का उनकी समस्याओं के प्रित सरकारी रवैये पर प्रभाव पड़ा।
1) अिधकािरयों को भूिम िरकॉडर् तैयार करने की आवश्यकता महसूस हुई तािक वे आिदवासी िहतों की रक्षा कर सकें।
2) सरकार ने इस उद्देश्य को प्राप्त करने के िलए 1902 और 1910 के बीच आिदवािसयों के िलए सवेर्क्षण और िनपटान अिभयान चलाए।
3) इसने अिनवायर् बेगार प्रणाली को समाप्त करने का िनणर्य िलया और 1903 का िकरायेदारी अिधिनयम पािरत िकया िजसने मुंडारी खुंटकट्टी
प्रणाली को मान्यता दी।
4) 'सरकार' ने 1908 में छोटानागपुर काश्तकारी अिधिनयम भी पािरत िकया।

Source: A BRIEF HISTORY OF MODERN INDIA BY RAJIV AHIR Ch-17

Question Attempted / Student

Question: 39 Mark Flag  Previous Next 

Correct Answer : b
Your Answer :

Consider the following pairs with regard to socio-religious reformers and their contributions during 19th and 20th century:

Reformer Literary work

1. Bipin Chandra Sattar Batsar


Pal

2. Sister Nivedita Cradle Tales of


Hinduism

3. Aurobindo Jnana Yoga


Ghose

https://academy.forumias.com/beta/lms/viewSolutions Page 66 of 180


Academy ForumIAS | Dashboard 18/02/24, 8:41 PM

How many of the above pairs are correctly matched?

19वीं और 20वीं शताब्दी के दौरान सामािजक-धािमर् क सुधारकों और उनके योगदान के संबंध में िनम्निलिखत युग्मों पर िवचार करें:

सुधारक सािहित्यक कायर्

1. िबिपन चंद्र पाल सत्तर बत्सर

2. िसस्टर िनवेिदता िहं दू धमर् की पालना


कहािनयाँ (Cradle
Tales)

3. अरिबं दो घोष ज्ञान योग

उपरोक्त में से िकतने युग्म सही सुमेिलत हैं?

a Only one केवल एक


b Only two केवल दो
c All three तीनों
d None कोई नहीं

Explanation
Option b is the correct answer.

Pair 1 is correct. Bipin Chandra Pal (1858-1932), a key figure in the early Indian independence movement, was part of the
Lal-Bal-Pal triumvirate. He authored "Sattar Batsar," an autobiography exploring India's contemporary history
from a revolutionary perspective. The book also delves into the educational systems of rural and urban India,
showcasing the coexistence of traditional institutions like tols, madrasas, and mastabas with modern centers of higher
education, primarily in 19th-century Calcutta. His impactful essays, articles, and speeches in publications like "The
Tribune" and "Vande Mataram" shaped the nationalist narrative, emphasizing cultural pride and self-determination in
India's quest for independence.

Pair 2 is correct. Sister Nivedita (1867 - 1911), born Margaret Elizabeth Noble, was an Irish social worker and disciple
of Swami Vivekananda. Her notable works include "Kali, the Mother," exploring the symbolism of the Hindu goddess,
and "Cradle Tales of Hinduism," introducing Hindu mythology to children. She also documented her experiences with
Swami Vivekananda in "Notes of Some Wanderings" and provided insightful perspectives on Indian society in works like
"The Web of Indian Life" and "Studies from an Eastern Home." Her contributions left a lasting impact on understanding
India's cultural heritage.

https://academy.forumias.com/beta/lms/viewSolutions Page 67 of 180


Academy ForumIAS | Dashboard 18/02/24, 8:41 PM

Pair 3 is incorrect. Sri Aurobindo or Aurobindo Ghose (1872-1950) was a philosopher, yogi, and nationalist leader. His
literary contributions are vast, covering diverse topics. His magnum opus is "The Life Divine," an extensive exploration of
metaphysical and spiritual ideas. "Savitri," an epic poem, delves into the soul's journey. Aurobindo's political writings,
including "Bande Mataram" during India's struggle for independence, showcased his vision for a spiritually awakened
society. His synthesis of yoga and philosophy is encapsulated in works like "The Synthesis of Yoga" and "Essays on the
Gita." Aurobindo's writings continue to inspire spiritual seekers and thinkers globally.

1) Jnana Yoga ('The Yoga of Knowledge') is a book of Swami Vivekananda's transcribed lectures. The lectures were
mostly given in New York and London.

Source: A BRIEF HISTORY OF MODERN INDIA BY RAJIV AHIR

https://amritmahotsav.nic.in/district-reopsitory-detail.htm?25375#:~:text=Bipin Chandra Pal, a multifaceted,the lens of


revolutionary thought.

https://nbtindia.gov.in/writereaddata/freebooks/pdf/Sister Nivedita.pdf Pg-104

https://www.sriaurobindoashram.org/sriaurobindo/writings.php

https://estudantedavedanta.net/Jnana-Yoga-by-Swami-Vivekananda.pdf

िवकल्प b सही उत्तर है।


युग्म 1 सही है. िबिपन चंद्र पाल (1858-1932), प्रारंिभक भारतीय स्वतंत्रता आं दोलन के एक प्रमुख व्यिक्त, लाल-बाल-पाल ितकड़ी का
िहस्सा थे। उन्होंने क्रांितकारी दृिष्टकोण से भारत के समकालीन इितहास की खोज करने वाली एक आत्मकथा "सत्तर बत्सर" िलखी। यह पुस्तक
ग्रामीण और शहरी भारत की शैिक्षक प्रणािलयों पर भी प्रकाश डालती है, जो मुख्य रूप से 19वीं सदी के कलकत्ता में उच्च िशक्षा के आधुिनक
केंद्रों के साथ टोल, मदरसा और मस्तबा जैसे पारंपिरक संस्थानों के सह-अिस्तत्व को दशार्ती है। "द िट्रब्यून" और "वंदे मातरम" जैसे प्रकाशनों में
उनके प्रभावशाली िनबंधों, लेखों और भाषणों ने राष्ट्रवादी आख्यान को आकार िदया, िजसमें भारत की स्वतंत्रता की तलाश में सांस्कृितक गौरव
और आत्मिनणर्य पर जोर िदया गया।
युग्म 2 सही है. िसस्टर िनवेिदता (1867 - 1911), जन्म मागर्रटे एिलजाबेथ नोबल, एक आयिरश सामािजक कायर्कतार् और स्वामी िववेकानन्द
की िशष्या थीं। उनके उल्लेखनीय कायोर्ं में "काली, द मदर", िहं दू देवी के प्रतीकवाद की खोज, और "क्रैडल टेल्स ऑफ िहं दू इज्म" शािमल हैं,
जो बच्चों को िहं दू पौरािणक कथाओं से पिरिचत कराते हैं। उन्होंने "नोट् स ऑफ सम वांडिरं ग्स" में स्वामी िववेकानन्द के साथ अपने अनुभवों को
भी प्रलेिखत िकया और "द वेब ऑफ इं िडयन लाइफ" और "स्टडीज फ्रॉम एन ईस्टनर् होम" जैसे कायोर्ं में भारतीय समाज पर व्यावहािरक
दृिष्टकोण प्रदान िकया। उनके योगदान ने भारत की सांस्कृितक िवरासत को समझने पर स्थायी प्रभाव छोड़ा।
युग्म 3 गलत है. श्री अरिबं दो या अरिबं दो घोष (1872-1950) एक दाशर्िनक, योगी और राष्ट्रवादी नेता थे। उनका सािहित्यक योगदान िवशाल
है, िजसमें िविवध िवषय शािमल हैं। उनकी महान रचना "द लाइफ िडवाइन" है, जो आध्याित्मक और आध्याित्मक िवचारों की व्यापक खोज है।
"सािवत्री," एक महाकाव्य किवता, आत्मा की यात्रा पर प्रकाश डालती है। भारत के स्वतंत्रता संग्राम के दौरान "बंदे मातरम" सिहत अरिबं दो के
राजनीितक लेखन ने आध्याित्मक रूप से जागृत समाज के िलए उनके दृिष्टकोण को प्रदिशर् त िकया। योग और दशर्न का उनका संश्लेषण "योग
का संश्लेषण" और "गीता पर िनबंध" जैसे कायोर्ं में समािहत है। अरिबं दो की रचनाएँ वैिश्वक स्तर पर आध्याित्मक साधकों और िवचारकों को
प्रेिरत करती रहती हैं।
1) ज्ञान योग ('ज्ञान का योग') स्वामी िववेकानन्द के िलिखत व्याख्यानों की एक पुस्तक है। अिधकांश व्याख्यान न्यूयॉकर् और लंदन में िदये गये।
Source: A BRIEF HISTORY OF MODERN INDIA BY RAJIV AHIR
https://amritmahotsav.nic.in/district-reopsitory-detail.htm?
25375#:~:text=Bipin%20Chandra%20Pal%2C%20a%20multifaceted,the%20lens%20of%20revolutionary%20thought
https://nbtindia.gov.in/writereaddata/freebooks/pdf/Sister%20Nivedita.pdf Pg-104
https://www.sriaurobindoashram.org/sriaurobindo/writings.php

https://academy.forumias.com/beta/lms/viewSolutions Page 68 of 180


Academy ForumIAS | Dashboard 18/02/24, 8:41 PM

https://estudantedavedanta.net/Jnana-Yoga-by-Swami-Vivekananda.pdf

Question Attempted / Student

Question: 40 Mark Flag  Previous Next 

Correct Answer : b
Your Answer :

Consider the following statements regarding the Wetland City Accreditation (WCA) scheme under the Ramsar Convention
on Wetlands:

1. It is voluntary for nations to join this scheme.

2. Cities recognized under the scheme are eligible for significant financial assistance from the Ramsar Small Grants Fund
(SGF).

3. Currently, no cities from India have been included in this scheme.

How many of the statements given above are correct?

वेटलैंड्स पर रामसर कन्वेंशन के तहत वेटलैंड िसटी प्रत्यायन (Wetland City Accreditation; WCA) योजना के संबंध में िनम्निलिखत
कथनों पर िवचार करें:
1. राष्ट्रों के िलए इस योजना में शािमल होना स्वैिच्छक है।
2. योजना के तहत मान्यता प्राप्त शहर रामसर लघु अनुदान िनिध (Small Grants Fund; SGF) से महत्वपूणर् िवत्तीय सहायता के िलए पात्र
हैं।
3. िफलहाल इस योजना में भारत के िकसी भी शहर को शािमल नहीं िकया गया है.

ऊपर िदए गए कथनों में से िकतने सही हैं?

a Only one केवल एक


b Only two केवल दो
c All three तीनों
d None कोई नहीं

Explanation
Option b is the correct answer.

https://academy.forumias.com/beta/lms/viewSolutions Page 69 of 180


Academy ForumIAS | Dashboard 18/02/24, 8:41 PM

Recently, the Ministry for Environment, Forest and Climate Change (MoEF&CC) has nominated Indore (Madhya
Pradesh), Bhopal (Madhya Pradesh), and Udaipur (Rajasthan) for Wetland City Accreditation (WCA) under the Ramsar
Convention on Wetlands.

Statement 1 is correct: The Wetland City Accreditation (WCA) scheme, introduced in 2015 under the Ramsar
Convention, operates on a voluntary basis. It serves as a recognition system for cities that have undertaken notable
efforts in the conservation and protection of urban and peri-urban wetlands. The primary aim of the scheme is to promote
the sustainable utilization of these wetlands, fostering socio-economic benefits for local communities.

Statement 2 is incorrect: The Wetland City Accreditation (WCA) scheme acknowledges the commendable efforts of
cities in conserving and managing their urban wetlands. The scheme does not provide any financial assistance from
the Ramsar Small Grants Fund (SGF). However, such an international recognition t could aid these cities in securing
financial support due to their renowned conservation efforts.

Statement 3 is correct: At present, no cities from India have been included in this scheme. Although the Ministry for
Environment, Forest and Climate Change (MoEF&CC) has nominated Indore, Bhopal and Udaipur it is yet to be
recognised as WCA cities by the Ramsar Convention.

Source: https://ddnews.gov.in/international/indore-bhopal-and-udaipur-nominated-international-wetland-city-accreditation

https://www.ramsar.org/our-work/activities/wetland-city-accreditation

िवकल्प b सही उत्तर है।


हाल ही में, पयार्वरण, वन और जलवायु पिरवतर्न मंत्रालय (MoEF&CC) ने वेटलैंड्स पर रामसर कन्वेंशन के तहत वेटलैंड िसटी एक्रेिडएशन
(WCA) के िलए इं दौर (मध्य प्रदेश), भोपाल (मध्य प्रदेश) और उदयपुर (राजस्थान) को नामांिकत िकया है।
कथन 1 सही है: रामसर कन्वेंशन के तहत 2015 में शुरू की गई वेटलैंड िसटी प्रत्यायन (Wetland City Accreditation; WCA) योजना
स्वैिच्छक आधार पर संचािलत होती है। यह उन शहरों के िलए एक मान्यता प्रणाली के रूप में कायर् करता है िजन्होंने शहरी और पिर-शहरीय
आद्रर्भूिम के संरक्षण और सुरक्षा में उल्लेखनीय प्रयास िकए हैं। योजना का प्राथिमक उद्देश्य इन आद्रर्भूिमयों के सतत उपयोग को बढ़ावा देना,
स्थानीय समुदायों के िलए सामािजक-आिथर् क लाभों को बढ़ावा देना है।
कथन 2 गलत है: वेटलैंड िसटी एिक्रिडटेशन (Wetland City Accreditation) योजना शहरों के शहरी वेटलैंड के संरक्षण और प्रबंधन में
सराहनीय प्रयासों को स्वीकार करती है। यह योजना रामसर लघु अनुदान िनिध (Small Grants Fund; SGF) से कोई िवत्तीय सहायता
प्रदान नहीं करती है। हालाँिक, ऐसी अंतरार्ष्ट्रीय मान्यता इन शहरों को उनके प्रिसद्ध संरक्षण प्रयासों के कारण िवत्तीय सहायता हािसल करने में
सहायता कर सकती है।

कथन 3 सही है: वतर्मान में, भारत के िकसी भी शहर को इस योजना में शािमल नहीं िकया गया है। हालाँिक पयार्वरण, वन और जलवायु
पिरवतर्न मंत्रालय (MoEF&CC) ने इं दौर, भोपाल और उदयपुर को नामांिकत िकया है, लेिकन अभी तक इसे रामसर कन्वेंशन द्वारा WCA
शहरों के रूप में मान्यता नहीं दी गई है।

https://academy.forumias.com/beta/lms/viewSolutions Page 70 of 180


Academy ForumIAS | Dashboard 18/02/24, 8:41 PM

Source: https://ddnews.gov.in/international/indore-bhopal-and-udaipur-nominated-international-wetland-city-
accreditation
https://www.ramsar.org/our-work/activities/wetland-city-accreditation

Question Attempted / Student

Question: 41 Mark Flag  Previous Next 

Correct Answer : a
Your Answer : c

With regard to the peasant movements, consider the following statements regarding the Tebhaga movement occurred in
1946:

1. The movement demanded to implement the Floud Commission recommendations of 1938 in Bengal region.

2. The British government passed Bengal Tenancy Act to pacify the movement.

3. Communist leaders and Krishak Samity leaders supported the jotedars in the Tebhaga movement.

How many of the above statements are correct?

िकसान आं दोलनों के संबंध में, 1946 में हुए तेभागा आं दोलन के संबंध में िनम्निलिखत कथनों पर िवचार करें:
1. आं दोलन ने बंगाल क्षेत्र में 1938 की फ्लाऊड आयोग (Floud Commission) की िसफािरशों को लागू करने की मांग की।
2. िब्रिटश सरकार ने आं दोलन को शांत करने के िलए बंगाल काश्तकारी अिधिनयम (टेनेंसी एक्ट) पािरत िकया।
3. कम्युिनस्ट नेताओं और कृषक सिमित के नेताओं ने तेभागा आं दोलन में जोतदारों का समथर्न िकया।

उपरोक्त में से िकतने कथन सही हैं?

https://academy.forumias.com/beta/lms/viewSolutions Page 71 of 180


Academy ForumIAS | Dashboard 18/02/24, 8:41 PM

a Only one केवल एक


b Only two केवल दो
c All three तीनों
d None कोई नहीं

Explanation
Option a is the correct answer.

The Tebhaga movement (1946) in Bengal demanded a two-thirds share for share-croppers, challenging the existing
one-half share. Led by communist cadres, it primarily involved Rajbanshis and Muslims. Despite repression and political
challenges, the movement dissipated, influencing agrarian discourse in post-independence India.

Statement 1 is correct. The Krishak Praja Party formed the first popular Ministry in Bengal in 1937. The Land Revenue
Commission (Floud Commission) appointed by it recommended in 1940 that "All bargadars should be treated as
tenants, that the share of the crops legally recoverable from them should be one-third, instead of half" (Vol. I, 1940:
69). The government did not show urgency to implement these recommendations. This prompted the All India Kisan
Sabha to radicalize its agrarian programme and in November 1946 the Bengal Kisan Sabha, its provincial branch,
passed a resolution in Calcutta for - Tebhaga (two thirds share of the produced crops) for the sharecroppers and land to
the tiller.

Statement 2 is incorrect. The Bengal Tenancy Act was not passed after the Tebhaga movement, rather in 1885. The
Bargadari Act of 1950 was passed after independence, recognizing bargadars' rights to a larger share of the crops
they produced, but it was never implemented. Later land reforms in 1967, as well as the subsequent "Operation
Barga" under the Left Front government, aimed to secure peasant tenancy rights by redistributing land beyond the
prescribed limit, effectively putting an end to the Tebhaga movement.

Statement 3 is incorrect. During the Tebhaga movement, Communist leaders and Krishak Samity leaders were on
the side of the poor peasants and landless agricultural laborers, not the jotedars (landlords). The movement aimed at
securing a two-thirds share of the harvested crops for the sharecroppers, challenging the existing practice where they had
to relinquish half of their harvest as rent.

Knowledge Base: The Bengal Tenancy Act of 1885 was enacted by the Governor-General of India in Council. It
established the rights and liabilities of zamindars and tenants in response to widespread peasant discontent. The act
amended and consolidated certain laws governing landlords and tenants in the territories administered by the Lieutenant-
Governor of Bengal.

Source: A BRIEF HISTORY OF MODERN INDIA BY RAJIV AHIR Ch-18

िवकल्प a सही उत्तर है।


बंगाल में तेभागा आं दोलन (1946) ने मौजूदा आधे िहस्से को चुनौती देते हुए बटाईदारों के िलए दो-ितहाई िहस्से की मांग की। कम्युिनस्ट
कायर्कतार्ओ ं के नेतृत्व में, इसमें मुख्य रूप से राजबंशी और मुिस्लम शािमल थे। दमन और राजनीितक चुनौितयों के बावजूद, आं दोलन ख़त्म हो
गया, िजससे आज़ादी के बाद के भारत में कृिष संबंधी चचार् प्रभािवत हुई।
कथन 1 सही है. कृषक प्रजा पाटीर् ने 1937 में बंगाल में पहला लोकिप्रय मंत्रालय बनाया।

https://academy.forumias.com/beta/lms/viewSolutions Page 72 of 180


Academy ForumIAS | Dashboard 18/02/24, 8:41 PM

इसके द्वारा िनयुक्त भूिम राजस्व आयोग (फ्लाउड कमीशन) ने 1940 में िसफािरश की िक "सभी बरगादारों को काश्तकारों के रूप में माना जाना
चािहए, उनसे कानूनी रूप से वसूली योग्य फसलों का िहस्सा आधे के बजाय एक ितहाई होना चािहए" (खंड I, 1940) : 69)। सरकार ने इन
िसफ़ािरशों को लागू करने में तत्परता नहीं िदखाई. इसने अिखल भारतीय िकसान सभा को अपने कृिष कायर्क्रम को उग्र रूख अपनाने के िलए
प्रेिरत िकया और नवंबर 1946 में बंगाल िकसान सभा, इसकी प्रांतीय शाखा, ने बटाईदारों के िलए 'तेभागा' (उत्पािदत फसलों का दो ितहाई
िहस्सा) और भूिम जोतने वालों के िलए कलकत्ता में एक प्रस्ताव पािरत िकया।
कथन 2 गलत है. बंगाल काश्तकारी अिधिनयम तेभागा आं दोलन के बाद पािरत नहीं िकया गया था, बिल्क 1885 में पािरत िकया गया था।
1950 का बरगादार अिधिनयम स्वतंत्रता के बाद पािरत िकया गया था, जो बरगादारों को उनके द्वारा उत्पािदत फसलों के एक बड़े िहस्से के
अिधकार को मान्यता देता था, लेिकन इसे कभी लागू नहीं िकया गया था। बाद में 1967 में भूिम सुधार, साथ ही वाम मोचार् सरकार के तहत
"ऑपरेशन बगार्" का उद्देश्य िनधार्िरत सीमा से अिधक भूिम का पुनिवर् तरण करके िकसानों के काश्तकारी अिधकारों को सुरिक्षत करना था,
िजससे तेभागा आं दोलन को प्रभावी ढंग से समाप्त िकया जा सके।
कथन 3 ग़लत है. तेभागा आं दोलन के दौरान, कम्युिनस्ट नेता और कृषक सिमित के नेता जोतदारों (जमींदारों) के नहीं, बिल्क गरीब िकसानों
और भूिमहीन खेितहर मजदू रों के पक्ष में थे। इस आं दोलन का उद्देश्य बटाईदारों के िलए कटी हुई फसलों का दो-ितहाई िहस्सा सुरिक्षत करना
था, साथ ही मौजूदा प्रथा को चुनौती देना था जहां उन्हें अपनी फसल का आधा िहस्सा िकराए के रूप में छोड़ना पड़ता था।
ज्ञानकोष: 1885 का बंगाल काश्तकारी अिधिनयम भारत के गवनर्र-जनरल द्वारा पिरषद में अिधिनयिमत िकया गया था। इसने व्यापक िकसान
असंतोष के जवाब में जमींदारों और िकरायेदारों के अिधकारों और देनदािरयों की स्थापना की। इस अिधिनयम ने बंगाल के उपराज्यपाल द्वारा
प्रशािसत क्षेत्रों में जमींदारों और काश्तकारों के िनयमन करने वाले कुछ कानूनों को संशोिधत और समेिकत िकया।

Source: A BRIEF HISTORY OF MODERN INDIA BY RAJIV AHIR Ch-18

Question Attempted / Student

Question: 42 Mark Flag  Previous Next 

Correct Answer : b
Your Answer : b

With reference to Indian National Social Conference established in 1887, consider the following statements:

1. It was founded by Dadabhai Naoroji and Gopal Krishan Gokhale.

2. It acted as the social reform cell of the Indian National Congress.

3. It condemned child marriages but was silent on inter-caste marriages.

4. As a part of this organization, Bharat Mahila Parishad was established by Ramabai Ranade.

How many of the above statements are correct?

1887 में स्थािपत भारतीय राष्ट्रीय सामािजक सम्मेलन के संदभर् में, िनम्निलिखत कथनों पर िवचार करें:
1. इसकी स्थापना दादाभाई नौरोजी और गोपाल कृष्ण गोखले ने की थी।
2. इसने भारतीय राष्ट्रीय कांग्रेस के सामािजक सुधार प्रकोष्ठ के रूप में कायर् िकया।

https://academy.forumias.com/beta/lms/viewSolutions Page 73 of 180


Academy ForumIAS | Dashboard 18/02/24, 8:41 PM

3. इसने बाल िववाह की िनं दा की लेिकन अंतजार्तीय िववाह पर चुप रहा।


4. इस संगठन के एक भाग के रूप में, भारत मिहला पिरषद की स्थापना रमाबाई रानाडे द्वारा की गई थी।

उपरोक्त में से िकतने कथन सही हैं?

a Only one केवल एक


b Only two केवल दो
c Only three केवल तीन
d All four सभी चार

Explanation
Option b is the correct answer.

The Indian National Social Conference, founded in 1887 by M.G. Ranade and Raghunath Rao, iut aimed to address
social issues and foster reforms in India. It focused on promoting socio-religious reforms and fostering a progressive and
inclusive society.

Statement 1 is incorrect. M.G. Ranade and Raghunath Rao founded the Indian (National) Social Conference. It was
essentially the social reform arm of the Indian National Congress. In December 1887, it held its first meeting in Madras.

Statement 2 is correct. Indian National Social Conference was essentially the Indian National Congress's social
reform cell. Its inaugural meeting was held in Madras in December 1887. The Conference met annually at the same
location as a subsidiary convention of the Indian National Congress and focused on social reform.

Statement 3 is incorrect. The Indian National Social Conference supported intercaste marriages while condemning
kulinism and polygamy. It launched the well-known "Pledge Movement" to encourage people to take an oath to oppose
child marriage.

Statement 4 is correct. The Ladies Social Conference (Bharat Mahila Parishad) was founded in 1904 in Bombay by
Ramabai Ranade. The organization was a part of the Indian National Social Conference, which was founded in 1887
by Ramabai's husband, Justice Mahadev Govind Ranade.

Source: A BRIEF HISTORY OF MODERN INDIA BY RAJIV AHIR, Appendices Pg. 833

िवकल्प b सही उत्तर है।


1887 में एम.जी.रानाडे और रघुनाथ राव द्वारा स्थािपत भारतीय राष्ट्रीय सामािजक सम्मेलन का उद्देश्य भारत में सामािजक मुद्दों को संबोिधत
करना और सुधारों को बढ़ावा देना था। इसने सामािजक-धािमर् क सुधारों को बढ़ावा देने और एक प्रगितशील और समावेशी समाज को बढ़ावा
देने पर ध्यान केंिद्रत िकया।
कथन 1 गलत है. एम.जी. रानाडे और रघुनाथ राव ने इं िडयन (नेशनल) सोशल कॉन्फ्रें स की स्थापना की। यह मूलतः भारतीय राष्ट्रीय कांग्रेस
की सामािजक सुधार शाखा थी। िदसंबर 1887 में इसकी पहली बैठक मद्रास में हुई।
कथन 2 सही है. भारतीय राष्ट्रीय सामािजक सम्मेलन मूलतः भारतीय राष्ट्रीय कांग्रेस का सामािजक सुधार प्रकोष्ठ था। इसकी उद्घाटन बैठक
िदसंबर 1887 में मद्रास में आयोिजत की गई थी। यह सम्मेलन हर साल भारतीय राष्ट्रीय कांग्रेस के सहायक सम्मेलन के रूप में उसी स्थान पर
आयोिजत िकया जाता था और सामािजक सुधार पर केंिद्रत होता था।

https://academy.forumias.com/beta/lms/viewSolutions Page 74 of 180


Academy ForumIAS | Dashboard 18/02/24, 8:41 PM

कथन 3 ग़लत है. भारतीय राष्ट्रीय सामािजक सम्मेलन ने कुलीनवाद और बहुिववाह की िनं दा करते हुए अंतरजातीय िववाह का समथर्न िकया।
इसने लोगों को बाल िववाह का िवरोध करने की शपथ लेने के िलए प्रोत्सािहत करने के िलए प्रिसद्ध "प्रितज्ञा आं दोलन (Pledge
Movement)" शुरू िकया।
कथन 4 सही है. लेडीज़ सोशल कॉन्फ्रें स (भारत मिहला पिरषद) की स्थापना 1904 में रमाबाई रानाडे द्वारा बॉम्बे में की गई थी। यह संगठन
भारतीय राष्ट्रीय सामािजक सम्मेलन का एक िहस्सा था, िजसकी स्थापना 1887 में रमाबाई के पित, न्यायमूितर् महादेव गोिवं द रानाडे द्वारा की
गई थी।

Source: A BRIEF HISTORY OF MODERN INDIA BY RAJIV AHIR, Appendices Pg. 833

Question Attempted / Student

Question: 43 Mark Flag  Previous Next 

Correct Answer : d
Your Answer : c

Consider the following statements with regard to the United Provinces Kisan Sabha (UPKS) founded in 1918:

1. It was set up by Gauri Shankar Mishra and Indra Narayan Dwivedi.

2. Home rule activist played instrumental role in setting up the UPKS.

3. Nehru rendered his support to UPKS by visiting the villages in United provinces.

4. Due to political differences within the UPKS, the split resulted in the formation of the Awadh Kisan Sabha in 1920.

How many of the above statements are correct?

1918 में स्थािपत संयुक्त प्रांत िकसान सभा (United Provinces Kisan Sabha; UPKS) के संबंध में िनम्निलिखत कथनों पर िवचार करें:
1. इसकी स्थापना गौरी शंकर िमश्र और इं द्र नारायण िद्ववेदी ने की थी।
2. होमरूल कायर्कतार्ओ ं ने UPKS की स्थापना में महत्वपूणर् भूिमका िनभाई।
3. नेहरू ने संयुक्त प्रांत के गांवों का दौरा करके UPKS को अपना समथर्न प्रदान िकया।
4. UPKS के भीतर राजनीितक मतभेदों के कारण िवभाजन के पिरणामस्वरूप 1920 में अवध िकसान सभा का गठन हुआ।

उपरोक्त में से िकतने कथन सही हैं?

a Only one केवल एक


b Only two केवल दो
c Only three केवल तीन
d All four सभी चार

https://academy.forumias.com/beta/lms/viewSolutions Page 75 of 180


Academy ForumIAS | Dashboard 18/02/24, 8:41 PM

Explanation
Option d is the correct answer.

After the 1857 revolt, Awadh taluqdars regained their lands, reinforcing the dominance of big landlords in agrarian society.
Peasants faced high rents, evictions, and illegal levies. Post-World War I, deteriorating conditions led to the organization of
kisan sabhas in UP, including the United Provinces Kisan Sabha in 1918. The UP Kisan Sabha was founded in 1918 in
Lucknow by Gauri Shankar Mishra, Indra Narayan Dwivedi, and Madan Mohan Malviya.

Statement 1 is correct. Madan Mohan Malviya rendered his support to Gauri Shankar Mishra and Indra Narayan
Dwivedi who played a leading role in setting up the UP Kisan Sabha. Other leaders involved in UP Kisan Sabha
Movement include Jhinguri Singh, Durgapal Singh, Baba Ramchandra, and Jawaharlal Nehru.

Statement 2 is correct. Home Rule Movement activists played a pivotal role in organizing United Provinces Kisan
Sabha . Their efforts culminated in the establishment of the United Provinces Kisan Sabha in February 1918 by Gauri
Shankar Mishra and Indra Narayan Dwivedi. Support from leaders like Madan Mohan Malaviya contributed to the rapid
expansion, reaching 450 branches by June 1919.

Statement 3 is correct. Jawaharlal Nehru actively supported the United Provinces Kisan Sabha (UPKS), personally
visiting villages in Uttar Pradesh's Pratapgarh region. His engagement was prompted by his connection to Pratapgarh,
which evolved from ties with neighboring Allahabad. Beginning in 1920, Nehru, a staunch socialist, concentrated on
uplifting peasants. His visits during the anti-colonial struggle demonstrated a commitment to understanding and addressing
the challenges that rural communities face.

Statement 4 is correct. The emergence of the Awadh Kisan Sabha in October 1920 was a consequence of political
disagreements within the United Provinces Kisan Sabha (UPKS). The split occurred among nationalists, leading to the
formation of the Awadh Kisan Sabha. This new Sabha aimed to combat the exploitation of farmers by landlords and
talukdars who imposed excessive rents and taxes.

Source: A BRIEF HISTORY OF MODERN INDIA BY RAJIV AHIR Ch-31

िवकल्प d सही उत्तर है।


1857 के िवद्रोह के बाद, अवध के तालुकदारों ने अपनी ज़मीनें वापस हािसल कर लीं, िजससे कृिष समाज में बड़े जमींदारों का प्रभुत्व मजबूत
हो गया। िकसानों को ऊंचे लगान, बेदखली और अवैध उगाही का सामना करना पड़ा। प्रथम िवश्व युद्ध के बाद, िबगड़ते हालात के कारण यूपी
में िकसान सभाओं का आयोजन हुआ, िजसमें 1918 में संयुक्त प्रांत िकसान सभा भी शािमल थी। यूपी िकसान सभा की स्थापना 1918 में
लखनऊ में गौरी शंकर िमश्रा, इं द्र नारायण िद्ववेदी और मदन मोहन मालवीय द्वारा की गई थी। .
कथन 1 सही है. मदन मोहन मालवीय ने गौरी शंकर िमश्रा और इं द्र नारायण िद्ववेदी को अपना समथर्न िदया िजन्होंने यूपी िकसान सभा की
स्थापना में अग्रणी भूिमका िनभाई। यूपी िकसान सभा आं दोलन में शािमल अन्य नेताओं में िझं गुरी िसं ह, दुगार्पाल िसं ह, बाबा रामचन्द्र और
जवाहरलाल नेहरू शािमल हैं।
कथन 2 सही है. होम रूल आं दोलन के कायर्कतार्ओ ं ने संयुक्त प्रांत िकसान सभा के आयोजन में महत्वपूणर् भूिमका िनभाई। उनके प्रयासों की
पिरणित फरवरी 1918 में गौरी शंकर िमश्र और इं द्र नारायण िद्ववेदी द्वारा संयुक्त प्रांत िकसान सभा की स्थापना के रूप में हुई। मदन मोहन
मालवीय जैसे नेताओं के समथर्न ने तेजी से िवस्तार में योगदान िदया, जो जून 1919 तक 450 शाखाओं तक पहुंच गया।
कथन 3 सही है. जवाहरलाल नेहरू ने संयुक्त प्रांत िकसान सभा (United Provinces Kisan Sabha; UPKS) का सिक्रय रूप से समथर्न
िकया और व्यिक्तगत रूप से उत्तर प्रदेश के प्रतापगढ़ क्षेत्र के गांवों का दौरा िकया। उनकी सहभािगता प्रतापगढ़ से उनके जुड़ाव के कारण प्रेिरत
हुई, जो पड़ोसी इलाहबाद के साथ संबंधों से िवकिसत हुआ। 1920 की शुरुआत में, कट्टर समाजवादी नेहरू ने िकसानों के उत्थान पर ध्यान

https://academy.forumias.com/beta/lms/viewSolutions Page 76 of 180


Academy ForumIAS | Dashboard 18/02/24, 8:41 PM

केंिद्रत िकया। उपिनवेशवाद िवरोधी संघषर् के दौरान उनकी यात्राओं ने ग्रामीण समुदायों के सामने आने वाली चुनौितयों को समझने और उनका
समाधान करने की प्रितबद्धता प्रदिशर् त की।
कथन 4 सही है. अक्टू बर 1920 में अवध िकसान सभा का उद्भव संयुक्त प्रांत िकसान सभा (UPKS) के भीतर राजनीितक असहमित का
पिरणाम था। राष्ट्रवािदयों के बीच िवभाजन हुआ, िजसके पिरणामस्वरूप अवध िकसान सभा का गठन हुआ। इस नई सभा का उद्देश्य अत्यिधक
लगान और कर लगाने वाले जमींदारों और तालुकदारों द्वारा िकसानों के शोषण का सामना करना था।

Source: A BRIEF HISTORY OF MODERN INDIA BY RAJIV AHIR Ch-31

Question Attempted / Student

Question: 44 Mark Flag  Previous Next 

Correct Answer : c
Your Answer : d

In the context of modern Indian history, consider the following regions:

1. Hyderabad

2. Madras

3. Mysore

4. Kashmir

How many of the above regions remained largely unaffected by the 1857 revolt?

आधुिनक भारतीय इितहास के संदभर् में, िनम्निलिखत क्षेत्रों पर िवचार करें:


1. हैदराबाद
2. मद्रास
3. मैसूर
4. कश्मीर

उपरोक्त में से िकतने क्षेत्र 1857 के िवद्रोह से काफी हद तक अप्रभािवत रहे?

a Only one केवल एक


b Only two केवल दो
c Only three केवल तीन
d All four सभी चार

https://academy.forumias.com/beta/lms/viewSolutions Page 77 of 180


Academy ForumIAS | Dashboard 18/02/24, 8:41 PM

Explanation
Option c is the correct answer.

During the Indian Rebellion of 1857, also known as the Sepoy Mutiny or the First War of Indian Independence, some areas
remained unaffected by the revolt. The reasons for this varied and included factors such as local loyalties, the presence of
British military garrisons, effective British administration, or strategic reasons.

Option 1, 3 and 4 are correct: The large princely states, Hyderabad, Mysore, Travancore, and Kashmir, as well as the
smaller ones of Rajputana, did not join the rebellion, serving the British, in the Governor-General Lord Canning's words,
as "breakwaters in a storm".

Option 2 is incorrect. Coastal regions such as Madras and Chingleput, as well as interior areas such as Coimbatore,
were disrupted by the Great Revolt of 1857. Chingleput, a coastal town south of Madras in Tamil Nadu, was a focus
of anti-colonial movement in the early days of 1857.

Source: A BRIEF HISTORY OF MODERN INDIA BY RAJIV AHIR Ch-7

https://indianculture.gov.in/node/2820052#:~:text=Chingleput, a coastal town south,similar towns with nationalist fervour.

https://amritmahotsav.nic.in/district-reopsitory-detail.htm?11269

https://archives.peoplesdemocracy.in/2007/0415/04152007_1857.html

िवकल्प c सही उत्तर है।


1857 के भारतीय िवद्रोह के दौरान, िजसे िसपाही िवद्रोह या भारतीय स्वतंत्रता का पहला युद्ध भी कहा जाता है, कुछ क्षेत्र िवद्रोह से अप्रभािवत
रहे। इसके कारण अलग-अलग थे और इसमें स्थानीय वफादारी, िब्रिटश सैन्य चौिकयों की उपिस्थित, प्रभावी िब्रिटश प्रशासन या रणनीितक
कारण जैसे कारक शािमल थे।
िवकल्प 1, 3 और 4 सही हैं: बड़ी िरयासतें, हैदराबाद, मैसूर, त्रावणकोर और कश्मीर, साथ ही राजपूताना की छोटी िरयासतें, गवनर्र-जनरल
लॉडर् कैिनं ग के शब्दों में, "तूफान में ब्रेकवाटर" के रूप में, िब्रिटशों की सेवा करते हुए, िवद्रोह में शािमल नहीं हुईं।
िवकल्प 2 ग़लत है. मद्रास और िचं गलपुट जैसे तटीय क्षेत्र, साथ ही कोयंबटू र जैसे आं तिरक क्षेत्र, 1857 के महान िवद्रोह से बािधत हो गए थे।
तिमलनाडु में मद्रास के दिक्षण में एक तटीय शहर िचं गलपुट, 1857 के शुरुआती िदनों में उपिनवेशवाद िवरोधी आं दोलन का केंद्र था।
Source: A BRIEF HISTORY OF MODERN INDIA BY RAJIV AHIR Ch-7
https://indianculture.gov.in/node/2820052#:~:text=Chingleput%2C%20a%20coastal%20town%20south,similar%20towns%
https://amritmahotsav.nic.in/district-reopsitory-detail.htm?11269

https://archives.peoplesdemocracy.in/2007/0415/04152007_1857.html

Question Attempted / Student

Question: 45 Mark Flag  Previous Next 

Correct Answer : b
https://academy.forumias.com/beta/lms/viewSolutions Page 78 of 180
Academy ForumIAS | Dashboard 18/02/24, 8:41 PM

Your Answer : b

With reference to the United Nation's Genocide Convention, consider the following statements:

1. It was the first human rights treaty adopted by the United Nations General Assembly.

2. It obligates member nations to enact appropriate legislation to give effect to its provisions.

3. It grants International Criminal Court (ICC) the power to prosecute individuals for genocide in non-member states.

4. India is not a party to this convention.

How many of the statements given above are correct?

संयुक्त राष्ट्र के नरसंहार सम्मेलन (Genocide Convention) के संदभर् में, िनम्निलिखत कथनों पर िवचार करें:
1. यह संयुक्त राष्ट्र महासभा द्वारा अपनाई गई पहली मानवािधकार संिध थी।
2. यह सदस्य राष्ट्रों को इसके प्रावधानों को प्रभावी बनाने के िलए उिचत कानून बनाने के िलए बाध्य करता है।
3. यह अंतरार्ष्ट्रीय आपरािधक न्यायालय (ICC) को गैर-सदस्य देशों में नरसंहार के िलए व्यिक्तयों पर मुकदमा चलाने की शिक्त प्रदान करता
है।
4. भारत इस सम्मेलन का सदस्य नहीं है।

ऊपर िदए गए कथनों में से िकतने सही हैं?

a Only one केवल एक


b Only two केवल दो
c Only three केवल तीन
d All four सभी चार

Explanation
Option b is the correct answer.

South Africa recently appealed to the International Court of Justice (ICJ) for an urgent ruling, asserting that Israel breached
its obligations under the 1948 Genocide Convention. South Africa alleged Israel of committing genocide during its current
military operations in Gaza against Palestinians.

Statement 1 is correct: The Convention on the Prevention and Punishment of the Crime of Genocide (Genocide
Convention) was the first human rights treaty adopted by the General Assembly of the United Nations on 9
December 1948. It signified the international community's commitment to 'never again' after the atrocities committed
during the Second World War.

Statement 2 is correct: This convention imposes three key obligations on its member nations.

1) Firstly, it mandates the prevention and punishment of genocide, regardless of whether it occurs during times of peace or
conflict.

https://academy.forumias.com/beta/lms/viewSolutions Page 79 of 180


Academy ForumIAS | Dashboard 18/02/24, 8:41 PM

2) Secondly, member states are required to enact domestic legislation that aligns with the provisions outlined in the
Genocide Convention.

3) Thirdly, signatory nations must ensure that individuals accused of genocide are brought to trial before competent
tribunals.

Statement 3 is incorrect: The Convention enables the International Court of Justice to investigate upon the crime of
genocide upon the request of any involved party. The ICC does not have the unilateral power to prosecute individuals
for the crime of aggression in non-member states. The United Nations Security Council (UNSC) holds significant
control over the ICC's jurisdiction, being the only entity capable of overriding the need for State consent. This authority
allows the UNSC to extend ICC jurisdiction to any state, unless vetoed by one of its five permanent members.

Statement 4 is incorrect: India ratified this convention and has been its party since 1959. Despite India's ratification
of the convention, there has been a notable absence of domestic legislation that penalizes genocide. For instance, India's
Penal Code does not explicitly criminalize acts of genocide or the intentional destruction of particular national, ethnic,
racial, or religious groups.

Source: https://indianexpress.com/article/explained/explained-global/icj-genocide-case-israel-south-africa-9105662/

https://thewire.in/law/genocide-ontario-1984

https://www.ohchr.org/en/instruments-mechanisms/instruments/convention-prevention-and-punishment-crime-genocide

https://guide-humanitarian-law.org/content/article/3/international-criminal-court-icc/#:~:text=has a high-,degree,-of
authority over

िवकल्प b सही उत्तर है।


दिक्षण अफ्रीका ने हाल ही में तत्काल फैसले के िलए अंतरार्ष्ट्रीय न्यायालय (ICJ) में अपील की, िजसमें कहा गया िक इज़राइल ने 1948 के
नरसंहार सम्मेलन के तहत अपने दाियत्वों का उल्लंघन िकया है। दिक्षण अफ़्रीका ने इज़राइल पर िफ़िलस्तीिनयों के िख़लाफ़ गाज़ा में अपने
मौजूदा सैन्य अिभयानों के दौरान नरसंहार करने का आरोप लगाया।
कथन 1 सही है: नरसंहार के अपराध की रोकथाम और सजा पर कन्वेंशन (जीनोसाइड कन्वेंशन) 9 िदसंबर 1948 को संयुक्त राष्ट्र की
महासभा द्वारा अपनाई गई पहली मानवािधकार संिध थी। इसने िद्वतीय िवश्व युद्ध के दौरान िकए गए अत्याचारों के बाद 'िफर कभी नहीं
(Never Again)' की अंतरार्ष्ट्रीय समुदाय की प्रितबद्धता को दशार्या।
कथन 2 सही है: यह सम्मेलन अपने सदस्य देशों पर तीन प्रमुख दाियत्व आरोिपत करता है।
1) सबसे पहले, यह नरसंहार की रोकथाम और सज़ा का आदेश देता है, भले ही यह शांित या संघषर् के समय में हो।
2) दू सरे, सदस्य राज्यों को घरेलू कानून बनाने की आवश्यकता है जो नरसंहार सम्मेलन में उिल्लिखत प्रावधानों के अनुरूप हो।
3) तीसरा, हस्ताक्षरकतार् राष्ट्रों को यह सुिनिश्चत करना चािहए िक नरसंहार के आरोपी व्यिक्तयों को सक्षम न्यायािधकरण के समक्ष मुकदमे में
लाया जाए।
कथन 3 गलत है: कन्वेंशन अंतरार्ष्ट्रीय न्यायालय को िकसी भी शािमल पक्ष के अनुरोध पर नरसंहार के अपराध की जांच करने में सक्षम बनाता
है। आईसीसी के पास गैर-सदस्य देशों में आक्रामकता के अपराध के िलए व्यिक्तयों पर मुकदमा चलाने की एकतरफा शिक्त नहीं है। संयुक्त
राष्ट्र सुरक्षा पिरषद (UNSC) ICC के अिधकार क्षेत्र पर महत्वपूणर् िनयंत्रण रखती है, यह एकमात्र इकाई है जो राज्य की सहमित की
आवश्यकता को खत्म करने में सक्षम है। यह प्रािधकरण UNSC को ICC क्षेत्रािधकार को िकसी भी राज्य तक िवस्तािरत करने की अनुमित
देता है, जब तक िक इसके पांच स्थायी सदस्यों में से एक द्वारा वीटो न िकया गया हो।
कथन 4 ग़लत है: भारत ने इस सम्मेलन का अनुसमथर्न िकया है और 1959 से इसका पक्ष बना हुआ है। भारत के सम्मेलन के अनुसमथर्न के
बावजूद, नरसंहार को दंिडत करने वाले घरेलू कानून की उल्लेखनीय अनुपिस्थित रही है। उदाहरण के िलए, भारत की दंड संिहता स्पष्ट रूप से
नरसंहार या िवशेष राष्ट्रीय, जातीय, नस्लीय या धािमर् क समूहों के जानबूझकर िवनाश के कृत्यों को अपराध नहीं मानती है।

https://academy.forumias.com/beta/lms/viewSolutions Page 80 of 180


Academy ForumIAS | Dashboard 18/02/24, 8:41 PM

Source: https://indianexpress.com/article/explained/explained-global/icj-genocide-case-israel-south-africa-
9105662/
https://thewire.in/law/genocide-ontario-1984
https://www.ohchr.org/en/instruments-mechanisms/instruments/convention-prevention-and-punishment-crime-
genocide

https://guide-humanitarian-law.org/content/article/3/international-criminal-court-
icc/#:~:text=has%20a%20high-,degree,-of%20authority%20over

Question Attempted / Student

Question: 46 Mark Flag  Previous Next 

Correct Answer : a
Your Answer : a

With regard to the Malabar/Moplah Rebellion of 1921, consider the following:

1. It was against the high rent fixed by upper caste Hindu landlords.

2. It was associated with Non-Cooperation movement.

3. It was resolved amicably by British government by passing Tenancy Reform Act.

4. Its communal overtone breached the Hindu-Muslim Unity.

How many of the above statements are incorrect?

1921 के मालाबार/मोपला िवद्रोह के संबंध में िनम्निलिखत पर िवचार करें:


1. यह उच्च जाित के िहं दू जमींदारों द्वारा िनधार्िरत उच्च लगान के िखलाफ था।
2. यह असहयोग आं दोलन से जुड़ा था।
3. िब्रिटश सरकार द्वारा काश्तकारी सुधार अिधिनयम (Tenancy Reform Act) पािरत करके इसे सौहादर्पूणर् ढंग से हल िकया गया।
4. इसके सांप्रदाियक स्वर ने िहं दू -मुिस्लम एकता को भंग कर िदया।

उपरोक्त में से िकतने कथन गलत हैं?

https://academy.forumias.com/beta/lms/viewSolutions Page 81 of 180


Academy ForumIAS | Dashboard 18/02/24, 8:41 PM

Only one केवल


a एक
Only two केवल
b दो
Only three केवल
c तीन
All four सभी
d चार

Explanation
Option a is the correct answer.

The Malabar Rebellion, also known as the Moplah Rebellion or Mappila Rebellion was a series of armed uprisings by
Mappila Muslims, who were mostly tenant farmers, against the British colonial authorities and upper-caste Hindu
landlords in 1921.

Statement 1 is correct. The Moplah tenants were Muslims and they agitated against the Hindu landlords and the British
government. The Moplah peasents worked as agriculturists who were tenants of Hindu landlords called Jenmis. The major
grievances of the Moplah tenants were

1) Insecurity due to unfavourable land tenure system due to which Moplahs could be ejected from their land without any
appropriate notice.

2) High renewal of fees fixed by the Jenmis.

3) High rent fixed for Moplah and discriminating them against Hindu Tenants.

Statement 2 is correct: The Moplah Rebellion and the Khilafat Movement merged in 1920 due to shared discontent
with British rule. Both the Moplahs in Malabar and the Khilafat Movement were dissatisfied with British colonial
policies. The Khilafat Movement aimed to protect the Ottoman Caliphate, which was threatened by British actions.
The Khilafat Movement's pan-Islamic cause resonated with the Moplahs' religious sentiments, facilitating their
collaboration. The Moplahs, facing agrarian issues and British injustices, sought an opportunity to voice their discontent.

Statement 3 is incorrect: The British government did not pass the tenancy reforms, rather it was suppressed forcefully.
The British government issued prohibitory orders against the Khilafat meetings in the beginning of 1921. In August 1921,
police raided the mosque at Tirurangadi to arrest a Khilafat leader and a highly respected priest. The police opened fire on
the unarmed crowd killing many persons. The Moplah toll was quite heavy and about 2400 persons lost their lives.

Statement 4 is correct: The Moplah uprising in Malabar, followed by other communal riots in various parts of the
subcontinent in 1922-23, there was a visible breach in the Hindu�Muslim alliance.

Source: A BRIEF HISTORY OF MODERN INDIA BY RAJIV AHIR, Ch-31

https://clpr.org.in/blog/moplah-rebellion-not-just-a-religious-uprising/

िवकल्प a सही उत्तर है।

https://academy.forumias.com/beta/lms/viewSolutions Page 82 of 180


Academy ForumIAS | Dashboard 18/02/24, 8:41 PM

मालाबार िवद्रोह, िजसे मोपला िवद्रोह या मिप्पला िवद्रोह के रूप में भी जाना जाता है, 1921 में िब्रिटश औपिनवेिशक अिधकािरयों और उच्च
जाित के िहं दू जमींदारों के िखलाफ मिप्पला मुसलमानों, जो ज्यादातर कृषक थे, द्वारा सशस्त्र िवद्रोह की एक श्रृंखला थी।
कथन 1 सही है. मोपला के काश्तकार मुिस्लम थे और उन्होंने िहं दू जमींदारों और िब्रिटश सरकार के िखलाफ आं दोलन िकया। मोपला के
िकसान कृिष श्रिमक के रूप में काम करते थे जो जेनमी नामक िहं दू जमींदारों के बटाईदार थे। मोपला काश्तकारों की प्रमुख िशकायतें थीं
1) प्रितकूल भूिम स्वािमत्व प्रणाली के कारण असुरक्षा िजसके कारण मोपलाओं को िबना िकसी उिचत सूचना के उनकी भूिम से बेदखल िकया
जा सकता था।
2) जेनमी द्वारा िनधार्िरत शुल्क का उच्च नवीनीकरण।
3) मोपलाओं के िलए उच्च लगान िनधार्िरत करना और िहं दू िकरायेदारों के साथ भेदभाव करना।
कथन 2 सही है: िब्रिटश शासन के साथ साझा असंतोष के कारण 1920 में मोपला िवद्रोह और िखलाफत आं दोलन का िवलय हो गया।
मालाबार में मोपला और िखलाफत आं दोलन दोनों िब्रिटश औपिनवेिशक नीितयों से असंतुष्ट थे। िखलाफत आं दोलन का उद्देश्य ओटोमन
खलीफा की रक्षा करना था, िजसे िब्रिटश कायोर्ं से खतरा था। िखलाफत आं दोलन का अिखल-इस्लािमक मुद्दा मोपलाओं की धािमर् क भावनाओं
के अनुरूप था, िजससे उनके सहयोग में आसानी हुई। कृिष संबंधी मुद्दों और िब्रिटश अन्यायों का सामना कर रहे मोपलाओं ने अपने असंतोष को
व्यक्त करने का अवसर मांगा।
कथन 3 गलत है: िब्रिटश सरकार ने काश्तकारी सुधारों को पािरत नहीं िकया, बिल्क इसे बलपूवर्क दबा िदया गया। िब्रिटश सरकार ने 1921
की शुरुआत में िखलाफत बैठकों के िखलाफ िनषेधाज्ञा आदेश जारी िकए। अगस्त 1921 में, पुिलस ने एक िखलाफत नेता और एक अत्यिधक
सम्मािनत पुजारी को िगरफ्तार करने के िलए ितरुरंगडी की मिस्जद पर छापा मारा। पुिलस ने िनहत्थी भीड़ पर गोिलयां चलायीं िजसमें कई लोग
मारे गये। मोपला में भारी तबाही हुई और लगभग 2400 लोगों की जान चली गई।
कथन 4 सही है: मालाबार में मोपला िवद्रोह, िजसके बाद 1922-23 में उपमहाद्वीप के िविभन्न िहस्सों में अन्य सांप्रदाियक दंगे हुए, िहं दू -मुिस्लम
गठबंधन में एक स्पष्ट दरार थी।
Source: A BRIEF HISTORY OF MODERN INDIA BY RAJIV AHIR, Ch-31

https://clpr.org.in/blog/moplah-rebellion-not-just-a-religious-uprising/

Question Attempted / Student

Question: 47 Mark Flag  Previous Next 

Correct Answer : d
Your Answer : d

Consider the following pairs with regard to the Dalit welfare organisations founded during the Indian National Movement:

Organization Founder

1. All India Anti- B.R Ambedkar


Untouchability League

2. All India Depressed Mahatma Gandhi


Classes Association

3. All India Scheduled E.V. Ramasamy


Caste Federation

https://academy.forumias.com/beta/lms/viewSolutions Page 83 of 180


Academy ForumIAS | Dashboard 18/02/24, 8:41 PM

How many of the above pairs are correctly matched?

भारतीय राष्ट्रीय आं दोलन के दौरान स्थािपत दिलत कल्याण संगठनों के संबंध में िनम्निलिखत युग्मों पर िवचार करें:

संगठन संस्थापक

1. अिखल भारतीय अस्पृश्यता बी आर अम्बेडकर


िवरोधी लीग

2. अिखल भारतीय दिलत वगर् महात्मा गांधी


संघ

3. अिखल भारतीय अनुसूिचत ई.वी. रामास्वामी


जाित महासंघ

उपरोक्त में से िकतने युग्म सही सुमेिलत हैं?

a Only one केवल एक


b Only two केवल दो
c All three तीनों
d None कोई नहीं

Explanation
Option d is the correct answer.

Pair 1 is incorrect. The All India Anti-Untouchability League was established in 1932 by Mahatma Gandhi. It was
later renamed the All India Harijan Sevak Sangh, which translates to "Servants of Harijan Society". It aimed

1) To eradicate untouchability in India by working for Harijan (Dalits) and the upliftment of India's Depressed Class.

2) To work with and help women and children.

3) To promote a nonviolent society created on love, self-sacrifice, brotherhood, cooperation, and service to humanity.

Pair 2 is incorrect. The All India Depressed Classes Association was established in 1930 by Dr. B.R. Ambedkar. The
association's goal was to ensure that the Scheduled Castes had fair representation in India's decision-making bodies.

The association's objectives included:

1) Safeguards for marginalized untouchables.

2) Direct representation of the depressed community in councils using proportional representation.

3) Reserved seats in educational institutions.

4) A separate electorate for electing Dalit representatives to legislative councils.

https://academy.forumias.com/beta/lms/viewSolutions Page 84 of 180


Academy ForumIAS | Dashboard 18/02/24, 8:41 PM

Pair 3 is incorrect. Dr B.R Ambedkar actively supported the rights of the Dalits. He established the All India
Scheduled Caste Federation in 1942. It aimed to put forth the issues of Dalits in an effective manner.

Knowledge Base:

1) E.V. Ramasamy, popularly known as Periyar founded the Self-Respect Movement in 1925. This movement aimed
at promoting self-respect and dignity among all sections of society, particularly marginalized groups such as Dalits. It
rejected caste-based discrimination and advocated for social equality.

Source: A BRIEF HISTORY OF MODERN INDIA BY RAJIV AHIR, Ch-19

िवकल्प d सही उत्तर है।


युग्म 1 गलत है. अिखल भारतीय अस्पृश्यता िवरोधी लीग ( All India Anti-Untouchability League) की स्थापना 1932 में महात्मा
गांधी द्वारा की गई थी। बाद में इसका नाम बदलकर अिखल भारतीय हिरजन सेवक संघ कर िदया गया। इसका लक्ष्य था-
1) हिरजन (दिलतों) के िलए काम करके और भारत के दिलत वगर् के उत्थान के द्वारा भारत में अस्पृश्यता को िमटाना।
2) मिहलाओं और बच्चों के साथ काम करना और उनकी मदद करना।
3) प्रेम, आत्म-बिलदान, भाईचारा, सहयोग और मानवता की सेवा पर आधािरत अिहं सक समाज को बढ़ावा देना।
युग्म 2 ग़लत है. अिखल भारतीय दिलत वगर् संघ (The All India Depressed Classes Association) की स्थापना 1930 में डॉ.
बी.आर.अम्बेडकर द्वारा की गई थी। एसोिसएशन का लक्ष्य यह सुिनिश्चत करना था िक भारत के िनणर्य लेने वाले िनकायों में अनुसूिचत
जाितयों को उिचत प्रितिनिधत्व िमले।
एसोिसएशन के उद्देश्यों में शािमल थे :
1) हािशये पर पड़े अछूतों के िलए सुरक्षा उपाय।
2) आनुपाितक प्रितिनिधत्व का उपयोग करके पिरषदों में दिलत समुदाय का प्रत्यक्ष प्रितिनिधत्व।
3) शैक्षिणक संस्थानों में आरिक्षत सीटें।
4) िवधान पिरषदों में दिलत प्रितिनिधयों को चुनने के िलए एक अलग िनवार्चन क्षेत्र।
युग्म 3 गलत है। डॉ. बी.आर. अम्बेडकर ने सिक्रय रूप से दिलतों के अिधकारों का समथर्न िकया। उन्होंने 1942 में ऑल इं िडया शेड्यूल्ड कास्ट
फेडरेशन की स्थापना की। इसका उद्देश्य दिलतों के मुद्दों को प्रभावी ढंग से सामने रखना था।
ज्ञानकोष:
1)ई.वी. रामास्वामी पेिरयार ने 1925 में आत्म-सम्मान आं दोलन की स्थापना की। इस आं दोलन का उद्देश्य समाज के सभी वगोर्ं, िवशेष रूप से
दिलतों जैसे हािशए पर रहने वाले समूहों के बीच आत्म-सम्मान और सम्मान को बढ़ावा देना था। इसने जाित-आधािरत भेदभाव को खािरज कर
िदया और सामािजक समानता की वकालत की।

Source: A BRIEF HISTORY OF MODERN INDIA BY RAJIV AHIR, Ch-19

Question Attempted / Student

Question: 48 Mark Flag  Previous Next 

Correct Answer : d

https://academy.forumias.com/beta/lms/viewSolutions Page 85 of 180


Academy ForumIAS | Dashboard 18/02/24, 8:41 PM

Your Answer :

With reference to the Modern Indian History, consider the following statements regarding the Indian Jury Act of 1826:

1. It allowed the Britishers to be tried in the Indian judicial court by any British or Indian Jury.

2. It was actively supported by Raja Ram Mohan Roy.

Which of the statements given above is/are correct?

आधुिनक भारतीय इितहास के संदभर् में, 1826 के भारतीय जूरी अिधिनयम (Indian Jury Act) के संबंध में िनम्निलिखत कथनों पर िवचार
करें:
1. इसने िब्रिटशों पर िकसी भी िब्रिटश या भारतीय जूरी द्वारा भारतीय न्याियक अदालत में मुकदमा चलाने की अनुमित दी।
2. इसका सिक्रय समथर्न राजा राम मोहन राय ने िकया था।

ऊपर िदए गए कथनों में से कौन सा/से सही है/हैं?

a 1 only केवल 1
b 2 only केवल 2
c Both 1 and 2 1 और 2 दोनों
d Neither 1 nor 2 न तो 1 और न ही 2

Explanation
Option d is the correct answer.

The Indian Jury Act of 1826 reflected the British government's religious superiority. It established religious distinctions in
the country's judicial system. The act went into effect in 1827. The act stated that every Hindu and Muslim would be tried
by Christian jurists. Non-Christian jurists, on the other hand, were not permitted to try Christians. The act also prohibited
the use of Indian jurors to try European offenders.

Option 1 is incorrect. The Indian Jury Act was passed in 1826 and went into effect in early 1827. The Act was the
manifestation of the British government's conception of religious superiority, which established the concept of religious
distinctions into the country's judicial system. According to the Act, only Christians were permitted to serve as jurors
in Christian cases. Christian jurors, on the other hand, had every right to see the cases involving Hindus and Muslims.

Option 2 is incorrect. Raja Ram Mohan Roy vehemently opposed the Indian Jury Act was passed in 1826, highlighting
the denial of honor to native Indians. His petition emphasized the lack of consultation with or understanding of the
sentiments of Indian subjects by the imperial Government.

Source: https://amritmahotsav.nic.in/district-reopsitory-detail.htm?10860#:~:text=The Indian Jury Act was,judicial system


of the country.

िवकल्प d सही उत्तर है।

https://academy.forumias.com/beta/lms/viewSolutions Page 86 of 180


Academy ForumIAS | Dashboard 18/02/24, 8:41 PM

1826 का भारतीय जूरी अिधिनयम िब्रिटश सरकार की धािमर् क श्रेष्ठता को दशार्ता था। इसने देश की न्याियक प्रणाली में धािमर् क भेदभाव
स्थािपत िकया। यह अिधिनयम 1827 में लागू हुआ। अिधिनयम में कहा गया िक प्रत्येक िहं दू और मुिस्लम पर ईसाई न्यायिवदों द्वारा मुकदमा
चलाया जाएगा। दू सरी ओर, गैर-ईसाई न्यायिवदों को ईसाइयों पर मुकदमा चलाने की अनुमित नहीं थी। इस अिधिनयम ने यूरोपीय अपरािधयों
पर मुकदमा चलाने के िलए भारतीय जूरी सदस्यों के उपयोग पर भी रोक लगा दी।
िवकल्प 1 ग़लत है. भारतीय जूरी अिधिनयम 1826 में पािरत हुआ और 1827 की शुरुआत में लागू हुआ। यह अिधिनयम िब्रिटश सरकार की
धािमर् क श्रेष्ठता की अवधारणा का प्रकटीकरण था, िजसने देश की न्याियक प्रणाली में धािमर् क भेदभाव की अवधारणा को स्थािपत िकया।
अिधिनयम के अनुसार, केवल ईसाइयों को ही ईसाई मामलों में जूरी सदस्य के रूप में सेवा करने की अनुमित थी। दू सरी ओर, ईसाई जूरी
सदस्यों को िहं दुओ ं और मुसलमानों से जुड़े मामलों को देखने का पूरा अिधकार था।
िवकल्प 2 ग़लत है. राजा राम मोहन राय ने 1826 में पािरत भारतीय जूरी अिधिनयम का जोरदार िवरोध िकया, िजसमें मूल भारतीयों को
सम्मान से वंिचत करने पर प्रकाश डाला गया। उनकी यािचका में शाही सरकार द्वारा भारतीय िवषयों की भावनाओं को समझने या उनके साथ
परामशर् की कमी पर जोर िदया गया।

Source: https://amritmahotsav.nic.in/district-reopsitory-detail.htm?
10860#:~:text=The%20Indian%20Jury%20Act%20was,judicial%20system%20of%20the%20country.

Question Attempted / Student

Question: 49 Mark Flag  Previous Next 

Correct Answer : c
Your Answer : c

With respect to the Vaikom Satyagraha movement held in 1924, consider the following personalities:

1. It was led by visionary leaders like T.K. Madhavan, K.P. Kesava Menon, and K. Kelappan.

2. Prominent leaders like Periyar participated in the movement.

3. Mahatma Gandhiji reached vaikom and held discussion in the support of the movement.

How many of the above are correct with regard to personalities associates with the movement?

1924 में आयोिजत वाइकोम सत्याग्रह आं दोलन के संबंध में िनम्निलिखत व्यिक्तत्वों पर िवचार करें:
1. इसका नेतृत्व टी.के.माधवन, के.पी.केशव मेनन और के.केलप्पन जैसे दू रदशीर् नेताओं ने िकया था।
2. पेिरयार जैसे प्रमुख नेताओं ने आं दोलन में भाग िलया।
3. महात्मा गांधीजी ने वाइकोम पहुंचकर आं दोलन के समथर्न में चचार् की।

आं दोलन से जुड़े व्यिक्तत्वों के संबंध में उपरोक्त में से िकतने सही हैं?

https://academy.forumias.com/beta/lms/viewSolutions Page 87 of 180


Academy ForumIAS | Dashboard 18/02/24, 8:41 PM

Only one केवल


a एक
Only two केवल
b दो
All three
c तीनों
None कोई
d नहीं

Explanation
Option c is the correct answer

The Temple Entry Movement was a social reform movement in India that campaigned for the right of all Hindus,
regardless of caste, to enter temples. The movement began in the early 20th century and gained momentum in the 1920s.

Vaikom, in the northern part of Travancore, became a centre of agitation for temple entry because it was a major
pilgrimage site for Hindus. The town was also home to a number of temples that were off-limits to lower-caste Hindus.
In 1924, the Vaikom Satyagraha was launched in Kerala demanding the throwing open of Hindu temples and roads to the
untouchables.

Statement 1 is correct: Visionary leaders such as T.K. Madhavan, K.P. Kesava Menon, and K. Kelappan spearheaded
the Vaikom Satyagraha.

Statement 2 is correct: The Vaikom Satyagraha attracted national attention and many prominent leaders came to
participate in the protest, including Periyar, who came to Vaikom at the request of barrister George Joseph, who had
already been jailed for leading the movement.

Statement 3 is correct: The Vaikom Satyagraha gained more power when Mahatma Gandhi reached Vaikom on
March 1925 and held discussions with leaders of various caste groups.

Source: A BRIEF HISTORY OF MODERN INDIA BY RAJIV AHIR - "Temple Entry Movement"

िवकल्प c सही उत्तर है।


मंिदर प्रवेश आं दोलन भारत में एक सामािजक सुधार आं दोलन था िजसने जाित की परवाह िकए िबना सभी िहं दुओ ं को मंिदरों में प्रवेश करने के
अिधकार के िलए अिभयान चलाया। यह आं दोलन 20वीं सदी की शुरुआत में शुरू हुआ और 1920 के दशक में गित पकड़ी। त्रावणकोर के
उत्तरी भाग में िस्थत वाइकोम , मंिदर में प्रवेश के िलए आं दोलन का केंद्र बन गया क्योंिक यह िहं दुओ ं का एक प्रमुख तीथर् स्थल था। यह शहर
कई मंिदरों का भी घर था जो िनचली जाित के िहं दुओ ं के िलए विजर् त थे। 1924 में, अछूतों के िलए िहं दू मंिदरों और सड़कों को खोलने की मांग
को लेकर केरल में वाइकोम सत्याग्रह शुरू िकया गया था।
कथन 1 सही है: दू रदशीर् नेता जैसे टी.के. माधवन, के.पी. केशव मेनन और के. केलप्पन ने वायकोम सत्याग्रह का नेतृत्व िकया।
कथन 2 सही है: वाइकोम सत्याग्रह ने राष्ट्रीय ध्यान आकिषर् त िकया और कई प्रमुख नेता िवरोध में भाग लेने आए, िजनमें पेिरयार भी शािमल
थे, जो बैिरस्टर जॉजर् जोसेफ के अनुरोध पर वाइकोम आए थे, जो पहले ही आं दोलन का नेतृत्व करने के िलए जेल जा चुके थे।
कथन 3 सही है: वाइकोम सत्याग्रह को तब और अिधक ताकत िमली जब महात्मा गांधी माचर् 1925 को वाइकोम पहुंचे और िविभन्न जाित
समूहों के नेताओं के साथ चचार् की।

https://academy.forumias.com/beta/lms/viewSolutions Page 88 of 180


Academy ForumIAS | Dashboard 18/02/24, 8:41 PM

Source: A BRIEF HISTORY OF MODERN INDIA BY RAJIV AHIR - "Temple Entry Movement"

Question Attempted / Student

Question: 50 Mark Flag  Previous Next 

Correct Answer : c
Your Answer : c

Which of the following best reflects the meaning of the term "digital sovereignty" often seen in the news?

िनम्निलिखत में से कौन अक्सर समाचारों में देखे जाने वाले शब्द "िडिजटल संप्रभुता (Digital sovereignty)" के अथर् को सबसे अच्छा दशार्ता
है?

a
Utilization of digital technologies by all citizens, including the most disadvantaged. सवार्िधक वंिचत वगर् सिहत सभी
नागिरकों द्वारा िडिजटल प्रौद्योिगिकयों का उपयोग।
b
Freedom to access the internet by citizens with minimal restriction from the government. सरकार की ओर से न्यूनतम
प्रितबंध के साथ नागिरकों को इं टरनेट का उपयोग करने की स्वतंत्रता।
c
Control of a nation's data infrastructure and online platforms by domestic entities. घरेलू संस्थाओं द्वारा देश के डेटा
अवसंरचना और ऑनलाइन प्लेटफामोर्ं का िनयंत्रण।
d
The increasing impact that tech giants have on shaping the political landscape of a specific nation. िकसी िविशष्ट राष्ट्र के
राजनीितक पिरदृश्य को आकार देने में तकनीकी िदग्गजों का बढ़ता प्रभाव।

Explanation
Option c is the correct answer.

The critical need for ethical governance of AI technology takes center stage amidst the complex challenges presented by
digital sovereignty, misinformation, and privacy concerns. Further the concept of territorial sovereignty is gradually
evolving into that of digital sovereignty, emphasizing the need for a holistic approach to ensure responsible and human-
centric AI advancements

Option c is correct: Digital Sovereignty is the nation's capacity to govern its digital future, encompassing control
over the complete AI supply chain, spanning from data to hardware and software. This shift towards digital sovereignty
finds resonance in the European Union's General Data Protection Regulation (GDPR), implemented in 2018. The

https://academy.forumias.com/beta/lms/viewSolutions Page 89 of 180


Academy ForumIAS | Dashboard 18/02/24, 8:41 PM

GDPR stands as a notable instance of a country asserting its digital sovereignty by establishing regulations that prioritize
data protection and privacy.

Source: https://indianexpress.com/article/opinion/columns/ai-is-changing-what-sovereignty-means-9095702/

https://www.brookings.edu/articles/the-geopolitics-of-ai-and-the-rise-of-digital-sovereignty/#:~:text=rise in
digital-,sovereignty,-, which refers to

िवकल्प c सही उत्तर है।


वतर्मान में िडिजटल संप्रभुता, गलत सूचना और िनजता संबंिधत मुद्दों द्वारा प्रस्तुत जिटल चुनौितयों के बीच AI तकनीक के नैितक शासन की
आवश्यकता मुख्य केंद्र में है। इसके अलावा क्षेत्रीय संप्रभुता की अवधारणा धीरे-धीरे िडिजटल संप्रभुता में िवकिसत हो रही है, जो िजम्मेदार
और मानव-केंिद्रत AI प्रगित सुिनिश्चत करने के िलए एक समग्र दृिष्टकोण की आवश्यकता पर बल देती है।
िवकल्प c सही है: िडिजटल संप्रभुता देश के िडिजटल भिवष्य को िनयंित्रत करने की क्षमता है, िजसमें डेटा से लेकर हाडर्वेयर और सॉफ्टवेयर
तक फैली संपूणर् एआई आपूितर् श्रृंखला पर िनयंत्रण शािमल है। िडिजटल संप्रभुता की ओर यह बदलाव 2018 में लागू िकए गए यूरोपीय संघ
के जनरल डेटा प्रोटेक्शन रेगुलेशन (GDPR) में प्रितध्विनत होता है। GDPR डेटा सुरक्षा और गोपनीयता को प्राथिमकता देने वाले िनयमों की
स्थापना करके अपनी िडिजटल संप्रभुता का दावा करने वाले देश का एक उल्लेखनीय उदाहरण है।
Source: https://indianexpress.com/article/opinion/columns/ai-is-changing-what-sovereignty-means-9095702/

https://www.brookings.edu/articles/the-geopolitics-of-ai-and-the-rise-of-digital-
sovereignty/#:~:text=rise%20in%20digital-,sovereignty,-%2C%20which%20refers%20to

Question Attempted / Student

Question: 51 Mark Flag  Previous Next 

Correct Answer : c
Your Answer :

Which one of the following statements correctly defines the term' Hali system'?

िनम्निलिखत में से कौन सा कथन 'हाली प्रणाली' शब्द को सही ढंग से पिरभािषत करता है?

https://academy.forumias.com/beta/lms/viewSolutions Page 90 of 180


Academy ForumIAS | Dashboard 18/02/24, 8:41 PM

It was a component of the jajmani system. यह जजमानी व्यवस्था का एक घटक


a था।
b
It was a land revenue agreement between the landlord and peasants. यह जमींदार और िकसानों के बीच एक भू-राजस्व
समझौता था।
It was a form of bonded labour prevalent in India. यह भारत में प्रचिलत बंधुआ मजदू री का एक रूप
c था।
d
It was an exploitative taxation method imposed on peasants. यह िकसानों पर थोपी गई एक शोषणकारी कराधान पद्धित
थी।

Explanation
Option c is the correct answer

Option c is correct: The Hali system was a form of bonded labour prevalent in India. Halis were like permanent
estate servants of their masters known as dhaniamas. They would become bonded labourer in perpetuity for a trifling
sum of money. They were like unpaid labourers who did all type of manual begar for the local rural magnates. M.B. Desai
estimated that in Surat district about one fifth of tribal labourers were halis. The upper caste women of landowning castes
like Anavils, Rajputs and Patidars could not work in the fields due to social taboo associated with manual labour. Such
groups, therefore, employed bonded labour on a large scale. Hali system of keeping bonded labour in south Gujarat was
permeated by notion of patronage and was based on use of labour-services of subordinate families in perpetuity by the
dominant landlords of that locality.

Source: https://egyankosh.ac.in/bitstream/123456789/22335/5/Unit-30.pdf

िवकल्प c सही उत्तर है।


िवकल्प c सही है: हाली प्रथा भारत में प्रचिलत बंधुआ मजदू री का एक रूप थी। हाली अपने स्वािमयों के स्थायी संपित्त सेवकों की तरह थे
िजन्हें धिनयामा के नाम से जाना जाता था। वे मामूली रकम के िलए हमेशा के िलए बंधुआ मजदू र बन जाते थे। वे अवैतिनक मजदू रों की तरह थे
जो स्थानीय ग्रामीण सरदारों के िलए सभी प्रकार की शारीिरक बेगार करते थे। एम.बी. देसाई ने अनुमान लगाया िक सूरत िजले में आिदवासी
मजदू रों का लगभग पांचवां िहस्सा हाली था। अनािवल, राजपूत और पाटीदार जैसी जमींदार जाितयों की मिहलाएं शारीिरक श्रम से जुड़ी
सामािजक वजर्नाओं के कारण खेतों में काम नहीं कर सकती थीं। इसिलए, ऐसे समूहों ने बड़े पैमाने पर बंधुआ मज़दू री करायी। दिक्षण गुजरात
में बंधुआ मज़दू री रखने की हाली प्रणाली संरक्षण की धारणा से व्याप्त थी और उस इलाके के प्रमुख जमींदारों द्वारा अधीनस्थ पिरवारों की श्रम-
सेवाओं के िनरंतर उपयोग पर आधािरत थी।
Source: https://egyankosh.ac.in/bitstream/123456789/22335/5/Unit-30.pdf

Question Attempted / Student

Question: 52 Mark Flag  Previous Next 

https://academy.forumias.com/beta/lms/viewSolutions Page 91 of 180


Academy ForumIAS | Dashboard 18/02/24, 8:41 PM

Correct Answer : c
Your Answer : c

Consider the following statements about the Gadhar Party, founded in 1913:

1. Its aim was to bring a simultaneous revolt in all British colonies by procuring arms and working among Indian troops
stationed abroad.

2. It assigned Rashbehari Bose and Sachin Sanyal to lead the revolutionary movement in India.

3. British suppressed the movement by enacting the the Defence of India Act, 1915.

How many of the above statements are correct?

1913 में स्थािपत गदर पाटीर् के बारे में िनम्निलिखत कथनों पर िवचार करें:
1. इसका उद्देश्य हिथयारों की खरीद और िवदेशों में तैनात भारतीय सैिनकों के बीच काम करके सभी िब्रिटश उपिनवेशों में एक साथ िवद्रोह
करना था।
2. इसने रासिबहारी बोस और सिचन सान्याल को भारत में क्रांितकारी आं दोलन का नेतृत्व करने की िजम्मेदारी सौंपी।
3. अंग्रेजों ने भारत रक्षा अिधिनयम, 1915 लागू करके आं दोलन को दबा िदया।

उपरोक्त में से िकतने कथन सही हैं?

Only one केवल


a एक
Only two केवल
b दो
All three
c तीनों
None कोई
d नहीं

Explanation
Option c is the correct answer

The Ghadr Party was a revolutionary organization that was founded in the United States in 1913. Its goal was to overthrow
British rule in India. The party's headquarters were located in San Francisco, and it had branches in other cities
across the United States. The moving spirits behind the Ghadr Party were Lala Hardayal, Ramchandra, Bhagwan Singh,
Kartar Singh Saraba,

Barkatullah, and Bhai Parmanand.

Statement 1 is correct: The Ghadr programme was to

1) organise assassinations of officials,

https://academy.forumias.com/beta/lms/viewSolutions Page 92 of 180


Academy ForumIAS | Dashboard 18/02/24, 8:41 PM

2) publish revolutionary and anti-imperialist literature,

3) work among Indian troops stationed abroad,

4) procure arms and bring about a simultaneous revolt in all British colonies.

Statement 2 is correct: The Ghadr party were encouraged by 1914 events the Komagata Maru incident and the outbreak
of the First World War. The Ghadr leaders decided to launch a violent attack to oust British rule in India. Rashbehari Bose
and Sachin Sanyal were asked to lead the Movement.

Statement 3 is correct: The British met the wartime threat with the Defence of India Act passed in March 1915 primarily
to smash the Ghadr movement. There were large-scale detentions without trial, special courts giving extremely severe
sentences, numerous court-martials of armymen.

Source: A BRIEF HISTORY OF MODERN INDIA BY RAJIV AHIR "First Phase of Revolutionary Activities (1907-
1917)"

िवकल्प c सही उत्तर है।


ग़दर पाटीर् एक क्रांितकारी संगठन थी िजसकी स्थापना 1913 में संयुक्त राज्य अमेिरका में हुई थी। इसका लक्ष्य भारत में िब्रिटश शासन को
उखाड़ फेंकना था। पाटीर् का मुख्यालय सैन फ्रांिसस्को में िस्थत था, और इसकी शाखाएँ संयुक्त राज्य भर के अन्य शहरों में थीं। ग़दर पाटीर् के
पीछे प्रेरक शिक्तयों में लाला हरदयाल, रामचन्द्र, भगवान िसं ह, करतार िसं ह सराबा,
बरकतुल्लाह, और भाई परमानंद जैसे व्यिक्तत्व थे।
कथन 1 सही है: ग़दर कायर्क्रम था
1) अिधकािरयों की हत्याएँ आयोिजत करना,
2) क्रांितकारी और साम्राज्यवाद िवरोधी सािहत्य प्रकािशत करें,
3) िवदेशों में तैनात भारतीय सैिनकों के बीच काम करना,
4) हिथयार ख़रीदें और सभी िब्रिटश उपिनवेशों में एक साथ िवद्रोह करें।
कथन 2 सही है: ग़दर पाटीर् को 1914 की घटनाओं, कामागाटा मारू घटना और प्रथम िवश्व युद्ध के फैलने से प्रोत्सािहत िकया गया था। ग़दर
नेताओं ने भारत में िब्रिटश शासन को उखाड़ फेंकने के िलए एक िहं सक हमला शुरू करने का फैसला िकया। रासिबहारी बोस और सिचन
सान्याल को आं दोलन का नेतृत्व करने के िलए कहा गया।
कथन 3 सही है: अंग्रेजों ने मुख्य रूप से ग़दर आं दोलन को कुचलने के िलए माचर् 1915 में पािरत भारत रक्षा अिधिनयम के साथ युद्धकालीन
खतरे का सामना िकया। िबना मुकदमे के बड़े पैमाने पर िहरासत में िलया गया, िवशेष अदालतें बेहद कड़ी सज़ाएं दे रही थीं, सेना के जवानों का
कई बार कोटर्-माशर्ल िकया गया।

Source: A BRIEF HISTORY OF MODERN INDIA BY RAJIV AHIR "First Phase of Revolutionary Activities
(1907-1917)

Question Attempted / Student

Question: 53 Mark Flag  Previous Next 

https://academy.forumias.com/beta/lms/viewSolutions Page 93 of 180


Academy ForumIAS | Dashboard 18/02/24, 8:41 PM

Correct Answer : b
Your Answer : c

With respect to modern Indian history, consider the following statements about the Swadeshi movement,1905:

1. Lokmanya Tilak spread movement to Poona and Bombay through Ganapati and Shivaji festivals.

2. Chidambaram Pillai spread the movement in Tamilnadu by organizing strike of the Tuticorin Coral Mill.

3. Surendranath Banerjea spread the movement in Bengal by setting up Swadesh Bandhab Samiti.

How many of the above statements are correct?

आधुिनक भारतीय इितहास के संबंध में, स्वदेशी आं दोलन, 1905 के बारे में िनम्निलिखत कथनों पर िवचार करें:
1. लोकमान्य ितलक ने गणपित और िशवाजी उत्सवों के माध्यम से पूना और बंबई तक आं दोलन का प्रसार िकया।
2. िचदम्बरम िपल्लई ने तूतीकोिरन कोरल िमल की हड़ताल आयोिजत करके तिमलनाडु में आं दोलन का प्रसार िकया ।
3. सुरद्र
ें नाथ बनजीर् ने स्वदेश बांधव सिमित की स्थापना कर बंगाल में आं दोलन का प्रसार िकया।

उपरोक्त में से िकतने कथन सही हैं?

Only one केवल


a एक
Only two केवल
b दो
All three
c तीनों
None कोई
d नहीं

Explanation
Option b is the correct answer

The Swadeshi movement was a social and economic campaign in India that originated in Bengal in 1905. It was a protest
against the partition of Bengal by the British Raj. The movement called for a boycott of British goods and the use of
Indian-made products instead.

Statement 1 is correct: Lokmanya Tilak propagated the swadeshi message in Poona and Bombay, orchestrating
Ganapati and Shivaji festivals to evoke patriotic sentiments. Emphasizing that swaraj was the goal of swadeshi, boycotts,
and national education, he established cooperative stores and led the Swadeshi Wastu Pracharini Sabha.

Statement 2 is correct: Chidambaram Pillai spread the movement to Madras and organised the strike of the
Tuticorin Coral Mill. He founded the Swadeshi Steam Navigation Company in Tuticorin on the east coast of the Madras
Province.

https://academy.forumias.com/beta/lms/viewSolutions Page 94 of 180


Academy ForumIAS | Dashboard 18/02/24, 8:41 PM

Statement 3 is incorrect: Ashwini Kumar Dutt, a school teacher, established the Swadesh Bandhab Samiti to promote
the Swadeshi Movement and guided the Muslim peasants of Barisal in their demonstrations.

Surendranath Banerjea, who held moderate nationalist views, initiated impactful press campaigns through newspapers
like The Bengalee and conducted large-scale public gatherings. He received support from Krishnakumar Mitra and
Narendra Kumar Sen.

Source: A BRIEF HISTORY OF MODERN INDIA BY RAJIV AHIR "Swadeshi and Boycott Movement"

िवकल्प b सही उत्तर है।


स्वदेशी आं दोलन भारत में एक सामािजक और आिथर् क अिभयान था िजसकी शुरुआत 1905 में बंगाल में हुई थी। यह िब्रिटश राज द्वारा बंगाल
के िवभाजन के िखलाफ एक िवरोध प्रदशर्न था। आं दोलन में िब्रिटश वस्तुओ ं के बिहष्कार और उसके स्थान पर भारतीय िनिमर् त उत्पादों के
उपयोग का आह्वान िकया गया।
कथन 1 सही है: लोकमान्य ितलक ने पूना और बॉम्बे में स्वदेशी संदेश का प्रचार िकया, देशभिक्त की भावनाएँ जगाने के िलए गणपित और
िशवाजी उत्सवों का आयोजन िकया। इस बात पर जोर देते हुए िक स्वराज, स्वदेशी बिहष्कार और राष्ट्रीय िशक्षा का लक्ष्य था, उन्होंने सहकारी
दुकानों की स्थापना की और स्वदेशी वस्तु प्रचािरणी सभा का नेतृत्व िकया।
कथन 2 सही है: िचदम्बरम िपल्लई ने आं दोलन को मद्रास तक फैलाया और तूतीकोिरन कोरल िमल की हड़ताल का आयोजन िकया। उन्होंने
मद्रास प्रांत के पूवीर् तट पर तूतीकोिरन में स्वदेशी स्टीम नेिवगेशन कंपनी की स्थापना की।
कथन 3 गलत है: एक स्कूली िशक्षक अिश्वनी कुमार दत्त ने स्वदेशी आं दोलन को बढ़ावा देने के िलए स्वदेश बांधव सिमित की स्थापना की
और उनके प्रदशर्नों में बािरसाल के मुिस्लम िकसानों का मागर्दशर्न िकया।
उदारवादी राष्ट्रवादी िवचार रखने वाले सुरद्र
ें नाथ बनजीर् ने द बंगाली जैसे समाचार पत्रों के माध्यम से प्रभावशाली प्रेस अिभयान शुरू िकया और
बड़े पैमाने पर सावर्जिनक सभाएँ आयोिजत कीं। उन्हें कृष्णकुमार िमत्र और नरेंद्र कुमार सेन का समथर्न िमला।

Source: A BRIEF HISTORY OF MODERN INDIA BY RAJIV AHIR "Swadeshi and Boycott Movement"

Question Attempted / Student

Question: 54 Mark Flag  Previous Next 

Correct Answer : b
Your Answer : d

Consider the following statements regarding the role of various women in the Indian freedom struggle:

1. Rani Gaidinliu led the Heraka religious movement which aimed to establish a Naga kingdom.

2. Matangini Hazra played an instrumental role during the Salt Satyagraha.

3. Pritilata Waddedar led various underground activities during the Quit India movement.

How many of the above statements are correct?

https://academy.forumias.com/beta/lms/viewSolutions Page 95 of 180


Academy ForumIAS | Dashboard 18/02/24, 8:41 PM

भारतीय स्वतंत्रता संग्राम में िविभन्न मिहलाओं की भूिमका के संबंध में िनम्निलिखत कथनों पर िवचार करें:
1. रानी िगडालू ने हेराका धािमर् क आं दोलन का नेतृत्व िकया िजसका उद्देश्य नागा साम्राज्य की स्थापना करना था।
2. मतांिगनी हजारा ने नमक सत्याग्रह के दौरान महत्वपूणर् भूिमका िनभाई।
3. प्रीितलता वाडेदार ने भारत छोड़ो आं दोलन के दौरान िविभन्न भूिमगत गितिविधयों का नेतृत्व िकया।

उपरोक्त में से िकतने कथन सही हैं?

a Only one केवल एक


b Only two केवल दो
c All three तीनों
d None कोई नहीं

Explanation
Option b is the correct answer.

Women played a pivotal role in the Indian freedom struggle, contributing as leaders, activists, and symbols of resistance.
Their involvement ranged from participating in nonviolent movements to leading protests, contributing significantly to the
fight against British colonial rule.

Statement 1 is correct. Rani Gaidinliu, born on January 26, 1915, in Manipur, was a key figure in the Indian
independence movement against British rule. She led the Heraka religious movement for the Zeliangrong Naga
people, advocating for a Naga kingdom. She was arrested in 1932 and initially sentenced to life imprisonment, public
pressure led to a commuted 14-year sentence, and she was released in 1947. Post-independence, Gaidinliu worked on
social and economic development for her community. Honored with the Padma Bhushan in 1982, she passed away on
February 17, 1993, leaving a lasting legacy as a fearless freedom fighter.

Statement 2 is correct. Matangini Hazra, also known as "Gandhi buri" (old lady Gandhi), was a remarkable Indian
freedom fighter who fearlessly and tirelessly contributed to the struggle for India's independence from British colonial
rule. Matangini Hazra played an instrumental role in the Salt Satyagraha, famously known as the Dandi March, led
by Mahatma Gandhi in 1930.

Statement 3 is incorrect. Pritilata Waddedar did not participate in the Quit India movement as she died in 1932.
Pritilata Waddedar became involved in revolutionary activities during the period of Chittagong Armoury Raid, an attempt
by Indian nationalists to raid the armoury of police and auxiliary forces in Chittagong. Though she didn't directly
participate in the raid, she was associated with its leaders. In 1932, Pritilata led an attack on the Pahartali European Club in
Chittagong as a symbolic protest against racial discrimination. Pritilata Waddedar sacrificed her life at a young age on
September 23, 1932, making her a martyr for the cause of Indian independence.

Source: A BRIEF HISTORY OF MODERN INDIA BY RAJIV AHIR, Ch-19, APPENDICES

िवकल्प b सही उत्तर है।

https://academy.forumias.com/beta/lms/viewSolutions Page 96 of 180


Academy ForumIAS | Dashboard 18/02/24, 8:41 PM

मिहलाओं ने भारतीय स्वतंत्रता संग्राम में नेतृत्त्व, कायर्कतार्ओ ं और प्रितरोध के प्रतीकों के रूप में योगदान देकर महत्वपूणर् भूिमका िनभाई। उनकी
भागीदारी अिहं सक आं दोलनों में भाग लेने से लेकर िवरोध प्रदशर्नों का नेतृत्व करने तक, िब्रिटश औपिनवेिशक शासन के िखलाफ लड़ाई में
महत्वपूणर् योगदान देने तक थी।
कथन 1 सही है. 26 जनवरी, 1915 को मिणपुर में जन्मी रानी िगडालू िब्रिटश शासन के िखलाफ भारतीय स्वतंत्रता आं दोलन में एक प्रमुख नेत्री
थीं। उन्होंने नागा साम्राज्य की वकालत करते हुए ज़ेिलयानग्रोंग नागा लोगों के िलए हेराका धािमर् क आं दोलन का नेतृत्व िकया। उन्हें 1932 में
िगरफ्तार िकया गया था और शुरू में आजीवन कारावास की सजा सुनाई गई थी, जनता के दबाव के कारण 14 साल की सजा कम कर दी गई
और 1947 में उन्हें िरहा कर िदया गया। आजादी के बाद, िगडालू ने अपने समुदाय के िलए सामािजक और आिथर् क िवकास पर काम िकया।
1982 में पद्म भूषण से सम्मािनत, एक िनडर स्वतंत्रता सेनानी के रूप में एक स्थायी िवरासत छोड़कर, 17 फरवरी 1993 को उनका िनधन हो
गया।
कथन 2 सही है. मतांिगनी हजारा, िजन्हें "गांधी बूढ़ी" (बूढ़ी मिहला गांधी) के नाम से भी जाना जाता है, एक उल्लेखनीय भारतीय स्वतंत्रता
सेनानी थीं, िजन्होंने िब्रिटश औपिनवेिशक शासन से भारत की आजादी के संघषर् में िनडर और अथक योगदान िदया। मतांिगनी हजारा ने 1930
में महात्मा गांधी के नेतृत्व में दांडी माचर् के नाम से प्रिसद्ध नमक सत्याग्रह में महत्वपूणर् भूिमका िनभाई थी।
कथन 3 ग़लत है. प्रीितलता वाडेदार ने भारत छोड़ो आं दोलन में भाग नहीं िलया क्योंिक 1932 में उनकी मृत्यु हो गई। प्रीितलता वाडेदार चटगांव
शस्त्रागार छापे की अविध के दौरान क्रांितकारी गितिविधयों में शािमल हो गईं, जो भारतीय राष्ट्रवािदयों द्वारा चटगांव में पुिलस और सहायक
बलों के शस्त्रागार पर छापा मारने का एक प्रयास था। हालाँिक उसने सीधे तौर पर छापे में भाग नहीं िलया था, लेिकन वह इसके नेताओं से जुड़ी
हुई थी। 1932 में, प्रीितलता ने नस्लीय भेदभाव के िखलाफ एक प्रतीकात्मक िवरोध के रूप में चटगांव में पहाड़ी यूरोिपयन क्लब पर हमले का
नेतृत्व िकया। प्रीितलता वाडेदार ने 23 िसतंबर, 1932 को कम उम्र में अपने जीवन का बिलदान िदया, िजससे वह भारतीय स्वतंत्रता के िलए
शहीद हो गईं।
Source: A BRIEF HISTORY OF MODERN INDIA BY RAJIV AHIR, Ch-19, APPENDICES

Question Attempted / Student

Question: 55 Mark Flag  Previous Next 

Correct Answer : a
Your Answer :

Consider the following pairs:

Regions sometimes Country


mentioned in the news

1. Sanaa Iran

2. Kerman Yemen

3. Haifa Israel

How many of the pairs given above are correctly matched?

https://academy.forumias.com/beta/lms/viewSolutions Page 97 of 180


Academy ForumIAS | Dashboard 18/02/24, 8:41 PM

िनम्निलिखत युग्मों पर िवचार करें:

क्षेत्र, िजनका समाचारों में दे श


कभी-कभी उल्लेख िकया
जाता है

1. सना (Sanaa) ईरान

2. करमन (Kerman) यमन

3. हाइफा/हैफा (Haifa) इजराइल

ऊपर िदए गए युग्मों में से िकतने सही सुमेिलत हैं?

a Only one केवल एक


b Only two केवल दो
c All three तीनों
d None कोई नहीं

Explanation
Option a is the correct answer.

In recent times, the world has observed the eruption of various conflicts adversely affecting the life and livelihood of
people. In this context, the cities Sanaa, Kerman, and Haifa, have become focal points of the ongoing turmoil.

Pair 1 is incorrect: The Houthi militia currently controls Sanaa, the capital of Yemen. Since November, they've
launched attacks on international ships in the Red Sea, claiming solidarity with Palestinians. These actions have resulted in
retaliatory strikes from the US and Britain in the past month. These attacks significantly disrupt maritime trade in the Red
Sea, a crucial global shipping lane, forcing companies to reroute cargo around Africa to avoid the Suez Canal.

Pair 2 is incorrect: Recently, the Islamic State (IS) claimed responsibility for bombings in Kerman, an Iranian city.
The twin blasts in the southeastern Iranian city of Kerman at a memorial for Qassem Soleimani, the Quds Force
commander whom the U.S. assassinated in Baghdad in January 2020, expose the security vulnerabilities of the Iranian
regime at a time when conflicts are spreading in West Asia. At least 84 were killed in the worst terror attack in the Islamic
Republic's history.

Pair 3 is correct: The recent reports from Israel indicate Hezbollah militants targeted cities Acre and Haifa with
drones and missiles, all of which were successfully neutralized by Israel's air defense systems, resulting in no reported
casualties. These events unfold amidst ongoing clashes between Israel and Hezbollah along the Lebanon-Israeli border
amidst the Gaza conflict.

Source: https://www.thehindu.com/opinion/editorial/terror-in-iran-on-the-blasts-in-irans-kerman-and-the-
impact/article67706200.ece

https://academy.forumias.com/beta/lms/viewSolutions Page 98 of 180


Academy ForumIAS | Dashboard 18/02/24, 8:41 PM

https://www.reuters.com/world/middle-east/yemen-houthi-leader-says-group-will-further-escalate-if-attacks-gaza-do-not-
stop-2024-02-06/

https://www.hindustantimes.com/videos/world-news/israels-port-cities-under-fire-from-lebanon-hezbollah-
strikes-with-drones-missiles-watch-101703776657203.html

िवकल्प a सही उत्तर है।


हाल के िदनों में, दुिनया ने िविभन्न संघषोर्ं के िवस्फोट को देखा है जो लोगों के जीवन और आजीिवका पर प्रितकूल प्रभाव डाल रहे हैं। इस संदभर्
में, सना, करमन और हाइफ़ा/हैफा शहर, चल रही उथल-पुथल के केंद्र िबं दु बन गए हैं।
युग्म 1 गलत है: हौथी लड़ाकू दल वतर्मान में यमन की राजधानी सना को िनयंित्रत करती है। नवंबर के बाद से, उन्होंने िफिलस्तीिनयों के साथ
एकजुटता का दावा करते हुए, लाल सागर में अंतरराष्ट्रीय जहाजों पर हमले शुरू कर िदए हैं। इन कारर्वाइयों के पिरणामस्वरूप िपछले महीने
अमेिरका और िब्रटेन की ओर से जवाबी हमले हुए हैं। इन हमलों से लाल सागर में समुद्री व्यापार काफी हद तक बािधत हो गया है, जो एक
महत्वपूणर् वैिश्वक नौवहनीय मागर् है, िजससे कंपिनयों को स्वेज नहर से बचने के िलए अफ्रीका के आसपास माल का मागर् बदलने के िलए
मजबूर होना पड़ता है।
युग्म 2 गलत है: हाल ही में, इस्लािमक स्टेट (IS) ने ईरानी शहर करमन में बम िवस्फोटों की िजम्मेदारी ली थी। कुद्स फोसर् (Quds Force)
के कमांडर कािसम सुलेमानी, िजनकी जनवरी 2020 में बगदाद में अमेिरका ने हत्या कर दी थी, के स्मारक पर दिक्षणपूवीर् ईरानी शहर करमान
में दोहरे िवस्फोट, ईरानी शासन की सुरक्षा कमजोिरयों को उजागर करते हैं, ऐसे समय में जब पिश्चम एिशया में संघषर् फैल रहे हैं। इस्लािमक
गणराज्य के इितहास के सबसे भीषण आतंकी हमले में कम से कम 84 लोग मारे गए।
युग्म 3 सही है: इज़राइल की हािलया िरपोटोर्ं से संकेत िमलता है िक िहज़्बुल्लाह आतंकवािदयों ने ड्रोन और िमसाइलों से एकर और हाइफ़ा
शहरों को िनशाना बनाया, िजनमें से सभी को इज़राइल की वायु रक्षा प्रणािलयों द्वारा सफलतापूवर्क िनिष्क्रय कर िदया गया, िजसके
पिरणामस्वरूप कोई हताहत नहीं हुआ। ये घटनाएं गाजा संघषर् के बीच लेबनान-इजरायल सीमा पर इजरायल और िहजबुल्लाह के बीच चल
रही झड़पों के बीच सामने आईं।
Source: https://www.thehindu.com/opinion/editorial/terror-in-iran-on-the-blasts-in-irans-kerman-and-the-
impact/article67706200.ece
https://www.reuters.com/world/middle-east/yemen-houthi-leader-says-group-will-further-escalate-if-attacks-
gaza-do-not-stop-2024-02-06/

https://www.hindustantimes.com/videos/world-news/israels-port-cities-under-fire-from-lebanon-hezbollah-
strikes-with-drones-missiles-watch-101703776657203.html

Question Attempted / Student

Question: 56 Mark Flag  Previous Next 

Correct Answer : a
Your Answer : a

With regard to the Indian Parliamentary Committee formed by Sir William Wedderburn and W. S. Caine in 1893, which of
the following was its primary purpose?

https://academy.forumias.com/beta/lms/viewSolutions Page 99 of 180


Academy ForumIAS | Dashboard 18/02/24, 8:41 PM

1893 में सर िविलयम वेडरबनर् और डब्ल्यू.एस. केन द्वारा गिठत भारतीय संसदीय सिमित के संबंध में, िनम्निलिखत में से इसका प्राथिमक
उद्देश्य क्या था?

a
To advocate in the House of Commons for Indian political reforms. भारतीय राजनीितक सुधारों के िलए हाउस ऑफ कॉमन्स में
वकालत करना।
b
To advocate for Indian participation in the Imperial bureaucracy. शाही नौकरशाही में भारतीयों की भागीदारी की वकालत
करना।
c
To campaign for India's complete independence from British rule. िब्रिटश शासन से भारत की पूणर् स्वतंत्रता के िलए अिभयान
चलाना।
d
To advocate for the entry of prominent Indians to the British Parliament. िब्रिटश संसद में प्रमुख भारतीयों के प्रवेश की
वकालत करना।

Explanation
Option a is the correct answer.

Option a is correct. Sir William Wedderburn was elected as a liberal member of the British Parliament in 1893. In 1895,
Sir William Wedderburn was a member of the Royal Commission on Indian Expenditure. Sir William Wedderburn was the
chairman of the "Indian Parliamentary Committee" from 1893 to 1900. Sir William Wedderburn and W. S. Caine
established the "Indian Parliamentary Committee" in 1893, along with Dadabhai Naoroji and other people sympathetic to
Indian cause. The Indian Parliamentary Committee's aimed to lobby the House of Commons for Indian political
reforms.

Source: A BRIEF HISTORY OF MODERN INDIA BY RAJIV AHIR

िवकल्प a सही उत्तर है।


िवकल्प a सही है. सर िविलयम वेडरबनर् को 1893 में िब्रिटश संसद के एक उदार सदस्य के रूप में चुना गया था। 1895 में, सर िविलयम
वेडरबनर् भारतीय व्यय पर रॉयल कमीशन के सदस्य थे। सर िविलयम वेडरबनर् 1893 से 1900 तक "भारतीय संसदीय सिमित" के अध्यक्ष थे।
सर िविलयम वेडरबनर् और डब्ल्यू.एस. केन ने दादाभाई नौरोजी और भारतीय िहतों के प्रित सहानुभूित रखने वाले अन्य लोगों के साथ िमलकर
1893 में "भारतीय संसदीय सिमित" की स्थापना की। भारतीय संसदीय सिमित का उद्देश्य भारतीय राजनीितक सुधारों के िलए हाउस ऑफ
कॉमन्स की पैरवी करना था।

Source: A BRIEF HISTORY OF MODERN INDIA BY RAJIV AHIR

Question Attempted / Student

https://academy.forumias.com/beta/lms/viewSolutions Page 100 of 180


Academy ForumIAS | Dashboard 18/02/24, 8:41 PM

Question: 57 Mark Flag  Previous Next 

Correct Answer : b
Your Answer : c

With regard to 'The Anarchical and Revolutionary Crimes Act of 1919', consider the following statements:

1. It aimed to suspend the writ of habeas corpus for those detained under its provisions.

2. Leaders such as Mohammed Ali Jinnah and Mazhar Ul Haq decided not to oppose the law.

3. The Jallianwala Bagh massacre occurred amid protests against the act.

How many of the above statements are correct?

'अराजकता और क्रांितकारी अपराध अिधिनयम 1919 (The Anarchical and Revolutionary Crimes Act)' के संबंध में, िनम्निलिखत
कथनों पर िवचार करें:
1. इसका उद्देश्य इसके प्रावधानों के तहत िहरासत में िलए गए लोगों के िलए बंदी प्रत्यक्षीकरण की िरट को िनलंिबत करना था।
2. मोहम्मद अली िजन्ना और मजहर उल हक जैसे नेताओं ने कानून का िवरोध न करने का फैसला िकया।
3. जिलयांवाला बाग हत्याकांड इस अिधिनयम के िवरोध के बीच हुआ।

उपरोक्त में से िकतने कथन सही हैं?

a Only one केवल एक


b Only two केवल दो
c All three तीनों
d None कोई नहीं

Explanation
Option b is the correct answer.

The Anarchical and Revolutionary Crimes Act of 1919, commonly known as the Rowlatt Act, was a legislation
passed by the British colonial government in India. Its primary purpose was to empower authorities to combat
revolutionary activities and suppress political dissent more effectively.

Statement 1 is correct. The Rowlatt Act suspended the writ of habeas corpus for those detained under its
provisions. The Act empowered the government to arrest and detain individuals suspected of being involved in
revolutionary or seditious activities without the need for a trial. The Act curtailed civil liberties by restricting freedom of
speech and assembly. The legislation included provisions that restricted freedom of the press.

https://academy.forumias.com/beta/lms/viewSolutions Page 101 of 180


Academy ForumIAS | Dashboard 18/02/24, 8:41 PM

Statement 2 is incorrect. Leaders such as Mohammed Ali Jinnah, Madan Mohan Malaviya and Mazhar Ul Haq
actively opposed the Rowlatt Act, and it was unanimously opposed by all the elected Indian members of the Imperial
Legislative Council. The leaders expressed strong dissent against the repressive measures of the act.

Statement 3 is correct. The Jallianwala Bagh massacre took place in Amritsar on April 13, 1919 during a public
gathering that was a part of widespread protests against the Rowlatt Act.

Source: A BRIEF HISTORY OF MODERN INDIA BY RAJIV AHIR, Ch-15

िवकल्प b सही उत्तर है।


1919 का अराजकता और क्रांितकारी अपराध अिधिनयम, िजसे आमतौर पर रोलेट एक्ट के रूप में जाना जाता है, भारत में िब्रिटश
औपिनवेिशक सरकार द्वारा पािरत एक कानून था। इसका प्राथिमक उद्देश्य अिधकािरयों को क्रांितकारी गितिविधयों से लड़ने और राजनीितक
असहमित को अिधक प्रभावी ढंग से दबाने के िलए सशक्त बनाना था।
कथन 1 सही है. रॉलेट एक्ट ने इसके प्रावधानों के तहत िहरासत में िलए गए लोगों के िलए बंदी प्रत्यक्षीकरण की िरट को िनलंिबत कर िदया।
इस अिधिनयम ने सरकार को क्रांितकारी या देशद्रोही गितिविधयों में शािमल होने के संदेह वाले व्यिक्तयों को िबना मुकदमे की आवश्यकता के
िगरफ्तार करने और िहरासत में लेने का अिधकार िदया। इस अिधिनयम ने भाषण और सभा की स्वतंत्रता को प्रितबंिधत करके नागिरक स्वतंत्रता
को कम कर िदया। कानून में ऐसे प्रावधान शािमल थे जो प्रेस की स्वतंत्रता को प्रितबंिधत करते थे।
कथन 2 गलत है. मोहम्मद अली िजन्ना, मदन मोहन मालवीय और मजहर उल हक जैसे नेताओं ने रोलेट एक्ट का सिक्रय रूप से िवरोध िकया
और इं पीिरयल लेिजस्लेिटव काउं िसल के सभी िनवार्िचत भारतीय सदस्यों ने सवर्सम्मित से इसका िवरोध िकया। नेताओं ने अिधिनयम के
दमनकारी उपायों के िखलाफ कड़ा असंतोष व्यक्त िकया।
कथन 3 सही है. जिलयांवाला बाग नरसंहार 13 अप्रैल, 1919 को अमृतसर में एक सावर्जिनक सभा के दौरान हुआ था जो रोलेट एक्ट के
िखलाफ व्यापक िवरोध का एक िहस्सा था।

Source: A BRIEF HISTORY OF MODERN INDIA BY RAJIV AHIR, Ch-15

Question Attempted / Student

Question: 58 Mark Flag  Previous Next 

Correct Answer : a
Your Answer : a

With regard to the Presidents of Indian National Congress during the colonial period, which of the following options is
correct?

औपिनवेिशक काल के दौरान भारतीय राष्ट्रीय कांग्रेस के अध्यक्षों के संबंध में, िनम्निलिखत में से कौन सा िवकल्प सही है?

https://academy.forumias.com/beta/lms/viewSolutions Page 102 of 180


Academy ForumIAS | Dashboard 18/02/24, 8:41 PM

a
Dadabhai Naoroji presided over the Indian National Congress session three times. दादाभाई नौरोजी ने तीन बार भारतीय
राष्ट्रीय कांग्रेस अिधवेशन की अध्यक्षता की।
b
Lala Lajpat Rai never became President of the Indian National Congress. लाला लाजपत राय कभी भी भारतीय राष्ट्रीय कांग्रेस
के अध्यक्ष नहीं बने।
c
Annie Besant was the first Indian woman to serve as the president of the Indian National Congress. एनी बेसेंट भारतीय
राष्ट्रीय कांग्रेस की अध्यक्ष के रूप में पदासीन होने वाली पहली भारतीय मिहला थीं।
d
Abul Kalam Azad was president of the Indian National Congress during the partition of India. भारत के िवभाजन के दौरान
अबुल कलाम आज़ाद भारतीय राष्ट्रीय कांग्रेस के अध्यक्ष थे।

Explanation
Option a is the correct answer.

The Indian National Congress (INC) was established on December 28, 1885. A.O. Hume, a retired British civil servant,
played a key role in the formation of the Congress with the objective of providing a platform for civic and political
dialogue among educated Indians.

Option a is correct. Dadabhai Naoroji served as the president of the Indian National Congress (INC) on three
occasions. He presided over the INC sessions in 1886, 1893 and 1906. Dadabhai Naoroji was an influential figure in the
early Indian nationalist movement and an advocate for India's self-rule.

Option b is incorrect. Lala Lajpat Rai was a socio-religious leader who worked for Indian welfare. His political career
advanced when he was elected president of the Indian National Congress in 1920.

1) Bal Gangadhar Tilak never became President of the Indian National Congress. In 1890, Tilak joined the Indian
National Congress. He was opposed to its moderate stance, particularly on the fight for self-government. Fellow Indian
nationalists Bipin Chandra Pal in Bengal and Lala Lajpat Rai in Punjab backed him up.

Option c is incorrect. Annie Besant was the first woman to serve as the president of the Indian National Congress,
but she was not Indian. She presided over the INC session in 1917. Annie Besant was of British nationality, and her
presidency marked a significant milestone as the first woman to hold this position in the history of the Indian National
Congress.

1) The first Indian woman to serve as the president of the Indian National Congress (INC) was Sarojini Naidu. She
presided over the Congress session in 1925.

Option d is incorrect. Abul Kalam Azad was the Indian National Congress president in 1923 and again from 1940
to 1946. Abul Kalam Azad, at the age of 35, became the youngest President of the Indian National Congress in 1923.

1) At the time of India's partition and independence, Acharya J.B Kriplani was the President of the Indian National
Congress. He served as the president of the Indian National Congress (INC) between 1946-47.

Source: A BRIEF HISTORY OF MODERN INDIA BY RAJIV AHIR

https://academy.forumias.com/beta/lms/viewSolutions Page 103 of 180


Academy ForumIAS | Dashboard 18/02/24, 8:41 PM

िवकल्प a सही उत्तर है।


भारतीय राष्ट्रीय कांग्रेस (INC) की स्थापना 28 िदसंबर, 1885 को हुई थी। ए.ओ. ह्यूम, एक सेवािनवृत्त िब्रिटश िसिवल सेवक, ने िशिक्षत
भारतीयों के बीच नागिरक और राजनीितक संवाद के िलए एक मंच प्रदान करने के उद्देश्य से कांग्रेस के गठन में महत्वपूणर् भूिमका िनभाई।
िवकल्प a सही है. दादाभाई नौरोजी ने तीन अवसरों पर भारतीय राष्ट्रीय कांग्रेस (INC) के अध्यक्ष के रूप में कायर् िकया। उन्होंने 1886, 1893
और 1906 में कांग्रेस सत्रों की अध्यक्षता की। दादाभाई नौरोजी प्रारंिभक भारतीय राष्ट्रवादी आं दोलन में एक प्रभावशाली व्यिक्त थे और भारत
के स्व-शासन के समथर्क थे।
िवकल्प b ग़लत है. लाला लाजपत राय एक सामािजक-धािमर् क नेता थे िजन्होंने भारतीय कल्याण के िलए काम िकया। उनका राजनीितक
किरयर तब आगे बढ़ा जब उन्हें 1920 में भारतीय राष्ट्रीय कांग्रेस का अध्यक्ष चुना गया।
1) बाल गंगाधर ितलक कभी भी भारतीय राष्ट्रीय कांग्रेस के अध्यक्ष नहीं बने। 1890 में, ितलक भारतीय राष्ट्रीय कांग्रेस में शािमल हो गये। वह
इसके उदारवादी रुख के िवरोधी थे, िवशेषकर स्वशासन की लड़ाई पर। बंगाल में साथी भारतीय राष्ट्रवािदयों िबिपन चंद्र पाल और पंजाब में
लाला लाजपत राय ने उनका समथर्न िकया।
िवकल्प c गलत है. एनी बेसेंट भारतीय राष्ट्रीय कांग्रेस की अध्यक्ष के रूप में पदासीन होने वाली पहली मिहला थीं, लेिकन वह भारतीय नहीं
थीं। उन्होंने 1917 में कांग्रेस सत्र की अध्यक्षता की। एनी बेसेंट िब्रिटश राष्ट्रीयता की थीं, और उनकी अध्यक्षता भारतीय राष्ट्रीय कांग्रेस के
इितहास में इस पद को संभालने वाली पहली मिहला के रूप में एक महत्वपूणर् मील का पत्थर सािबत हुई।
1) भारतीय राष्ट्रीय कांग्रेस (INC) के अध्यक्ष के रूप में पदासीन होने वाली पहली भारतीय मिहला सरोिजनी नायडू थीं। उन्होंने 1925 में कांग्रेस
अिधवेशन की अध्यक्षता की।
िवकल्प d गलत है. अबुल कलाम आज़ाद 1923 में और िफर 1940 से 1946 तक भारतीय राष्ट्रीय कांग्रेस के अध्यक्ष रहे। 1923 में 35 साल
की उम्र में अबुल कलाम आज़ाद भारतीय राष्ट्रीय कांग्रेस के सबसे कम उम्र के अध्यक्ष बने।
1) भारत के िवभाजन और स्वतंत्रता के समय, आचायर् जे.बी कृपलानी भारतीय राष्ट्रीय कांग्रेस के अध्यक्ष थे। उन्होंने 1946-47 के बीच भारतीय
राष्ट्रीय कांग्रेस (INC) के अध्यक्ष के रूप में कायर् िकया।

Source: A BRIEF HISTORY OF MODERN INDIA BY RAJIV AHIR

Question Attempted / Student

Question: 59 Mark Flag  Previous Next 

Correct Answer : c
Your Answer : a

With regard to the Indian national movement, consider the following statements about revolutionary activities abroad in
the early 20th century:

1. Lala Har Dayal formed the 'Hindi Association of the Pacific Coast' in the USA to assert influence over the Indian
immigrant community.

2. Taraknath Das and G.D. Kumar established the United India House in Seattle to promote Indian independence.

3. Madame Bhikaji Cama established the Paris Indian Society to champion Indian freedom abroad.

How many of the above statements are correct?

https://academy.forumias.com/beta/lms/viewSolutions Page 104 of 180


Academy ForumIAS | Dashboard 18/02/24, 8:41 PM

भारतीय राष्ट्रीय आं दोलन के संबंध में, 20वीं सदी की शुरुआत में िवदेशों में क्रांितकारी गितिविधयों के बारे में िनम्निलिखत कथनों पर िवचार
करें:
1. लाला हर दयाल ने भारतीय आप्रवासी समुदाय पर प्रभाव जमाने के िलए संयुक्त राज्य अमेिरका में 'िहं दी एसोिसएशन ऑफ द पेिसिफक
कोस्ट' का गठन िकया।
2. तारकनाथ दास और जी.डी. कुमार ने भारतीय स्वतंत्रता को बढ़ावा देने के िलए िसएटल में यूनाइटेड इं िडया हाउस की स्थापना की।
3. मैडम भीकाजी कामा ने िवदेशों में भारतीय स्वतंत्रता की वकालत करने के िलए पेिरस इं िडयन सोसाइटी की स्थापना की।
उपरोक्त में से िकतने कथन सही हैं?

a Only one केवल एक


b Only two केवल दो
c All three तीनों
d None कोई नहीं

Explanation
Option c is the correct answer.

In the early 20th century, several revolutionary activities involving Indian nationalists unfolded abroad. These activities
were often centered around key figures, and notable locations included the United States and Europe.

Statement 1 is correct. The Ghadar Party, also known as the Hindi Association of the Pacific Coast (originally
called the Hindustani Workers of the Pacific Coast), was established in 1913 by Indian immigrants in Astoria,
Oregon. The party was formed by Lala Har Dayal, Sant Baba Wasakha Singh Dadehar, Baba Jawala Singh,
Santokh Singh, and Sohan Singh Bhakna, who served as President The aim of the association was to liberate India from
the rule of the British. In May 1913, the organization was renamed the Hindustan Ghadar Party.

Statement 2 is correct. Taraknath Das and G.D. Kumar established the United India House in Seattle in 1910. The
house was a center for Indian nationalist activities that aimed to promote Indian independence from British rule.

Statement 3 is correct. The Paris Indian Society was an Indian nationalist organisation founded in 1905 at Paris
under the patronage of Madam Bhikaji Rustom Cama, Munchershah Burjorji Godrej and S. R. Rana. The Paris
Indian Society served as a refuge for revolutionaries escaping London in 1909. The Society also published nationalist
newspapers such as Bande Mataram and financed the publication of Virendranath Chattopadhyay's newspaper Talvar from
Berlin. Madame Bhikaji Cama made history by hoisting the Indian flag in a foreign land at the International
Socialist Conference in Stuttgart, Germany, in 1907.

Source: A BRIEF HISTORY OF MODERN INDIA BY RAJIV AHIR, Ch-13

https://amritmahotsav.nic.in/unsung-heroes-detail.htm?55#:~:text=Madam Bhikaiji Cama became the,over the Indian sub-


continent.

https://egyankosh.ac.in/bitstream/123456789/89552/3/Unit-9.pdf

https://amritmahotsav.nic.in/district-reopsitory-detail.htm?13680

https://academy.forumias.com/beta/lms/viewSolutions Page 105 of 180


Academy ForumIAS | Dashboard 18/02/24, 8:41 PM

िवकल्प c सही उत्तर है।


20वीं सदी की शुरुआत में, भारतीय राष्ट्रवािदयों से जुड़ी कई क्रांितकारी गितिविधयाँ िवदेशों में सामने आईं। ये गितिविधयाँ अक्सर प्रमुख
हिस्तयों के आसपास केंिद्रत थीं, और उल्लेखनीय स्थानों में संयुक्त राज्य अमेिरका और यूरोप शािमल थे।
कथन 1 सही है. गदर पाटीर्, िजसे प्रशांत तट के िहं दी एसोिसएशन (मूल रूप से प्रशांत तट के िहं दुस्तानी कायर्कतार् कहा जाता है) के रूप में भी
जाना जाता है, की स्थापना 1913 में एस्टोिरया, ओरेगॉन में भारतीय प्रवािसयों द्वारा की गई थी। पाटीर् का गठन लाला हर दयाल, संत बाबा
वसाखा िसं ह ददेहर, बाबा ज्वाला िसं ह, संतोख िसं ह और सोहन िसं ह भाकना ने िकया था, िजन्होंने राष्ट्रपित के रूप में कायर् िकया था।
एसोिसएशन का उद्देश्य भारत को अंग्रेजों के शासन से मुक्त कराना था। मई 1913 में, संगठन का नाम बदलकर िहं दुस्तान ग़दर पाटीर् कर िदया
गया।
कथन 2 सही है. तारकनाथ दास और जी.डी. कुमार ने 1910 में िसएटल में यूनाइटेड इं िडया हाउस की स्थापना की। यह घर भारतीय राष्ट्रवादी
गितिविधयों का एक केंद्र था, िजसका उद्देश्य िब्रिटश शासन से भारतीय स्वतंत्रता को बढ़ावा देना था।
कथन 3 सही है. पेिरस इं िडयन सोसाइटी एक भारतीय राष्ट्रवादी संगठन था िजसकी स्थापना 1905 में पेिरस में मैडम भीकाजी रुस्तम कामा,
मुंचेरशाह बुजोर्रजी गोदरेज और एस. आर. राणा के संरक्षण में की गई थी। पेिरस इं िडयन सोसाइटी ने 1909 में लंदन से भागने वाले
क्रांितकािरयों के िलए शरणस्थली के रूप में काम िकया। सोसाइटी ने बंदे मातरम जैसे राष्ट्रवादी समाचार पत्र भी प्रकािशत िकए और बिलर् न से
वीरेंद्रनाथ चट्टोपाध्याय के समाचार पत्र तलवार के प्रकाशन को िवत्तपोिषत िकया। मैडम भीकाजी कामा ने 1907 में जमर्नी के स्टू टगाटर् में
अंतरार्ष्ट्रीय समाजवादी सम्मेलन में िवदेशी भूिम पर भारतीय ध्वज फहराकर इितहास रचा।
Source: A BRIEF HISTORY OF MODERN INDIA BY RAJIV AHIR, Ch-13
https://amritmahotsav.nic.in/unsung-heroes-detail.htm?
55#:~:text=Madam%20Bhikaiji%20Cama%20became%20the,over%20the%20Indian%20sub%2Dcontinent.
https://egyankosh.ac.in/bitstream/123456789/89552/3/Unit-9.pdf
https://amritmahotsav.nic.in/district-reopsitory-detail.htm?13680

Question Attempted / Student

Question: 60 Mark Flag  Previous Next 

Correct Answer : b
Your Answer : b

Which of the following recovery types best represents the scenario where certain segments of the economy rebound from a
recession while others continue to decline?

िनम्निलिखत में से कौन सा पुनप्रार्िप्त प्रकार (Recovery Types) उस पिरदृश्य का सबसे अच्छा प्रितिनिधत्व करता है जहां अथर्व्यवस्था के
कुछ खंड मंदी से उबरते हैं जबिक अन्य में िगरावट जारी रहती है?

https://academy.forumias.com/beta/lms/viewSolutions Page 106 of 180


Academy ForumIAS | Dashboard 18/02/24, 8:41 PM

a V-shaped recovery V-आकार की िरकवरी


b K-shaped recovery K-आकार की िरकवरी
c U-shaped recovery U-आकार की िरकवरी
d W-shaped recovery W-आकार की िरकवरी

Explanation
Option b is the correct answer.

A recent study conducted by the Economic Research Department of the State Bank of India challenges the notion of a K-
shaped recovery in India's domestic economy post-pandemic. The study suggests that this debate may be flawed and
biased, potentially serving the interests of certain groups who are uncomfortable with India's significant economic
progress.

Option a is incorrect: A V-shaped recovery is defined by a rapid downturn succeeded by a swift and equally sharp
rebound, often returning to pre-recession levels.

Option b is correct: K-shaped recovery describes the uneven economic recovery across different sectors, industries, and
groups of people in the economy. Thus K-shaped economic recovery best represents the scenario where certain
segments of the economy rebound from a recession while others continue to decline.

Option c is incorrect: U-shaped recovery is a form of economic recovery where the recession portion typically lasts
for several quarters with a slow return to growth. It is measured by economic indicators like GDP, inflation,
employment levels, and industrial output.

Option d is incorrect: A W-shaped recovery occurs when an economy experiences a recession, enters a brief recovery,
and then promptly enters another recession. This pattern resembles the shape of the letter "W" on economic performance
charts. Also known as a double-dip recession, this type of downturn can mislead investors into premature re-entry due to
the temporary recovery phase.

Source: https://www.livemint.com/economy/sbi-study-questions-flawed-narrative-of-k-shaped-recovery-highlights-five-
major-trends-of-indian-economy-11704702238245.html

https://www.investopedia.com/k-shaped-recovery-5080086#:~:text=A K-shaped recovery is,while others continue to


decline.

https://academy.forumias.com/beta/lms/viewSolutions Page 107 of 180


Academy ForumIAS | Dashboard 18/02/24, 8:41 PM

िवकल्प b सही उत्तर है।


भारतीय स्टेट बैंक के आिथर् क अनुसंधान िवभाग द्वारा िकए गए एक हािलया अध्ययन में महामारी के बाद भारत की घरेलू अथर्व्यवस्था में K-
आकार की िरकवरी की धारणा को चुनौती दी गई है। अध्ययन से पता चलता है िक यह बहस त्रुिटपूणर् और पक्षपातपूणर् हो सकती है, जो
संभािवत रूप से कुछ समूहों के िहतों की पूितर् कर सकती है जो भारत की महत्वपूणर् आिथर् क प्रगित से असहज हैं।
िवकल्प a गलत है: V-आकार की िरकवरी को एक तीव्र मंदी से पिरभािषत िकया जाता है िजसके बाद एक तेज और समान रूप से तेज
पलटाव होता है, जो अक्सर पूवर्-मंदी के स्तर पर लौट आता है।

िवकल्प b सही है: K-आकार की िरकवरी अथर्व्यवस्था में िविभन्न क्षेत्रों, उद्योगों और लोगों के समूहों में असमान आिथर् क िरकवरी का वणर्न
करती है। इस प्रकार K-आकार की आिथर् क िरकवरी उस पिरदृश्य का सबसे अच्छा प्रितिनिधत्व करती है जहां अथर्व्यवस्था के कुछ खंड मंदी से
उबर रहे हैं जबिक अन्य में िगरावट जारी है।

िवकल्प c गलत है: U-आकार की िरकवरी आिथर् क िरकवरी का एक रूप है जहां मंदी का िहस्सा आम तौर पर िवकास में धीमी वापसी के साथ
कई ितमािहयों तक रहता है। इसे जीडीपी, मुद्रास्फीित, रोजगार स्तर और औद्योिगक उत्पादन जैसे आिथर् क संकेतकों द्वारा मापा जाता है।
िवकल्प d गलत है: W-आकार की िरकवरी तब होती है जब एक अथर्व्यवस्था मंदी का अनुभव करती है, एक संिक्षप्त िरकवरी में प्रवेश करती
है और िफर तुरत
ं दू सरी मंदी में प्रवेश करती है। यह पैटनर् आिथर् क प्रदशर्न चाटर् पर "W" अक्षर के आकार जैसा िदखता है। इसे डबल-िडप मंदी
के रूप में भी जाना जाता है, इस प्रकार की मंदी अस्थायी पुनप्रार्िप्त चरण के कारण िनवेशकों को समय से पहले पुनः प्रवेश के िलए गुमराह कर
सकती है।
Source: https://www.livemint.com/economy/sbi-study-questions-flawed-narrative-of-k-shaped-recovery-
highlights-five-major-trends-of-indian-economy-11704702238245.html
https://www.investopedia.com/k-shaped-recovery-
5080086#:~:text=A%20K%2Dshaped%20recovery%20is,while%20others%20continue%20to%20decline.

Question Attempted / Student

Question: 61 Mark Flag  Previous Next 

Correct Answer : c

https://academy.forumias.com/beta/lms/viewSolutions Page 108 of 180


Academy ForumIAS | Dashboard 18/02/24, 8:41 PM

Your Answer : c

With respect to modern Indian history, Consider the following events:

1. Appointments of Disorders Inquiry Committee

2. Renunciations of the knighthood by Rabindranath Tagore

3. Mahatma Gandhi giving up the title Kaiser-I-Hind

4. Chettur Sankaran Nair resigning from the Viceroy's Executive Council.

How many of the above events occurred in response to the Jallianwala Bagh Massacre in 1919?

आधुिनक भारतीय इितहास के संबंध में, िनम्निलिखत घटनाओं पर िवचार करें:


1. अव्यवस्था जाँच सिमित की िनयुिक्त।
2. रवीन्द्रनाथ टैगोर द्वारा नाइटहुड का त्याग।
3. महात्मा गांधी द्वारा कैसर-ए-िहं द की उपािध का त्यागा जाना।
4. चेट्टूर शंकरन नायर का वायसराय की कायर्कारी पिरषद से इस्तीफा।

उपरोक्त में से िकतनी घटनाएँ 1919 में जिलयाँवाला बाग नरसंहार की प्रितिक्रया में घिटत हुईं?

a Only one केवल एक


b Only two केवल दो
c Only three केवल तीन
d All four सभी चार

Explanation
Option c is the correct answer.

On April 13, 1919, the Jallianwala Bagh Massacre, also known as the Amritsar Massacre, took place. Colonel Reginald
Dyer, commanding the British Indian Army, opened fire on a mob of Baisakhi pilgrims. The firing lasted ten minutes, or
until all of the ammunition was exhausted. Several hundred individuals were killed and many more were injured in the
massacre.

Option 1 is incorrect. The Disorders Inquiry Committee, formed in 1919 to investigate the Jallianwala Bagh massacre,
which is known as the Hunter Committee. In the final report submitted in March 1920, the Committee unanimously
condemned Dyer's actions.' However, the Hunter Committee did not impose any penal or disciplinary action against
General Dyer. The Committee observed Dyer's actions as a response to a perceived threat to public order, rather
than a premeditated act of violence.

Options 2 and 3 are correct. After the Jallianwala Bagh massacre Rabindranath Tagore renounced his knighthood
in protest and Mahatma Gandhi gave up the title of Kaiser-i-Hind, bestowed by the British for his work during the
Boer War (1899 to 1902).

https://academy.forumias.com/beta/lms/viewSolutions Page 109 of 180


Academy ForumIAS | Dashboard 18/02/24, 8:41 PM

Option 4 is correct. Chettur Sankaran Nair (1857-1934), became the first Malayali president of the Indian National
Congress. In 1915, he became a member of the Viceroy's Executive Council. He resigned from the Viceroy's
Executive Council in 1919, after the Jallianwala Bagh Massacre. When Lord Chelmsford asked him to propose his
successor to the Council, he nominated his servant in protest.

Source: A BRIEF HISTORY OF MODERN INDIA BY RAJIV AHIR

https://amritmahotsav.nic.in/unsung-heroes-detail.htm?3619

िवकल्प c सही उत्तर है।


13 अप्रैल, 1919 को जिलयाँवाला बाग नरसंहार, िजसे अमृतसर नरसंहार के नाम से भी जाना जाता है, हुआ। िब्रिटश भारतीय सेना की कमान
संभाल रहे कनर्ल रेिजनाल्ड डायर ने बैसाखी तीथर्याित्रयों की भीड़ पर गोिलयां चला दीं। गोलीबारी दस िमनट तक या तब तक चली जब तक
सारा गोला-बारूद ख़त्म नहीं हो गया। इस नरसंहार में कई सौ लोग मारे गए और कई घायल हुए।
िवकल्प 1 ग़लत है. जिलयांवाला बाग हत्याकांड की जांच के िलए 1919 में िवकार जांच सिमित का गठन िकया गया, िजसे हंटर सिमित के
नाम से जाना जाता है। माचर् 1920 में प्रस्तुत अंितम िरपोटर् में, सिमित ने सवर्सम्मित से डायर के कायोर्ं की िनं दा की। हालाँिक, हंटर कमेटी ने
जनरल डायर के िखलाफ कोई दंडात्मक या अनुशासनात्मक कारर्वाई नहीं की। सिमित ने डायर की हरकतों को िहं सा के पूवर्-िनधार्िरत कृत्य के
बजाय सावर्जिनक व्यवस्था के िलए एक किथत खतरे की प्रितिक्रया के रूप में देखा।
िवकल्प 2 और 3 सही हैं। जिलयांवाला बाग हत्याकांड के बाद रवींद्रनाथ टैगोर ने िवरोध में अपनी नाइटहुड की उपािध त्याग दी और महात्मा
गांधी ने बोअर युद्ध (1899 से 1902) के दौरान उनके काम के िलए अंग्रेजों द्वारा दी गई कैसर-ए-िहं द की उपािध छोड़ दी।
िवकल्प 4 सही है. चेट्टूर शंकरन नायर (1857-1934), भारतीय राष्ट्रीय कांग्रेस के पहले मलयाली अध्यक्ष बने। 1915 में वे वायसराय की
कायर्कारी पिरषद के सदस्य बने। 1919 में जिलयांवाला बाग नरसंहार के बाद उन्होंने वायसराय की कायर्कारी पिरषद से इस्तीफा दे िदया। जब
लॉडर् चेम्सफोडर् ने उनसे अपने उत्तरािधकारी को पिरषद में प्रस्तािवत करने के िलए कहा, तो उन्होंने िवरोध में अपने नौकर को नािमत िकया।
Source: A BRIEF HISTORY OF MODERN INDIA BY RAJIV AHIR

https://amritmahotsav.nic.in/unsung-heroes-detail.htm?3619

Question Attempted / Student

Question: 62 Mark Flag  Previous Next 

Correct Answer : b
Your Answer : c

Consider the following statements with regard to the Madras Mahajana Sabha:

1. It was formed by C. Rajagopalachari and K.T Telang.

2. It advocated for the concurrent Civil Service exams in England and in India.

3. It launched salt satyagraha in 1930 at Madras in parallel with Civil Disobedient Movement.

How many of the above statements are correct?

https://academy.forumias.com/beta/lms/viewSolutions Page 110 of 180


Academy ForumIAS | Dashboard 18/02/24, 8:41 PM

मद्रास महाजन सभा के संबंध में िनम्निलिखत कथनों पर िवचार करें:


1. इसका गठन सी. राजगोपालाचारी और के.टी. तेलंग ने िकया था।
2. इसने इं ग्लैंड और भारत में एक साथ िसिवल सेवा परीक्षाओं की वकालत की।
3. इसने सिवनय अवज्ञा आं दोलन के समानांतर 1930 में मद्रास में नमक सत्याग्रह शुरू िकया।

उपरोक्त में से िकतने कथन सही हैं?

Only one केवल


a एक
Only two केवल
b दो
All three
c तीनों
None कोई
d नहीं

Explanation
Option b is the correct answer.

The Madras Mahajan Sabha was formed to oppose government policies peacefully. Members of the Mahajan Sabha saw
the need to build an All India organization to relieve and free the nation from the grips of British control, as well as to
tackle Indian problems.

Statement 1 is incorrect. M Veeraraghavachariar, G Subramania Iyer, and P Ananda Charlu formed the Madras
Mahajana Sabha, a regional political organization and forerunner of the Indian National Congress, in May 1884 in
Madras district (present-day Chennai).

1) K. T. Telang was associated with the establishment of the Bombay Presidency Association, founded in 1885.

2) Chakravarti Rajagopalachari (1878-1972) was an Indian statesman, writer, lawyer, and independence activist. He was
also a member of the Indian National Congress and the Swatantra (Independent) Party, which he founded and led in 1959.

Statement 2 is correct. The goal of the Madras Mahajana Sabha founded in 1884 to free the country from British
rule and to resolve national issues. The Sabha has advocated for a variety of measures while remaining moderate,
such as tax cuts, reductions in government and military spending, the abolition of the Council of India in London,
concurrent Civil Service exams in England and India, and fundamental rights for its fellow citizens, such as national
freedom.

Statement 3 is correct: In April 1930, the Madras Mahajana Sabha launched the Salt Satyagraha in George Town,
Esplanade, High Court, and Beach neighbourhoods (all localities in Madras) in parallel with Civil disobedient
movement .

Source: A BRIEF HISTORY OF MODERN INDIA BY RAJIV AHIR

https://academy.forumias.com/beta/lms/viewSolutions Page 111 of 180


Academy ForumIAS | Dashboard 18/02/24, 8:41 PM

िवकल्प b सही उत्तर है।


मद्रास महाजन सभा का गठन सरकारी नीितयों का शांितपूवर्क िवरोध करने के िलए िकया गया था। महाजन सभा के सदस्यों ने देश को िब्रिटश
िनयंत्रण से मुक्त कराने के साथ-साथ भारतीय समस्याओं से िनपटने के िलए एक अिखल भारतीय संगठन बनाने की आवश्यकता महसूस की।
कथन 1 गलत है. एम वीराराघवाचारी, जी सुब्रमण्यम अय्यर और पी आनंद चारलू ने मई 1884 में मद्रास िजले (वतर्मान चेन्नई) में मद्रास
महाजन सभा, एक क्षेत्रीय राजनीितक संगठन और भारतीय राष्ट्रीय कांग्रेस के अग्रदू त का गठन िकया।
1) के. टी. तेलंग 1885 में स्थािपत बॉम्बे प्रेसीडेंसी एसोिसएशन की स्थापना से जुड़े थे।
2) चक्रवतीर् राजगोपालाचारी (1878-1972) एक भारतीय राजनेता, लेखक, वकील और स्वतंत्रता कायर्कतार् थे। वह भारतीय राष्ट्रीय कांग्रेस
और स्वतंत्र पाटीर् के भी सदस्य थे, िजसकी उन्होंने 1959 में स्थापना और नेतृत्व िकया था।
कथन 2 सही है. 1884 में स्थािपत मद्रास महाजन सभा का लक्ष्य देश को िब्रिटश शासन से मुक्त कराना और राष्ट्रीय मुद्दों का समाधान करना
था। सभा ने उदारवादी रहते हुए कई तरह के उपायों की वकालत की है, जैसे कर में कटौती, सरकारी और सैन्य खचर् में कटौती, लंदन में भारतीय
पिरषद की समािप्त, इं ग्लैंड और भारत में समवतीर् िसिवल सेवा परीक्षा और अपने साथी नागिरकों के िलए मौिलक अिधकार , जैसे राष्ट्रीय
स्वतंत्रता।
कथन 3 सही है: अप्रैल 1930 में, मद्रास महाजन सभा ने सिवनय अवज्ञा आं दोलन के समानांतर जॉजर् टाउन, स्प्लेनेड, उच्च न्यायालय और
समुद्र तट के इलाकों (मद्रास के सभी इलाकों) में नमक सत्याग्रह शुरू िकया।

Source: A BRIEF HISTORY OF MODERN INDIA BY RAJIV AHIR

Question Attempted / Student

Question: 63 Mark Flag  Previous Next 

Correct Answer : c
Your Answer : c

With reference to the Chittagong Armoury Raid launched under the leadership of Surya Sen in 1930 consider the
following:

1. Surya Sen launched the Indian Republican Army to accomplish the Chittagong Armoury Raid.

2. Surya Sen proclaimed Provisional Government in Chittagong after the successful initial raid in Chittagong.

3. Surya Sen's leadership witnessed active participation of young women during the Chittagong raid.

How many of the above statements are correct?

1930 में सूयर् सेन के नेतृत्व में शुरू िकए गए चटगांव शस्त्रागार छापे के संदभर् में िनम्निलिखत पर िवचार करें:
1. सूयर् सेन ने चटगांव शस्त्रागार छापे को पूरा करने के िलए भारतीय िरपिब्लकन सेना की शुरुआत की।
2. चटगांव में सफल प्रारंिभक छापेमारी के बाद सूयर् सेन ने चटगांव में अनंितम सरकार की घोषणा की।
3. सूयर् सेन के नेतृत्व में चटगांव छापे के दौरान युवा मिहलाओं की सिक्रय भागीदारी देखी गई।

https://academy.forumias.com/beta/lms/viewSolutions Page 112 of 180


Academy ForumIAS | Dashboard 18/02/24, 8:41 PM

उपरोक्त में से िकतने कथन सही हैं?

Only one केवल


a एक
Only two केवल
b दो
All three
c तीनों
None कोई
d नहीं

Explanation
Option c is the correct answer.

The Chittagong Armoury Raid was a daring revolutionary action on April 18, 1930, led by Surya Sen and his group of
freedom fighters in Chittagong (now in Bangladesh). The objective was to seize arms from the British armoury to initiate
an armed struggle against colonial rule.

Statement 1 is correct: The raid was conducted in April 1930 and involved 65 activists under the banner of Indian
Republican Army-Chittagong Branch. The raid was quite successful; Sen hoisted the national flag, took salute and
proclaimed a provisional revolutionary government.

Statement 2 is correct: The Chittagong Armoury Raid was carried out by the Indian Republican Army in
Chittagong. The revolutionaries gathered outside the Polia Armoury, and Surya Sen was declared the President of
the Provisional Government formed in Chittagong. They took down the British flag and raised the Indian National
Flag, chanting patriotic slogans like "Vande Matram" and "Inquilab Zindabad".

Statement 3 is correct. The Chittagong Armoury Raid involved the active participation of young women who played
crucial roles in Surya Sen's group. These women provided shelter, acted as messengers, carried arms, and even fought
with guns. Some notable women during the Chittagong Armoury Raid include Pritilata Waddedar and Kalpana Datta
(received a life sentence after being arrested with Surya Sen).

1) Pritilata Waddedar(1911- 1932) joined the Deepali Sangha and later the Indian Revolutionary Army. She overcame
gender bias, participated in the Chittagong Armoury Raid aftermath, and executed crucial tasks for the movement with
skill and bravery.

2) Kalpana Datta (1913-95) was an Indian independence movement activist and freedom fighter. She was part of Master
Da Surya Sen's revolutionary group that carried out the 1930 Chittagong armoury raid. Datta was also a member of the
Chattri Sangha, a semi-revolutionary women's organization in Kolkata.

Source: A BRIEF HISTORY OF MODERN INDIA BY RAJIV AHIR

https://egyankosh.ac.in/bitstream/123456789/20043/1/Unit-24.pdf

https://indianculture.gov.in/node/2790410

https://academy.forumias.com/beta/lms/viewSolutions Page 113 of 180


Academy ForumIAS | Dashboard 18/02/24, 8:41 PM

िवकल्प c सही उत्तर है।


चटगांव शस्त्रागार छापा 18 अप्रैल, 1930 को चटगांव (अब बांग्लादेश में) में सूयर् सेन और उनके स्वतंत्रता सेनािनयों के समूह के नेतृत्व में एक
साहसी क्रांितकारी कारर्वाई थी। इसका उद्देश्य औपिनवेिशक शासन के िखलाफ सशस्त्र संघषर् शुरू करने के िलए िब्रिटश शस्त्रागार से हिथयार
जब्त करना था।
कथन 1 सही है: छापेमारी अप्रैल 1930 में की गई थी और इसमें भारतीय िरपिब्लकन आमीर्-चटगांव शाखा के बैनर तले 65 कायर्कतार् शािमल
थे। छापेमारी काफी सफल रही;
सेन ने राष्ट्रीय ध्वज फहराया, सलामी ली और एक अस्थायी क्रांितकारी सरकार की घोषणा की।
कथन 2 सही है: चटगांव शस्त्रागार पर छापा भारतीय िरपिब्लकन सेना द्वारा चटगांव में चलाया गया था। क्रांितकारी पोिलया शस्त्रागार के बाहर
एकत्र हुए और सूयर् सेन को चटगांव में गिठत अनंितम सरकार का अध्यक्ष घोिषत िकया गया। उन्होंने िब्रिटश ध्वज को उतार िदया और भारतीय
राष्ट्रीय ध्वज फहराया और "वंदे मातरम" और "इं कलाब िजं दाबाद" जैसे देशभिक्त के नारे लगाए।
कथन 3 सही है. चटगांव शस्त्रागार छापे में युवा मिहलाओं की सिक्रय भागीदारी शािमल थी िजन्होंने सूयर् सेन के समूह में महत्वपूणर् भूिमका
िनभाई। इन मिहलाओं ने आश्रय प्रदान िकया, दू त के रूप में काम िकया, हिथयार उठाए और यहां तक िक बंदूकों से भी लड़ाई की। चटगांव
शस्त्रागार छापे के दौरान कुछ उल्लेखनीय मिहलाओं में प्रीितलता वाडेदार और कल्पना दत्ता (सूयर् सेन के साथ िगरफ्तार होने के बाद आजीवन
कारावास की सजा िमली) शािमल हैं।
1) प्रीितलता वाडेदार (1911-1932) दीपाली संघ और बाद में भारतीय क्रांितकारी सेना में शािमल हुईं। उन्होंने लैंिगक पूवार्ग्रहों से परे जाकर
चटगांव शस्त्रागार छापे में भाग िलया और कौशल और बहादुरी के साथ आं दोलन के िलए महत्वपूणर् कायोर्ं को अंजाम िदया।
2) कल्पना दत्ता (1913-95) एक भारतीय स्वतंत्रता आं दोलन कायर्कतार् और स्वतंत्रता सेनानी थीं। वह मास्टर दा सूयार् सेन के क्रांितकारी समूह
का िहस्सा थीं, िजसने 1930 में चटगांव शस्त्रागार पर छापा मारा था। दत्ता कोलकाता में एक अधर्-क्रांितकारी मिहला संगठन, छत्री संघ
(Chattri Sangha) की सदस्य भी थीं।
Source: A BRIEF HISTORY OF MODERN INDIA BY RAJIV AHIR
https://egyankosh.ac.in/bitstream/123456789/20043/1/Unit-24.pdf

https://indianculture.gov.in/node/2790410

Question Attempted / Student

Question: 64 Mark Flag  Previous Next 

Correct Answer : a
Your Answer : c

With regard to the Indian Freedom Struggle, consider following statements about Barrah dacoity in the early twentieth
century:

1. The Barrah Dacoity was a planned action to acquire funds for revolutionary activities against the British.

2. It was organized by Ram Prasad Bismil and Ashfaqullah Khan.

3. The Hindustan Socialist Republican Association played a key role in organizing the Barrah dacoity.

How many of the above statements are correct?

https://academy.forumias.com/beta/lms/viewSolutions Page 114 of 180


Academy ForumIAS | Dashboard 18/02/24, 8:41 PM

भारतीय स्वतंत्रता संग्राम के संबंध में, बीसवीं सदी की शुरुआत में बरार् डकैती के बारे में िनम्निलिखत कथनों पर िवचार करें:
1. बरार् डकैती अंग्रेजों के िखलाफ क्रांितकारी गितिविधयों के िलए धन प्राप्त करने की एक योजनाबद्ध कारर्वाई थी।
2. इसका आयोजन राम प्रसाद िबिस्मल और अशफाकउल्ला खान ने िकया था।
3. िहं दुस्तान सोशिलस्ट िरपिब्लकन एसोिसएशन ने बरार् डकैती के आयोजन में महत्वपूणर् भूिमका िनभाई।

उपरोक्त में से िकतने कथन सही हैं?

a Only one केवल एक


b Only two केवल दो
c All three तीनों
d None कोई नहीं

Explanation
Option a is the correct answer.

The Barrah dacoity was the one of the significant event of revolutionary terrorists in Dhaka, East Bengal, in 1908.

Option 1 is correct. The Barrah Dacoity was a planned action in the early 20th century to acquire funds for
revolutionary activities against British colonial rule in India, demonstrating the resistance against British oppression.

Option 2 is incorrect. Pulin Das organised the Deccan Anushilan, with the Barrah dacoity as its first major venture.

1) Ram Prasad Bismil and Ashfaqullah Khan were associated with the Kakori Train Action of 1925. The Kakori
train robbery was a notable event where members of the Hindustan Socialist Republican Association (HSRA)

Option 3 is incorrect. The Hindustan Socialist Republican Association (HSRA) was founded in 1928 at a meeting in
Delhi to overcome the Kakori setback, whereas the Barrah dacoity took place in 1908. Hence, The HSRA was not
associated with the Barrah dacoity rather Dacca Anushilan Samiti was associated with it.

Source: A BRIEF HISTORY OF MODERN INDIA BY RAJIV AHIR

िवकल्प a सही उत्तर है।


बरार् डकैती 1908 में ढाका, पूवीर् बंगाल में क्रांितकािरयों की महत्वपूणर् घटनाओं में से एक थी।
िवकल्प 1 सही है. बरार् डकैती 20वीं सदी की शुरुआत में भारत में िब्रिटश औपिनवेिशक शासन के िखलाफ क्रांितकारी गितिविधयों के िलए धन
हािसल करने के िलए एक योजनाबद्ध कारर्वाई थी, जो िब्रिटश उत्पीड़न के िखलाफ प्रितरोध का प्रदशर्न करती थी।
िवकल्प 2 ग़लत है. पुिलन दास ने दक्कन अनुशीलन का आयोजन िकया, िजसका पहला प्रमुख उद्यम बरार् डकैती थी।
1) राम प्रसाद िबिस्मल और अशफाकउल्ला खान 1925 के काकोरी ट्रेन एक्शन से जुड़े थे। काकोरी ट्रेन डकैती एक उल्लेखनीय घटना थी
िजसमें िहं दुस्तान सोशिलस्ट िरपिब्लकन एसोिसएशन (HSRA) के सदस्य शािमल थे।
िवकल्प 3 ग़लत है. िहं दुस्तान सोशिलस्ट िरपिब्लकन एसोिसएशन (HSRA) की स्थापना 1928 में काकोरी झटके से उबरने के िलए िदल्ली में
एक बैठक में की गई थी, जबिक बरार् डकैती 1908 में हुई थी। इसिलए, HSRA बरार् डकैती से जुड़ा नहीं था, बिल्क ढाका अनुशीलन सिमित
इससे जुड़ी थी।

https://academy.forumias.com/beta/lms/viewSolutions Page 115 of 180


Academy ForumIAS | Dashboard 18/02/24, 8:41 PM

Source: A BRIEF HISTORY OF MODERN INDIA BY RAJIV AHIR

Question Attempted / Student

Question: 65 Mark Flag  Previous Next 

Correct Answer : b
Your Answer :

With reference to the biopesticide 'Tricolime', recently seen in news, consider the following statements:

1. It has the genes of a soil bacterium that can kill the specific virus and fungi that can affect the plants.

2. It promotes the growth of plants by neutralizing the soil acidity.

3. It has been developed by the Indian Institute of Spices Research (IISR).

How many of the statements given above are correct?

हाल ही में समाचारों में देखे गए जैव कीटनाशक 'ट्राइकोिलम' के संदभर् में, िनम्निलिखत कथनों पर िवचार करें:
1. इसमें िमट्टी के जीवाणु के जीन होते हैं जो पौधों को प्रभािवत करने वाले िविशष्ट वायरस और कवक को मार सकते हैं।
2. यह िमट्टी की अम्लता को िनिष्क्रय करके पौधों के िवकास को बढ़ावा देता है।
3. इसे भारतीय मसाला अनुसंधान संस्थान (Indian Institute of Spices Research; IISR) द्वारा िवकिसत िकया गया है।

ऊपर िदए गए कथनों में से िकतने सही हैं?

a Only one केवल एक


b Only two केवल दो
c All three तीनों
d None कोई नहीं

Explanation
Option b is the correct answer.

Tricholime serves as a bio-pesticide and bio-fertiliser in crop production. Tricholime integrates a fungi called
'Trichoderma' and lime into a single product, making the application easier for the farmers.

https://academy.forumias.com/beta/lms/viewSolutions Page 116 of 180


Academy ForumIAS | Dashboard 18/02/24, 8:41 PM

Statement 1 is incorrect: 'Tricholime' is a bio pesticides and bio fertilizers that utilizes the living organism known as
Trichoderma fungi, which is commonly found in all types of soil. Therefore the statement that it has the genes of a soil
bacterium is incorrect.

Statement 2 is correct: Tricholime is a lime-based formulation that neutralizes the soil acidity while promoting plant
growth and shields crops from soil-borne pathogens. It also benefits the crop by improving the physical condition of the
soil, enhancing secondary nutrient availability and by boosting soil microbial activity.

Statement 3 is correct: The Indian Council of Agricultural Research (ICAR) -Indian Institute of Spices Research
(IISR), Kozhikode has developed a new granular lime-based Trichoderma biopesticide known as Tricholime.

Source: https://www.thehindu.com/news/cities/kozhikode/iisr-develops-new-granular-lime-based-trichoderma-bio-
pesticide-fertiliser/article67706359.ece

https://icar.org.in/tricholime-new-granular-lime-based-trichoderma-formulation

िवकल्प b सही उत्तर है।


ट्राइकोिलम फसल उत्पादन में जैव-कीटनाशक और जैव-उवर्रक के रूप में कायर् करता है। ट्राइकोिलम 'ट्राइकोडमार्' नामक कवक और चूने को
एक ही उत्पाद में एकीकृत करता है, िजससे िकसानों के िलए इसका उपयोग आसान हो जाता है।
कथन 1 गलत है: 'ट्राइकोिलम' एक जैव कीटनाशक और जैव उवर्रक है जो ट्राइकोडमार् कवक नामक जीिवत जीव का उपयोग करता है, जो
आमतौर पर सभी प्रकार की िमट्टी में पाया जाता है। इसिलए यह कथन गलत है िक इसमें सॉइल बैक्टीिरयम के जीन हैं।
कथन 2 सही है: ट्राइकोिलम एक चूना-आधािरत सूत्रीकरण है जो पौधों के िवकास को बढ़ावा देने के साथ-साथ िमट्टी की अम्लता को बेअसर
करता है और फसलों को िमट्टी से पैदा होने वाले रोगजनकों से बचाता है। यह िमट्टी की भौितक िस्थित में सुधार करके, िद्वतीयक पोषक तत्वों
की उपलब्धता को बढ़ाकर और िमट्टी की सूक्ष्मजीव गितिविध को बढ़ाकर फसल को लाभ पहुंचाता है।
कथन 3 सही है: भारतीय कृिष अनुसंधान पिरषद (ICAR) -भारतीय मसाला अनुसंधान संस्थान (IISR), कोिझकोड ने एक नया दानेदार चूना-
आधािरत ट्राइकोडमार् बायोपेस्टीसाइड िवकिसत िकया है िजसे ट्राइकोिलम के नाम से जाना जाता है।
Source: https://www.thehindu.com/news/cities/kozhikode/iisr-develops-new-granular-lime-based-trichoderma-
bio-pesticide-fertiliser/article67706359.ece

https://icar.org.in/tricholime-new-granular-lime-based-trichoderma-formulation

Question Attempted / Student

Question: 66 Mark Flag  Previous Next 

Correct Answer : b
Your Answer : b

Consider the following statement with regard to the Indian Independence Committee formed at Berlin, 1915:

1. The committee was associated with Lala Hardayal and Bupendra Nath Dutta.

https://academy.forumias.com/beta/lms/viewSolutions Page 117 of 180


Academy ForumIAS | Dashboard 18/02/24, 8:41 PM

2. It organized a "provisional Indian government" in Kabul to stir anti-British feelings.

3. It actively participated and supported the Non-Cooperation Movement.

How many of the above statements are correct?

बिलर् न, 1915 में गिठत भारतीय स्वतंत्रता सिमित के संबंध में िनम्निलिखत कथन पर िवचार करें:
1. सिमित लाला हरदयाल और भूपेन्द्र नाथ दत्त से सम्बंिधत थी।
2. इसने िब्रिटश िवरोधी भावनाओं को भड़काने के िलए काबुल में एक "अस्थायी भारतीय सरकार" का आयोजन िकया।
3. इसने असहयोग आं दोलन में सिक्रय रूप से भाग िलया और समथर्न िकया।

उपरोक्त में से िकतने कथन सही हैं?

a Only one केवल एक


b Only two केवल दो
c All three तीनों
d None कोई नहीं

Explanation
Option b is the correct answer.

The Indian Independence Committee at Berlin(originally called the Berlin-Indian Committee), was established in 1915 by
Indian nationalists living in Germany during World War I. The committee's goal was to seek Germany's support for India's
independence from British rule.

Statement 1 is correct. Some famous Indians associated with the Berlin Committee were: Virendranath
Chattopadhyaya, Bupendra Nath Dutta and Lala Hardayal.

Statement 2 is correct. With the aid of Germany, the Indian Independence Committee devised several plans to oppose
British rule in India. The committee sent missions to the Middle Eastern cities of Istanbul, Baghdad, and Kabul to
incite anti-British feelings there. In one mission, Raja Mahendra Pratap Singh, Barkatullah, and Obaidullah Sindhi
organized a "provisional Indian government" in Kabul, Afghanistan.

Statement 3 is incorrect. The committee was formally disbanded in November 1918, with most of the members
shifting their attention to the nascent Soviet Russia whereas the Non Cooperation Movement took place in 1920

Source: A BRIEF HISTORY OF MODERN INDIA BY RAJIV AHIR

िवकल्प b सही उत्तर है।


बिलर् न में भारतीय स्वतंत्रता सिमित (मूल रूप से बिलर् न-भारतीय सिमित कहा जाता है) की स्थापना 1915 में प्रथम िवश्व युद्ध के दौरान जमर्नी में
रहने वाले भारतीय राष्ट्रवािदयों द्वारा की गई थी। सिमित का लक्ष्य िब्रिटश शासन से भारत की आजादी के िलए जमर्नी का समथर्न प्राप्त करना
था।
कथन 1 सही है. बिलर् न सिमित से जुड़े कुछ प्रिसद्ध भारतीय थे: वीरेंद्रनाथ चट्टोपाध्याय, भूपेंद्र नाथ दत्ता और लाला हरदयाल।

https://academy.forumias.com/beta/lms/viewSolutions Page 118 of 180


Academy ForumIAS | Dashboard 18/02/24, 8:41 PM

कथन 2 सही है. जमर्नी की सहायता से, भारतीय स्वतंत्रता सिमित ने भारत में िब्रिटश शासन का िवरोध करने के िलए कई योजनाएँ तैयार कीं।
सिमित ने मध्य पूवीर् शहरों इस्तांबुल, बगदाद और काबुल में िब्रिटश िवरोधी भावनाओं को भड़काने के िलए िमशन भेजे। एक िमशन में, राजा
महेंद्र प्रताप िसं ह, बरकतुल्लाह और ओबैदुल्ला िसं धी ने काबुल, अफगािनस्तान में एक "अनंितम भारतीय सरकार" का आयोजन िकया।
कथन 3 ग़लत है. नवंबर 1918 में सिमित को औपचािरक रूप से भंग कर िदया गया, अिधकांश सदस्यों ने अपना ध्यान उभरते सोिवयत रूस
की ओर केंिद्रत कर िदया, जबिक असहयोग आं दोलन 1920 में हुआ था।

Source: A BRIEF HISTORY OF MODERN INDIA BY RAJIV AHIR

Question Attempted / Student

Question: 67 Mark Flag  Previous Next 

Correct Answer : c
Your Answer : b

With regard to the impact and consequence of World War II on India, consider the following statements:

1. Increased foreign investment in the country.

2. Resignation of the Congress ministries from the office

3. Situation of high inflation the country

4. the development of exchange control

How many of the above statements are correct?

भारत पर िद्वतीय िवश्व युद्ध के प्रभाव और पिरणाम के संबंध में, िनम्निलिखत कथनों पर िवचार करें:
1. देश में िवदेशी िनवेश बढ़ना।
2. कांग्रेस मंित्रमंडलों का कायार्लय से इस्तीफा
3. देश में उच्च मुद्रास्फीित की िस्थित
4. िविनमय िनयंत्रण (Exchange Control) का िवकास

उपरोक्त में से िकतने कथन सही हैं?

a Only one केवल एक


b Only two केवल दो
c Only three केवल तीन
d All four सभी चार

https://academy.forumias.com/beta/lms/viewSolutions Page 119 of 180


Academy ForumIAS | Dashboard 18/02/24, 8:41 PM

Explanation
Option c is the correct answer.

World War II, lasting from 1939 to 1945, was a global conflict involving major powers across multiple continents. It arose
from unresolved issues of World War I, including territorial disputes and economic instability. The war witnessed
significant military engagements, atrocities, and the Holocaust, ultimately reshaping the geopolitical landscape.

Option 1 is incorrect. World War II generally led to decreased foreign investment in most countries, including
India, due to economic uncertainties and disruptions caused by the war.

Option 2 is correct. In October and November 1939, the Indian National Congress-led ministries in the provinces
resigned as a powerful protest against Viceroy Lord Linlithgow's decision to involve India in the Second World War
without consulting its people. The unilateral move, taken without seeking the consent of Indian leaders, was viewed as a
violation of India's right to self-determination.

Option 3 is incorrect. World War II profoundly impacted India, leading to economic imbalances, high inflation, and
currency devaluation. The Sterling Balance problem, with 1600 crores in Sterling securities, fueled uncontrolled
inflation. The war fueled India's independence movement, and the Bengal Famine strengthened nationalist resolve.
Post-war, the Labour Party in Britain initiated the process of granting India independence in 1947.

Option 4 is correct. Scarcity of Foreign Exchange in India led to its control since the beginning of World War II. India
introduced exchange control on September 3, 1939, on a temporary basis, under the Defense of India Rules.

Source: A BRIEF HISTORY OF MODERN INDIA BY RAJIV AHIR, Ch-15

https://www.rbi.org.in/scripts/FS_Overview.aspx?fn=5#:~:text=Exchange control was introduced in,Foreign Exchange


Regulation Act, 1973.

िवकल्प c सही उत्तर है।


िद्वतीय िवश्व युद्ध, जो 1939 से 1945 तक चला, एक वैिश्वक संघषर् था िजसमें कई महाद्वीपों की प्रमुख शिक्तयाँ शािमल थीं। यह क्षेत्रीय
िववादों और आिथर् क अिस्थरता सिहत प्रथम िवश्व युद्ध के अनसुलझे मुद्दों से उत्पन्न हुआ। युद्ध में महत्वपूणर् सैन्य संघषर्, अत्याचार और नरसंहार
देखा गया, िजसने अंततः भू-राजनीितक पिरदृश्य को नया आकार िदया।
िवकल्प 1 ग़लत है. िद्वतीय िवश्व युद्ध के कारण आम तौर पर आिथर् क अिनिश्चतताओं और युद्ध के कारण उत्पन्न व्यवधानों के कारण भारत
सिहत अिधकांश देशों में िवदेशी िनवेश में कमी आई।
िवकल्प 2 सही है. अक्टू बर और नवंबर 1939 में, प्रांतों में भारतीय राष्ट्रीय कांग्रेस के नेतृत्व वाले मंित्रमंडलों ने अपने लोगों से परामशर् िकए
िबना भारत को िद्वतीय िवश्व युद्ध में शािमल करने के वायसराय लॉडर् िलनिलथगो के फैसले के िखलाफ एक शिक्तशाली िवरोध के रूप में
इस्तीफा दे िदया। भारतीय नेताओं की सहमित के िबना उठाए गए इस एकतरफा कदम को भारत के आत्मिनणर्य के अिधकार के उल्लंघन के
रूप में देखा गया।
िवकल्प 3 ग़लत है. िद्वतीय िवश्व युद्ध ने भारत पर गहरा प्रभाव डाला, िजससे आिथर् क असंतुलन, उच्च मुद्रास्फीित और मुद्रा अवमूल्यन हुआ।
स्टिलर्ं ग बैलेंस समस्या, स्टिलर्ं ग िसक्योिरटीज में 1600 करोड़ के साथ, अिनयंित्रत मुद्रास्फीित को बढ़ावा िदया। युद्ध ने भारत के स्वतंत्रता
आं दोलन को बढ़ावा िदया और बंगाल के अकाल ने राष्ट्रवादी संकल्प को मजबूत िकया। युद्ध के बाद, िब्रटेन में लेबर पाटीर् ने 1947 में भारत को
स्वतंत्रता देने की प्रिक्रया शुरू की।
िवकल्प 4 सही है. िद्वतीय िवश्व युद्ध की शुरुआत से ही भारत में िवदेशी मुद्रा की कमी के कारण इस पर िनयंत्रण हो गया। भारत ने 3 िसतंबर,
1939 को भारत की रक्षा िनयमों के तहत अस्थायी आधार पर िविनमय िनयंत्रण की शुरुआत की।
Source: A BRIEF HISTORY OF MODERN INDIA BY RAJIV AHIR, Ch-15

https://academy.forumias.com/beta/lms/viewSolutions Page 120 of 180


Academy ForumIAS | Dashboard 18/02/24, 8:41 PM

https://www.rbi.org.in/scripts/FS_Overview.aspx?
fn=5#:~:text=Exchange%20control%20was%20introduced%20in,Foreign%20Exchange%20Regulation%20Act%2C%201

Question Attempted / Student

Question: 68 Mark Flag  Previous Next 

Correct Answer : a
Your Answer : a

Consider the following pairs with regard to the Conspiracy cases during Indian freedom struggle movement:

Conspiracy case Event occurred

1. Delhi Conspiracy Bombs thrown at DH


Case Kingsford's vehicle

2. Alipore Bomb Attempt to assassinate Lord


Conspiracy Case Hardinge

3. Kanpur Bolshevik Conspiring against the


Conspiracy Case Government.

How many of the above pairs are correctly matched?

भारतीय स्वतंत्रता संग्राम आं दोलन के दौरान षड्यंत्र के मामलों के संबंध में िनम्निलिखत युग्मों पर िवचार करें:

षड़यंत्र का मामला घिटत घटना

1. िदल्ली षडयंत्र केस िकंग्सफोडर् की गाड़ी पर


बम फेंके जाना

2. अलीपुर बम षडयंत्र लॉडर् हािडर्ं ग की हत्या का


केस प्रयास

3. कानपुर बोल्शेिवक सरकार के िखलाफ


षडयंत्र केस सािजश रची जाना

उपरोक्त में से िकतने युग्म सही सुमेिलत हैं?

https://academy.forumias.com/beta/lms/viewSolutions Page 121 of 180


Academy ForumIAS | Dashboard 18/02/24, 8:41 PM

a Only one केवल एक


b Only two केवल दो
c All three तीनों
d None कोई नहीं

Explanation
Option a is the correct answer.

The Indian Independence Revolutionary Movement was a vital component of the broader struggle for freedom,
encompassing the efforts of both known and unknown contributors. The movement involved diverse approaches, with
Indian youth participating in moderate, extremist, and revolutionary activities to challenge the rule of the East India
Company and pave the way for India's liberation.

Option 1 is incorrect. The Delhi-Lahore Conspiracy Case (1912), also known as the Delhi Conspiracy Case,
involved an assassination attempt on Lord Hardinge, the Viceroy of India at that time. It was led by Rash Behari Bose,
revolutionaries executed the plot during a ceremonial procession in Delhi, marking the transfer of the British capital from
Calcutta to Delhi. In the trial of this Delhi Conspiracy Case, Basant Kumar Biswas, Amir Chand, and Avadh Behari were
convicted and executed.

Option 2 is incorrect. The Alipore Bomb Conspiracy Case (1908), also known as the Muraripukur or Manicktolla
bomb conspiracy, targeted British Chief Magistrate Douglas Kingsford. The Ghosh brothers faced charges of
'conspiracy' or 'waging war against the King,' equivalent to high treason and punishable by hanging. Chittaranjan Das
defended Aurobindo, who was acquitted as the judge criticized the flimsy evidence. Barindra Ghosh, head of the
revolutionary secret society, and Ullaskar Dutt, bomb maker, initially received the death penalty, later commuted to life
imprisonment. During the trial, Narendra Gosain, an approver, was killed by co-accused Satyendranath Bose and Kanailal
Dutta in jail.

Option 3 is correct. In the Kanpur Bolshevik Conspiracy Case (1924 AD), the British Government executed India's
recently formed communists. The government apprehended and prosecuted Muzaffar Ahamed, S A Dange, Shaukat
Usmani and Nalini Gupta for conspiring against the government.

Source: A BRIEF HISTORY OF MODERN INDIA BY RAJIV AHIR, Pg-346

िवकल्प a सही उत्तर है।


भारतीय स्वतंत्रता क्रांितकारी आं दोलन (Indian Independence Revolutionary Movement) स्वतंत्रता के िलए व्यापक संघषर् का एक
महत्वपूणर् घटक था, िजसमें ज्ञात और अज्ञात दोनों योगदानकतार्ओ ं के प्रयास शािमल थे। इस आं दोलन में िविवध दृिष्टकोण शािमल थे, िजसमें
भारतीय युवाओं ने ईस्ट इं िडया कंपनी के शासन को चुनौती देने और भारत की मुिक्त का मागर् प्रशस्त करने के िलए उदारवादी, उग्रवादी और
क्रांितकारी गितिविधयों में भाग िलया।
िवकल्प 1 ग़लत है. िदल्ली-लाहौर षडयंत्र केस (1912), िजसे िदल्ली षडयंत्र केस के नाम से भी जाना जाता है, में उस समय भारत के
वायसराय लॉडर् हािडर्ं ग पर हत्या का प्रयास शािमल था। इसका नेतृत्व रासिबहारी बोस ने िकया था, क्रांितकािरयों ने िदल्ली में एक औपचािरक
जुलूस के दौरान इस सािजश को अंजाम िदया, जो िब्रिटश राजधानी को कलकत्ता से िदल्ली स्थानांतिरत करने का प्रतीक था। इस िदल्ली
षडयंत्र मामले की सुनवाई में बसंत कुमार िवश्वास, अमीर चंद और अवध िबहारी को दोषी ठहराया गया और फाँसी दे दी गई।

https://academy.forumias.com/beta/lms/viewSolutions Page 122 of 180


Academy ForumIAS | Dashboard 18/02/24, 8:41 PM

िवकल्प 2 ग़लत है. अलीपुर बम षडयंत्र केस (1908), िजसे मुरारीपुकुर या मािनकटोला बम षडयंत्र के नाम से भी जाना जाता है, में िब्रिटश
मुख्य मिजस्ट्रेट डगलस िकंग्सफोडर् को िनशाना बनाया गया था। घोष बंधुओ ं पर 'षड्यंत्र' या 'राजा के िखलाफ युद्ध छे ड़ने' का आरोप लगाया
गया, जो उच्च राजद्रोह के बराबर था और फांसी की सजा हो सकती थी। िचत्तरंजन दास ने अरिबं दो का बचाव िकया, िजन्हें न्यायाधीश द्वारा
कमज़ोर सबूतों की आलोचना के कारण बरी कर िदया गया था। क्रांितकारी गुप्त समाज के प्रमुख बािरन्द्र घोष और बम िनमार्ता उल्लासकर दत्त
को शुरू में मौत की सज़ा िमली, िजसे बाद में आजीवन कारावास में बदल िदया गया। मुकदमे के दौरान, एक सरकारी गवाह नरेंद्र गोसाईं की
जेल में सह-अिभयुक्त सत्येन्द्रनाथ बोस और कन्हैया लाल दत्त ने हत्या कर दी।
िवकल्प 3 सही है. कानपुर बोल्शेिवक षडयंत्र केस (1924 ई.) में िब्रिटश सरकार ने भारत के नवगिठत कम्युिनस्टों को फाँसी दे दी। सरकार ने
मुजफ्फर अहमद, एस ए डांग,े शौकत उस्मानी और निलनी गुप्ता को सरकार के िखलाफ सािजश रचने के आरोप में िगरफ्तार िकया और उन पर
मुकदमा चलाया।

Source: A BRIEF HISTORY OF MODERN INDIA BY RAJIV AHIR, Pg-346

Question Attempted / Student

Question: 69 Mark Flag  Previous Next 

Correct Answer : c
Your Answer : c

Consider the following developments with regard to the colonial period:

1. Establishment of the Durand Commission

2. Appointment of a Police Commission

3. The Ancient Monuments Preservation Act

4. The Younghusband's Mission to Tibet

How many of the above developments/reforms were carried out during the tenure of Lord Curzon?

औपिनवेिशक काल के संबंध में िनम्निलिखत िवकास पर िवचार करें:


1. डू रंड आयोग की स्थापना
2. पुिलस आयोग की िनयुिक्त
3. प्राचीन स्मारक संरक्षण अिधिनयम
4. यंगहसबैंड का ितब्बत िमशन

उपरोक्त में से िकतने िवकास/सुधार लॉडर् कजर्न के कायर्काल में िकये गये?

https://academy.forumias.com/beta/lms/viewSolutions Page 123 of 180


Academy ForumIAS | Dashboard 18/02/24, 8:41 PM

a Only one केवल एक


b Only two केवल दो
c Only three केवल तीन
d All four सभी चार

Explanation
Option c is the correct answer.

Lord Curzon succeeded Lord Elgin as Viceroy of India from 1899 to 1905. At the age of 39, he became India's youngest
Viceroy. He was one of the most controversial and influential people to hold that position. Curzon had visited India (four
times), Ceylon, and Afghanistan before becoming governor general and viceroy. China, Persia, Turkestan, Japan, and
Korea are among the countries involved.

Option 1 is incorrect. The Durand Commission was established in 1893 to demarcate the border between British
India and Afghanistan. The commission was established during Lord Lansdowne's tenure as Viceroy of India from
1888 to 1894.

Option 2 is correct. The Police Commission was appointed in 1902 during Lord Curzon's tenure. Lord Curzon
recognized the need for police reforms and appointed the Police Commission under Sir Andrew Frazer to review and
reorganize the police system in India. The commission's recommendations aimed at improving the efficiency, training, and
organization of the police force.

Option 3 is correct. The Ancient Monuments Preservation Act was enacted in 1904, during Lord Curzon's
administration. This act aimed at preserving and protecting ancient monuments and archaeological sites of historical and
cultural significance. It provided legal provisions for the maintenance and conservation of these monuments.

Option 4 is correct. The Younghusband's Mission to Tibet took place in 1903-1904 under the leadership of Sir
Francis Younghusband, who was a British military officer. The mission to Tibet occurred during Lord Curzon's
time, which aimed at establishing British influence in Tibet.

Source: A BRIEF HISTORY OF MODERN INDIA BY RAJIV AHIR, Pg-820

िवकल्प c सही उत्तर है।


लॉडर् कजर्न 1899 से 1905 तक भारत के वायसराय के रूप में लॉडर् एिल्गन के उत्तरािधकारी बने। 39 वषर् की आयु में, वह भारत के सबसे कम
उम्र के वायसराय बने। वह उस पद पर रहने वाले सबसे िववादास्पद और प्रभावशाली लोगों में से एक थे। गवनर्र जनरल और वायसराय बनने से
पहले कजर्न ने भारत (चार बार), सीलोन और अफगािनस्तान का दौरा िकया था। इसमें शािमल देशों में चीन, फारस, तुिकर्स्तान, जापान और
कोिरया शािमल हैं।
िवकल्प 1 ग़लत है. िब्रिटश भारत और अफगािनस्तान के बीच सीमा का सीमांकन करने के िलए 1893 में डू रंड आयोग की स्थापना की गई
थी। आयोग की स्थापना 1888 से 1894 तक भारत के वायसराय के रूप में लॉडर् लैंसडाउन के कायर्काल के दौरान की गई थी।
िवकल्प 2 सही है. 1902 में लॉडर् कजर्न के कायर्काल में पुिलस आयोग की िनयुिक्त की गई। लॉडर् कजर्न ने पुिलस सुधारों की आवश्यकता
को पहचाना और भारत में पुिलस प्रणाली की समीक्षा और पुनगर्ठन के िलए सर एं ड्रयू फ्रेज़र के तहत पुिलस आयोग िनयुक्त िकया। आयोग की
िसफािरशों का उद्देश्य पुिलस बल की दक्षता, प्रिशक्षण और संगठन में सुधार करना है।

https://academy.forumias.com/beta/lms/viewSolutions Page 124 of 180


Academy ForumIAS | Dashboard 18/02/24, 8:41 PM

िवकल्प 3 सही है. प्राचीन स्मारक संरक्षण अिधिनयम 1904 में लॉडर् कजर्न के प्रशासन के दौरान अिधिनयिमत िकया गया था। इस अिधिनयम
का उद्देश्य ऐितहािसक और सांस्कृितक महत्व के प्राचीन स्मारकों और पुराताित्वक स्थलों को संरिक्षत और संरिक्षत करना है। इसने इन स्मारकों
के रखरखाव और संरक्षण के िलए कानूनी प्रावधान प्रदान िकए।
िवकल्प 4 सही है. यंगहसबैंड का ितब्बत िमशन 1903-1904 में सर फ्रांिसस यंगहसबैंड के नेतृत्व में हुआ, जो एक िब्रिटश सैन्य अिधकारी थे।
ितब्बत िमशन लॉडर् कजर्न के समय में हुआ, िजसका उद्देश्य ितब्बत में िब्रिटश प्रभाव स्थािपत करना था।

Source: A BRIEF HISTORY OF MODERN INDIA BY RAJIV AHIR, Pg-820

Question Attempted / Student

Question: 70 Mark Flag  Previous Next 

Correct Answer : d
Your Answer : d

With reference to 'direct-to-cell satellite technology', consider the following statements:

1. While it enables direct transfer of data from satellite to mobile phones, it cannot directly transmit voice data.

2. It is immune to cyber threats as it does not require any ground-based infrastructure for communication purposes.

Which of the statements given above is/are correct?

'डायरेक्ट-टू -सेल सैटेलाइट टेक्नोलॉजी' के संदभर् में, िनम्निलिखत कथनों पर िवचार करें:
1. हालांिक यह सैटेलाइट से मोबाइल फोन पर सीधे डेटा ट्रांसफर करने में सक्षम बनाता है, लेिकन यह वॉयस डेटा सीधे ट्रांसिमट नहीं कर
सकता है।
2. यह साइबर खतरों से प्रितरिक्षत है क्योंिक इसे संचार उद्देश्यों के िलए िकसी धरातल आधािरत अवसंरचनात्मक ढांचे की आवश्यकता नहीं है।

ऊपर िदए गए कथनों में से कौन सा/से सही है/हैं?

a 1 only केवल 1
b 2 only केवल 2
c Both 1 and 2 1 और 2 दोनों
d Neither 1 nor 2 न तो 1 और न ही 2

Explanation
Option d is the correct answer.

https://academy.forumias.com/beta/lms/viewSolutions Page 125 of 180


Academy ForumIAS | Dashboard 18/02/24, 8:41 PM

Recently, SpaceX successfully launched the first set of Starlink satellites equipped with direct-to-cell capabilities.
These Starlink satellites can beam phone signals from space directly to smartphones.

Statement 1 is incorrect: Direct-to-cell technology allows for the direct transmission of communication signals,
including text messages, voice, and data, from space-based satellites to mobile devices (cell phones), without
conventional ground-based infrastructure. These satellites once in orbit instantly connect over a laser backhaul to Starlink's
low-earth-orbit (LEO) constellation to provide global connectivity. Starlink satellites, loaded with the specific modems,
will then directly beam internet to smartphones in any location on the planet.

Statement 2 is incorrect: While direct-to-cell satellite technology may be less vulnerable to certain types of cyber
threats due to its reduced reliance on ground-based infrastructure, it is not entirely immune to cyber threats. For instance
Internet of Things (IoT) devices that utilizes this technology make it vulnerable for cyber attacks.

Source: https://economictimes.indiatimes.com/news/et-explains/et-explainer-what-is-starlinks-direct-to-cell-technology-
does-it-work-with-existing-handsets/articleshow/106542052.cms?from=mdr

https://www.archonsecure.com/blog/satellite-cybersecurity#:~:text=that utilize satellite-,communications,-pose additional


potential

िवकल्प d सही उत्तर है।


हाल ही में, स्पेसएक्स ने डायरेक्ट-टू -सेल (Direct-to-cell) क्षमताओं से लैस स्टारिलं क उपग्रहों का पहला सेट सफलतापूवर्क लॉन्च िकया। ये
स्टारिलं क उपग्रह अंतिरक्ष से फोन िसग्नल को सीधे स्माटर्फोन तक पहुंचा सकते हैं।
कथन 1 गलत है: डायरेक्ट-टू -सेल तकनीक पारंपिरक धरातल-आधािरत अवसंरचनात्मक ढांचे के िबना, अंतिरक्ष-आधािरत उपग्रहों से मोबाइल
उपकरणों (सेल फोन) तक पाठ संदेश, आवाज और डेटा सिहत संचार संकेतों के सीधे प्रसारण की अनुमित देती है। एक बार कक्षा में पहुंचने के
बाद ये उपग्रह वैिश्वक कनेिक्टिवटी प्रदान करने के िलए तुरत
ं लेजर बैकहॉल से स्टारिलं क की िनम्न-पृथ्वी-कक्षा (LEO) तारामंडल से जुड़ जाते
हैं। िविशष्ट मॉडेम से लोड िकए गए स्टारिलं क उपग्रह, ग्रह पर िकसी भी स्थान पर स्माटर्फोन पर सीधे इं टरनेट प्रसािरत करेंगे।
कथन 2 गलत है: जबिक डायरेक्ट-टू -सेल उपग्रह प्रौद्योिगकी जमीनी-आधािरत बुिनयादी ढांचे पर कम िनभर्रता के कारण कुछ प्रकार के साइबर
खतरों के प्रित कम संवेदनशील हो सकती है, लेिकन यह साइबर खतरों से पूरी तरह से प्रितरिक्षत नहीं है। उदाहरण के िलए, इं टरनेट ऑफ िथं ग्स
(IoT) उपकरण जो इस तकनीक का उपयोग करते हैं, इसे साइबर हमलों के िलए असुरिक्षत बनाते हैं।
Source: https://economictimes.indiatimes.com/news/et-explains/et-explainer-what-is-starlinks-direct-to-cell-
technology-does-it-work-with-existing-handsets/articleshow/106542052.cms?from=mdr

https://www.archonsecure.com/blog/satellite-
cybersecurity#:~:text=that%20utilize%20satellite-,communications,-pose%20additional%20potential

Question Attempted / Student

Question: 71 Mark Flag  Previous Next 

Correct Answer : d
Your Answer : d

https://academy.forumias.com/beta/lms/viewSolutions Page 126 of 180


Academy ForumIAS | Dashboard 18/02/24, 8:41 PM

Consider the following pairs with regard to various British administrative policies and the persons associated with them.

Policy Person associated

1. Policy of Masterly Lord Wellesley


Inactivity

2. Policy of Proud Lord Auckland


Reserve

3. Policy of Ring-Fence Lord Dalhousie

How many of the above pairs are correctly matched?

िविभन्न िब्रिटश प्रशासिनक नीितयों और उनसे जुड़े व्यिक्तयों के संबंध में िनम्निलिखत युग्मों पर िवचार करें।

नीित सम्बंिधत व्यिक्त

1. कुशल अकमर्ण्यता की लॉडर् वेलेजली


नीित

2. प्राउड िरजवर् की नीित लॉडर् ऑकलैंड

3. िरं ग-फेंस की नीित लॉडर् डलहौजी

उपरोक्त में से िकतने युग्म सही सुमेिलत हैं?

a Only one केवल एक


b Only two केवल दो
c All three तीनों
d None कोई नहीं

Explanation
Option d is the correct answer.

British expansion policies in India resulted in territorial control, economic exploitation, cultural transformation, and social
disruptions. Colonization impacted Indian society, governance, and economy, leaving lasting legacies and challenges.

Option 1 is incorrect. John Lawrence (1864-1869) started a policy of masterly inactivity which was a reaction to the
disasters of the First Afghan War and an outcome of practical common sense and an intimate knowledge of the frontier
problem and of Afghan passion for independence.

Option 2 is incorrect. Lord Lytton, the Viceroy of India from 1876 to 1880, carried out the Policy of Proud Reserve.
The policy sought to define scientific boundaries and preserve "spheres of influence." It also aimed to establish British
dominance and control over its territories, as well as to exert influence over neighboring regions.

Option 3 is incorrect. Warren Hastings established the Ring-Fence Policy from 1765 to 1813 to protect the East India
Company's borders. The policy called for the Company to defend the borders of its neighbors in order to protect their own
territories. The East India Company would send troops to strengthen their allies' defenses, and the rulers of those states

https://academy.forumias.com/beta/lms/viewSolutions Page 127 of 180


Academy ForumIAS | Dashboard 18/02/24, 8:41 PM

would pay for their maintenance.

Knowledge Base:

1) Auckland who came to India as the governor-general in 1836, advocated a forward policy. This implied that the
Company government in India itself had to take initiatives to protect the boundary of British India from a probable Russian
attack.

2) In 1798, Lord Wellesley established the Subsidiary Alliance. The British East India Company and the Indian states
formed an alliance to provide military protection. The alliance allowed the Company to become more involved in the
internal politics of the Indian kingdoms. The princely states ceded their sovereignty to the British in exchange for
protection. The princely states were not permitted to have their own armed forces and were required to pay for the
subsidiary forces.

3) The Doctrine of Lapse was introduced by Lord Dalhousie, the Governor-General of India from 1848 to 1856. The
policy stated that if an Indian ruler died without a male heir, his kingdom would automatically pass to the British.

Source: A BRIEF HISTORY OF MODERN INDIA BY RAJIV AHIR,Pg-816

िवकल्प d सही उत्तर है।


भारत में िब्रिटश िवस्तार नीितयों के पिरणामस्वरूप क्षेत्रीय िनयंत्रण, आिथर् क शोषण, सांस्कृितक पिरवतर्न और सामािजक व्यवधान उत्पन्न हुए।
उपिनवेशीकरण ने भारतीय समाज, शासन और अथर्व्यवस्था को प्रभािवत िकया और स्थायी िवरासतें और चुनौितयाँ छोड़ दीं।
िवकल्प 1 गलत है| जॉन लॉरेंस (1864-1869) ने कुशल अकमर्ण्यता की नीित शुरू की जो प्रथम अफगान युद्ध की आपदाओं की प्रितिक्रया
थी और व्यावहािरक सामान्य ज्ञान और सीमांत समस्या के गहन ज्ञान और स्वतंत्रता के िलए अफगान जुनून का पिरणाम थी।
िवकल्प 2 गलत है| 1876 से 1880 तक भारत के वायसराय लॉडर् िलटन ने प्राउड िरजवर् की नीित लागू की। नीित ने वैज्ञािनक सीमाओं को
पिरभािषत करने और "प्रभाव के क्षेत्रों" को संरिक्षत करने की मांग की। इसका उद्देश्य अपने क्षेत्रों पर िब्रिटश प्रभुत्व और िनयंत्रण स्थािपत करना
और साथ ही पड़ोसी क्षेत्रों पर प्रभाव डालना भी था।
िवकल्प 3 गलत है| वॉरेन हेिस्टं ग्स ने ईस्ट इं िडया कंपनी की सीमाओं की सुरक्षा के िलए 1765 से 1813 तक िरं ग-फेंस नीित की स्थापना की।
नीित में कंपनी से अपने क्षेत्रों की रक्षा के िलए अपने पड़ोिसयों की सीमाओं की रक्षा करने का आह्वान िकया गया। ईस्ट इं िडया कंपनी अपने
सहयोिगयों की सुरक्षा को मजबूत करने के िलए सेना भेजेगी, और उन राज्यों के शासक उनके रखरखाव के िलए भुगतान करेंगे।
ज्ञानधार:
1) 1836 में गवनर्र-जनरल बनकर भारत आए ऑकलैंड ने फॉरवडर् पॉिलसी की वकालत की। इसका तात्पयर् यह था िक भारत में कंपनी सरकार
को संभािवत रूसी हमले से िब्रिटश भारत की सीमा की रक्षा के िलए स्वयं पहल करनी होगी।
2) 1798 में लॉडर् वेलेस्ली ने सहायक गठबंधन की स्थापना की। िब्रिटश ईस्ट इं िडया कंपनी और भारतीय राज्यों ने सैन्य सुरक्षा प्रदान करने के
िलए एक गठबंधन बनाया। गठबंधन ने कंपनी को भारतीय राज्यों की आं तिरक राजनीित में और अिधक शािमल होने की अनुमित दी। िरयासतों
ने सुरक्षा के बदले में अपनी संप्रभुता अंग्रेजों को सौंप दी। िरयासतों को अपनी स्वयं की सशस्त्र सेना रखने की अनुमित नहीं थी और उन्हें
सहायक सेनाओं के िलए भुगतान करना पड़ता था।
3) 1848 से 1856 तक भारत के गवनर्र-जनरल लॉडर् डलहौजी द्वारा डॉिक्ट्रन ऑफ लैप्स की शुरुआत की गई थी। नीित में कहा गया था िक
यिद कोई भारतीय शासक िबना िकसी पुरुष उत्तरािधकारी के मर जाता है, तो उसका राज्य स्वचािलत रूप से अंग्रेजों के पास चला जाएगा।

Source: A BRIEF HISTORY OF MODERN INDIA BY RAJIV AHIR,Pg-816

Question Attempted / Student

https://academy.forumias.com/beta/lms/viewSolutions Page 128 of 180


Academy ForumIAS | Dashboard 18/02/24, 8:41 PM

Question: 72 Mark Flag  Previous Next 

Correct Answer : c
Your Answer :

With regard to the Anglo-Mysore War in Modern Indian History, consider the following pairs:

War Event occured

1. First Anglo-Mysore Attempt to Capture


War Mahe by English

2. Second Anglo- Treaty of Mangalore


Mysore War was signed

3. Third Anglo- Lord Cornwallis headed


Mysore War the British Army

4. Fourth Anglo- Imposition of


Mysore War subsidiary alliance
system on Mysore

How many of the above statements are correct?

आधुिनक भारतीय इितहास में आं ग्ल-मैसूर युद्ध के संबंध में, िनम्निलिखत युग्मों पर िवचार करें:

!"# $%&' () $&*+

1. प्रथम आं ग्ल-मैसूर युद्ध अंग्रेजों द्वारा माहे पर कब्ज़ा करने का


प्रयास

2. िद्वतीय आं ग्ल-मैसूर युद्ध मैंगलोर की संिध पर हस्ताक्षर िकये


गये

3. तृतीय आं ग्ल-मैसूर युद्ध लॉडर् कॉनर्वािलस ने िब्रिटश सेना का


नेतृत्व िकया

4. चतुथर् आं ग्ल-मैसूर युद्ध मैसूर पर सहायक गठबंधन प्रणाली


लागू करना

उपरोक्त में से िकतने कथन सही हैं?

a Only one केवल एक


b Only two केवल दो
c Only three केवल तीन
d All four सभी चार

https://academy.forumias.com/beta/lms/viewSolutions Page 129 of 180


Academy ForumIAS | Dashboard 18/02/24, 8:41 PM

Explanation
Option c is the correct answer.

The Anglo-Mysore Wars were a series of four military confrontations between the British East India Company and the
Kingdom of Mysore in South India during the 18th century.

Pair 1 is incorrect. During the Second Anglo-Mysore War (1780-1784) not in First Anglo-Mysore War, Haidar Ali
accused the English of violating the Treaty of Madras by failing to aid him when attacked by the Marathas in 1771. The
English's inadequate support contrasted with the French, who proved more helpful in supplying guns, saltpetre, and lead
through Mahe on the Malabar coast. With the American War of Independence unfolding, where the French sided against
the English, Haidar Ali's allegiance to the French heightened English apprehensions. When the English attempted to
capture Mahe, considered under Haidar Ali's protection, it exacerbated tensions, viewed by him as a direct
challenge to his authority.

Pair 2 is correct. The Second Anglo-Mysore War (1780-1784) was concluded with the Treaty of Mangalore in 1784.

Pair 3 is correct. During the Third Anglo-Mysore War (1790-1792), the English aligned with Travancore to launch an
attack on Tipu Sultan. In the initial phase, Tipu emerged victorious in 1790, defeating the English forces led by General
Meadows. However, in 1791, Cornwallis assumed command, leading a substantial army through Ambur and Vellore
to capture Bangalore in March 1791. Continuing their advance, the English forces, under Cornwallis's leadership,
proceeded to the capital city of Seringapatam.

Pair 4 is correct. The Fourth Anglo-Mysore War unfolded from April 17, 1799, to May 4, 1799, culminating in the fall of
Seringapatam. Tipu Sultan faced successive defeats, initially by English General Stuart and later by General Harris. Arthur
Wellesley, brother of Lord Wellesley, played a role in the conflict. Maratha and Nizam support bolstered the English, with
the Marathas promised half of Tipu's territory. The Nizam, having signed the Subsidiary Alliance, also joined forces. Tipu
valiantly perished in battle; his family was interned at Vellore, and English forces confiscated his treasures. A young heir
from Mysore's former Hindu royal family became the new maharaja, subjected to the subsidiary alliance.

Source: A BRIEF HISTORY OF MODERN INDIA BY RAJIV AHIR,Ch-5

https://academy.forumias.com/beta/lms/viewSolutions Page 130 of 180


Academy ForumIAS | Dashboard 18/02/24, 8:41 PM

िवकल्प c सही उत्तर है।


आं ग्ल-मैसूर युद्ध 18वीं शताब्दी के दौरान िब्रिटश ईस्ट इं िडया कंपनी और दिक्षण भारत में मैसूर साम्राज्य के बीच चार सैन्य टकरावों की एक
श्रृंखला थी।
युग्म 1 गलत है| प्रथम आं ग्ल-मैसूर युद्ध के अलावा दू सरे आं ग्ल-मैसूर युद्ध (1780-1784) के दौरान, हैदर अली ने अंग्रेजों पर 1771 में मराठों
द्वारा हमला िकए जाने पर उनकी सहायता करने में िवफल रहने के कारण मद्रास की संिध का उल्लंघन करने का आरोप लगाया। फ्रांसीसी, जो
मालाबार तट पर माहे के माध्यम से बंदूकें, शोरा और सीसा की आपूितर् में अिधक मददगार सािबत हुए। अमेिरकी स्वतंत्रता संग्राम शुरू होने के
साथ जहां फ्रांसीसी, अंग्रेजों के िखलाफ थे, हैदर अली की फ्रांसीिसयों के प्रित िनष्ठा ने अंग्रेजी आशंकाओं को बढ़ा िदया। जब अंग्रेजों ने माहे
पर कब्जा करने का प्रयास िकया, िजसे हैदर अली के संरक्षण में माना जाता था, तो इससे तनाव बढ़ गया, िजसे उन्होंने अपने अिधकार के िलए
सीधी चुनौती के रूप में देखा।
युग्म 2 सही है| िद्वतीय आं ग्ल-मैसूर युद्ध (1780-1784), 1784 में मैंगलोर की संिध के साथ संपन्न हुआ।
युग्म 3 सही है| तीसरे आं ग्ल-मैसूर युद्ध (1790-1792) के दौरान, अंग्रेजों ने टीपू सुल्तान पर हमला शुरू करने के िलए त्रावणकोर के साथ
गठबंधन िकया। प्रारंिभक चरण में, टीपू 1790 में जनरल मीडोज के नेतृत्व वाली अंग्रेजी सेना को हराकर िवजयी हुए। हालाँिक, 1791 में,
कॉनर्वािलस ने कमान संभाली, और माचर् 1791 में बैंगलोर पर कब्ज़ा करने के िलए अंबुर और वेल्लोर के माध्यम से एक बड़ी सेना का नेतृत्व
िकया। अपनी प्रगित जारी रखते हुए, कॉनर्वािलस के नेतृत्व में अंग्रेजी सेना, राजधानी शहर श्रीरंगपट्टम की ओर आगे बढ़ी।

युग्म 4 सही है| चौथा आं ग्ल-मैसूर युद्ध 17 अप्रैल, 1799 से 4 मई, 1799 तक चला, िजसकी पिरणित श्रीरंगपट्टम के पतन में हुई। टीपू
सुल्तान को लगातार हार का सामना करना पड़ा, शुरुआत में अंग्रेज जनरल स्टुअटर् से और बाद में जनरल हैिरस से। लॉडर् वेलेस्ली के भाई आथर्र
वेलेस्ली ने संघषर् में भूिमका िनभाई। मराठों और िनज़ाम के समथर्न से अंग्रेज़ों को बल िमला, मराठों ने टीपू के क्षेत्र का आधा िहस्सा देने का
वादा िकया। सहायक संिध पर हस्ताक्षर करने के बाद िनज़ाम भी सेना में शािमल हो गया। टीपू युद्ध में वीरतापूवर्क मारा गया; उनके पिरवार को
वेल्लोर में नजरबंद कर िदया गया और अंग्रेजी सेना ने उनके खजाने को जब्त कर िलया। मैसूर के पूवर् िहं दू शाही पिरवार का एक युवा
उत्तरािधकारी सहायक गठबंधन के अधीन नया महाराजा बन गया।

Source: A BRIEF HISTORY OF MODERN INDIA BY RAJIV AHIR,Ch-5

Question Attempted / Student

Question: 73 Mark Flag  Previous Next 

https://academy.forumias.com/beta/lms/viewSolutions Page 131 of 180


Academy ForumIAS | Dashboard 18/02/24, 8:41 PM

Correct Answer : a
Your Answer : a

Consider the following statements regarding the Third Battle of Panipat:

Statement-I: The Third Battle of Panipat halted the expansion of the Maratha Empire.

Statement-II: The defeat had a profound impact on the Maratha Empire, leading to a decline in their influence in North
India.

Which one of the following is correct in respect of the above statements?

पानीपत की तीसरी लड़ाई के संबंध में िनम्निलिखत कथनों पर िवचार करें:


कथन-I: पानीपत की तीसरी लड़ाई ने मराठा साम्राज्य के िवस्तार को रोक िदया।
कथन-II: हार का मराठा साम्राज्य पर गहरा प्रभाव पड़ा, िजससे उत्तर भारत में उनका प्रभाव कम हो गया।

उपरोक्त कथनों के संबंध में िनम्निलिखत में से कौन सा सही है?

a
Both Statement-I and Statement-II are correct and Statement-II is the correct explanation of Statement-I कथन-I और
कथन-II दोनों सही हैं और कथन-II कथन-I की सही व्याख्या है
b
Both Statement-I and Statement-II are correct and Statement-II is not the correct explanation of Statement-I कथन-I
और कथन-II दोनों सही हैं और कथन-II कथन-I की सही व्याख्या नहीं है
Statement-I is correct but Statement-II is incorrect कथन-I सही है लेिकन कथन-II गलत
c है
Statement-I is incorrect but Statement-II is correct कथन-I गलत है लेिकन कथन-II सही
d है

Explanation
Option a is the correct answer.

The Third Battle of Panipat was fought on January 14, 1761, between a northern expeditionary force of the Maratha
Empire and a coalition of the King of Afghanistan, Ahmad Shah Durrani, and two Indian Muslim allies-the Rohilla
Afghans of the Doab, and Shuja-ud-Daula, the Nawab of Oudh.

Statement-I is correct. The Third Battle of Panipat in 1761 halted the Maratha Empire's expansion for approximately ten
years. The Marathas were accustomed to guerrilla warfare and were unprepared for an open battle with Afghan invader
Ahmed Shah Abdali. The Marathas lost the battle, which contributed to their decline in power.

https://academy.forumias.com/beta/lms/viewSolutions Page 132 of 180


Academy ForumIAS | Dashboard 18/02/24, 8:41 PM

Statement-II is correct and is the explanation of Statement-I. The defeat in the Third Battle of Panipat had profound
and far-reaching consequences for the Maratha Empire. The loss not only resulted in a considerable reduction in
Maratha military strength but also had economic and political repercussions. The defeat weakened the Maratha
Empire's hold on North India, and they faced challenges in maintaining their dominance in the region.

Source: A BRIEF HISTORY OF MODERN INDIA BY RAJIV AHIR, Chapter-4

िवकल्प a सही उत्तर है।


पानीपत की तीसरी लड़ाई 14 जनवरी, 1761 को मराठा साम्राज्य के उत्तरी अिभयान दल और अफगािनस्तान के राजा, अहमद शाह दुरार्नी और
दो भारतीय मुिस्लम सहयोिगयों-दोआब के रोिहल्ला अफगानों और शुजा-उद-दौला, अवध के नवाब के गठबंधन के बीच लड़ी गई थी।
कथन-I सही है। 1761 में पानीपत की तीसरी लड़ाई ने मराठा साम्राज्य के िवस्तार को लगभग दस वषोर्ं तक रोक िदया। मराठा गुिरल्ला युद्ध
के आदी थे और अफगान आक्रमणकारी अहमद शाह अब्दाली के साथ खुली लड़ाई के िलए तैयार नहीं थे। मराठा लड़ाई हार गए, िजससे
उनकी शिक्त में कमी आई।
कथन-II सही है और कथन-I की व्याख्या है। पानीपत की तीसरी लड़ाई में हार का मराठा साम्राज्य पर गहरा और दू रगामी प्रभाव पड़ा। इस
नुकसान से न केवल मराठा सैन्य ताकत में काफी कमी आई, बिल्क आिथर् क और राजनीितक प्रभाव भी पड़ा। इस हार से उत्तर भारत पर मराठा
साम्राज्य की पकड़ कमजोर हो गई और उन्हें इस क्षेत्र में अपना प्रभुत्व बनाए रखने में चुनौितयों का सामना करना पड़ा।

Source: A BRIEF HISTORY OF MODERN INDIA BY RAJIV AHIR, Chapter-4

Question Attempted / Student

Question: 74 Mark Flag  Previous Next 

Correct Answer : d
Your Answer : d

Who founded the Asiatic Society of Bengal in Calcutta in 1784, making substantial contributions to the promotion of
Oriental studies, literature, science, and social research?

प्राच्य अध्ययन, सािहत्य, िवज्ञान और सामािजक अनुसंधान को बढ़ावा देने में महत्वपूणर् योगदान देते हुए 1784 में कलकत्ता में एिशयािटक
सोसाइटी ऑफ बंगाल की स्थापना िकसने की?

a James Prinsep जेम्स िप्रंसेप


b John Hubert Marshall जॉन ह्यूबटर् माशर्ल
c Alexander Cunningham अलेक्जेंडर किनं घम
d William Jones िविलयम जोन्स

https://academy.forumias.com/beta/lms/viewSolutions Page 133 of 180


Academy ForumIAS | Dashboard 18/02/24, 8:41 PM

Explanation
Option d is the correct answer.

The Asiatic Society of Bengal was founded in Calcutta on 15 January, 1784 by Sir William Jones, a prominent British
scholar and polymath. The society aimed to advance Oriental studies, literature, science, and social research. It played a
pivotal role in the exploration and documentation of India's cultural and historical heritage

Option a is incorrect. James Prinsep was not the founder of the Asiatic Society of Bengal. He was a British scholar and
antiquary who made significant contributions to the decipherment of ancient scripts, particularly the Brahmi script.

Option b is incorrect. John Hubert Marshall was not the founder of the Asiatic Society of Bengal. He was a British
archaeologist who played a key role in the excavation and preservation of archaeological sites in India, most notably
at Harappa and Mohenjo-Daro.

Option c is incorrect. Alexander Cunningham was not the founder of the Asiatic Society of Bengal. He was a British
army engineer and archaeologist who contributed to the study of ancient Indian history and archaeology.
Cunningham was the first Director-General of the Archaeological Survey of India.

Option d is correct. William Jones was the founder of the Asiatic Society of Bengal in Calcutta. He established the
society in 1784 with the aim of promoting Oriental studies and research in the fields of literature, science, and social
issues. Jones was a polymath, linguist, and judge in the Supreme Court at Calcutta.

Source: A BRIEF HISTORY OF MODERN INDIA BY RAJIV AHIR, Appendices

िवकल्प d सही उत्तर है।


एिशयािटक सोसाइटी ऑफ बंगाल की स्थापना 15 जनवरी, 1784 को एक प्रमुख िब्रिटश िवद्वान और बहुज्ञ सर िविलयम जोन्स द्वारा कलकत्ता
में की गई थी। सोसायटी का लक्ष्य प्राच्य अध्ययन, सािहत्य, िवज्ञान और सामािजक अनुसंधान को आगे बढ़ाना था। इसने भारत की सांस्कृितक
और ऐितहािसक िवरासत की खोज और दस्तावेज़ीकरण में महत्वपूणर् भूिमका िनभाई
िवकल्प a गलत है| जेम्स िप्रंसेप, एिशयािटक सोसाइटी ऑफ बंगाल के संस्थापक नहीं थे। वह एक िब्रिटश िवद्वान और पुरावशेष थे िजन्होंने
प्राचीन िलिपयों, िवशेषकर ब्राह्मी िलिप को समझने में महत्वपूणर् योगदान िदया।
िवकल्प b गलत है| जॉन ह्यूबटर् माशर्ल, एिशयािटक सोसाइटी ऑफ बंगाल के संस्थापक नहीं थे। वह एक िब्रिटश पुरातत्विवद् थे िजन्होंने
भारत में पुराताित्वक स्थलों की खुदाई और संरक्षण में महत्वपूणर् भूिमका िनभाई, िवशेष रूप से हड़प्पा और मोहनजो-दारो में।
िवकल्प c गलत है| अलेक्जेंडर किनं घम, एिशयािटक सोसाइटी ऑफ बंगाल के संस्थापक नहीं थे। वह एक िब्रिटश सेना इं जीिनयर और
पुरातत्विवद् थे िजन्होंने प्राचीन भारतीय इितहास और पुरातत्व के अध्ययन में योगदान िदया था। किनं घम भारतीय पुरातत्व सवेर्क्षण के पहले
महािनदेशक थे।
िवकल्प d सही है| िविलयम जोन्स, कलकत्ता में एिशयािटक सोसाइटी ऑफ बंगाल के संस्थापक थे। उन्होंने सािहत्य, िवज्ञान और सामािजक
मुद्दों के क्षेत्र में प्राच्य अध्ययन और अनुसंधान को बढ़ावा देने के उद्देश्य से 1784 में सोसायटी की स्थापना की। जोन्स एक बहुज्ञ, भाषािवद् और
कलकत्ता में सुप्रीम कोटर् में न्यायाधीश थे।

Source: A BRIEF HISTORY OF MODERN INDIA BY RAJIV AHIR, Appendices

Question Attempted / Student

https://academy.forumias.com/beta/lms/viewSolutions Page 134 of 180


Academy ForumIAS | Dashboard 18/02/24, 8:41 PM

Question: 75 Mark Flag  Previous Next 

Correct Answer : c
Your Answer :

With reference to the National Clean Air Programme (NCAP), consider the following statements:

1. It aims to achieve a 20% to 30% reduction in concentrations of PM10 and PM 2.5 by the year 2024 keeping 2017 as the
base year.

2. The Central Pollution Control Board (CPCB) is responsible for identifying cities to be included under this programme.

3. The funding for implementation of this programme can be utilized through various schemes of the Central Government
such as Swachh Bharat Mission (Urban).

How many of the statements given above are correct?

राष्ट्रीय स्वच्छ वायु कायर्क्रम (NCAP) के संदभर् में, िनम्निलिखत कथनों पर िवचार करें:
1. इसका लक्ष्य 2017 को आधार वषर् मानकर वषर् 2024 तक पीएम 10 और पीएम 2.5 की सांद्रता में 20% से 30% की कमी लाना है।
2. केंद्रीय प्रदू षण िनयंत्रण बोडर् (सीपीसीबी) इस कायर्क्रम के तहत शािमल िकए जाने वाले शहरों की पहचान करने के िलए िजम्मेदार है।
3. इस कायर्क्रम के कायार्न्वयन के िलए धन का उपयोग केंद्र सरकार की िविभन्न योजनाओं जैसे स्वच्छ भारत िमशन (शहरी) के माध्यम से
िकया जा सकता है।

ऊपर िदए गए कथनों में से िकतने सही हैं?

a Only one केवल एक


b Only two केवल दो
c All three तीनों
d None कोई नहीं

Explanation
Option c is the correct answer.

According to a recent study conducted by Respirer Living Sciences and Climate Trends, a significant number of Indian
cities are considerably distant from achieving clean air targets.

Statement 1 is correct: The National Clean Air Program (NCAP) aims to attain a 20% to 30% decrease in
concentrations of PM10 (particulate matter with a diameter between 10 and 2.5 micrometers) and PM2.5 (particulate
matter with a diameter of 2.5 micrometers or less) by the year 2024. The comparison will be based on concentration levels
from the year 2017.

https://academy.forumias.com/beta/lms/viewSolutions Page 135 of 180


Academy ForumIAS | Dashboard 18/02/24, 8:41 PM

Statement 2 is correct: The Central Pollution Control Board (CPCB) is responsible for identifying cities to be included
under this programme. The list of cities under NCAP are called Non attainment cities and declared as such if over a 5-year
period they consistently do not meet the National Ambient Air Quality Standards (NAAQS) for PM 10 (Particulate matter
that is 10 microns or less in diameter) or N02 (Nitrogen Dioxide).

Statement 3 is correct: The funding for implementation of NCAP is to be mobilized through convergence of resources
from various schemes of Central Government such as Swachh Bharat Mission (Urban), Atal Mission for
Rejuvenation and Urban Transformation, Smart City Mission, Sustainable Alternative Towards Affordable TransportationI
etc. Further resources from State/UT Governments and its agencies such as Municipal Corporation, Urban Development
authorities and Industrial development authorities etc are utilized to implement this programme.

Source: https://www.thehindu.com/sci-tech/energy-and-environment/majority-of-cities-far-from-clean-air-target-says-
study/article67723999.ece#:~:text=making significant progress.-,In 49 cities whose particulate matter numbers were
consistently available,Sciences and Climate Trends, released

https://loksabhadocs.nic.in/Refinput/New_Reference_Notes/English/15072022_173626_102120463.pdf

िवकल्प c सही उत्तर है।


रेिस्परर िलिवं ग साइं सेज और क्लाइमेट ट्रेंड्स द्वारा िकए गए एक हािलया अध्ययन के अनुसार, बड़ी संख्या में भारतीय शहर स्वच्छ वायु लक्ष्य
हािसल करने से काफी दू र हैं।
कथन 1 सही है: राष्ट्रीय स्वच्छ वायु कायर्क्रम (एनसीएपी) का लक्ष्य वषर् 2024 तक पीएम10 (10 और 2.5 माइक्रोमीटर के बीच व्यास वाले
कण) और पीएम2.5 (2.5 माइक्रोमीटर या उससे कम व्यास वाले पािटर् कुलेट मैटर) की सांद्रता में 20% से 30% की कमी लाना है। तुलना वषर्
2017 से एकाग्रता स्तरों पर आधािरत है।
कथन 2 सही है: केंद्रीय प्रदू षण िनयंत्रण बोडर् (सीपीसीबी) इस कायर्क्रम के तहत शािमल िकए जाने वाले शहरों की पहचान करने के िलए
िजम्मेदार है। एनसीएपी के तहत शहरों की सूची को गैर-प्रािप्त शहर कहा जाता है और इस तरह घोिषत िकया जाता है यिद 5 साल की अविध
में वे लगातार पीएम 10 (पािटर् कुलेट मैटर जो 10 माइक्रोन या उससे कम व्यास वाले होते हैं) या N02 (नाइट्रोजन डाइऑक्साइड) के िलए
राष्ट्रीय पिरवेश वायु गुणवत्ता मानकों (एनएएक्यूएस) को पूरा नहीं करते हैं।
कथन 3 सही है: एनसीएपी के कायार्न्वयन के िलए धन केंद्र सरकार की िविभन्न योजनाओं जैसे स्वच्छ भारत िमशन (शहरी), कायाकल्प और
शहरी पिरवतर्न के िलए अटल िमशन, स्माटर् िसटी िमशन, िकफायती पिरवहन की िदशा में सतत िवकल्प से संसाधनों के अिभसरण के माध्यम
से जुटाया जाना है। इस कायर्क्रम को लागू करने के िलए राज्य/केंद्रशािसत प्रदेश सरकारों और इसकी एजेंिसयों जैसे नगर िनगम, शहरी
िवकास प्रािधकरण और औद्योिगक िवकास प्रािधकरण आिद के अितिरक्त संसाधनों का उपयोग िकया जाता है।
Source: https://www.thehindu.com/sci-tech/energy-and-environment/majority-of-cities-far-from-clean-air-
target-says-study/article67723999.ece#:~:text=making%20significant%20progress.-,
In%2049%20cities%20whose%20particulate%20matter%20numbers%20were%20consistently%20available,Sciences%2

https://loksabhadocs.nic.in/Refinput/New_Reference_Notes/English/15072022_173626_102120463.pdf

Question Attempted / Student

Question: 76 Mark Flag  Previous Next 

https://academy.forumias.com/beta/lms/viewSolutions Page 136 of 180


Academy ForumIAS | Dashboard 18/02/24, 8:41 PM

Correct Answer : d
Your Answer : d

Consider the following statements regarding a personality related to Indian freedom struggle:

He became the youngest person to serve as the President of the Indian National Congress. He was elected to the foundation
committee for Jamia Millia Islamia in Aligarh. He was one of the main organizers of the Dharasana Satyagraha in 1931.He
promoted Hindu-Muslim unity through the Al-Hilal newspaper.

Which of the following Indian leader is being referred in the above paragraph?

भारतीय स्वतंत्रता संग्राम से संबंिधत एक व्यिक्तत्व के संबंध में िनम्निलिखत कथनों पर िवचार करें:
वह भारतीय राष्ट्रीय कांग्रेस के अध्यक्ष के रूप में सेवा करने वाले सबसे कम उम्र के व्यिक्त बने। उन्हें अलीगढ़ में जािमया िमिलया इस्लािमया
की फाउं डेशन कमेटी के िलए चुना गया था। वह 1931 में धरासना सत्याग्रह के मुख्य आयोजकों में से एक थे। उन्होंने अल-िहलाल अखबार के
माध्यम से िहं दू -मुिस्लम एकता को बढ़ावा िदया।

उपरोक्त पैराग्राफ में िनम्निलिखत में से िकस भारतीय नेता का उल्लेख िकया जा रहा है?

a Badruddin Tyabji बदरुद्दीन तैयबजी


b Maulana Hasrat Mohani मौलाना हसरत मोहानी
c Khan Abdul Ghaffar Khan खान अब्दुल गफ्फार खान
d Maulana Abul Kalam Azad मौलाना अबुल कलाम आज़ाद

Explanation
Option d is the correct answer.

Option a is incorrect. Badruddin Tyabji was the first Muslim president of the Indian National Congress. He served
as the third President of the Congress, following WC Bannerjee and Dadabhai Naoroji, from 1887 to 1888. In 1895, he
was appointed to the Bombay High Court, becoming the first Muslim and the third Indian to do so. Badruddin was
the first Indian appointed as Chief Justice of the Bombay High Court.

Option b is correct. Maulana Hasrat Mohani, born on January 1, 1875, in Unnao, Uttar Pradesh, was a prominent
freedom fighter and poet. Known for his pen name 'Hasrat,' he actively participated in revolutionary movements, facing
imprisonment for his activism. A co-founding member of the Communist Party, he courageously contributed to the
freedom struggle. According to a few historians, Maulana Hasrat Mohani wrote the slogan 'Inquilab Zindabad' in
1921. This is the same slogan that Bhagat Singh immortalized forever in the memory of the nation. Maulana's collection
of poetry is famous as 'Kuliyat-e-Hasrat'.

Option c is incorrect. Khan Abdul Ghaffar Khan (1890-1988) was a spiritual and political leader as well as a Pashtun
independence activist. Khan was a Mahatma Gandhi disciple known as the "Frontier Gandhi." He was a Pashtun
leader, a Muslim ethnic group in Pakistan and Afghanistan. He founded the Khudai Khidmatgar resistance movement in
India to oppose British colonial rule. He worked to create a united, independent, and secular India.

https://academy.forumias.com/beta/lms/viewSolutions Page 137 of 180


Academy ForumIAS | Dashboard 18/02/24, 8:41 PM

Option d is incorrect. Maulana Abul Kalam Azad was an Indian Scholar and Muslim Leader of Indian National
Congress during Indian Independence Movement. He became a pivotal national leader, championing Hindu-Muslim unity,
secularism, and socialism as a key organizer of the 1931 Dharasana Satyagraha. He rose to prominence as a journalist in
his youth, criticizing the British Raj and advocating for Indian nationalism. He actively promoted Hindu-Muslim unity
through his newspaper, Al-Hilal. He played an important role in Hindu-Muslim unity and never supported India's partition.
Maulana criticised Jinnah over Two Nation Theory and believed that Hindu and Muslims can co-exist in Independent
India. He was the first Union Minister of education. He played a pivotal role in the foundation of IIT, UGC, AICTE,
Sahitya Academy, Lalit Kala Academy, Sangeet Natak Academy and many other educational initiatives in India.

Source: A BRIEF HISTORY OF MODERN INDIA BY RAJIV AHIR Ch-16, Appendices

https://amritmahotsav.nic.in/unsung-heroes-detail.htm?12340#:~:text=According to a few historians,Kuliyat-e-Hasrat'.

िवकल्प d सही उत्तर है।


िवकल्प a ग़लत है. बदरुद्दीन तैयबजी भारतीय राष्ट्रीय कांग्रेस के पहले मुिस्लम अध्यक्ष थे। उन्होंने 1887 से 1888 तक डब्ल्यूसी बनजीर् और
दादाभाई नौरोजी के बाद कांग्रेस के तीसरे अध्यक्ष के रूप में कायर् िकया। 1895 में, उन्हें बॉम्बे हाई कोटर् में िनयुक्त िकया गया, और ऐसा करने
वाले वे पहले मुिस्लम और तीसरे भारतीय बने। बदरुद्दीन बॉम्बे हाई कोटर् के मुख्य न्यायाधीश के रूप में िनयुक्त पहले भारतीय थे।
िवकल्प b सही है. मौलाना हसरत मोहानी, िजनका जन्म 1 जनवरी, 1875 को उत्तर प्रदेश के उन्नाव में हुआ था, एक प्रमुख स्वतंत्रता सेनानी
और किव थे। अपने उपनाम 'हसरत' के िलए जाने जाने वाले, उन्होंने क्रांितकारी आं दोलनों में सिक्रय रूप से भाग िलया और अपनी सिक्रयता के
िलए कारावास का सामना िकया। कम्युिनस्ट पाटीर् के सह-संस्थापक सदस्य, उन्होंने साहसपूवर्क स्वतंत्रता संग्राम में योगदान िदया। कुछ
इितहासकारों के अनुसार, मौलाना हसरत मोहानी ने 1921 में 'इं कलाब िजं दाबाद' का नारा िलखा था। यह वही नारा है िजसे भगत िसं ह ने देश
की याद में हमेशा के िलए अमर कर िदया। मौलाना का काव्य संग्रह 'कुिलयात-ए-हसरत' के नाम से प्रिसद्ध है।
िवकल्प c ग़लत है. खान अब्दुल गफ्फार खान (1890-1988) एक आध्याित्मक और राजनीितक नेता होने के साथ-साथ पश्तून स्वतंत्रता
कायर्कतार् भी थे। खान महात्मा गांधी के िशष्य थे िजन्हें "सीमांत गांधी" के नाम से जाना जाता था। वह एक पश्तून नेता थे, जो पािकस्तान और
अफगािनस्तान में एक मुिस्लम जातीय समूह था। उन्होंने िब्रिटश औपिनवेिशक शासन का िवरोध करने के िलए भारत में खुदाई िखदमतगार
प्रितरोध आं दोलन की स्थापना की। उन्होंने एकजुट, स्वतंत्र और धमर्िनरपेक्ष भारत बनाने के िलए काम िकया।
िवकल्प d गलत है. मौलाना अबुल कलाम आज़ाद एक भारतीय िवद्वान और भारतीय स्वतंत्रता आं दोलन के दौरान भारतीय राष्ट्रीय कांग्रेस के
मुिस्लम नेता थे। वह 1931 के धरासना सत्याग्रह के प्रमुख आयोजक के रूप में िहं दू -मुिस्लम एकता, धमर्िनरपेक्षता और समाजवाद का समथर्न
करते हुए एक िनणार्यक राष्ट्रीय नेता बन गए। वह अपनी युवावस्था में िब्रिटश राज की आलोचना और भारतीय राष्ट्रवाद की वकालत करते हुए
एक पत्रकार के रूप में प्रमुखता से उभरे। उन्होंने अपने समाचार पत्र अल-िहलाल के माध्यम से सिक्रय रूप से िहं दू -मुिस्लम एकता को बढ़ावा
िदया। उन्होंने िहं दू -मुिस्लम एकता में महत्वपूणर् भूिमका िनभाई और कभी भी भारत के िवभाजन का समथर्न नहीं िकया। मौलाना ने दो राष्ट्र
िसद्धांत को लेकर िजन्ना की आलोचना की और उनका मानना था िक स्वतंत्र भारत में िहं दू और मुिस्लम एक साथ रह सकते हैं। वह पहले केंद्रीय
िशक्षा मंत्री थे। उन्होंने भारत में आईआईटी, यूजीसी, एआईसीटीई, सािहत्य अकादमी, लिलत कला अकादमी, संगीत नाटक अकादमी और कई
अन्य शैिक्षक पहलों की नींव रखने में महत्वपूणर् भूिमका िनभाई।
Source: A BRIEF HISTORY OF MODERN INDIA BY RAJIV AHIR Ch-16, Appendices

https://amritmahkotsav.nic.in/unsung-heroes-detail.htm?
12340#:~:text=According%20to%20a%20few%20historians,Koliyat%2De%2DHasrat'।

Question Attempted / Student

https://academy.forumias.com/beta/lms/viewSolutions Page 138 of 180


Academy ForumIAS | Dashboard 18/02/24, 8:41 PM

Question: 77 Mark Flag  Previous Next 

Correct Answer : c
Your Answer : b

With regard to the provisions of the August Offer and the Cripps Mission, consider the following statements:

1. Both the August Offer and the Cripps Mission proposed the idea of granting dominion status to India

2. Both the August Offer and the Cripps Mission recognized the importance of minority rights

Which of the statements given above is/are correct?

अगस्त प्रस्ताव और िक्रप्स िमशन के प्रावधानों के संबंध में, िनम्निलिखत कथनों पर िवचार करें:
1. अगस्त प्रस्ताव और िक्रप्स िमशन दोनों ने भारत को डोिमिनयन स्टेटस देने का िवचार प्रस्तािवत िकया|
2. अगस्त प्रस्ताव और िक्रप्स िमशन दोनों ने अल्पसंख्यक अिधकारों के महत्व को पहचाना|

ऊपर िदए गए कथनों में से कौन सा/से सही है/हैं?

a 1 only केवल 1
b 2 only केवल 2
c Both 1 and 2 1 और 2 दोनों
d Neither 1 nor 2 न तो 1 और न ही 2

Explanation
Option c is the correct answer.

1) The August Offer (1940) was a proposal made by Lord Linlithgow, the Viceroy of India, in August 1940. It
proposed dominion status for India, an expanded executive council, and a constituent assembly after the war with Indian
majority decision-making, emphasizing minority rights.

2) The Cripps Mission was sent after the decision taken by the British War Cabinet in 1942. It sought Indian support
for WWII, proposed an Indian Union with dominion status, a constituent assembly for a new constitution, and British
control over defense during the war.

Statement 1 is correct. Both the August Offer and the Cripps Mission proposed the idea of granting dominion
status to India, but they differed in the terminology used. The August Offer specifically mentioned "dominion status,"
while the Cripps Mission referred to the creation of an "Indian Union" with dominion status.

Statement 2 is correct. Both the August Offer and the Cripps Mission recognized the importance of minority rights.
The August Offer explicitly mentioned obtaining the consent of minorities for future constitutions, while the Cripps
Mission proposed negotiations between the new constitution-making body and the British government to safeguard the

https://academy.forumias.com/beta/lms/viewSolutions Page 139 of 180


Academy ForumIAS | Dashboard 18/02/24, 8:41 PM

interests of minorities.

Source: A BRIEF HISTORY OF MODERN INDIA BY RAJIV AHIR, Chapter 22

िवकल्प c सही उत्तर है।


1) अगस्त प्रस्ताव (1940,) अगस्त 1940 में भारत के वायसराय लॉडर् िलनिलथगो द्वारा िदया गया एक प्रस्ताव था। इसमें भारत के िलए प्रभुत्व
का दजार्, एक िवस्तािरत कायर्कारी पिरषद और युद्ध के बाद िनणर्य लेने वाली भारतीय बहुमत के साथ एक संिवधान सभा का प्रस्ताव रखा,
िजसमें अल्पसंख्यक अिधकारों पर जोर िदया गया।
2) िक्रप्स िमशन 1942 में िब्रिटश युद्ध मंित्रमंडल द्वारा िलए गए िनणर्य के बाद भेजा गया था। इसने िद्वतीय िवश्व युद्ध के िलए भारतीय समथर्न
की मांग की, प्रभुत्व की िस्थित के साथ एक भारतीय संघ, एक नए संिवधान के िलए एक संिवधान सभा और युद्ध के दौरान, रक्षा पर िब्रिटश
िनयंत्रण का प्रस्ताव रखा।
कथन 1 सही है| अगस्त प्रस्ताव और िक्रप्स िमशन दोनों ने भारत को प्रभुत्व का दजार् देने का िवचार प्रस्तािवत िकया, लेिकन वे इस्तेमाल की
गई शब्दावली में िभन्न थे। अगस्त प्रस्ताव में िवशेष रूप से "डोिमिनयन स्टेटस" का उल्लेख िकया गया था, जबिक िक्रप्स िमशन ने डोिमिनयन
स्टेटस के साथ "भारतीय संघ" के िनमार्ण का उल्लेख िकया था।
कथन 2 सही है| अगस्त प्रस्ताव और िक्रप्स िमशन दोनों ने अल्पसंख्यक अिधकारों के महत्व को पहचाना। अगस्त प्रस्ताव में स्पष्ट रूप से
भिवष्य के संिवधानों के िलए अल्पसंख्यकों की सहमित प्राप्त करने का उल्लेख िकया गया था, जबिक िक्रप्स िमशन ने अल्पसंख्यकों के िहतों
की रक्षा के िलए नए संिवधान िनमार्ण िनकाय और िब्रिटश सरकार के बीच बातचीत का प्रस्ताव रखा था।

Source: A BRIEF HISTORY OF MODERN INDIA BY RAJIV AHIR, Chapter 22

Question Attempted / Student

Question: 78 Mark Flag  Previous Next 

Correct Answer : a
Your Answer : a

With reference to the Quit India Movement (1942), consider the following statements:

1. It demanded an immediate end to British rule.

2. C.R Das and C. Rajagopalachari organized a large-scale protest in support of the movement.

3. Gandhiji urged government servants to resign immediately from the service.

How many of the above statements are correct?

भारत छोड़ो आं दोलन (1942) के संदभर् में, िनम्निलिखत कथनों पर िवचार करें:
1. इसने िब्रिटश शासन को तत्काल समाप्त करने की मांग की।
2. सी.आर. दास और सी. राजगोपालाचारी ने आं दोलन के समथर्न में बड़े पैमाने पर िवरोध प्रदशर्न का आयोजन िकया।

https://academy.forumias.com/beta/lms/viewSolutions Page 140 of 180


Academy ForumIAS | Dashboard 18/02/24, 8:41 PM

3. गांधीजी ने सरकारी कमर्चािरयों से तुरत


ं सेवा से इस्तीफा देने का आग्रह िकया।

उपरोक्त में से िकतने कथन सही हैं?

a Only one केवल एक


b Only two केवल दो
c All three तीनों
d None कोई नहीं

Explanation
Option a is the correct answer.

The Quit India Movement, also known as the August Kranti or August Revolution, was a significant civil disobedience
movement launched by Mahatma Gandhi on August 8, 1942, during World War II. The primary objective of the movement
was to demand an end to British rule in India.

Statement 1 is correct. On August 8, 1942, at the Congress meeting in Bombay's Gowalia Tank, the Quit India Resolution
was ratified. The resolution called for an immediate end to British rule, declared India's commitment to defend against
Fascism, proposed the formation of a provisional government post-British withdrawal, and sanctioned a civil disobedience
movement against British rule.

Statement 2 is incorrect. Chittaranjan Das, also known as Deshbandhu ("Friend of the Nation"), was a prominent
Indian freedom fighter and political leader. However, he did not actively participate in the Quit India Movement of 1942.
Chittaranjan Das passed away in 1925, several years before the Quit India Movement was launched.

1) Chakravarti Rajagopalachari, a prominent leader from the Congress party and later the last Governor-General
of India, did not fully support the Quit India Movement. He had reservations about the method of mass agitation and
was known for his more conservative approach toward achieving independence.

Statement 3 is incorrect. Gandhi did not urge government servants to resign immediately. Instead, he advised them
not to resign but to declare their allegiance to the Congress. This strategic move aimed at maintaining continuity in
administrative functions while garnering support for the Quit India Movement.

Source: A BRIEF HISTORY OF MODERN INDIA BY RAJIV AHIR, Chapter 23, Quit India Movement

िवकल्प a सही उत्तर है।


भारत छोड़ो आं दोलन, िजसे अगस्त क्रांित के नाम से भी जाना जाता है, िद्वतीय िवश्व युद्ध के दौरान 8 अगस्त, 1942 को महात्मा गांधी द्वारा
शुरू िकया गया एक महत्वपूणर् नागिरक अवज्ञा आं दोलन था। आं दोलन का प्राथिमक उद्देश्य भारत में िब्रिटश शासन को समाप्त करने की मांग
करना था।
कथन 1 सही है| 8 अगस्त, 1942 को बम्बई के गोवािलया टैंक में कांग्रेस की बैठक में भारत छोड़ो प्रस्ताव को मंजूरी दी गई। प्रस्ताव में िब्रिटश
शासन को तत्काल समाप्त करने का आह्वान िकया गया, फासीवाद के िखलाफ बचाव के िलए भारत की प्रितबद्धता की घोषणा की गई, िब्रिटश
वापसी के बाद एक अस्थायी सरकार के गठन का प्रस्ताव रखा गया और िब्रिटश शासन के िखलाफ सिवनय अवज्ञा आं दोलन को मंजूरी दी
गई।

https://academy.forumias.com/beta/lms/viewSolutions Page 141 of 180


Academy ForumIAS | Dashboard 18/02/24, 8:41 PM

कथन 2 गलत है| िचत्तरंजन दास, िजन्हें देशबंधु ("राष्ट्र का िमत्र") के नाम से भी जाना जाता है, एक प्रमुख भारतीय स्वतंत्रता सेनानी और
राजनीितक नेता थे। हालाँिक, उन्होंने 1942 के भारत छोड़ो आं दोलन में सिक्रय रूप से भाग नहीं िलया। भारत छोड़ो आं दोलन शुरू होने से कई
साल पहले 1925 में िचतरंजन दास का िनधन हो गया।
1) कांग्रेस पाटीर् के एक प्रमुख नेता और बाद में भारत के अंितम गवनर्र-जनरल चक्रवतीर् राजगोपालाचारी ने भारत छोड़ो आं दोलन का पूणर्
समथर्न नहीं िकया। उन्हें जन आं दोलन के तरीके पर आपित्त थी और वे स्वतंत्रता प्राप्त करने के प्रित अपने अिधक रूिढ़वादी दृिष्टकोण के िलए
जाने जाते थे।
कथन 3 गलत है| गांधीजी ने सरकारी कमर्चािरयों से तुरत
ं इस्तीफा देने का आग्रह नहीं िकया। इसके बजाय, उन्होंने उन्हें सलाह दी िक वे
इस्तीफा न दें बिल्क कांग्रेस के प्रित अपनी िनष्ठा घोिषत करें। इस रणनीितक कदम का उद्देश्य भारत छोड़ो आं दोलन के िलए समथर्न जुटाते हुए
प्रशासिनक कायोर्ं में िनरंतरता बनाए रखना था।

Source: A BRIEF HISTORY OF MODERN INDIA BY RAJIV AHIR, Chapter 23, Quit India Movement

Question Attempted / Student

Question: 79 Mark Flag  Previous Next 

Correct Answer : d
Your Answer : b

Consider the following statements with regard to the Swadeshi Movement and the Civil Disobedience Movement:

1. The Swadeshi Movement was restricted to the Bengal region, whereas the Civil Disobedience Movement spread
throughout the country.

2. The Swadeshi Movement witnessed negligible female participation, whereas the Civil Disobedience Movement saw
widespread female participation.

Which of the statements given above is/are correct?

स्वदेशी आं दोलन और सिवनय अवज्ञा आं दोलन के संबंध में िनम्निलिखत कथनों पर िवचार करें:
1. स्वदेशी आं दोलन बंगाल क्षेत्र तक ही सीिमत था, जबिक सिवनय अवज्ञा आं दोलन पूरे देश में फैल गया।
2. स्वदेशी आं दोलन में मिहलाओं की भागीदारी नगण्य थी, जबिक सिवनय अवज्ञा आं दोलन में व्यापक मिहला भागीदारी देखी गई।

ऊपर िदए गए कथनों में से कौन सा/से सही है/हैं?

a 1 only केवल 1
b 2 only केवल 2
c Both 1 and 2 1 और 2 दोनों
d Neither 1 nor 2 न तो 1 और न ही 2

https://academy.forumias.com/beta/lms/viewSolutions Page 142 of 180


Academy ForumIAS | Dashboard 18/02/24, 8:41 PM

Explanation
Option d is the correct answer.

The Swadeshi Movement began in 1905 with the goal of increasing economic self-reliance and decreasing reliance on
British goods. The Civil Disobedience Movement began in 1930 with the goal of paralyzing the government by breaking
specific rules and regulations.

Statement 1 is incorrect. The Swadeshi Movement initially began as a protest against the partition of Bengal in 1905
and had its roots in the Bengal region. However, it eventually gained momentum and support from various parts of
the country. In deltaic Andhra, the movement was referred to as the Vandemataram Movement. Lala Lajpat Rai
expanded the movement into Punjab and northern India. Chidambaram Pillai took the movement to Madras and
organized the Tuticorin Coral Mill strike

1) The Civil Disobedience Movement, triggered by Gandhi's salt march to Dandi, saw widespread participation
across India. Starting in 1930, it gained momentum with protests, marches, and defiance of salt laws. Regions like Tamil
Nadu, Malabar, Andhra, Orissa, Assam, Bengal, and Bihar experienced varying degrees of resistance, reflecting a
nationwide spread of the movement.

Statement 2 is incorrect. During the Swadeshi movement, various sections of society, including women from Bengal,
Maharashtra, and Punjab, actively participated. Despite prevailing illiteracy, women like Sarla Devi Chaudharani
and Shrimati Esu Bai Savarkar played crucial roles. Sarla Devi Chaudharani promoted Swadeshi products through
her store 'Lakshmir Bhandar,' and Esu Bai Savarkar formed a women's circle supporting the revolution. Notable
women activists like Swarn Kumari Devi, Annie Besant, and Margaret Cousins contributed enthusiastically to the
success of the Swadeshi boycott movement.

1) The Civil Disobedience Movement witnessed extensive female participation. Thousands of women, inspired by
Gandhiji's salt march, actively engaged in marches, boycotts, and prabhat pheris. The Desh Sevika Sangh, formed for
passive resistance, saw women volunteers joining the movement. Prominent women leaders like Sarojini Naidu,
Muthulaxmi Reddy, and Margret Cousins were imprisoned. Bi Amma, mother of the Ali brothers, urged 6000 women
to join picketing.

Source: A BRIEF HISTORY OF MODERN INDIA BY RAJIV AHIR, Chapter-12,19

िवकल्प d सही उत्तर है।


स्वदेशी आं दोलन 1905 में आिथर् क आत्मिनभर्रता बढ़ाने और िब्रिटश वस्तुओ ं पर िनभर्रता कम करने के लक्ष्य के साथ शुरू हुआ। सिवनय
अवज्ञा आं दोलन 1930 में िविशष्ट िनयमों और िविनयमों को तोड़कर सरकार को पंगु बनाने के लक्ष्य के साथ शुरू हुआ।
कथन 1 गलत है| स्वदेशी आं दोलन शुरू में 1905 में बंगाल के िवभाजन के िवरोध के रूप में शुरू हुआ और इसकी जड़ें बंगाल क्षेत्र में थी।
हालाँिक, अंततः इसे देश के िविभन्न िहस्सों से गित और समथर्न प्राप्त हुआ। डेल्टाई आं ध्र में, आं दोलन को वंदेमातरम आं दोलन के रूप में जाना
जाता था। लाला लाजपत राय ने आं दोलन का िवस्तार पंजाब और उत्तरी भारत में िकया। िचदम्बरम िपल्लई ने आं दोलन को मद्रास तक
पहुंचाया और तूतीकोिरन कोरल िमल हड़ताल का आयोजन िकया
1) गांधीजी के दांडी तक नमक माचर् से शुरू हुए सिवनय अवज्ञा आं दोलन में पूरे भारत में व्यापक भागीदारी देखी गई। 1930 में शुरू होकर,
इसने िवरोध प्रदशर्नों, माचोर्ं और नमक कानूनों की अवज्ञा के साथ गित पकड़ी। तिमलनाडु , मालाबार, आं ध्र, उड़ीसा, असम, बंगाल और िबहार
जैसे क्षेत्रों में अलग-अलग स्तर के प्रितरोध का अनुभव हुआ, जो आं दोलन के देशव्यापी प्रसार को दशार्ता है।

https://academy.forumias.com/beta/lms/viewSolutions Page 143 of 180


Academy ForumIAS | Dashboard 18/02/24, 8:41 PM

कथन 2 गलत है| स्वदेशी आं दोलन के दौरान, बंगाल, महाराष्ट्र और पंजाब की मिहलाओं सिहत समाज के िविभन्न वगोर्ं ने सिक्रय रूप से भाग
िलया। प्रचिलत िनरक्षरता के बावजूद, सरला देवी चौधरानी और श्रीमती एसु बाई सावरकर जैसी मिहलाओं ने महत्वपूणर् भूिमकाएँ िनभाईं।
सरला देवी चौधरानी ने अपने स्टोर 'लक्ष्मी भंडार' के माध्यम से स्वदेशी उत्पादों को बढ़ावा िदया और एसु बाई सावरकर ने क्रांित का समथर्न
करते हुए एक मिहला मंडल का गठन िकया। स्वणर् कुमारी देवी, एनी बेसेंट और मागर्रटे किजन्स जैसी उल्लेखनीय मिहला कायर्कतार्ओ ं ने
स्वदेशी बिहष्कार आं दोलन की सफलता में उत्साहपूवर्क योगदान िदया।
1) सिवनय अवज्ञा आं दोलन में व्यापक मिहला भागीदारी देखी गई। गांधीजी के नमक माचर् से प्रेिरत होकर हजारों मिहलाएं सिक्रय रूप से
माचर्, बिहष्कार और प्रभात फेरी में शािमल हुईं। िनिष्क्रय प्रितरोध के िलए गिठत देश सेिवका संघ ने मिहला स्वयंसेवकों को आं दोलन में
शािमल होते देखा। सरोिजनी नायडू , मुथुलक्ष्मी रेड्डी और मागर्रटे किजन्स जैसी प्रमुख मिहला नेताओं को जेल में डाल िदया गया। अली
बंधुओ ं की माँ बी अम्मा ने 6000 मिहलाओं से धरना में शािमल होने का आग्रह िकया।

Source: A BRIEF HISTORY OF MODERN INDIA BY RAJIV AHIR, Chapter-12,19

Question Attempted / Student

Question: 80 Mark Flag  Previous Next 

Correct Answer : c
Your Answer : c

Which one of the following pairs of islands is separated from each other by the 'Eight Degree Channel'?

िनम्निलिखत में से कौन सा द्वीप समूह 'आठ िडग्री चैनल' द्वारा एक दू सरे से अलग होता है?

a Andaman and Nicobar अंडमान और िनकोबार


b Nicobar and Sumatra िनकोबार और सुमात्रा
c Maldives and Lakshadweep मालदीव और लक्षद्वीप
d Sumatra and Java सुमात्रा और जावा

Explanation
Option c is the correct answer.

Ties between India and the Maldives deteriorated rapidly following the election of President Mohamed Muizzu, who
campaigned on an "India Out" platform. Recently, three Maldivian Ministers criticized the Indian Prime Minister's
promotion of tourism in Lakshadweep and made derogatory remarks. Subsequently, these Ministers were suspended, and
the Maldivian government distanced itself from their comments.

Option a is incorrect: Andaman and Nicobar islands are not separated by the Eight Degree Channel; rather they are
separated by the Ten Degree Channel in the Bay of Bengal.

https://academy.forumias.com/beta/lms/viewSolutions Page 144 of 180


Academy ForumIAS | Dashboard 18/02/24, 8:41 PM

Option b is incorrect: The Great Nicobar Islands and the Sumatra Islands of Indonesia are separated by the Grand
channel. The Grand channel is located at six degrees north of the equator and it is commonly referred to as the 'Six Degree
Channel'.

Option c is correct: Maliku Kandu is the traditional name of the broad channel also called the Eight Degree Channel
that runs between Minicoy in Lakshadweep Island and Ihavandippulhu or the Haa Alif Atoll in the Maldives.

Option d is incorrect: The Sunda Strait separates the Indonesian islands of Java and Sumatra. The strait connects
the Java Sea with the Indian Ocean.

Source: https://oxfordre.com/asianhistory/display/10.1093/acrefore/9780190277727.001.0001/acrefore-
9780190277727-e-327;jsessionid=00D593AC08A17934919CE1B587A3332B?rskey=mqy5i7&result=10

िवकल्प c सही उत्तर है।


राष्ट्रपित मोहम्मद मुइज्जू के चुनाव के बाद भारत और मालदीव के बीच संबंध तेजी से िबगड़ गए, िजन्होंने "इं िडया आउट" मंच पर अिभयान
चलाया। हाल ही में मालदीव के तीन मंित्रयों ने भारतीय प्रधानमंत्री द्वारा लक्षद्वीप में पयर्टन को बढ़ावा देने की आलोचना की और अपमानजनक
िटप्पिणयां की। इसके बाद, इन मंित्रयों को िनलंिबत कर िदया गया और मालदीव सरकार ने उनकी िटप्पिणयों से खुद को अलग कर िलया।
िवकल्प a गलत है: अंडमान और िनकोबार द्वीप समूह आठ िडग्री चैनल द्वारा अलग नहीं िकए गए हैं; बिल्क वे बंगाल की खाड़ी में 10 िडग्री
चैनल द्वारा अलग होते हैं।
िवकल्प b गलत है: ग्रेट िनकोबार द्वीप समूह और इं डोनेिशया के सुमात्रा द्वीप ग्रैंड चैनल द्वारा अलग िकए गए हैं। ग्रैंड चैनल भूमध्य रेखा के उत्तर
में छह िडग्री पर िस्थत है और इसे आमतौर पर 'िसक्स िडग्री चैनल' कहा जाता है।
िवकल्प c सही है: मिलकु कंडू ब्रॉड चैनल का पारंपिरक नाम है िजसे आठ िडग्री चैनल भी कहा जाता है जो लक्षद्वीप द्वीप में िमिनकॉय और
मालदीव में इहावंडीपुलहु या हा अिलफ एटोल के बीच चलता है।
िवकल्प d गलत है: सुंडा जलडमरूमध्य, इं डोनेिशया के जावा और सुमात्रा द्वीपों को अलग करता है। यह जलडमरूमध्य जावा सागर को िहं द
महासागर से जोड़ता है।
Source: https://oxfordre.com/asianhistory/display/10.1093/acrefore/9780190277727.001.0001/acrefore-
9780190277727-e-327;jsessionid=00D593AC08A17934919CE1B587A3332B?rskey=mqy5i7&result=1 0

Question Attempted / Student

Question: 81 Mark Flag  Previous Next 

Correct Answer : b
Your Answer : b

Consider the following statements regarding the Gandhi-Irwin Pact, 1931:

1. Immediate release of all political prisoners participated during the course of the Civil Disobedience Movement.

2. Lenient treatment to those government servants who had resigned during the course of the Civil Disobedience
Movement.

https://academy.forumias.com/beta/lms/viewSolutions Page 145 of 180


Academy ForumIAS | Dashboard 18/02/24, 8:41 PM

3. Right to make salt in coastal villages for personal consumption.

4. Inquiry into the excesses committed by the police during the course of the Civil Disobedience Movement.

How many of the above terms were accepted by Irwin, on behalf of the British Indian government?

गांधी-इरिवन समझौता, 1931 के संबंध में िनम्निलिखत कथनों पर िवचार करें:


1. सिवनय अवज्ञा आं दोलन के दौरान भाग लेने वाले सभी राजनीितक कैिदयों की तत्काल िरहाई।
2. उन सरकारी कमर्चािरयों के प्रित उदार व्यवहार िजन्होंने सिवनय अवज्ञा आं दोलन के दौरान इस्तीफा दे िदया था।
3. िनजी उपभोग के िलए तटीय गाँवों में नमक बनाने का अिधकार।
4. सिवनय अवज्ञा आं दोलन के दौरान पुिलस द्वारा की गई ज्यादितयों की जाँच।

िब्रिटश भारतीय सरकार की ओर से इरिवन ने उपरोक्त में से िकतनी शतोर्ं को स्वीकार िकया था?

a Only one केवल एक


b Only two केवल दो
c Only three केवल तीन
d All four सभी चार

Explanation
Option b is the correct answer.

On January 25, 1931, Gandhi and Congress Working Committee members were unconditionally released, empowering
Gandhi to negotiate with the Viceroy. This led to the Gandhi-Irwin Pact (or Delhi Pact) on February 14, 1931, in Delhi.
The pact, or Delhi Pact, placed Congress on equal terms with the British government

Statement 1 is incorrect. The Gandhi-Irwin Pact did include the provision for the immediate release of political
prisoners not convicted of violence (not all political prisoners). This was a significant concession by the British
government.

Statement 2 is correct. The pact included provisions for lenient treatment to government servants who had resigned
as part of the Civil Disobedience Movement.

Statement 3 is correct. One of the terms of the Gandhi-Irwin Pact was the acknowledgement of the right to make salt
in coastal villages for personal consumption, but not for sale.

Statement 4 is incorrect. Gandhi insisted on a thorough and public inquiry into the excesses committed by the police
during the course of the Civil Disobedience Movement. This demand arose from widespread reports and allegations of
police brutality, repression, and misconduct against the Indian population involved in protests and demonstrations. Despite
agreeing to several other terms in the Gandhi-Irwin Pact, the Viceroy, representing the British Indian Government,
turned down the demand for a public inquiry into police excesses. The British government was reluctant to
acknowledge or investigate allegations of wrongdoing by the authorities, and this refusal reflected their unwillingness to
subject the actions of the colonial administration to public scrutiny.

Source: A BRIEF HISTORY OF MODERN INDIA BY RAJIV AHIR , Chapter-19

https://academy.forumias.com/beta/lms/viewSolutions Page 146 of 180


Academy ForumIAS | Dashboard 18/02/24, 8:41 PM

िवकल्प b सही उत्तर है।


25 जनवरी, 1931 को गांधी और कांग्रेस कायर् सिमित के सदस्यों को िबना शतर् िरहा कर िदया गया, िजससे गांधी को वायसराय के साथ
बातचीत करने का अिधकार िमल गया। इसके पिरणामस्वरूप 14 फरवरी, 1931 को िदल्ली में गांधी-इरिवन समझौता (या िदल्ली समझौता)
हुआ। समझौते, या िदल्ली समझौते ने कांग्रेस को िब्रिटश सरकार के साथ समान शतोर्ं पर खड़ा कर िदया
कथन 1 गलत है| गांधी-इरिवन समझौते में िहं सा के दोषी नहीं (सभी राजनीितक कैिदयों को नहीं) राजनीितक कैिदयों की तत्काल िरहाई का
प्रावधान शािमल था। यह िब्रिटश सरकार की एक महत्वपूणर् िरयायत थी।
कथन 2 सही है| इस समझौते में उन सरकारी कमर्चािरयों के प्रित उदार व्यवहार के प्रावधान शािमल थे िजन्होंने सिवनय अवज्ञा आं दोलन के
िहस्से के रूप में इस्तीफा दे िदया था।
कथन 3 सही है| गांधी-इरिवन समझौते की शतोर्ं में से एक िनजी उपभोग के िलए तटीय गांवों में नमक बनाने के अिधकार की स्वीकृित थी,
लेिकन िबक्री के िलए नहीं।
कथन 4 गलत है| गांधीजी ने सिवनय अवज्ञा आं दोलन के दौरान पुिलस द्वारा की गई ज्यादितयों की गहन और सावर्जिनक जांच पर जोर िदया।
यह मांग िवरोध प्रदशर्नों और प्रदशर्नों में शािमल भारतीय आबादी के िखलाफ पुिलस की बबर्रता, दमन और कदाचार की व्यापक िरपोटोर्ं और
आरोपों से उठी।
गांधी-इरिवन समझौते में कई अन्य शतोर्ं पर सहमत होने के बावजूद, िब्रिटश भारत सरकार का प्रितिनिधत्व करने वाले वायसराय ने पुिलस
ज्यादितयों की सावर्जिनक जांच की मांग को ठु करा िदया। िब्रिटश सरकार अिधकािरयों द्वारा गलत काम के आरोपों को स्वीकार करने या
उनकी जांच करने में अिनच्छु क थी, और यह इनकार औपिनवेिशक प्रशासन के कायोर्ं को सावर्जिनक जांच के अधीन करने की उनकी अिनच्छा
को दशार्ता था।

Source: A BRIEF HISTORY OF MODERN INDIA BY RAJIV AHIR , Chapter-19

Question Attempted / Student

Question: 82 Mark Flag  Previous Next 

Correct Answer : b
Your Answer : b

Consider the following options regarding Nehru Report of 1928:

1. Complete independence for India

2. Continuation of separate electorates

3. Creation of linguistic provinces

4. Complete dissociation of the state from religion

How many of the above recommendations were made in the Nehru Report of 1928?

https://academy.forumias.com/beta/lms/viewSolutions Page 147 of 180


Academy ForumIAS | Dashboard 18/02/24, 8:41 PM

1928 की नेहरू िरपोटर् के संबंध में िनम्निलिखत िवकल्पों पर िवचार करें:


1. भारत को पूणर् स्वतंत्रता
2. पृथक िनवार्चक मंडल का जारी रहना
3. भाषाई प्रांतों का िनमार्ण
4. राज्य का धमर् से पूणर् पृथक्करण

उपरोक्त में से िकतनी िसफ़ािरशें 1928 की नेहरू िरपोटर् में की गई थीं?

a Only one केवल एक


b Only two केवल दो
c Only three केवल तीन
d All four सभी चार

Explanation
Option b is the correct answer.

In response to Lord Birkenhead's challenge, the Nehru Committee, chaired by Motilal Nehru, drafted the Nehru Report in
August 1928. It marked the first major attempt by Indians to formulate a constitutional framework.

Option 1 is incorrect. The Nehru Report of 1928 did not explicitly demand complete independence for India.
Instead, it sought increased self-governance and constitutional reforms within the framework of the British Empire.
The primary emphasis was on achieving Dominion status for India, similar to the self-governing dominions like Canada
and Australia.

Option 2 is incorrect. The Nehru Report rejected separate electorates and instead proposed joint electorates with
reserved seats for Muslims in proportion to their population.

Option 3 is correct. The Nehru Report did propose the creation of linguistic provinces to address regional and
linguistic diversity.

Option 4 is correct. The Nehru Report indeed recommended the complete dissociation of the state from religion,
reflecting a commitment to secular governance.

Source: A BRIEF HISTORY OF MODERN INDIA BY RAJIV AHIR, Chapter-18

िवकल्प b सही उत्तर है।


लॉडर् बकेर्नहेड की चुनौती के जवाब में, मोतीलाल नेहरू की अध्यक्षता में नेहरू सिमित ने अगस्त 1928 में नेहरू िरपोटर् का मसौदा तैयार िकया।
यह संवैधािनक ढांचा तैयार करने के िलए भारतीयों द्वारा िकए गए पहले बड़े प्रयास को िचिह्नत करता है।
िवकल्प 1 गलत है| 1928 की नेहरू िरपोटर् में स्पष्ट रूप से भारत के िलए पूणर् स्वतंत्रता की मांग नहीं की गई थी। इसके बजाय, इसने िब्रिटश
साम्राज्य के ढांचे के भीतर स्वशासन और संवैधािनक सुधारों को बढ़ाने की मांग की। प्राथिमक जोर कनाडा और ऑस्ट्रेिलया जैसे स्वशासी
प्रभुत्व के समान भारत के िलए डोिमिनयन िस्थित प्राप्त करने पर था।
िवकल्प 2 गलत है| नेहरू िरपोटर् ने अलग िनवार्चन क्षेत्रों को खािरज कर िदया और इसके बजाय मुसलमानों के िलए उनकी आबादी के अनुपात
में आरिक्षत सीटों के साथ संयुक्त िनवार्चन क्षेत्रों का प्रस्ताव रखा।

https://academy.forumias.com/beta/lms/viewSolutions Page 148 of 180


Academy ForumIAS | Dashboard 18/02/24, 8:41 PM

िवकल्प 3 सही है| नेहरू िरपोटर् ने क्षेत्रीय और भाषाई िविवधता को संबोिधत करने के िलए भाषाई प्रांतों के िनमार्ण का प्रस्ताव िदया था।
िवकल्प 4 सही है| नेहरू िरपोटर् ने वास्तव में धमर्िनरपेक्ष शासन के प्रित प्रितबद्धता को दशार्ते हुए राज्य को धमर् से पूरी तरह अलग करने की
िसफािरश की थी।

Source: A BRIEF HISTORY OF MODERN INDIA BY RAJIV AHIR, Chapter-18

Question Attempted / Student

Question: 83 Mark Flag  Previous Next 

Correct Answer : c
Your Answer : c

Which organization, associated with Bal Gangadhar Tilak, played a key role in advocating self-governance in India and
later became "Swarajya Sabha" in 1920?

बाल गंगाधर ितलक से जुड़े िकस संगठन ने भारत में स्वशासन की वकालत करने में महत्वपूणर् भूिमका िनभाई और बाद में 1920 में "स्वराज्य
सभा" बन गया?

a Deccan Education Society डेक्कन एजुकेशन सोसायटी


b Poona Sarvajanik Sabha पूना सावर्जिनक सभा
c All India Home Rule League ऑल इं िडया होम रूल लीग
d Hindu Mahasabha िहं दू महासभा

Explanation
Option c is the correct answer.

The organization associated with Bal Gangadhar Tilak that played a key role in advocating self-governance and
later became "Swarajya Sabha" in 1920 was the All India Home Rule League.

Option a is incorrect. Deccan Education Society founded in 1884, the Deccan Education Society aimed at promoting
education in Pune, Maharashtra. While Bal Gangadhar Tilak was associated with the Deccan Education Society, it was
primarily focused on educational reforms rather than political advocacy.

Option b is incorrect. Poona Sarvajanik Sabha established in 1870, the Poona Sarvajanik Sabha was a public
association in Pune that addressed social and political issues. Tilak was actively involved in the Sabha, and it served as
a platform for early nationalist activities. However, it wasn't primarily focused on the demand for self-governance.

https://academy.forumias.com/beta/lms/viewSolutions Page 149 of 180


Academy ForumIAS | Dashboard 18/02/24, 8:41 PM

Option c is correct. The All India Home Rule League, founded in 1916 by Annie Besant and Bal Gangadhar Tilak,
was a significant organization in the Indian independence movement. It aimed at advocating for self-governance or home
rule for India within the British Empire. The league played a key role in mobilizing public opinion and demanding political
reforms.

Option d is incorrect. The Hindu Mahasabha is a right-wing Hindu nationalist organization. While Tilak was
associated with Hindu nationalism, the Mahasabha wasn't directly linked to the demand for self-governance. It
primarily focused on issues related to Hindu identity and interests.

Source: A BRIEF HISTORY OF MODERN INDIA BY RAJIV AHIR, Chapter-14

िवकल्प c सही उत्तर है।


बाल गंगाधर ितलक से जुड़ा संगठन िजसने स्वशासन की वकालत में महत्वपूणर् भूिमका िनभाई और बाद में 1920 में "स्वराज्य सभा" बन गया,
वह अिखल भारतीय होम रूल लीग था।
िवकल्प a गलत है| डेक्कन एजुकेशन सोसाइटी की स्थापना 1884 में हुई, डेक्कन एजुकेशन सोसाइटी का उद्देश्य पुण,े महाराष्ट्र में िशक्षा को
बढ़ावा देना था। जबिक बाल गंगाधर ितलक डेक्कन एजुकेशन सोसाइटी से जुड़े थे, यह मुख्य रूप से राजनीितक वकालत के बजाय शैिक्षक
सुधारों पर केंिद्रत था।
िवकल्प b गलत है| पूना सावर्जिनक सभा की स्थापना 1870 में हुई, पूना सावर्जिनक सभा पुणे में एक सावर्जिनक संस्था थी जो सामािजक
और राजनीितक मुद्दों को संबोिधत करती थी। ितलक सभा में सिक्रय रूप से शािमल थे, और इसने प्रारंिभक राष्ट्रवादी गितिविधयों के िलए एक
मंच के रूप में कायर् िकया। हालाँिक, यह मुख्य रूप से स्वशासन की मांग पर केंिद्रत नहीं था।
िवकल्प c सही है| एनी बेसेंट और बाल गंगाधर ितलक द्वारा 1916 में स्थािपत ऑल इं िडया होम रूल लीग, भारतीय स्वतंत्रता आं दोलन में एक
महत्वपूणर् संगठन था। इसका उद्देश्य िब्रिटश साम्राज्य के भीतर भारत के िलए स्वशासन या घरेलू शासन की वकालत करना था। लीग ने जनमत
जुटाने और राजनीितक सुधारों की मांग करने में महत्वपूणर् भूिमका िनभाई।
िवकल्प d गलत है| िहं दू महासभा एक दिक्षणपंथी िहं दू राष्ट्रवादी संगठन है। जबिक ितलक िहं दू राष्ट्रवाद से जुड़े थे, महासभा सीधे तौर पर
स्वशासन की मांग से नहीं जुड़ी थी। यह मुख्य रूप से िहं दू पहचान और िहतों से संबंिधत मुद्दों पर केंिद्रत था।

Source: A BRIEF HISTORY OF MODERN INDIA BY RAJIV AHIR, Chapter-14

Question Attempted / Student

Question: 84 Mark Flag  Previous Next 

Correct Answer : c
Your Answer : b

Consider the following statements with regard to the Champaran Satyagraha, 1917:

1. Babu Brajkishore Prasad and Mazhar-ul-Haq supported Mahatma Gandhi in the Satyagraha.

2. Its objective was to address the exploitation and grievances faced by indigo farmers.

https://academy.forumias.com/beta/lms/viewSolutions Page 150 of 180


Academy ForumIAS | Dashboard 18/02/24, 8:41 PM

3. The tinkathia system in Champaran was abolished as a result of the Champaran Satyagraha.

How many of the above statements are correct?

चंपारण सत्याग्रह, 1917 के संबंध में िनम्निलिखत कथनों पर िवचार करें:


1. बाबू ब्रजिकशोर प्रसाद और मजहर-उल-हक ने सत्याग्रह में महात्मा गांधी का साथ िदया।
2. इसका उद्देश्य नील िकसानों के शोषण और िशकायतों का समाधान करना था।
3. चंपारण सत्याग्रह के पिरणामस्वरूप चंपारण में ितनकिठया प्रणाली को समाप्त कर िदया गया।

उपरोक्त में से िकतने कथन सही हैं?

a Only one केवल एक


b Only two केवल दो
c All three तीनों
d None कोई नहीं

Explanation
Option c is the correct answer.

Champaran Satyagraha (1917) was led by Mahatma Gandhi, addressing exploitation of indigo farmers in Bihar. Defying
orders, Gandhi's civil disobedience compelled the authorities to appoint a committee. As result of satyagraha, It led to the
abolition of the tinkathia system, compensation, and departure of European planters.

Statement 1 is correct. Important leaders associated with the Champaran Satyagraha included Rajendra Prasad,
Mazhar-ul-Haq, Narahari Parekh, JB Kriplani Mahadev Desai, Brajkishore Prasad, Anugrah Narayan Sinha,
Ramnavmi Prasad, and Shambhusharan Varma.

1) Babu Brajkishore Prasad was a local leader and lawyer who supported Mahatma Gandhi during the Champaran
Satyagraha. Babu Brajkishore Prasad played a crucial role in facilitating the legal aspects of the protest.

2) Mazhar-ul-Haq was a lawyer who supported Mahatma Gandhi in providing legal assistance and representation during
the Champaran Satyagraha. His involvement was important in addressing legal issues related to the protest.

Statement 2 is correct. The main objective of the Champaran Satyagraha was to address the exploitation and
grievances faced by indigo farmers in the Champaran district of Bihar under the British indigo plantation system.
The indigo farmers were forced to cultivate indigo and were subjected to oppressive practices by British indigo planters,
including the tinkathia system (forcing farmers to plant indigo on a specific portion of their land).

Statement 3 is correct. The Champaran Satyagraha marked a significant success in addressing the issues faced by
indigo farmers. The British government appointed the Champaran Agrarian Committee in response to the agitation, and
its recommendations led to the end of the tinkathia system and other oppressive practices.

Source: A BRIEF HISTORY OF MODERN INDIA BY RAJIV AHIR, Chapter-15

https://academy.forumias.com/beta/lms/viewSolutions Page 151 of 180


Academy ForumIAS | Dashboard 18/02/24, 8:41 PM

िवकल्प c सही उत्तर है।


िबहार में नील िकसानों के शोषण को संबोिधत करते हुए चंपारण सत्याग्रह (1917) का नेतृत्व महात्मा गांधी ने िकया था। आदेशों की
अवहेलना करते हुए, गांधी की सिवनय अवज्ञा ने अिधकािरयों को एक सिमित िनयुक्त करने के िलए मजबूर िकया। सत्याग्रह के
पिरणामस्वरूप, ितनकिठया प्रणाली का उन्मूलन, मुआवज़ा और यूरोपीय बागान मािलकों का प्रस्थान हुआ।
कथन 1 सही है| चंपारण सत्याग्रह से जुड़े महत्वपूणर् नेताओं में राजेंद्र प्रसाद, मजहर-उल-हक, नरहिर पारेख, जेबी कृपलानी महादेव देसाई,
ब्रजिकशोर प्रसाद, अनुग्रह नारायण िसन्हा, रामनवमी प्रसाद और शंभूशरण वमार् शािमल थे।
1) बाबू ब्रजिकशोर प्रसाद एक स्थानीय नेता और वकील थे िजन्होंने चंपारण सत्याग्रह के दौरान महात्मा गांधी का समथर्न िकया था। बाबू
ब्रजिकशोर प्रसाद ने िवरोध के कानूनी पहलुओ ं को सुिवधाजनक बनाने में महत्वपूणर् भूिमका िनभाई।
2) मजहर-उल-हक एक वकील थे, िजन्होंने चंपारण सत्याग्रह के दौरान कानूनी सहायता और प्रितिनिधत्व प्रदान करने में महात्मा गांधी का
समथर्न िकया था। िवरोध से संबंिधत कानूनी मुद्दों को संबोिधत करने में उनकी भागीदारी महत्वपूणर् थी।
कथन 2 सही है| चंपारण सत्याग्रह का मुख्य उद्देश्य िब्रिटश नील बागान प्रणाली के तहत िबहार के चंपारण िजले में नील िकसानों के शोषण
और िशकायतों को संबोिधत करना था। नील िकसानों को नील की खेती करने के िलए मजबूर िकया गया और उन्हें िब्रिटश नील बागान
मािलकों द्वारा दमनकारी प्रथाओं का सामना करना पड़ा, िजसमें ितनकिठया प्रणाली (िकसानों को अपनी भूिम के एक िविशष्ट िहस्से पर नील
की खेती करने के िलए मजबूर करना) भी शािमल थी।
कथन 3 सही है| चंपारण सत्याग्रह ने नील िकसानों के सामने आने वाली समस्याओं के समाधान में महत्वपूणर् सफलता हािसल की। िब्रिटश
सरकार ने आं दोलन के जवाब में चंपारण कृिष सिमित की िनयुिक्त की और इसकी िसफािरशों के कारण ितनकिठया प्रणाली और अन्य
दमनकारी प्रथाओं का अंत हुआ।

Source: A BRIEF HISTORY OF MODERN INDIA BY RAJIV AHIR, Chapter-15

Question Attempted / Student

Question: 85 Mark Flag  Previous Next 

Correct Answer : b
Your Answer :

With reference to the Goods and Services Tax (GST) Composition Scheme, consider the following statements:

1. It primarily aims to promote the hassle free movement of goods between States by streamlining tax procedures.

2. Taxpayers enrolled in this scheme are required to file quarterly returns instead of monthly returns.

3. Individuals who supply goods through an e-commerce platform cannot avail benefits under this scheme

How many of the statements given above are correct?

वस्तु एवं सेवा कर (जीएसटी) संरचना योजना के संदभर् में, िनम्निलिखत कथनों पर िवचार करें:
1. इसका मुख्य उद्देश्य कर प्रिक्रयाओं को सुव्यविस्थत करके राज्यों के बीच वस्तु की परेशानी मुक्त आवाजाही को बढ़ावा देना है।
2. इस योजना में नामांिकत करदाताओं को मािसक िरटनर् के बजाय त्रैमािसक िरटनर् दािखल करना आवश्यक है।

https://academy.forumias.com/beta/lms/viewSolutions Page 152 of 180


Academy ForumIAS | Dashboard 18/02/24, 8:41 PM

3. जो व्यिक्त ई-कॉमसर् प्लेटफॉमर् के माध्यम से सामान की आपूितर् करते हैं, वे इस योजना के तहत लाभ नहीं उठा सकते हैं

ऊपर िदए गए कथनों में से िकतने सही हैं?

a Only one केवल एक


b Only two केवल दो
c All three तीनों
d None कोई नहीं

Explanation
Option b is the correct answer.

Even though the Goods and services tax (GST) has been streamlined the tax procedures in India by implementing the
uniform tax rates across India, numerous businesses still face challenges in its compliance. This is primarily due to the
varying tax rates and the multitude of compliances involved and the GST composition scheme offers relief by easing
their burdens.

Statement 1 is incorrect: A person making inter-state supplies of goods are not eligible to avail benefits under the
GST composition scheme. Thus the primary aim of the project being the hassle free movement of goods between States
by streamlining tax procedures across borders is incorrect

Statement 2 is correct: Composition taxpayers are required to file one quarterly return and one annual return. On the
other hand, normal taxpayers are required to file three monthly returns and one annual return under the GST regime.

Statement 3 is correct: The manufacturing businesses making an annual turnover up to Rs 1.5 crore and service
businesses making turnover up to 50 lakhs are eligible to avail benefits under the GST composition scheme. But the
individuals or business engaged in supplying goods through an e-commerce platform cannot avail benefits under this
scheme.

Source: https://www.livemint.com/money/personal-finance/how-small-businesses-can-benefit-from-gst-composition-
scheme-11704733578253.html

https://www.bajajfinserv.in/gst-composition-scheme

िवकल्प b सही उत्तर है।


भले ही वस्तु एवं सेवा कर (जीएसटी) ने पूरे भारत में एक समान कर दरों को लागू करके भारत में कर प्रिक्रयाओं को सुव्यविस्थत कर िदया है,
िफर भी कई व्यवसायों को इसके अनुपालन में चुनौितयों का सामना करना पड़ता है। यह मुख्य रूप से अलग-अलग कर दरों और इसमें शािमल
अनुपालनों की भीड़ के कारण है और जीएसटी संरचना योजना उनके बोझ को कम करके राहत प्रदान करती है।
कथन 1 गलत है: माल की अंतर-राज्यीय आपूितर् करने वाला व्यिक्त जीएसटी संरचना योजना के तहत लाभ प्राप्त करने के िलए पात्र नहीं है।
इस प्रकार इस पिरयोजना का प्राथिमक उद्देश्य सीमाओं के पार कर प्रिक्रयाओं को सुव्यविस्थत करके राज्यों के बीच वस्तुओ ं की परेशानी मुक्त
आवाजाही करना गलत है|
कथन 2 सही है: संरचना, करदाताओं को एक त्रैमािसक िरटनर् और एक वािषर् क िरटनर् दािखल करना आवश्यक है। दू सरी ओर, सामान्य
करदाताओं को जीएसटी व्यवस्था के तहत तीन मािसक िरटनर् और एक वािषर् क िरटनर् दािखल करना आवश्यक है।

https://academy.forumias.com/beta/lms/viewSolutions Page 153 of 180


Academy ForumIAS | Dashboard 18/02/24, 8:41 PM

कथन 3 सही है: 1.5 करोड़ रुपये तक का वािषर् क कारोबार करने वाले िविनमार्ण व्यवसाय और 50 लाख तक का कारोबार करने वाले सेवा
व्यवसाय, जीएसटी संरचना योजना के तहत लाभ प्राप्त करने के पात्र हैं। लेिकन ई-कॉमसर् प्लेटफॉमर् के जिरए सामान की आपूितर् करने वाले
व्यिक्त या व्यवसाय इस योजना के तहत लाभ नहीं उठा सकते हैं।
Source: https://www.livemint.com/money/personal-finance/how-small-businesses-can-benefit-from-gst-
composition-scheme-11704733578253.html

https://www.bajajfinserv.in/gst-composition-scheme

Question Attempted / Student

Question: 86 Mark Flag  Previous Next 

Correct Answer : c
Your Answer : c

With regard to the colonial period in India, consider the following committee/commission:

1. Hunter Commission

2. Hartog Committee

3. Sargent Plan

4. Aitchison Commission

How many of the above were formulated for the development of education in India?

भारत में औपिनवेिशक काल के संबंध में िनम्निलिखत सिमित/आयोग पर िवचार करें:
1. हंटर कमीशन
2. हाटोर्ग सिमित
3. साजेर्ंट योजना
4. एिचसन आयोग

उपरोक्त में से िकतने भारत में िशक्षा के िवकास के िलए तैयार िकये गये थे?

a Only one केवल एक


b Only two केवल दो
c Only three केवल तीन
d All four सभी चार

https://academy.forumias.com/beta/lms/viewSolutions Page 154 of 180


Academy ForumIAS | Dashboard 18/02/24, 8:41 PM

Explanation
Option c is the correct answer.

During the colonial period, the British introduced a Western-influenced education system to India. English, mathematics,
science, and social sciences were among the subjects covered in the curriculum. The Charter Act of 1813 was the first step
toward establishing education as a government objective.

Option 1 is correct. The Hunter Commission (1882-1883), chaired by Sir William Wilson Hunter, reviewed the
progress of education in India. It emphasized primary education, recommended the separation of secondary education in
literary and vocational stream and addressed issues related to lack of female education. Another Hunter Commission was
an investigation committee formed in 1919 to look into the Jallianwala Bagh massacre. The committee's official name was
the Disorders Inquiry Committee, but it became known as the Hunter Commission after its chairman, Lord William
Hunter.

Option 2 is correct. The Hartog Committee (1929), chaired by Sir Philip Hartog, focused on university education. It
recommended measures for improving the quality of university education, including changes in curriculum, examination
systems, and governance.

Option 3 is correct. Sargent Plan (1944) also known as the Sergeant Report, this plan was formulated by J.W.
Sargent. It focused on post-war educational development in India and proposed measures to expand and improve
education, especially at the university level.

Option 4 is incorrect. The Aitchison Commission was formed in 1886 to investigate how Indians could be appointed
to higher levels of government service. Sir Charles Umpherston Aitchison, the lieutenant governor of Punjab, presided
over the commission.

Source: A BRIEF HISTORY OF MODERN INDIA BY RAJIV AHIR, Chapter-30

िवकल्प c सही उत्तर है।


औपिनवेिशक काल के दौरान, अंग्रेजों ने भारत में पिश्चमी-प्रभािवत िशक्षा प्रणाली शुरू की। पाठ्यक्रम में शािमल िवषयों में अंग्रेजी, गिणत,
िवज्ञान और सामािजक िवज्ञान शािमल थे। 1813 का चाटर्र अिधिनयम िशक्षा को सरकारी उद्देश्य के रूप में स्थािपत करने की िदशा में पहला
कदम था।
िवकल्प 1 सही है| सर िविलयम िवल्सन हंटर की अध्यक्षता में हंटर कमीशन (1882-1883) ने भारत में िशक्षा की प्रगित की समीक्षा की।
इसने प्राथिमक िशक्षा पर जोर िदया, माध्यिमक िशक्षा को सािहित्यक और व्यावसाियक धारा में अलग करने की िसफािरश की और मिहला
िशक्षा की कमी से संबंिधत मुद्दों को संबोिधत िकया।
एक अन्य हंटर कमीशन 1919 में जिलयांवाला बाग हत्याकांड की जांच के िलए गिठत एक जांच सिमित थी। सिमित का आिधकािरक नाम
िवकार जांच सिमित था, लेिकन इसके अध्यक्ष लॉडर् िविलयम हंटर के बाद इसे हंटर आयोग के नाम से जाना जाने लगा।
िवकल्प 2 सही है| सर िफिलप हाटोर्ग की अध्यक्षता में हाटोर्ग सिमित (1929) ने िवश्विवद्यालय िशक्षा पर ध्यान केंिद्रत िकया। इसने
िवश्विवद्यालय िशक्षा की गुणवत्ता में सुधार के िलए उपायों की िसफािरश की, िजसमें पाठ्यक्रम, परीक्षा प्रणाली और शासन में बदलाव
शािमल हैं।
िवकल्प 3 सही है| साजेर्ंट योजना (1944) को साजेर्ंट िरपोटर् के नाम से भी जाना जाता है, यह योजना जे.डब्ल्यू. साजेर्ंट द्वारा तैयार की गई थी।
इसने भारत में युद्ध के बाद के शैिक्षक िवकास पर ध्यान केंिद्रत िकया और िवशेष रूप से िवश्विवद्यालय स्तर पर िशक्षा के िवस्तार और सुधार
के िलए प्रस्तािवत उपाय िकए।

https://academy.forumias.com/beta/lms/viewSolutions Page 155 of 180


Academy ForumIAS | Dashboard 18/02/24, 8:41 PM

िवकल्प 4 गलत है| भारतीयों को सरकारी सेवा के उच्च स्तर पर कैसे िनयुक्त िकया जा सकता है, इसकी जांच के िलए 1886 में एिचसन
आयोग का गठन िकया गया था। पंजाब के लेिफ्टनेंट गवनर्र सर चाल्सर् अम्फरस्टन एिचसन ने आयोग की अध्यक्षता की।

Source: A BRIEF HISTORY OF MODERN INDIA BY RAJIV AHIR, Chapter-30

Question Attempted / Student

Question: 87 Mark Flag  Previous Next 

Correct Answer : a
Your Answer : a

Consider the following statements with regard to the Treaty of Amritsar, 1809:

1. It fixed the Sutlej River as the boundary between the dominions of British East India Company and Maharaja Ranjit
Singh.

2. It is recognized for the conclusion of the First Anglo-Sikh War.

3. It marked the beginning of Dogra rule in Kashmir region.

How many of the above statements are correct?

अमृतसर की संिध, 1809 के संबंध में िनम्निलिखत कथनों पर िवचार करें:


1. इसने सतलज नदी को िब्रिटश ईस्ट इं िडया कंपनी और महाराजा रणजीत िसं ह के प्रभुत्व के बीच की सीमा के रूप में तय िकया।
2. इसे प्रथम आं ग्ल-िसख युद्ध के िनष्कषर् के िलए जाना जाता है।
3. यह कश्मीर क्षेत्र में डोगरा शासन की शुरुआत का प्रतीक था।

उपरोक्त में से िकतने कथन सही हैं?

a Only one केवल एक


b Only two केवल दो
c All three तीनों
d None कोई नहीं

Explanation
Option a is the correct answer.

https://academy.forumias.com/beta/lms/viewSolutions Page 156 of 180


Academy ForumIAS | Dashboard 18/02/24, 8:41 PM

The Treaty of Amritsar was signed on April 25, 1809, by the British East India Company and Ranjit Singh, the ruler of the
Sikh kingdom of Punjab. For a generation, the treaty ended Indo-Sikh relations. The British established Punjab as a buffer
state to protect themselves from invasions from the north.

Statement 1 is correct. The Treaty of Amritsar, signed in 1809, established the Sutlej River as the boundary between
the British East India Company and Maharaja Ranjit Singh. The treaty was significant because it prevented Ranjit
Singh from ruling over the entire Sikh population.

Statement 2 is incorrect. The Treaty of Amritsar (1809) is not associated with the conclusion of the First Anglo-Sikh
War. The first Anglo-Sikh war is said to have begun on December 11, 1845, when the Sikh army crossed the Sutlej River.
The Treaty of Lahore, signed on March 8, 1846 was associated with the first Anglo-Sikh war.

Statement 3 is incorrect. The Treaty of Lahore, signed on March 8, 1846, officially established Dogra rule in
Kashmir. The treaty established Jammu and Kashmir as a princely state under the British Indian Empire. The treaty
bestowed the title of Maharaja of Jammu and Kashmir on Maharaja Gulab Singh and granted him control over lands west
of the Ravi River, including Chamba. Gulab Singh purchased the land from the British government for 75 lakh
rupees. From 1846 to 1947, the Dogra dynasty ruled Jammu and Kashmir.

Source: A BRIEF HISTORY OF MODERN INDIA BY RAJIV AHIR, Chapter-5

िवकल्प a सही उत्तर है।


िब्रिटश ईस्ट इं िडया कंपनी और पंजाब के िसख साम्राज्य के शासक रणजीत िसं ह द्वारा 25 अप्रैल, 1809 को अमृतसर की संिध पर हस्ताक्षर
िकए गए थे। इस संिध ने एक पीढ़ी के िलए भारत-िसख संबंधों को समाप्त कर िदया। अंग्रेजों ने खुद को उत्तर से होने वाले आक्रमणों से बचाने
के िलए पंजाब को एक बफर राज्य के रूप में स्थािपत िकया।
कथन 1 सही है| 1809 में हस्ताक्षिरत अमृतसर की संिध ने िब्रिटश ईस्ट इं िडया कंपनी और महाराजा रणजीत िसं ह के बीच सतलज नदी को
सीमा के रूप में स्थािपत िकया। यह संिध महत्वपूणर् थी क्योंिक इसने रणजीत िसं ह को संपूणर् िसख आबादी पर शासन करने से रोक िदया था।
कथन 2 गलत है| अमृतसर की संिध (1809) प्रथम आं ग्ल-िसख युद्ध के िनष्कषर् से जुड़ी नहीं है। ऐसा कहा जाता है िक पहला एं ग्लो-िसख युद्ध
11 िदसंबर, 1845 को शुरू हुआ था, जब िसख सेना ने सतलुज नदी को पार िकया था। 8 माचर्, 1846 को हस्ताक्षिरत लाहौर की संिध प्रथम
आं ग्ल-िसख युद्ध से सम्बंिधत थी।
कथन 3 गलत है| 8 माचर्, 1846 को हस्ताक्षिरत लाहौर की संिध ने आिधकािरक तौर पर कश्मीर में डोगरा शासन की स्थापना की। संिध ने
जम्मू और कश्मीर को िब्रिटश भारतीय साम्राज्य के तहत एक िरयासत के रूप में स्थािपत िकया। संिध ने महाराजा गुलाब िसं ह को जम्मू और
कश्मीर के महाराजा की उपािध प्रदान की और उन्हें चंबा सिहत रावी नदी के पिश्चम की भूिम पर िनयंत्रण प्रदान िकया। गुलाब िसं ह ने िब्रिटश
सरकार से 75 लाख रुपये में जमीन खरीदी। 1846 से 1947 तक डोगरा राजवंश ने जम्मू-कश्मीर पर शासन िकया।

Source: A BRIEF HISTORY OF MODERN INDIA BY RAJIV AHIR, Chapter-5

Question Attempted / Student

Question: 88 Mark Flag  Previous Next 

Correct Answer : c

https://academy.forumias.com/beta/lms/viewSolutions Page 157 of 180


Academy ForumIAS | Dashboard 18/02/24, 8:41 PM

Your Answer : b

Consider the following Indian National Movement events from 1940 to 1947 and arrange them in chronological order:

1. Cabinet Mission Plan

2. Shimla Conference

3. Mountbatten Plan

4. Direct Action Day

Select the correct answer using the codes given below:

1940 से 1947 तक िनम्निलिखत भारतीय राष्ट्रीय आं दोलन की घटनाओं पर िवचार करें और उन्हें कालानुक्रिमक क्रम में व्यविस्थत करें:
1. कैिबनेट िमशन योजना
2. िशमला सम्मेलन
3. माउं टबेटन योजना
4. प्रत्यक्ष कायर्वाही िदवस

नीचे िदए गए कोड का उपयोग करके सही उत्तर चुन:ें

a 1-2-3-4
b 2-1-3-4
c 2-1-4-3
d 1-2-4-3

Explanation
Option c is the correct answer.

The correct chronological order of the event is- 2-1-4-3

Shimla Conference- Cabinet Mission Plan- Direct Action Day-Mountbatten Plan.

1) The Shimla Conference took place in 1945 and brought together the Viceroy of India and the major political leaders
of British India. The Wavell Plan for Indian self-government was the topic of the conference. Lord Wavell, the Viceroy of
India, inspired the Wavell Plan. The purpose of the conference was to discuss the proposal for a new executive council and
a new constitution for India following the war.

2) On March 24, 1946, the Cabinet Mission arrived in India to discuss the transfer of power from the British
government to the Indian political leadership. Clement Atlee, then Prime Minister, dispatched the Cabinet Mission, which
included Sir Stafford Cripps, Pethick Lawrence, and A. V. Alexander.

3) Muhammad Ali Jinnah, the Muslim League's founder, declared August 16, 1946, "Direct Action Day" and urged
Muslims globally to "suspend all trade." The League's main goal was to create a new country with a Muslim majority. The
day was also known as the Calcutta Killings in 1946, and it was a day of nationwide communal riots.

https://academy.forumias.com/beta/lms/viewSolutions Page 158 of 180


Academy ForumIAS | Dashboard 18/02/24, 8:41 PM

4) The Mountbatten Plan was announced on June 3, 1947, and went into effect with the Indian Independence Act on
July 18, 1947. The plan resulted in the formation of two new countries, India and Pakistan.

Source: A BRIEF HISTORY OF MODERN INDIA BY RAJIV AHIR, Chapter-22-24

िवकल्प c सही उत्तर है।


घटना का सही कालानुक्रिमक क्रम है- 2-1-4-3
िशमला सम्मेलन-कैिबनेट िमशन योजना-प्रत्यक्ष कायर् िदवस-माउं टबेटन योजना।
1) िशमला सम्मेलन 1945 में हुआ और इसमें भारत के वायसराय और िब्रिटश भारत के प्रमुख राजनीितक नेता एक साथ आये। भारतीय
स्वशासन के िलए वेवेल योजना सम्मेलन का िवषय था। वेवेल योजना को भारत के वायसराय लॉडर् वेवेल ने प्रेिरत िकया। सम्मेलन का उद्देश्य
युद्ध के बाद भारत के िलए एक नई कायर्कारी पिरषद और एक नए संिवधान के प्रस्ताव पर चचार् करना था।
2) 24 माचर्, 1946 को िब्रिटश सरकार से भारतीय राजनीितक नेतृत्व को सत्ता हस्तांतरण पर चचार् करने के िलए कैिबनेट िमशन भारत आया।
तत्कालीन प्रधान मंत्री क्लेमेंट एटली ने कैिबनेट िमशन भेजा, िजसमें सर स्टैफ़ोडर् िक्रप्स, पेिथक लॉरेंस और ए. वी. अलेक्जेंडर शािमल थे।
3) मुिस्लम लीग के संस्थापक मुहम्मद अली िजन्ना ने 16 अगस्त, 1946 को "प्रत्यक्ष कारर्वाई िदवस" घोिषत िकया और िवश्व स्तर पर
मुसलमानों से "सभी व्यापार को िनलंिबत करने" का आग्रह िकया। लीग का मुख्य लक्ष्य मुिस्लम बहुमत वाला एक नया देश बनाना था। उस
िदन को 1946 में कलकत्ता हत्याओं के रूप में भी जाना जाता था, और यह देशव्यापी सांप्रदाियक दंगों का िदन था।
4) माउं टबेटन योजना की घोषणा 3 जून, 1947 को की गई थी और यह 18 जुलाई, 1947 को भारतीय स्वतंत्रता अिधिनयम के साथ लागू
हुई। इस योजना के पिरणामस्वरूप दो नए देशों, भारत और पािकस्तान का गठन हुआ।

Source: A BRIEF HISTORY OF MODERN INDIA BY RAJIV AHIR, Chapter-22-24

Question Attempted / Student

Question: 89 Mark Flag  Previous Next 

Correct Answer : d
Your Answer :

With regard to the Indian national movement, the 'Deepavali Declaration,' issued in 1929 was related to-

भारतीय राष्ट्रीय आन्दोलन के सम्बन्ध में 1929 में जारी 'दीपावली घोषणापत्र' िकससे सम्बिन्धत था-

a Ensuring equal voting rights for all सभी के िलए समान मतदान अिधकार सुिनिश्चत करना
b Advocating for religious freedom धािमर् क स्वतंत्रता की वकालत करना
c Advocating for women's education मिहलाओं की िशक्षा की वकालत करना
d Establishing Dominion status for India भारत के िलए डोिमिनयन स्टेटस की स्थापना

https://academy.forumias.com/beta/lms/viewSolutions Page 159 of 180


Academy ForumIAS | Dashboard 18/02/24, 8:41 PM

Explanation
Option d is the correct answer.

Option a is incorrect. The 'Deepavali Declaration' did not focus on equal voting rights.

Option b is incorrect. The 'Deepavali Declaration' did not advocate for religious freedom.

Option c is incorrect. The 'Deepavali Declaration' did not focus on advocating for women's education.

Option d is correct. The Deepavali Declaration, also known as the Irwin Declaration, was issued on October 31, 1929 by
Lord Irwin, Viceroy of India. The declaration was about India's place in the British Empire and the British
government's goal of granting India Dominion status.

Source: A BRIEF HISTORY OF MODERN INDIA BY RAJIV AHIR Pg 822

िवकल्प d सही उत्तर है।


िवकल्प a गलत है। 'दीपावली घोषणा' में समान मतािधकार पर ध्यान केंिद्रत नहीं िकया गया था।
िवकल्प b गलत है। 'दीपावली घोषणापत्र' में धािमर् क स्वतंत्रता की वकालत नहीं की गई थी।
िवकल्प c गलत है। 'दीपावली घोषणा' में मिहलाओं की िशक्षा की वकालत पर ध्यान केंिद्रत नहीं िकया गया था।

िवकल्प d सही है। दीपावली घोषणा, िजसे इरिवन घोषणा के रूप में भी जाना जाता है, 31 अक्टू बर, 1929 को भारत के वायसराय लॉडर्
इरिवन द्वारा जारी िकया गया था। यह घोषणा िब्रिटश साम्राज्य में भारत के स्थान और भारत को डोिमिनयन का दजार् देने के िब्रिटश सरकार के
लक्ष्य के बारे में थी। Source: A BRIEF HISTORY OF MODERN INDIA BY RAJIV AHIR Pg 822

Question Attempted / Student

Question: 90 Mark Flag  Previous Next 

Correct Answer : b
Your Answer : c

The recent study claimed that lead exposure caused millions to lose health across the world and removing it from petrol
was a global achievement. In this context, consider the following statements regarding lead pollution:

1. Traditional medicines act as one of the sources of lead pollution in India.

2. Babesiosis is a disease that occurs mainly among children due to their high exposure to lead poisoning.

3. Pregnant women exposed to lead poisoning transmit the same to their developing fetuses.

How many of the statements given above are correct?

https://academy.forumias.com/beta/lms/viewSolutions Page 160 of 180


Academy ForumIAS | Dashboard 18/02/24, 8:41 PM

हाल के अध्ययन में दावा िकया गया है िक सीसे के संपकर् में आने से दुिनया भर में लाखों लोगों का स्वास्थ्य खराब हुआ और इसे पेट्रोल से
हटाना एक वैिश्वक उपलिब्ध थी। इस संदभर् में, सीसा प्रदू षण के संबंध में िनम्निलिखत कथनों पर िवचार करें:
1. पारंपिरक औषिधयाँ भारत में सीसा प्रदू षण के स्रोतों में से एक के रूप में कायर् करती हैं।
2. बेबेिसयोिसस एक ऐसी बीमारी है जो मुख्य रूप से बच्चों में सीसा िवषाक्तता के अिधक संपकर् के कारण होती है।
3. सीसे की िवषाक्तता के संपकर् में आने वाली गभर्वती मिहलाएं इसे अपने िवकासशील भ्रूणों में संचािरत करती हैं।

ऊपर िदए गए कथनों में से िकतने सही हैं?

a Only one केवल एक


b Only two केवल दो
c All three तीनों
d None कोई नहीं

Explanation
Option b is the correct answer.

Lead, a naturally-occurring toxic metal present in the Earth's crust, has caused extensive environmental
contamination and human exposure due to its widespread use.

Statement 1 is incorrect: Babesiosis is a disease that occurs due to the parasitic infection of red blood cells. It is
primarily transmitted to humans and babies through the bite of infected ticks, particularly the black-legged or deer
tick. Childrens are more prone to lead exposure than adults. Compared to Adults, children absorb 4-5 times of lead by
ingestion from a given source due to their innate curiosity and hand-to-mouth behavior.

Statement 2 is correct: The report, jointly prepared by the government think tank Niti Aayog and the Council of
Scientific & Industrial Research (CSIR), identifies traditional medicines, cosmetics, and adulterated spices as
inconspicuous sources of lead. It also states that battery recycling, occupational sources such as lead mining and
scrapping of automobiles as major sources of lead pollution.

Statement 3 is correct: Thehuman body stores lead in the teeth and bones and lead stored in bone may be released
into the blood during pregnancy, thus exposing the fetus. Exposure of pregnant women to high levels of lead can cause
miscarriage, stillbirth, premature birth and low birth weight

Source: https://timesofindia.indiatimes.com/lead-exposure-caused-millions-to-lose-health-removing-it-from-petrol-was-a-
global-achievement/articleshow/106582655.cms?from=mdr#:~:text='Lead exposure caused millions to,India News - Times
of India

https://www.who.int/news-room/fact-sheets/detail/lead-poisoning-and-health#:~:text=Lead exposure also causes


anaemia,are believed to be irreversible.

https://www.downtoearth.org.in/news/health/india-bears-world-s-highest-health-economic-burden-due-to-lead-poisoning-
centre-85428#:~:text=spices, cosmetics and-,traditional,-medicines.

िवकल्प b सही उत्तर है।

https://academy.forumias.com/beta/lms/viewSolutions Page 161 of 180


Academy ForumIAS | Dashboard 18/02/24, 8:41 PM

सीसा, पृथ्वी की परत में मौजूद एक प्राकृितक रूप से पाई जाने वाली जहरीली धातु है, िजसके व्यापक उपयोग के कारण व्यापक पयार्वरणीय
प्रदू षण और मानव जोिखम हुआ है।
कथन 1 गलत है: बेबेिसयोिसस एक बीमारी है जो लाल रक्त कोिशकाओं के परजीवी संक्रमण के कारण होती है। यह मुख्य रूप से संक्रिमत
िकलनी, िवशेषकर काले पैर वाली या िहरण िकलनी के काटने से मनुष्यों और िशशुओ ं में फैलता है। वयस्कों की तुलना में बच्चों में सीसा के
संपकर् में आने की संभावना अिधक होती है। वयस्कों की तुलना में, बच्चे अपनी सहज िजज्ञासा और हाथ से मुँह के व्यवहार के कारण िकसी
िदए गए स्रोत से 4-5 गुना सीसा अवशोिषत करते हैं।
कथन 2 सही है: सरकारी िथं क टैंक नीित आयोग और वैज्ञािनक एवं औद्योिगक अनुसंधान पिरषद (सीएसआईआर) द्वारा संयुक्त रूप से तैयार
की गई िरपोटर्, पारंपिरक दवाओं, सौंदयर् प्रसाधनों और िमलावटी मसालों को सीसे के असंगत स्रोतों के रूप में पहचानती है। इसमें यह भी कहा
गया है िक बैटरी रीसाइिक्लं ग, व्यावसाियक स्रोत जैसे सीसा खनन और ऑटोमोबाइल की स्क्रै िपं ग सीसा प्रदू षण के प्रमुख स्रोत हैं।
कथन 3 सही है: मानव शरीर दांतों और हिड्डयों में सीसा जमा करता है और हड्डी में जमा सीसा गभार्वस्था के दौरान रक्त में छोड़ा जा सकता है,
िजससे भ्रूण उजागर हो सकता है। गभर्वती मिहलाओं के सीसे के उच्च स्तर के संपकर् में आने से गभर्पात, मृत बच्चे का जन्म, समय से पहले
जन्म और जन्म के समय कम वजन हो सकता है।
Source: https://timesofindia.indiatimes.com/lead-exposure-caused-millions-to-lose-health-removing-it-from-
petrol-was-a-global-achievement/articleshow/106582655.cms?
from=mdr#:~:text='Lead%20exposure%20caused%20millions%20to,India%20News%20%2D%20Times%20of%20India
https://www.who.int/news-room/fact-sheets/detail/lead-poisoning-and-
health#:~:text=Lead%20exposure%20also%20causes%20anaemia,are%20believed%20to%20be%20irreversible.

https://www.downtoearth.org.in/news/health/india-bears-world-s-highest-health-economic-burden-due-to-lead-
poisoning-centre-85428#:~:text=spices%2C%20cosmetics%20and-,traditional,-medicines.

Question Attempted / Student

Question: 91 Mark Flag  Previous Next 

Correct Answer : a
Your Answer : d

Consider the following statements about the evolution of demand of dominion status during the colonial period:

1. The demand for dominion status for India was initially put forth by Sir Tej Bahadur Sapru and M. R. Jayakar.

2. The Lahore Resolution of 1940 explicitly called for the immediate grant of dominion status to India.

3. The proposal for dominion status was finally accepted by the Indian leaders in Cripps mission.

How many of the above statements are correct?

औपिनवेिशक काल के दौरान डोिमिनयन स्टेटस (dominion status) की मांग के िवकास के बारे में िनम्निलिखत कथनों पर िवचार करें:
1. भारत के िलए डोिमिनयन स्टेटस की मांग सबसे पहले सर तेज बहादुर सप्रू और एम. आर. जयकर ने रखी थी।

https://academy.forumias.com/beta/lms/viewSolutions Page 162 of 180


Academy ForumIAS | Dashboard 18/02/24, 8:41 PM

2. 1940 के लाहौर प्रस्ताव में स्पष्ट रूप से भारत को तत्काल प्रभुत्व का दजार् देने का आह्वान िकया गया था।
3. िक्रप्स िमशन में भारतीय नेताओं द्वारा डोिमिनयन स्टेटस के प्रस्ताव को अंततः स्वीकार कर िलया गया।

उपरोक्त में से िकतने कथन सही हैं?

a Only one केवल एक


b Only two केवल दो
c All three तीनों
d None कोई नहीं

Explanation
Option a is the correct answer.

The Dominion status, initially granted through acts like the Dominion of Canada Act in 1867, provided countries within
the British Empire, such as Canada, with significant self-governance while remaining part of the empire. India sought a
similar status for independence. The Indian National Congress made the demand in 1917, later incorporating it into the
Nehru Report of 1928, proposing constitutional reforms. This marked a key step in India's pursuit of self-rule within the
British Empire.

Statement 1 is correct. Sir Tej Bahadur Sapru and M. R. Jayakar were prominent Indian leaders who initially
advocated for dominion status for India during the colonial period. Dominion status was seen as a transitional step
towards full self-governance and independence. Their efforts aimed to secure greater autonomy for India within the British
Empire.

Statement 2 is incorrect. The Lahore Resolution, also known as the Pakistan Resolution, was passed during the All-
India Muslim League session in 1940. Contrary to the statement, the resolution focused on the demand for the creation of
a separate Muslim state rather than calling for the immediate grant of dominion status to the entire India. It laid the
groundwork for the eventual formation of Pakistan in 1947.

Statement 3 is incorrect. The dominion status proposal was not accepted by Indian leaders through the Cripps
Mission. The Cripps Mission, which visited India in 1942 during World War II, did propose constitutional reforms but fell
short of meeting Indian aspirations for immediate self-governance and full dominion status. The Indian National Congress
rejected the Cripps proposals, as they did not provide for the immediate transfer of power and were perceived as
inadequate.

Source: A BRIEF HISTORY OF MODERN INDIA BY RAJIV AHIR

िवकल्प a सही उत्तर है।


डोिमिनयन का दजार्, शुरुआत में 1867 में डोिमिनयन ऑफ कनाडा अिधिनयम जैसे अिधिनयमों के माध्यम से प्रदान िकया गया था, जो कनाडा
जैसे िब्रिटश साम्राज्य के देशों को साम्राज्य का िहस्सा रहते हुए महत्वपूणर् स्वशासन प्रदान करता था। भारत ने स्वतंत्रता के िलए समान िस्थित
की मांग की। भारतीय राष्ट्रीय कांग्रेस ने 1917 में मांग की, बाद में इसे 1928 की नेहरू िरपोटर् में शािमल िकया गया, िजसमें संवैधािनक
सुधारों का प्रस्ताव रखा गया। यह िब्रिटश साम्राज्य के भीतर भारत के स्व-शासन की खोज में एक महत्वपूणर् कदम था।

https://academy.forumias.com/beta/lms/viewSolutions Page 163 of 180


Academy ForumIAS | Dashboard 18/02/24, 8:41 PM

कथन 1 सही है| सर तेज बहादुर सप्रू और एम. आर. जयकर प्रमुख भारतीय नेता थे, िजन्होंने शुरू में औपिनवेिशक काल के दौरान भारत के
िलए प्रभुत्व की िस्थित की वकालत की थी। डोिमिनयन िस्थित को पूणर् स्वशासन और स्वतंत्रता की िदशा में एक संक्रमणकालीन कदम के रूप
में देखा गया था। उनके प्रयासों का उद्देश्य िब्रिटश साम्राज्य के भीतर भारत के िलए अिधक स्वायत्तता सुरिक्षत करना था।
कथन 2 गलत है| लाहौर प्रस्ताव, िजसे पािकस्तान प्रस्ताव के रूप में भी जाना जाता है, 1940 में अिखल भारतीय मुिस्लम लीग सत्र के दौरान
पािरत िकया गया था। बयान के िवपरीत, प्रस्ताव पूरे भारत को तत्काल डोिमिनयन स्टेटस देने की मांग के बजाय एक अलग मुिस्लम राज्य के
िनमार्ण की मांग पर केंिद्रत था। इसने 1947 में पािकस्तान के अंितम गठन की नींव रखी।
कथन 3 गलत है| िक्रप्स िमशन के माध्यम से डोिमिनयन स्टेटस प्रस्ताव को भारतीय नेताओं ने स्वीकार नहीं िकया। िद्वतीय िवश्व युद्ध के दौरान
1942 में भारत का दौरा करने वाले िक्रप्स िमशन ने संवैधािनक सुधारों का प्रस्ताव िदया था, लेिकन तत्काल स्वशासन और पूणर् प्रभुत्व की
िस्थित के िलए भारतीय आकांक्षाओं को पूरा करने में िवफल रहा। भारतीय राष्ट्रीय कांग्रेस ने िक्रप्स प्रस्तावों को अस्वीकार कर िदया, क्योंिक
उनमें सत्ता के तत्काल हस्तांतरण का प्रावधान नहीं था और उन्हें अपयार्प्त माना गया।

Source: A BRIEF HISTORY OF MODERN INDIA BY RAJIV AHIR

Question Attempted / Student

Question: 92 Mark Flag  Previous Next 

Correct Answer : b
Your Answer : b

With reference to the administration under the Maratha empire, consider the following statements:

1. Shivaji's system of administration was influenced largely by the administrative practices of the Deccan states.

2. Chauth and sardeshmukhi were the taxes collected not in the Maratha kingdom but in the neighbouring territories.

3. The post of Peshwa became hereditary under Shivaji.

How many of the above statements are correct?

मराठा साम्राज्य के तहत प्रशासन के संदभर् में, िनम्निलिखत कथनों पर िवचार करें:
1. िशवाजी की प्रशासन प्रणाली काफी हद तक दक्कन राज्यों की प्रशासिनक प्रथाओं से प्रभािवत थी।
2. चौथ और सरदेशमुखी कर मराठा साम्राज्य में नहीं बिल्क पड़ोसी क्षेत्रों में वसूले जाते थे।
3. िशवाजी के अधीन पेशवा का पद वंशानुगत हो गया।

उपरोक्त में से िकतने कथन सही हैं?

https://academy.forumias.com/beta/lms/viewSolutions Page 164 of 180


Academy ForumIAS | Dashboard 18/02/24, 8:41 PM

a Only one केवल एक


b Only two केवल दो
c All three तीनों
d None कोई नहीं

Explanation
Option b is the correct answer.

Statement 1 is correct: Shivaji's system of administration was largely borrowed from the administrative practices of the
Deccani states. Although he designated eight ministers, sometimes called the Ashtapradhan, it was not in the nature of a
council of ministers, each minister being directly responsible to the ruler.

Statement 2 is correct: Chauth and sardeshmukhi were the taxes collected not in the Maratha kingdom but in the
neighbouring territories of the Mughal Empire or Deccan sultanates. The right to assess and collect Sardeshmukhi
was asserted first by Shivaji on grounds that his family was hereditary tax collectors in Maharashtra. The sardeshmukhi
was an additional 10% levy on top of the chauth. Chauth was one fourth of the land revenue paid to the Marathas in order
to avoid the Maratha raids.

Statement 3 is incorrect: During Shivaji's administration king was the pivot of the government, who was assisted by a
council of ministers called the Ashtapradhan. Each one was directly responsible to Shivaji and headed a department.
Under Shivaji, these offices were neither hereditary nor permanent and were also frequently transferred.

Knowledge Base:

Shivaji preferred to give cash salaries to the regular soldiers, though sometimes the chiefs received revenue grants
(saranjam). Strict discipline was maintained in the army, no women or dancing girls being allowed to accompany the army.
The plunder taken by each soldier during campaigns was strictly accounted for. The revenue system seems to have been
patterned on the system of Malik Ambar. A new revenue assessment was completed by Annaji Datto in 1679.

Source: History of Medieval India by Satish Chandra - Chapter 19 - Climax and Disintegration of the Mughal Empire—II.

िवकल्प b सही उत्तर है।


कथन 1 सही है: िशवाजी की प्रशासन प्रणाली काफी हद तक दक्कनी राज्यों की प्रशासिनक प्रथाओं से उधार ली गई थी। हालाँिक उन्होंने
आठ मंित्रयों को नािमत िकया, िजन्हें कभी-कभी अष्टप्रधान भी कहा जाता था, यह मंित्रपिरषद की प्रकृित में नहीं था, प्रत्येक मंत्री सीधे शासक
के प्रित उत्तरदायी था।
कथन 2 सही है: चौथ और सरदेशमुखी कर मराठा साम्राज्य में नहीं बिल्क मुगल साम्राज्य या दक्कन सल्तनत के पड़ोसी क्षेत्रों में एकत्र िकए
गए कर थे।
सरदेशमुखी का मूल्यांकन और संग्रह करने का अिधकार सबसे पहले िशवाजी द्वारा इस आधार पर िदया गया था िक उनका पिरवार महाराष्ट्र में
वंशानुगत कर संग्राहक था। सरदेशमुखी, चौथ के ऊपर 10% अितिरक्त कर था। मराठा छापों से बचने के िलए, चौथ मराठों को िदए जाने वाले
भू-राजस्व का एक चौथाई था।
कथन 3 गलत है: िशवाजी के प्रशासन के दौरान राजा सरकार की धुरी था, िजसे अष्टप्रधान नामक मंित्रपिरषद द्वारा सहायता प्रदान की जाती
थी। प्रत्येक व्यिक्त सीधे तौर पर िशवाजी के प्रित उत्तरदायी था और एक िवभाग का प्रमुख था। िशवाजी के अधीन, ये कायार्लय न तो
वंशानुगत थे और न ही स्थायी और अक्सर स्थानांतिरत भी होते थे।

https://academy.forumias.com/beta/lms/viewSolutions Page 165 of 180


Academy ForumIAS | Dashboard 18/02/24, 8:41 PM

ज्ञानधार:
िशवाजी िनयिमत सैिनकों को नकद वेतन देना पसंद करते थे, हालाँिक कभी-कभी प्रमुखों को राजस्व अनुदान (सरंजम) िमलता था। सेना में
सख्त अनुशासन बनाए रखा गया, िकसी भी मिहला या नाचने वाली लड़की को सेना के साथ जाने की अनुमित नहीं थी। अिभयानों के दौरान
प्रत्येक सैिनक द्वारा की गई लूट का सख्ती से िहसाब-िकताब िकया जाता था। ऐसा प्रतीत होता है िक राजस्व प्रणाली मिलक अम्बर की
प्रणाली पर आधािरत थी। 1679 में अन्नाजी दत्तो द्वारा एक नया राजस्व मूल्यांकन पूरा िकया गया।

Source: History of Medieval India by Satish Chandra - Chapter 19 - Climax and Disintegration of the Mughal
Empire—II.

Question Attempted / Student

Question: 93 Mark Flag  Previous Next 

Correct Answer : c
Your Answer : b

With reference to 'Anushilan Samiti', consider the following statements:

1. It was founded by Promothanath Mitter.

2. The Samiti was influenced by Bankim Chandra Chatterjee's 'Anandamath'.

3. The Samiti published a weekly Yugantar to advocate revolutionary activities.

How many of the above statements are correct?

'अनुशीलन सिमित' के संदभर् में, िनम्निलिखत कथनों पर िवचार करें:


1. इसकी स्थापना प्रोमोथनाथ िमत्तर ने की थी।
2. सिमित, बंिकम चंद्र चटजीर् के 'आनंदमठ' से प्रभािवत थी।
3. सिमित ने क्रांितकारी गितिविधयों की वकालत के िलए एक साप्तािहक युगांतर प्रकािशत िकया।

उपरोक्त में से िकतने कथन सही हैं?

a Only one केवल एक


b Only two केवल दो
c All three तीनों
d None कोई नहीं

Explanation
https://academy.forumias.com/beta/lms/viewSolutions Page 166 of 180
Academy ForumIAS | Dashboard 18/02/24, 8:41 PM

Option c is the correct answer.

Statement 1 is correct. Anushilan Samiti was founded by Promothanath Mitter in 1901.The Samiti includes
Jatindranath Banerjee; Barindra Kumar Ghosh; Bhupendra Nath Dutta; Aurobindo Ghosh, C.R. Das and others.

Statement 2 is correct. The revolutionary movement in Bengal derived its inspiration from the works of Bankim
Chandra Chatterjee. The greeting Bande Mataram (Hail Mother) of Anand Math became a war cry of the extremist
party in Bengal. He gave a religious significance to the idea of motherland by declaring that in the image of the benign
goddess Durga could be seen the future greatness of the Motherland. The new revolutionary gospel or cult of Shakti
called upon the Bengalis to shed the blood of their oppressors.

Statement 3 is correct. In April 1906, an inner circle within Anushilan (Barindra Kumar Ghosh, Bhupendranath Dutta)
started the weekly Yugantar. It started advocating revolutionary violence. For instance, after severe police brutalities on
participants of the Barisal Conference (April 1906), the Yugantar wrote: "The remedy lies with the people. The 30 crore
people inhabiting India must raise their 60 crore hands to stop this curse of oppression. Force must be stopped by force."

Source: https://mu.ac.in/wp-content/uploads/2014/04/FYBA-History-Paper-I-History-of-Modern-India-

Revised-Syllabus-2018-19.pdf

SPECTRUM(CH-First Phase of Revolutionary Activities)

https://www.egyankosh.ac.in/bitstream/123456789/44305/1/Unit-13.pdf

िवकल्प c सही उत्तर है।


कथन 1 सही है| अनुशीलन सिमित की स्थापना 1901 में प्रोमोथनाथ िमत्तर द्वारा की गई थी। सिमित में जतींद्रनाथ बनजीर्; बिरन्द्र कुमार घोष;
भूपेन्द्र नाथ दत्ता; अरिबं दो घोष, सी.आर. दास और अन्य शािमल हैं।
कथन 2 सही है| बंगाल में क्रांितकारी आं दोलन को बंिकम चंद्र चटजीर् के कायोर्ं से प्रेरणा िमली। आनंद मठ का अिभनंदन बंदे मातरम् बंगाल में
उग्रवादी दल का युद्धघोष बन गया। उन्होंने यह घोषणा करके मातृभूिम के िवचार को धािमर् क महत्व िदया िक सौम्य देवी दुगार् की छिव में
मातृभूिम की भिवष्य की महानता देखी जा सकती है। नए क्रांितकारी सुसमाचार या शिक्त के पंथ ने बंगािलयों से अपने उत्पीड़कों का खून बहाने
का आह्वान िकया।
कथन 3 सही है| अप्रैल 1906 में, अनुशीलन (बारीन्द्र कुमार घोष, भूपेन्द्रनाथ दत्ता) के भीतर एक आं तिरक मंडल ने साप्तािहक युगान्तर शुरू
िकया। इसने क्रांितकारी िहं सा की वकालत शुरू कर दी। उदाहरण के िलए, बािरसल सम्मेलन (अप्रैल 1906) के प्रितभािगयों पर गंभीर पुिलस
बबर्रता के बाद, युगांतर ने िलखा: “उपाय लोगों के पास है। भारत में रहने वाले 30 करोड़ लोगों को उत्पीड़न के इस अिभशाप को रोकने के िलए
अपने 60 करोड़ हाथ उठाने होंगे। शिक्त को बलपूवर्क रोका जाना चािहए।”
Source: https://mu.ac.in/wp-content/uploads/2014/04/FYBA-History-Paper-I-History-of-Modern-India-
Revised-Syllabus-2018-19.pdf
SPECTRUM(CH-First Phase of Revolutionary Activities)

https://www.egyankosh.ac.in/bitstream/123456789/44305/1/Unit-13.pdf

Question Attempted / Student

Question: 94 Mark Flag  Previous Next 

https://academy.forumias.com/beta/lms/viewSolutions Page 167 of 180


Academy ForumIAS | Dashboard 18/02/24, 8:41 PM

Correct Answer : c
Your Answer : c

With reference to Ancient Indian History, consider the following events:

1. Kanishka's Accession to the throne

2. 'Milind Panho' written by Nagasena

3. Invasion of North-Western India by Alexander

4. Junagarh Rock inscription under Rudradaman I

What is the correct chronological sequence of the above given events?

प्राचीन भारतीय इितहास के संदभर् में िनम्निलिखत घटनाओं पर िवचार करें:


1. किनष्क का िसं हासन पर आसीन होना
2. नागसेना द्वारा िलिखत 'िमिलं द पान्हो'
3. िसकन्दर द्वारा उत्तर-पिश्चमी भारत पर आक्रमण
4. रुद्रदामन प्रथम के अधीन जूनागढ़ िशलालेख

उपरोक्त दी गई घटनाओं का सही कालानुक्रिमक क्रम क्या है?

a 3-4-1-2
b 4-2-3-1
c 3-2-1-4
d 4-3-2-1

Explanation
Option c is the correct answer.

In the four centuries following the death of Emperor Asoka and the resulting decline of the Mauryan Empire, parts of India
were subject to the invasion of the Indo-Greeks, Sakas and Kushanas from West and Central Asia. All of them
established themselves as rulers over large parts of India.

The correct chronological sequence of the above given events are as follows:

1) Invasion of north-western India by Alexander (327-325 BCE): India's interaction with the Greeks began with the
invasion of north-western India by Alexander (327-325 BCE) and his conquest of the Punjab region. When he began his
return march to the West, he left the conquered territories under provincial governors.

https://academy.forumias.com/beta/lms/viewSolutions Page 168 of 180


Academy ForumIAS | Dashboard 18/02/24, 8:41 PM

2) 'Milind Panho' written by Nagasena: Menander (c.165/145 BCE) was the best known of the Indo-Greek kings. He is
said to have ruled a large kingdom in the north west India. Menander is mainly remembered as the eponymous hero of the
Buddhist text, Milinda-pinha (questions of Milinda), in which he is engaged in a question-and-answer discussion on
Buddhism with the teacher Nagasena. He is believed to have become a Buddhist and promoted Buddhism.

3) Kanishka's Accession to the throne (78 CE): The best known of the Kushana kings was Kanishka, who is thought to
have ruled from 78 CE till 101 or 102 CE. Ironically, 78 CE is held to be the beginning of the 'Saka era' in the Indian
calendar. Kanishka was an ardent follower of Buddhism and hosted the fourth Buddhist mahasangha or council (the third
council had been held in Pataliputra during Asoka's reign). By now Mahayana Buddhism had become the dominant sect,
and Kanishka supported the missions sent to China to preach Buddhism

4) Junagarh Rock inscription under Rudradaman I: One of the most famous of the Saka kshatrapas was Rudradaman
(130-150 CE). His exploits are celebrated in the famous rock inscription of Junagadh. He's credited to have issued the 1st
long inscription in chaste Sanskrit.

5) It mentions the repairs he undertook to improve the Sudershana Lake. Junagarh Rock also contains inscriptions from
Ashoka and Skandgupta (from the Gupta Period almost 800 years later).

Source: Tamil Nadu 11th Textbook Lesson 6 Polity and Society in Post-Mauryan Period

िवकल्प c सही उत्तर है।


सम्राट अशोक की मृत्यु और उसके पिरणामस्वरूप मौयर् साम्राज्य के पतन के बाद की चार शतािब्दयों में, भारत के कुछ िहस्सों पर पिश्चम और
मध्य एिशया से इं डो-यूनानी, शक और कुषाणों का आक्रमण हुआ। उन सभी ने खुद को भारत के बड़े िहस्से पर शासक के रूप में स्थािपत
िकया।
ऊपर दी गई घटनाओं का सही कालानुक्रिमक क्रम इस प्रकार है:
· िसकंदर द्वारा उत्तर-पिश्चमी भारत पर आक्रमण (327-325 ईसा पूवर्): यूनािनयों के साथ भारत का संपकर् िसकंदर (327-325 ईसा पूवर्)
द्वारा उत्तर-पिश्चमी भारत पर आक्रमण और पंजाब क्षेत्र पर उसकी िवजय के साथ शुरू हुआ। जब उन्होंने पिश्चम की ओर वापसी यात्रा शुरू
की, तो उन्होंने िविजत क्षेत्रों को प्रांतीय गवनर्रों के अधीन छोड़ िदया।
· नागसेना द्वारा िलिखत 'िमिलं द पान्हो': मेनेंडर (लगभग 165/145 ईसा पूव)र् इं डो-ग्रीक राजाओं में सबसे प्रिसद्ध थे। ऐसा कहा जाता है िक
उसने उत्तर पिश्चम भारत में एक बड़े साम्राज्य पर शासन िकया था। मेनेंडर को मुख्य रूप से बौद्ध ग्रंथ, 'िमिलं द पान्हो' (िमिलं डा के प्रश्न) के
नामांिकत नायक के रूप में याद िकया जाता है, िजसमें वह िशक्षक नागसेना के साथ बौद्ध धमर् पर प्रश्न-उत्तर चचार् में लगे हुए हैं। ऐसा माना
जाता है िक वह बौद्ध बन गये और उन्होंने बौद्ध धमर् का प्रचार िकया।
· किनष्क का िसं हासन पर बैठना (78 ई.पू.): कुषाण राजाओं में सबसे प्रिसद्ध किनष्क था, िजसके बारे में माना जाता है िक उसने 78 ई.पू. से
101 या 102 ई. तक शासन िकया था। िवडंबना यह है िक भारतीय कैलेंडर में 78 ईस्वी को "शक युग" की शुरुआत माना जाता है। किनष्क
बौद्ध धमर् का प्रबल अनुयायी था और उसने चौथे बौद्ध महासंघ या पिरषद की मेजबानी की थी (तीसरी पिरषद अशोक के शासनकाल के दौरान
पाटिलपुत्र में आयोिजत की गई थी)। अब तक महायान बौद्ध धमर् प्रमुख संप्रदाय बन गया था और किनष्क ने बौद्ध धमर् का प्रचार करने के िलए
चीन भेजे गए िमशनों का समथर्न िकया था
· रुद्रदामन प्रथम के अधीन जूनागढ़ िशलालेख: शक क्षत्रपों में सबसे प्रिसद्ध में से एक रुद्रदामन (130-150 ई.पू.) था। जूनागढ़ के प्रिसद्ध
िशलालेख में उनके कायोर्ं का गुणगान िकया गया है। उन्हें शुद्ध संस्कृत में पहला लंबा िशलालेख जारी करने का श्रेय िदया जाता है।
इसमें सुदशर्न झील को बेहतर बनाने के िलए उनके द्वारा की गई मरम्मत का उल्लेख है। जूनागढ़ रॉक में अशोक और स्कंदगुप्त (लगभग 800
साल बाद गुप्त काल से) के िशलालेख भी शािमल हैं।

Source: Tamil Nadu 11th Textbook Lesson 6 Polity and Society in Post-Mauryan Period

https://academy.forumias.com/beta/lms/viewSolutions Page 169 of 180


Academy ForumIAS | Dashboard 18/02/24, 8:41 PM

Question Attempted / Student

Question: 95 Mark Flag  Previous Next 

Correct Answer : b
Your Answer :

With reference to Indian Economy, consider the following statements regarding the Virtual Digital Assets (VDAs):

1. VDAs refer to digitally transacted and stored assets such as foreign currencies.

2. Regardless of their physical presence, persons providing VDA service in India must register themselves with the
Financial Intelligence Unit India (FIU IND).

3. Transactions of VDAs are regulated by the Prevention of Money Laundering Act (PMLA) 2002.

How many of the statements given above are correct?

भारतीय अथर्व्यवस्था के संदभर् में, वचुर्अल िडिजटल एसेट्स (VDAs) के संबंध में िनम्निलिखत कथनों पर िवचार करें:
1. वीडीए, िडिजटल रूप से लेनदेन और संग्रहीत संपित्तयों जैसे िवदेशी मुद्राओं को संदिभर् त करता है।
2. अपनी भौितक उपिस्थित के बावजूद, भारत में वीडीए सेवा प्रदान करने वाले व्यिक्तयों को खुद को फाइनेंिशयल इं टेिलजेंस यूिनट इं िडया
(एफआईयू आईएनडी) के साथ पंजीकृत करना होगा।
3. वीडीए के लेनदेन को धन शोधन िनवारण अिधिनयम (पीएमएलए), 2002 द्वारा िविनयिमत िकया जाता है|

ऊपर िदए गए कथनों में से िकतने सही हैं?

a Only one केवल एक


b Only two केवल दो
c All three तीनों
d None कोई नहीं

Explanation
Option b is the correct answer.

Recently, the Financial Intelligence Unit India (FIU IND) has issued Show Cause Notices to 9 offshore Virtual Digital
Assets Service Providers (VDA-SPs) for not complying with the PMLA regulations. Statement 1 is incorrect: Foreign
currencies and Indian currency are excluded from the category of Virtual Digital Assets (VDAs), whether digitally
transferred or not. VDAs specifically encompass digital assets like cryptocurrencies, Non-Fungible Tokens (NFTs),

https://academy.forumias.com/beta/lms/viewSolutions Page 170 of 180


Academy ForumIAS | Dashboard 18/02/24, 8:41 PM

gaming coins, etc., which are distinct from both Indian and foreign currencies. The Financial Action Task Force (FATF)
defines a virtual asset as 'A digital representation of value that can be digitally traded, transferred and used for payment or
investment purposes'.

Statement 2 is correct: In India, regardless of their physical presence, persons providing VDA service in India must
register themselves with the Financial Intelligence Unit India (FIU IND). Failure to register themselves amounts to
non compliance by the concerned party and FIU IND has the authority to take appropriate actions to ensure compliance.

Statement 3 is correct: The provisions of the Prevention of Money-laundering Act, 2002 will extend to cover
exchange between virtual digital assets and fiat currencies, transfer of virtual digital assets. Hence the transactions of
VDAs are regulated by the Prevention of Money Laundering Act (PMLA) 2002.

Source: https://www.thehindu.com/business/Economy/why-did-fiu-ind-act-against-virtual-asset-
providers/article67695217.ece

िवकल्प b सही उत्तर है।


हाल ही में, फाइनेंिशयल इं टेिलजेंस यूिनट इं िडया (FIU IND) ने PMLA िनयमों का अनुपालन नहीं करने के िलए 9 ऑफशोर वचुर्अल
िडिजटल एसेट्स सिवर् स प्रोवाइडसर् (VDA-SPs) को कारण बताओ नोिटस जारी िकया है। कथन 1 गलत है: िवदेशी मुद्राओं और भारतीय
मुद्रा को वचुर्अल िडिजटल एसेट्स (वीडीए) की श्रेणी से बाहर रखा गया है, चाहे िडिजटल रूप से हस्तांतिरत िकया गया हो या नहीं। वीडीए में
िवशेष रूप से िक्रप्टोकरेंसी, अपूरणीय टोकन (एनएफटी), गेिमं ग िसक्के आिद जैसी िडिजटल संपित्तयां शािमल हैं, जो भारतीय और िवदेशी
दोनों मुद्राओं से अलग हैं। िवत्तीय कारर्वाई कायर् बल (एफएटीएफ) एक आभासी संपित्त को "मूल्य का एक िडिजटल प्रितिनिधत्व िजसे
िडिजटल रूप से व्यापार, हस्तांतिरत और भुगतान या िनवेश उद्देश्यों के िलए उपयोग िकया जा सकता है" के रूप में पिरभािषत करता है।
कथन 2 सही है: भारत में, उनकी भौितक उपिस्थित की परवाह िकए िबना, भारत में वीडीए सेवा प्रदान करने वाले व्यिक्तयों को िवत्तीय खुिफया
इकाई भारत (एफआईयू आईएनडी) के साथ खुद को पंजीकृत करना होगा। स्वयं को पंजीकृत करने में िवफलता संबंिधत पक्ष द्वारा गैर-
अनुपालन के समान है और FIU IND के पास अनुपालन सुिनिश्चत करने के िलए उिचत कारर्वाई करने का अिधकार है।
कथन 3 सही है: धन-शोधन िनवारण अिधिनयम, 2002 के प्रावधानों का िवस्तार आभासी िडिजटल संपित्तयों और िफएट मुद्राओं के बीच
आदान-प्रदान, आभासी िडिजटल संपित्तयों के हस्तांतरण तक होगा। इसिलए वीडीए के लेनदेन को धन शोधन िनवारण अिधिनयम
(पीएमएलए), 2002 द्वारा िनयंित्रत िकया जाता है

Source: https://www.thehindu.com/business/Economy/why-did-fiu-ind-act-against-virtual-asset-
providers/article67695217.ece

Question Attempted / Student

Question: 96 Mark Flag  Previous Next 

Correct Answer : b
Your Answer : b

Consider the following statements regarding Chalcolithic period:

https://academy.forumias.com/beta/lms/viewSolutions Page 171 of 180


Academy ForumIAS | Dashboard 18/02/24, 8:41 PM

1. Black and red ware pottery were used during this period.

2. There was large scale use of iron for making tools and weapons.

3. Stone and Copper were used by agricultural communities.

How many of the above statements are correct?

ताम्रपाषाण काल के संबंध में िनम्निलिखत कथनों पर िवचार करें:


1. इस काल में काले और लाल बतर्नों का प्रयोग िकया जाता था।
2. औज़ार और हिथयार बनाने के िलए लोहे का बड़े पैमाने पर उपयोग होता था।
3. पत्थर और तांबे का उपयोग कृिष समुदायों द्वारा िकया जाता था।

उपरोक्त में से िकतने कथन सही हैं?

a Only one केवल एक


b Only two केवल दो
c All three तीनों
d None कोई नहीं

Explanation
Option b is the correct answer.

The Chalcolithic period, which refers to the period when copper and stone were used, was designated as the stone-copper
phase. These civilizations have a specific regional identity that distinguishes them.

Statement 1 is correct: The feature of the pottery during Chalcolithic period is the black colour inside and near the rim
on outside, and red colour, over the rest of the body. The pottery is mostly wheel turned, though some pots are also
handmade. It is made of the clay and has a fine fabric with thin walls. Black and Red Ware pottery with paintings has
been found at sites in Rajasthan, Madhya Pradesh, Bihar and West Bengal. But in the Black and red ware of the doab
area there is absence of paintings.

Statement 2 is incorrect: Use of iron at large scale had not been started in making of various tools and weapons
during Chalcolithic period. Though use of iron had been started but not in intensive way. It is copper and stone tools and
weapons that were used in intensively in the period of Chalcolithic period.

Statement 3 is correct: Stone and Copper were used by agricultural communities which have been reported from
eastern India. The people of the Chalcolithic phase produced wheat and rice, they also cultivated bajra. They also produced
several pulses such as lentil (masur), black gram, green gram, and grass pea.

Source: Unit-10.pdf (egyankosh.ac.in)

िवकल्प b सही उत्तर है।

https://academy.forumias.com/beta/lms/viewSolutions Page 172 of 180


Academy ForumIAS | Dashboard 18/02/24, 8:41 PM

ताम्रपाषाण काल, जो उस काल को संदिभर् त करता है जब तांबे और पत्थर का उपयोग िकया जाता था, को पाषाण-तांबा चरण के रूप में
नािमत िकया गया था। इन सभ्यताओं की एक िविशष्ट क्षेत्रीय पहचान होती है जो उन्हें अलग करती है।
कथन 1 सही है: ताम्रपाषाण काल के दौरान िमट्टी के बतर्नों की िवशेषता अंदर और बाहर िरम के पास काला रंग और के बाकी के िहस्सों पर
लाल रंग है। िमट्टी के बतर्न अिधकतर पिहए पर बने होते हैं, हालाँिक कुछ बतर्न हस्तिनिमर् त भी होते हैं। यह िमट्टी से बना है और इसमें पतली
दीवारों के साथ बिढ़या कपड़ा है। राजस्थान, मध्य प्रदेश, िबहार और पिश्चम बंगाल के स्थलों पर िचत्रों के साथ काले और लाल मृदभांड के
बतर्न पाए गए हैं। लेिकन दोआब क्षेत्र के काले और लाल बतर्नों में िचत्रकला का अभाव है।
कथन 2 गलत है: ताम्रपाषाण काल के दौरान िविभन्न उपकरणों और हिथयारों के िनमार्ण में बड़े पैमाने पर लोहे का उपयोग शुरू नहीं िकया गया
था। यद्यिप लोहे का प्रयोग प्रारम्भ हो चुका था परन्तु गहनता से नहीं। यह तांबे और पत्थर के उपकरण और हिथयार हैं िजनका उपयोग
ताम्रपाषाण काल में गहनता से िकया गया था।
कथन 3 सही है: पत्थर और तांबे का उपयोग कृिष समुदायों द्वारा िकया जाता था जो पूवीर् भारत से िरपोटर् िकया गया है। ताम्रपाषाण काल के
लोग गेहूँ और चावल का उत्पादन करते थे, वे बाजरे की भी खेती करते थे। उन्होंने कई दालें भी पैदा कीं जैसे मसूर, काला चना, हरा चना और
घास मटर।

Source: Unit-10.pdf (egyankosh.ac.in)

Question Attempted / Student

Question: 97 Mark Flag  Previous Next 

Correct Answer : b
Your Answer :

Which of the following statements correctly explains the term 'Karmaprabhrita and Kashayaprabhrita'?

िनम्निलिखत में से कौन सा कथन 'कमर्प्राभृत और कषायप्राभृत' शब्द की सही व्याख्या करता है?

a
They are treasury head officers under revenue department of Mauryan empire. वे मौयर् साम्राज्य के राजस्व िवभाग के अंतगर्त
प्रमुख राजकोष अिधकारी थे।
These are the holy texts of Digambar sect of Jainism. ये जैन धमर् के िदगंबर संप्रदाय के पिवत्र ग्रंथ
b हैं।
c
These are songs written for marriages by the women in Maharashtra region. ये महाराष्ट्र क्षेत्र की मिहलाओं द्वारा िववाह के
िलए िलखे गए गीत हैं।
d
These are inscriptions of Chola empire that give detailed account of its village administration. ये चोल साम्राज्य के
िशलालेख हैं जो इसके ग्राम प्रशासन का िवस्तृत िववरण देते हैं।

https://academy.forumias.com/beta/lms/viewSolutions Page 173 of 180


Academy ForumIAS | Dashboard 18/02/24, 8:41 PM

Explanation
Option b is the correct answer.

'Karmaprabhrita' and 'Kashayaprabhrita' are the sacred Jain texts related to the Digambaras sect. Karmaprabhrita
(Chapters on Karma), also called Shatkhandagama (Scripture of Six Sections) composed by Pushpadanta and
Bhutabalin and based on the principles of karma, and the Kashayaprabhrita (Chapters on the Kashayas) compiled by
Gunadhara and it deals with the passions (kashaya) that defile and bind the soul.

Option a is incorrect: 'Sannidata' were the treasury head officers in revenue department in Mauryan empire.

1) Samaharta- Revenue collector under revenue department

2) Council of ministers- Mantriparishad (Purohita, Mahamantri, Senapati, Yuvaraja)

3) Amatyas- civil servants- looks day to day administration.

4) Adyakshas- regulate foreign trade and custom duties

Option c is incorrect: Ovi is a form of music is from Maharashtra and Goa. They are usually the songs of women i.e.;
they are sung by women during leisure time and when they are completing their household work. They usually contain
four small lines of poetry. These are usually songs written for marriages, pregnancy and also lullabies for children.

Option d is incorrect: It is Uttaramerur inscription that speaks much about Chola administration specially of its
village administration made under reign of Parantaka-I. It mentions the qualifications, mode of election, disqualification
criteria and constitution of committee for local governance. It also mentions village autonomy with assemblies (Sabha,
Urs and Nagara) developed during this period.

Source: Jainism - Festivals | Britannica

Class 11 history NCERT- Mauryan administration

िवकल्प b सही उत्तर है।


'कमर्प्राभृत' और 'कषायप्राभृत' िदगंबर संप्रदाय से संबंिधत पिवत्र जैन ग्रंथ हैं। कमर्प्राभृत (कमर् पर अध्याय), िजसे षट् खंडागम (छह खंडों का
धमर्ग्रंथ) भी कहा जाता है, पुष्पदंत और भूतबिलन द्वारा रिचत और कमर् के िसद्धांतों पर आधािरत है, और गुणधर द्वारा संकिलत कषायप्रभृत
(कषायों पर अध्याय) और यह जुनून (कषाय) से संबंिधत है। जो आत्मा को अशुद्ध और बांधते हैं।
िवकल्प a गलत है: 'सिन्नदता' मौयर् साम्राज्य में राजस्व िवभाग में राजकोष के प्रमुख अिधकारी थे।
· समाहतार् - राजस्व िवभाग के अंतगर्त राजस्व संग्रहकतार्
· मंित्रपिरषद- मंित्रपिरषद (पुरोिहत, महामन्त्री, सेनापित, युवराज)

· अमात्य- िसिवल सेवक- दैिनक प्रशासन देखते हैं।


· अद्यक्ष - िवदेशी व्यापार और सीमा शुल्क को िविनयिमत करते हैं
िवकल्प c गलत है: ओवी, संगीत का एक रूप है जो महाराष्ट्र और गोवा का है। वे आम तौर पर मिहलाओं के गीत हैं यानी; इन्हें मिहलाओं द्वारा
ख़ाली समय के दौरान और जब वे अपना घरेलू काम पूरा कर रही होती हैं, गाया जाता है। उनमें आमतौर पर किवता की चार छोटी पंिक्तयाँ
होती हैं। ये आमतौर पर िववाह, गभार्वस्था और बच्चों के िलए लोरी के िलए िलखे गए गीत हैं।
िवकल्प d गलत है: यह उत्तरमेरूर िशलालेख है जो चोल प्रशासन, िवशेष रूप से परांतक-प्रथम के शासनकाल के तहत बनाए गए उसके ग्राम
प्रशासन के बारे में बहुत कुछ बताता है। इसमें स्थानीय शासन के िलए योग्यता, चुनाव का तरीका, अयोग्यता मानदंड और सिमित के गठन का
उल्लेख है। इसमें इस अविध के दौरान िवकिसत सभाओं (सभा, उसर् और नागारा) के साथ ग्राम स्वायत्तता का भी उल्लेख है।

https://academy.forumias.com/beta/lms/viewSolutions Page 174 of 180


Academy ForumIAS | Dashboard 18/02/24, 8:41 PM

Source: Jainism - Festivals | Britannica

Class 11 history NCERT- Mauryan administration

Question Attempted / Student

Question: 98 Mark Flag  Previous Next 

Correct Answer : b
Your Answer : c

Consider the following:

1. Issuance of silver currency known as Rupaiya.

2. Construction of Grand Trunk road connecting Sonargaon with Takshila.

3. Abolition of custom duty that was levied at the time of sale of goods.

4. Codification of the Islamic laws.

How many of the above reforms were carried out by the Sher Shah Suri?

िनम्निलिखत पर िवचार करें:


1. चाँदी की मुद्रा जारी करना िजसे रुपया कहा जाता है।
2. सोनारगांव को तक्षिशला से जोड़ने वाली ग्रैंड ट्रंक रोड का िनमार्ण।
3. माल की िबक्री के समय लगने वाली कस्टम ड्यूटी को खत्म करना।
4. इस्लामी कानूनों का संिहताकरण|

शेरशाह सूरी द्वारा उपरोक्त सुधारों में से िकतने िकए गए थे?

a Only one केवल एक


b Only two केवल दो
c Only three केवल तीन
d All four सभी चार

Explanation
Option b is the correct answer.

https://academy.forumias.com/beta/lms/viewSolutions Page 175 of 180


Academy ForumIAS | Dashboard 18/02/24, 8:41 PM

Statement 1 is correct: At the time of Sher Shah Suri to the throne of Delhi. Different types of mixed currencies were in
circulation. Sher Shah Suri abolished all these currencies and issued two new currencies made up of silver & copper.
Silver currency issued by him is known as Rupaiya. The currency reforms of Sher Shah were so lasting i.e. rupaiya
continued throughout the Mughal period as well as East India Company for a long time (till 1883).

Statement 2 is correct: Sher Shah constructed several roads to connect various parts of India. The most important road
built by Sher Shah connected Sonar Gaon in Bengal with Takshila. At present, it is known as the Grand Trunk (GT)
road. One road was built to connect Agra with Burhanpur & other road connected Agra with Jodhpur. The 4th road built
by Sher Shah connected Lahore to Multan. Rest houses (Sarai) were constructed for benefit of travelers.

Statement 3 is incorrect: Sher Shah also introduced other reforms to promote the growth of trade and commerce. In his
entire empire, customs duty for goods were paid only at two places: goods produced in Bengal or imported from outside
paid customs duty at the border of Bengal and Bihar at Sikrigali, and goods coming from West and Central Asia paid
customs duty at the Indus. No one was allowed to levy customs at roads, ferries or towns anywhere else. Duty was paid a
second time at the time of sale of goods.

Statement 4 is incorrect: Sher Shah's son and successor, Islam Shah. codified the laws, thus doing away with the
necessity of depending on a special set of people who could interpret the Islamic law. Islam Shah also tried to curb the
powers and privileges of the nobles, and to pay cash salaries to soldiers.

Knowledge Base:

1) Sher Shah Suri initiated various welfare measures for benefit of the people. Land grants were issued by him to the man
of learning and religion belonging to the communities of Hindus as well as Muslims. Free kitchens were established by
him to feed the poor & hungry.

2) Sher Shah set up a strong army in order to administer his vast empire. He dispensed with tribal levies under tribal chiefs,
and recruited soldiers directly, after verifying their character.

3) Every soldier had his descriptive roll (chehra) recorded, and his horse branded with the imperial sign so that horses of
inferior quality may not be substituted. Sher Shah seems to have borrowed this system, known as the dagh (branding)
system, from the military reforms of Alauddin Khalji.

4) Sher Shah introduced a new system of land revenue known as the Zabti system.

5) Sher Shah reformed the postal system as well. The Sarai was used as Dak Chauki (PO). A relay system was used for the
transportation of posts. The in-charge of the post office was known as Daroga-i-Dag Chauki.

6) For communication of secret information, spies were appointed by Sher Shah.

Source: History of Medieval India by Satish Chandra - Chapter 12 - Struggle for Empire in North India—II Mughals and
Afghans (1525-1555).

िवकल्प b सही उत्तर है।


कथन 1 सही है: शेरशाह सूरी के समय िदल्ली की गद्दी पर। िविभन्न प्रकार की िमिश्रत मुद्राएँ प्रचलन में थीं। शेरशाह सूरी ने इन सभी मुद्राओं
को समाप्त कर िदया और चांदी और तांबे से बनी दो नई मुद्राएँ जारी कीं। उनके द्वारा जारी चांदी की मुद्रा को रुपया के नाम से जाना जाता है।
शेरशाह के मुद्रा सुधार इतने स्थायी थे अथार्त रुपया पूरे मुगल काल के साथ-साथ ईस्ट इं िडया कंपनी में भी लंबे समय तक (1883 तक) जारी
रहा।

https://academy.forumias.com/beta/lms/viewSolutions Page 176 of 180


Academy ForumIAS | Dashboard 18/02/24, 8:41 PM

कथन 2 सही है: शेरशाह ने भारत के िविभन्न िहस्सों को जोड़ने के िलए कई सड़कों का िनमार्ण कराया। शेरशाह द्वारा बनवाई गई सबसे
महत्वपूणर् सड़क बंगाल के सोनार गाँव को तक्षिशला से जोड़ती थी। वतर्मान में इसे ग्रैंड ट्रंक (जीटी) रोड के नाम से जाना जाता है।
एक सड़क आगरा को बुरहानपुर से जोड़ने के िलए बनाई गई थी और दू सरी सड़क आगरा को जोधपुर से जोड़ने के िलए बनाई गई थी। शेरशाह
द्वारा बनवाई गई चौथी सड़क लाहौर को मुल्तान से जोड़ती थी। याित्रयों के लाभ के िलए िवश्राम गृह (सराय) का िनमार्ण िकया गया।
कथन 3 गलत है: शेरशाह ने व्यापार और वािणज्य के िवकास को बढ़ावा देने के िलए अन्य सुधार भी पेश िकए। उनके पूरे साम्राज्य में, माल के
िलए सीमा शुल्क का भुगतान केवल दो स्थानों पर िकया जाता था: बंगाल में उत्पािदत या बाहर से आयाितत माल पर बंगाल और िबहार की
सीमा पर िसकरीगली में सीमा शुल्क का भुगतान िकया जाता था, और पिश्चम और मध्य एिशया से आने वाले माल पर सीमा शुल्क का भुगतान
िकया जाता था। िसं धु। िकसी को भी सड़कों, घाटों या कस्बों में कहीं और सीमा शुल्क लगाने की अनुमित नहीं थी। माल की िबक्री के समय
दू सरी बार शुल्क का भुगतान िकया गया।
कथन 4 गलत है: शेरशाह का पुत्र और उत्तरािधकारी, इस्लाम शाह। कानूनों को संिहताबद्ध िकया, इस प्रकार इस्लामी कानून की व्याख्या करने
वाले लोगों के एक िवशेष समूह पर िनभर्र रहने की आवश्यकता समाप्त हो गई। इस्लाम शाह ने अमीरों की शिक्तयों और िवशेषािधकारों पर
अंकुश लगाने और सैिनकों को नकद वेतन देने का भी प्रयास िकया।
ज्ञानधार:
1) शेरशाह सूरी ने लोगों के लाभ के िलए िविभन्न कल्याणकारी कदम उठाए। उनके द्वारा िहं दुओ ं के साथ-साथ मुसलमानों के समुदायों से
संबंिधत िशक्षा और धमर् के लोगों को भूिम अनुदान जारी िकए गए थे। गरीबों और भूखों को खाना िखलाने के िलए उनके द्वारा िनःशुल्क रसोई
की स्थापना की गई।
2) शेरशाह ने अपने िवशाल साम्राज्य का प्रबंधन करने के िलए एक मजबूत सेना की स्थापना की। उन्होंने जनजातीय प्रमुखों के अधीन
जनजातीय करों को समाप्त कर िदया और उनके चिरत्र की पुिष्ट करने के बाद सीधे सैिनकों की भतीर् की।
3) प्रत्येक सैिनक का वणर्नात्मक रोल (चेहरा) दजर् िकया गया था, और उसके घोड़े पर शाही िचन्ह अंिकत िकया गया था तािक घिटया गुणवत्ता
के घोड़ों को प्रितस्थािपत न िकया जा सके। ऐसा प्रतीत होता है िक शेरशाह ने इस प्रणाली को, िजसे दाग (ब्रांिडं ग) प्रणाली के नाम से जाना
जाता है, अलाउद्दीन िखलजी के सैन्य सुधारों से उधार ली थी।
4) शेरशाह ने भू-राजस्व की एक नई प्रणाली शुरू की िजसे ज़ब्ती प्रणाली के नाम से जाना जाता है।
5) शेरशाह ने डाक व्यवस्था में भी सुधार िकया। सराय का उपयोग डाक चौकी (पीओ) के रूप में िकया जाता था। पदों के पिरवहन के िलए
एक िरले प्रणाली का उपयोग िकया गया था। डाकघर के प्रभारी को दरोगा-ए-दाग चौकी के नाम से जाना जाता था।
6) गुप्त सूचनाओं के संचार के िलए शेरशाह द्वारा जासूसों की िनयुिक्त की जाती थी।

Source: History of Medieval India by Satish Chandra - Chapter 12 - Struggle for Empire in North India—II
Mughals and Afghans (1525–1555).

Question Attempted / Student

Question: 99 Mark Flag  Previous Next 

Correct Answer : c
Your Answer : c

Consider the following statements with reference to the Khond Uprising:

1. Khonds were the tribals of the hilly tracts who revolted against the Company rule.

https://academy.forumias.com/beta/lms/viewSolutions Page 177 of 180


Academy ForumIAS | Dashboard 18/02/24, 8:41 PM

2. Khonds were joined by the Ghumsar and Kalahandi tribals.

3. The revolt was against the decision of colonial administration to stop human sacrifices.

How many of the above statements are correct?

खोंड िवद्रोह के संदभर् में िनम्निलिखत कथनों पर िवचार करें:


1. खोंड, पहाड़ी इलाकों के आिदवासी थे िजन्होंने कंपनी शासन के िखलाफ िवद्रोह िकया था।
2. खोंड घुमसर और कालाहांडी आिदवािसयों से जुड़े हुए थे।
3. यह िवद्रोह मानव बिल रोकने के औपिनवेिशक प्रशासन के फैसले के िखलाफ था।

उपरोक्त में से िकतने कथन सही हैं?

a Only one केवल एक


b Only two केवल दो
c All three तीनों
d None कोई नहीं

Explanation
Option c is the correct answer.

Statement 1 is correct: From the year 1837 to 1856 the Khonds, who were the tribals of the hilly tracts extending from
Odisha to the Srikakulam and Visakhapatnam districts of Andhra Pradesh revolted against Company rule.

Statement 2 is correct: Chakra Bisoi was a young raja who led the Khonds who were joined by the Ghumsar,
Kalahandi and other tribals to oppose the suppression of human sacrifice, new taxes, and the entry of zamindars
into their areas. With Chakra Bisoi's disappearance, the uprising came to an end.

Statement 3 is correct: The specific problem among the Khonds centered on the determination of the colonial
administration to stop the human sacrifices associated with the 'meriah' sacrifice. The new pressures and uncertainties
forced the Khonds to appease their gods for good harvests of turmeric, but the colonial administration tried to stop this
practice. Some colonial officials hoped that the improvement of material conditions of the Khonds would serve to liquidate
the 'meriah' sacrifice

Source: https://egyankosh.ac.in/bitstream/123456789/19939/1/Unit-7.pdf

Brief history of modern India. Spectrum. Chapter name- People's Resistance Against British Before 1857. Page no - 157

िवकल्प c सही उत्तर है।


कथन 1 सही है: वषर् 1837 से 1856 तक खोंड, जो ओिडशा से लेकर आं ध्र प्रदेश के श्रीकाकुलम और िवशाखापत्तनम िजलों तक फैले पहाड़ी
इलाकों के आिदवासी थे, ने कंपनी शासन के िखलाफ िवद्रोह िकया।

https://academy.forumias.com/beta/lms/viewSolutions Page 178 of 180


Academy ForumIAS | Dashboard 18/02/24, 8:41 PM

कथन 2 सही है: चक्र िबसोई एक युवा राजा थे िजन्होंने खोंडों का नेतृत्व िकया था, जो मानव बिल के दमन, नए करों और अपने क्षेत्रों में
जमींदारों के प्रवेश का िवरोध करने के िलए घुमसर, कालाहांडी और अन्य आिदवािसयों से जुड़े थे। चक्र िबसोई के गायब होने के साथ ही
िवद्रोह समाप्त हो गया।
कथन 3 सही है: खोंडों के बीच िविशष्ट समस्या 'मेिरया' बिलदान से जुड़े मानव बिलदानों को रोकने के िलए औपिनवेिशक प्रशासन के दृढ़
संकल्प पर केंिद्रत थी। नए दबावों और अिनिश्चतताओं ने खोंडों को हल्दी की अच्छी फसल के िलए अपने देवताओं को खुश करने के िलए
मजबूर िकया, लेिकन औपिनवेिशक प्रशासन ने इस प्रथा को रोकने की कोिशश की। कुछ औपिनवेिशक अिधकािरयों को उम्मीद थी िक खोंडों
की भौितक िस्थितयों में सुधार से 'मेिरया' बिलदान को ख़त्म करने में मदद िमलेगी
Source: https://egyankosh.ac.in/bitstream/123456789/19939/1/Unit-7.pdf

Brief history of modern India. Spectrum. Chapter name- People’s Resistance Against British Before 1857.
Page no – 157

Question Attempted / Student

Question: 100 Mark Flag  Previous Next 

Correct Answer : a
Your Answer : a

Which one of the following best describes the meaning of the term 'Inverted Duty Structure', recently seen in sews?

िनम्निलिखत में से कौन सा हाल ही में समाचारों में देखे गए 'इनवटेर्ड ड्यूटी स्ट्रक्चर' शब्द के अथर् का सबसे अच्छा वणर्न करता है?

a
It is a duty structure where inputs are taxed at higher rates than finished products. यह एक शुल्क संरचना है, जहां तैयार
उत्पादों की तुलना में इनपुट पर अिधक दर से कर लगाया जाता है।
b
It is a duty structure which allows producers to offset the taxes paid on inputs. यह एक शुल्क संरचना है, जो उत्पादकों को
इनपुट पर भुगतान िकए गए करों की भरपाई करने की अनुमित देती है।
c
It is a duty structure wherein taxes are waived on exports but not on imported goods. यह एक शुल्क संरचना है, िजसमें
िनयार्त पर कर माफ िकया जाता है लेिकन आयाितत वस्तुओ ं पर नहीं।
d
It involves imposition of higher import duties primarily to discourage imports to safeguard the domestic industries.
इसमें मुख्य रूप से घरेलू उद्योगों की सुरक्षा के िलए आयातों को हतोत्सािहत करने के िलए उच्च आयात शुल्क लगाना शािमल है।

Explanation

https://academy.forumias.com/beta/lms/viewSolutions Page 179 of 180


Academy ForumIAS | Dashboard 18/02/24, 8:41 PM

Option a is the correct answer.

Recently, the Union Ministry of Commerce and Industry has asked the Union Finance Ministry to address the inverted
duty structure in the forthcoming Union Budget for 2024-25. TheMinistry noted that inverted duty structures are not
economically efficient and was hopeful that the Budget 2024-25 would address it.

Option a is correct: An inverted duty structure occurs when the taxation on inputs exceeds that of finished
products, potentially making domestic industry import dependent. This goes against the objective of the Government's
'Make in India' as it incentivizes the import of finished goods over raw materials. Furthermore such duty structures can be
exacerbated by Free Trade Agreement (FTA) pacts, such as those with ASEAN countries, where duty elimination under
the agreement may render finished product imports duty-free while maintaining import taxes on intermediates.

Source: https://www.thehindubusinessline.com/economy/commerce-ministry-submits-list-of-items-to-finmin-
for-removal-of-inverted-duties/article67638973.ece

िवकल्प a सही उत्तर है।


हाल ही में, केंद्रीय वािणज्य और उद्योग मंत्रालय ने केंद्रीय िवत्त मंत्रालय से 2024-25 के आगामी केंद्रीय बजट में 'इनवटेर्ड ड्यूटी स्ट्रक्चर' पर
ध्यान देने को कहा है। मंत्रालय ने कहा िक इनवटेर्ड ड्यूटी संरचनाएं आिथर् क रूप से कुशल नहीं हैं और उम्मीद है िक बजट 2024-25 इसका
समाधान करेगा।
िवकल्प a सही है: 'इनवटेर्ड ड्यूटी स्ट्रक्चर' तब होती है जब इनपुट पर कराधान तैयार उत्पादों से अिधक हो जाता है, िजससे संभािवत रूप से
घरेलू उद्योग आयात पर िनभर्र हो जाता है। यह सरकार के 'मेक इन इं िडया' के उद्देश्य के िखलाफ है क्योंिक यह कच्चे माल के बजाय तैयार
माल के आयात को प्रोत्सािहत करता है। इसके अलावा ऐसी शुल्क संरचनाओं को मुक्त व्यापार समझौते (एफटीए) संिधयों द्वारा बढ़ाया जा
सकता है, जैसे िक आिसयान देशों के साथ, जहां समझौते के तहत शुल्क उन्मूलन मध्यवतीर् उत्पादों पर आयात करों को बनाए रखते हुए तैयार
उत्पाद आयात को शुल्क मुक्त बना सकता है।

Source: https://www.thehindubusinessline.com/economy/commerce-ministry-submits-list-of-items-to-finmin-for-
removal-of-inverted-duties/article67638973.ece

Question Attempted / Student

 Go Back

https://academy.forumias.com/beta/lms/viewSolutions Page 180 of 180

You might also like